Apostila de Provas EEAr COMPLETA

April 2, 2018 | Author: Luis Gustavo | Category: Subject (Grammar), Loneliness, Time, Metaphor, Suicide


Comments



Description

1ORTUGUÊS - EEAR 1/2001-TURMA A Um dedo na multidão (Dorival Coutinho da Silva) Um homem vai se atirar do sétimo andar de um edifício em pleno centro de São Paulo. Não é o primeiro. Nem será o último. Afinal nem todos os homens são poetas. Que os poetas desafogam na lira os desencantos. Um homem comum vai jogar as migalhas da vida Aos pombos famintos Que se aglomeram. O grupo que ensaia capoeira na praça Ao som do berimbau Deixa de ser atração. Todo mundo quer ver o homem que vai pular Do sétimo andar. Doida escalada aos prédios vizinhos, O melhor ângulo, Posições estratégicas... Quem sabe escapará algum detalhe Nos noticiários de amanhã? O estafeta liga para a repartição: ―Que venham logo! Pois um homem vai pular do sétimo andar. E tem até TV!‖ Expectativa pesada. Olhos fincados no sétimo andar. Uma palavrinha ao companheiro do lado, Um acocorar para prender os cadarços, Uma visita aos sanitários... Nem cogitar! Tampouco amendoins e pipocas Antecipam os louros na espera. Mas agora um dedo na multidão, Entre mil indicadores algozes, eretos, Pressiona os três dígitos Da salvação. E só depois de uma operação súbita Audaciosa Precisa Remove-se o trágico misantropo Para o interior do prédio (Mas não de si mesmo). Cortinas cerradas. Silêncio na praça. O homem não se jogou do sétimo andar E a multidão se dispersa Melancólica Desapontada Porque a vida continua.... As questões de 01 a 04 referem-se ao texto acima. 01 – ―Mas agora um dedo na multidão, Entre mil indicadores algozes, eretos, Pressiona os três dígitos Da salvação.‖ A idéia deduzida no trecho acima encontra-se na alternativa: a) Entre aqueles mil indicadores, somente um apontava seu dedo para a salvação do homem. b) Entre mil dedos na multidão, apenas um se compadeceu daquele homem comum e acionou o corpo de bombeiros. c) Naquela multidão, somente uma pessoa continuava indiferentemente a pressionar os dígitos de seu telefone celular. d) Uma pessoa apenas, no meio daquela multidão de algozes, recusou-se a pressionar os dígitos da salvação. 02 – O tom predominante no texto é: a) crítico, pois, sob o pretexto de mostrar a possibilidade de um suicídio, o autor transmite sua visão de mundo, que pode ser depreendida das ações atribuídas aos personagens. b) melancólico, visto o tema central versar sobre o estado depressivo em que se encontram as pessoas que vivem nas grandes cidades. c) pessimista, pois é evidente a disposição de espírito do autor, que encara tudo pelo lado negativo, como comprova a predominância de substantivos e adjetivos desse mesmo valor. d) trágico, já que a ação do homem suicida infunde piedade e terror na multidão que se aglomerava na praça. 03 – A última estrofe do texto permite a seguinte leitura: a) A indiferença da multidão desmotivou o homem anônimo a jogar-se do sétimo andar. b) A melancolia e o desapontamento da multidão se devem ao fato de a vida voltar à rotina, visto que o homem não se jogou do sétimo andar. c) A multidão, dispersa devido à melancolia do dia-a-dia, não se importa com o homem que tenta se jogar do sétimo andar. d) A vida só continua porque o homem não se suicidou. 04 – Diz-se do texto, quanto à estrutura, que: se caracteriza como descritiva, pois detém-se na observação dos detalhes do centro de São Paulo. a) não se caracteriza como narrativa, uma vez que foi escrito em verso e detém-se ao lirismo do poeta. b) se caracteriza como narrativa, que se utiliza de recursos descritivos e reflexivos para reforçar seu conflito. c) não se pode defini-la, pois apresenta trechos descritivos e dissertativos. 05 – ―Do Paraíba as águas a rolar A tua história bem longe irão levar Dos teus alunos e mestres desta casa Aparecida há de sempre se orgulhar.‖ O texto utiliza-se de uma figura de linguagem que se caracteriza como conotativa. É ela: a) prosopopéia. b) eufemismo. c) hipérbole. d) metáfora. 06 – No período ―O Rio é um feriado. São Paulo é uma segunda-feira.‖ (Arnaldo Jabor), a figura de linguagem encontrada é a: a) catacrese. b) metáfora. c) onomatopéia. d) prosopopéia. 07 – Em ―As águas do rio gemiam alto, soluçando entre seixos‖ (Guimarães Rosa), temos a figura de linguagem: a) catacrese. b) metonímia. c) prosopopéia. d) onomatopéia. 08 – Ler os enunciados abaixo e assinalar a opção que preenche, respectivamente, as lacunas: I – A _________ do IBGE de fazer um ___________ é de grande utilidade para o País. II – A __________ de terras aos posseiros foi decidida pela Assembléia Legislativa na última ___________ . III – __________________ as razões pelas quais cometera o crime, o réu acabou sendo _______________ pelo tribunal, que o julgou inocente. a) tenção – censo / cessão – sessão / Discriminadas – descriminado b) tenção – senso / seção – sessão / Descriminadas – discriminado c) tensão – censo / cessão – sessão / Descriminadas – discriminado d) tensão – senso / sessão – seção / Discriminadas – descriminado P 2 09 – Marcar a opção em que as palavras seguem, respectivamente, as mesmas regras de acentuação das palavras destacadas em: ―É incrível como certos políticos exercem forte influência sobre grande parte da população.‖ a) util – forceps – assembleia b) rouxinol – insolito – benção c) automovel – exercito – femea d) carretel – revolucionario – etiope 10 – No texto ―Onde estão os meus verdes? Os meus azuis? O Arranha-Céu comeu!‖ (Mário Quintana) Quanto ao processo de formação de palavra, o termo destacado acima caracteriza-se como: a) derivação imprópria. b) derivação regressiva. c) justaposição. d) parassíntese. 11 – O termo sublinhado é um modificador do verbo em: a) O jogo levou meio dia para acabar. b) Naquela época, andava cheio de esperança. c) O resultado da prova foi melhor do que eu esperava. d) É um privilégio trabalhar próximo a pessoas tão cultas. 12 – Apresento-lhe Maria Lúcia. Faço tudo pela felicidade de Maria Lúcia. Se juntarmos as duas sentenças numa única, usando o pronome relativo, teremos: a) Apresento-lhe Maria Lúcia, por cuja felicidade faço tudo. b) Apresento-lhe Maria Lúcia, cuja felicidade faço tudo por ela. c) Apresento-lhe Maria Lúcia, a qual faço tudo por sua felicidade. d) Apresento-lhe Maria Lúcia, a quem faço tudo pela felicidade dela. 13 – Assinalar a única opção em que a colocação pronominal é inadmissível de acordo com a norma culta: a) Acusaram-no de caluniador. b) Não respondiam, embora me ouvissem. c) Nos últimos dias, me aconteceram coisas incríveis. d) Ele não se conformou por nos opormos a suas idéias. 14 – Observando a norma culta da linguagem, qual das opções abaixo não permite voz passiva? a) A imobiliária do meu tio alugou todas as casas velhas da vila. b) Napoleão queria a suas tropas como a um irmão. c) Os criminosos recebem o merecido castigo. d) Nós arrumamos o salão para a festa. 15 – Indicar a opção em que se verifica a voz passiva: a) Após um longo período de recessão, admitiram-se novos empregados naquela fábrica. b) Cumprimentaram-se os adversários antes de a partida iniciar. c) Os amantes despediram-se no aeroporto entre beijos e abraços. d) Os candidatos queixaram-se da extensão da prova. 16 – Marcar a frase em que a omissão da pontuação acarreta mudança de sentido: a) A chuva de ontem foi tão forte, que os bairros periféricos ficaram inundados. b) As obras governamentais são regularmente interrompidas, já que não há uma continuidade na realização dos projetos. c) Os manifestantes fizeram uma rebelião, a fim de que as autoridades tomassem as devidas providências quanto ao reajuste salarial. d) Todos acreditavam no mestre, que lecionava há mais de quinze anos. 17 – Observar as frases: I – Se eu atirar, te faço um furo no peito seu ordinário. Agora mexe aí para ver só, se eu não te mando pro inferno. II – Naquela noite, quando se preparava para ir pro cinema, a empregada veio correndo lá de dentro, apavorada. III – O Almirante, satisfeito consigo mesmo, guardou a arma e foi pro cinema. Quando voltou, o sabiá tinha desaparecido. IV – — Pois tu vais botar o sabiá na gaiola outra vez, vagabundo. Vai botar o sabiá lá, vai me pedir, desculpas por tentar roubá-lo. V – — Agora, suma-se, mas lembre, sempre, que esta arma é 45. Eu explodo essa sua cabeça, se o vir perto de minha casa outra vez. Assinalar a opção cujas frases estão corretamente pontuadas: a) I, III e IV. b) II, III e V. c) II, III, IV e V. d) I, II, III, IV e V. 18 – ―A vida é combate Que aos fracos abate E aos fortes e aos bravos Só pode exaltar.‖ (Gonçalves Dias) Quanto à função sintática, o termo grifado, no trecho acima, classifica-se como: a) objeto direto. b) núcleo do sujeito. c) adjunto adnominal. d) predicativo do sujeito. 19 – Assinalar a alternativa incorreta quanto à concordância verbal: a) Acontece que não chegaram as cartas. b) Daquela massa inerte é que nascerá todos os seres viventes. c) Haviam conseguido cargos importantes na política. d) Seis bilhões de pessoas habitam o planeta Terra. 20 – ―Para entender os problemas brasileiros, precisamos ter uma visão ampla de nossa situação no contexto mundial.‖ No período acima, a oração sublinhada expressa a idéia de: a) causa. b) concessão. c) conclusão. d) finalidade. 21 – Quanto à concordância verbal, assinalar a forma correta: a) A maioria do povo parece ignorarem a medida de sua mediocridade. b) A maioria do povo parecem ignorarem a medida de sua mediocridade. c) A maioria do povo parece ignorar a medida de sua mediocridade. d) A maioria do povo parecem ignorar a medida de sua mediocridade. 22 – Observar a oração: ―Tarde da noite, entrou pé ante pé.‖. O verbo destacado é intransitivo. Assinalar a alternativa que apresenta também um verbo intransitivo: a) Trabalhou a escultura com afinco. b) Os operários pareciam felizes ontem. c) ―Quebrei a imagem dos meus próprios sonhos.‖ d) Existem reformas pendentes nas áreas política e econômica. 23 – ANULADA 24 – Assinalar a alternativa em que a regência está correta: a) As pessoas, naquele dia, foram amáveis; simpatizei-me com elas. b) Depois de tantos fracassos, ainda custei a reconhecer o problema. c) Nunca me esquecerei o seminarista batendo na porta da sala do reitor. d) Procedeu-se à apuração e o vencedor foi mesmo o Dr. Lupicínio. 3 25 – Assinalar a opção que preenche corretamente as lacunas das frases abaixo: I – Vamos _____ outras partes. II – Vossa Senhoria não pode prender-me _____ toa. III – O abaixo-assinado vem dar os parabéns _____ Vossa Senhoria. IV – Era melhor pensar numa cama igual _____ de Seu Tomás. a) as – à – a – à b) às – a – à – à c) às – à – a – à d) às – a – a – a 1/2001-TURMA B Da solidão Há muitas pessoas que sofrem do mal da solidão. Basta que em redor delas se arme o silêncio, que não se manifeste aos seus olhos nenhuma presença humana, para que delas se apodere imensa angústia: como se o peso do céu desabasse sobre sua cabeça, como se dos horizontes se levantasse o anúncio do fim do mundo. No entanto, haverá na terra verdadeira solidão? Não estamos todos cercados por inúmeros objetos, por infinitas formas da natureza e o nosso mundo particular não está cheio de lembranças, de sonhos, de raciocínios, de idéias que impedem uma total solidão? Tudo é vivo e tudo fala, em redor de nós, embora com vida e voz que não são humanas, mas que podemos aprender a escutar porque muitas vezes essa linguagem secreta ajuda a esclarecer o nosso próprio mistério. Como aquele sultão Mamude, que entendia a fala dos pássaros, podemos aplicar toda nossa sensibilidade a esse aparente vazio de solidão: e pouco a pouco nos sentiremos enriquecidos. Pintores e fotógrafos andam em volta dos objetos à procura de ângulos, jogos de luz, eloqüência de formas, para revelarem aquilo que lhes parece não só o mais estético dos seus aspectos, mas também o mais comunicável, o mais rico de sugestões, o mais capaz de transmitir aquilo que excede dos limites físicos desses objetos, constituindo, de certo modo, seu espírito e sua alma. Façamo-nos também desse modo videntes: olhemos devagar para a cor das paredes, o desenho das cadeiras, a transparência das vidraças, os dóceis panos tecidos sem maiores pretensões. Não procuremos neles a beleza que arrebata logo o olhar, o equilíbrio de linhas, a graça das proporções, muitas vezes seu aspecto ÷ como o das criaturas humanas ÷ é inábil e desajeitado. Mas não é isso que procuramos, apenas: é o seu sentido íntimo que tentamos discernir. Amemos nessas humildes coisas a carga de experiência que representam, e a repercussão, nelas sensível, de tanto trabalho humano, por infindáveis séculos. Cecília Meireles As questões de 01 a 03 referem-se ao texto acima. 01 – De acordo com o ponto de vista adotado no texto, é correto afirmar que: a) a verdadeira solidão existe somente no nosso mundo particular, pois lembranças, sonhos, raciocínios e idéias são conceitos abstratos. b) a solidão é um estado de espírito em que algumas pessoas se encontram quando estão angustiadas. c) a solidão é, para muitas pessoas, a ausência de manifestação humana ou simplesmente a presença do silêncio. d) a solidão é o estado ou a situação ideal para pintores e fotógrafos transmitirem o espírito e a alma dos objetos. 02 – O texto utilizado como unidade de análise é uma dissertação subjetiva sobre a solidão. Assinalar a alternativa que não se aplica diretamente ao enunciador do texto: a) ―Há muitas pessoas que sofrem do mal da solidão. Basta que em redor delas se arme o silêncio, que não se manifeste aos seus olhos nenhuma presença humana, ...‖ b) ―Tudo é vivo e tudo fala, em redor de nós, embora com vida e voz que não são humanas, mas que podemos aprender a escutar porque muitas vezes essa linguagem secreta ajuda a esclarecer nosso próprio mistério.‖ c) ― ... podemos aplicar toda a nossa sensibilidade a esse aparente vazio de solidão: e pouco a pouco nos sentiremos enriquecidos.‖ d) ―Façamo-nos também desse modo videntes: olhemos devagar para a cor das paredes, o desenho das cadeiras, a transparência das vidraças, os dóceis panos tecidos sem maiores pretensões.‖ 03 – Com relação ao conteúdo do último parágrafo, é possível se fazer a seguinte interpretação: a) o verdadeiro sentido das coisas encontra-se em suas formas e cores; ou seja, em seu valor estético. b) a harmonia das proporções e a beleza exterior das coisas muitas vezes não corresponde ao seu verdadeiro sentido. c) assim como os pintores e fotógrafos, devemos desenvolver nossa faculdade de visão sobrenatural por meio da observação. d) é preciso saber que a carga de experiências contidas nas coisas humildes independe do trabalho humano. 04 – A frase que apresenta um defeito de estilo – a ambigüidade – é: a) Diante do ocorrido entre você e seu filho, fiquei sem saber se o que eu disse concorreu para agravar a situação. b) A clareza é essencial a todo escritor, pois facilita para quem lê a percepção rápida do pensamento. c) A prosa ou o poema devem ser extravasados numa linguagem espontânea, colorida como a fala. d) Meu pai recebeu uma carta em que o irmão lhe contava como sua mulher sofrera um acidente de automóvel. 05 – Observar: I – ―Ando tão à flor da pele que qualquer beijo de novela me faz chorar ando tão à flor da pele que teu olhar flor na janela me faz morrer.‖ (Zeca Baleiro) O eu-lírico se apresenta muito sensível, provavelmente por estar apaixonado ou carente. II – ―Passas sem ver teu vigia catando a poesia que entornas no chão.‖ (Chico Buarque) O eu-lírico expressa a indiferença da amada, que o inspira a criar. III – ―Se a noite e o dia passassem como nuvens, sem cadeias, e os instantes da memória fossem vento nas areias!‖ (C. Meireles) O eu-lírico expressa o desejo de que o dia e os instantes sejam mais duradouros. IV – ―Eu deixo a vida como deixa o tédio Do deserto o poento caminheiro.‖ (Álvares de Azevedo) O eu-lírico sofre por deixar a vida. Quanto à linguagem conotativa, estão corretas as afirmativas: a) I e II. c) II e IV. b) I e III. d) III e IV. 06 – ―Mesmo com toda fama Com toda Brahma Com toda a cama Com toda a lama A gente vai levando A gente vai levando A gente vai levando essa chama.‖ (Chico Buarque) As palavras grifadas estão usadas no sentido conotativo, constituindo: a) duas metonímias. b) uma metáfora e uma hipérbole. c) uma metonímia e uma metáfora. d) uma metáfora e uma prosopopéia. 4 07 – ―O cafezal é a soldadesca verde que salta morros na distância iluminada um dois, um dois, de batalhão em batalhão, na sua arremetida acelerada contra o sertão.‖ (Cassiano Ricardo) Nos versos assinalados, ocorrem, respectivamente: a) metáfora e prosopopéia. b) comparação e prosopopéia. c) comparação e hipérbole. d) metáfora e eufemismo. 08 – Colocar C para certo e E para errado com relação à transposição do discurso direto para o indireto. I – ( ) Amável, a senhora me perguntou: ÷ Seu pai como vai? Amável, a senhora me perguntou como ia meu pai. II – ( ) Ela disse ontem a seu filho: ÷ É necessário que você faça as compras. Ela disse ontem a seu filho que era necessário que ele fizesse as compras. III – ( ) ÷ Conta-me o que há, minha filha ÷ pedia-lhe o pai na carta. O pai pediu, na carta, que a filha lhe conte tudo o que há. IV – ( ) Antonieta lhe comunicou inesperadamente: ÷ Irei embora desta casa. Antonieta lhe comunicou inesperadamente que iria embora daquela casa. Assinalar a alternativa que contém a seqüência correta: a) E – C – C – E c) E – C – E – E b) C – E – C – C d) C – C – E – C 09 – Assinalar a alternativa que apresenta erro de concordância verbal: a) No Congresso, mais de um parlamentar se ofenderam na tumultuada sessão de ontem. b) Luxo, riqueza, dinheiro, nada o tentavam naquele momento de decisão. c) ―As Memórias do Cárcere‖ são indispensáveis a quem acredita na dignidade humana. d) O Amazonas é um dos rios que cortam a floresta equatorial. 10 – Em cada par, há uma oração certa e uma errada em relação à sintaxe de colocação pronominal. I – A - Em se tratando de doença grave, o melhor é levá-lo para o hospital. B - Convidar-me-iam para a recepção se encontrassem-me. II – A - Nada contentá-lo-á enquanto não tiver a paz interior. B - Se conseguisse chorar um pouco, sentir-se-ia mais aliviado. III – A - Se se vive triste sem amor, por que não amar? B - A festa, que realizar-se-á na próxima semana, promete ser um sucesso. Estão corretas: a) IA, IIA e IIIB. c) IB, IIA e IIIB. b) IB, IIB e IIIA. d) IA, IIB e IIIA. 11 – Assinalar a alternativa em cuja frase a colocação pronominal segue a norma padrão: a) ―Me dá um cigarro.‖ b) ―Amo-te como se ama todo o bem/ Que o grande mal da vida traz consigo.‖ c) E eu direi-lhe, no entanto, que não aceito sua posição. d) Ah! Não roubou-me tudo a negra sorte! Inda tenho este abrigo. 12 – Assinalar a opção que completa correta e respectivamente as lacunas abaixo: O pai, _____ era fortemente preso às tradições e _____ princípios defendia veementemente, prendia as meninas ao sair para trabalhar, _____, não tendo como cuidar delas, temia por sua honra. a) que – cujos – pois b) o qual – cujos os – já que c) que – de cujos – pois d) o qual – por cujos – que 13 – Assinalar a alternativa em que o conectivo ―e‖ não pode ser interpretado com valor adversativo: a) Amou muito e não foi amada. b) O telefone tocou e, como estava magoada, ela não atendeu. c) Estudou Português muitos anos e desconhece o léxico da Língua. d) Escreveu uma carta para matar a saudade e colocou-a no correio imediatamente. 14 – Estabelecer a correlação entre o sentido e o termo grifado. Em seguida, assinalar a alternativa que apresenta a seqüência correta: 1 ÷ Causa 2 ÷ Comparação 3 ÷ Conformidade ( ) Confesso que eu escrevo de palpite, como outras pessoas tocam piano de ouvido. ( ) Vinham em bandos, descansavam e, como em redor não havia comida, seguiam viagem para o Sul. ( ) Como era muito seco de maneiras, tinha inimigos que chegavam a acusá-lo de bárbaro. ( ) Não me respeitava a adolescência , como não respeitava a batina do irmão... ( ) O diabo não é tão feio como o pintam. a) 2 ÷ 3 ÷ 1 ÷ 1 ÷ 2 c) 2 ÷ 1 ÷ 1 ÷ 2 ÷ 3 b) 1 ÷ 2 ÷ 3 ÷ 2 ÷ 3 d) 3 ÷ 1 ÷ 2 ÷ 1 ÷ 1 15 – Preencher as lacunas com a forma adequada dos verbos entre parênteses. A seguir, asssinalar a alternativa que contém a seqüência correta: I – Se eu os ____ amanhã, poderei dar-lhes o recado. (ver) II – Se nós as ____ amanhã, poderíamos dar-lhes o recado. (ver) III ÷ ____, por meio desta, informar-lhe sobre o assunto vigente. (vir) IV – Quando ____ para cá, não sabíamos que a situação seria essa. (vir) a) vir – víssemos – vimos – viemos b) ver – víssemos – viemos – vimos c) visse – virmos – vimos – viemos d) vir – virmos – viemos – vimos 16 – Considerar os períodos I, II e III pontuados de duas maneiras diferentes. I – Carlos, o professor de História, pediu-me o livro. Carlos, o professor de História pediu-me o livro. II ÷ O piloto nervoso cometeu um erro grave durante a corrida. O piloto, nervoso, cometeu um erro grave durante a corrida. III ÷ A poluição ambiental, infelizmente, vem crescendo nas grandes cidades. A poluição ambiental infelizmente vem crescendo nas grandes cidades. Com a alteração da pontuação, houve mudança de sentido em: a) I e III. c) I e II. b) II e III. d) I, II e III. 17 – Observar: ― ... não sei se é esta toalha.‖ Qual das alternativas abaixo apresenta oração com a mesma classificação da destacada acima? a) Se for possível, gostaria de acompanhá-lo. b) Ficaremos decepcionados, se a prova for adiada. c) Os pedidos serão atendidos, se bem elaborado o ofício. d) Na verdade, gostaria de perguntar se ainda me considera seu amigo. 18 – Observar a concordância nominal nas orações abaixo: I ÷ Anos depois, bastantes verdadeiros se tornaram também outros avisos de meu pai. II ÷ Bastantes verdades experimentei anos depois do aviso que meu pai me deu. III ÷ São estudiosos as alunas e os alunos deste curso. IV ÷ O advogado considerou perigosos o argumento e a decisão. Estão corretas as frases: a) II e IV apenas. c) I, II e III. b) II, III e IV. d) I, II, III e IV. 5 19 – Assinalar a alternativa em que os vocábulos estão corretamente acentuados: a) Sabíamos que vocês têm dinheiro para reconstruir o edifício. b) Todos crêem que a inocência vai guia-lo. c) Convém que só lhe deêm o cartão de crédito após a maioridade. d) Eles vão propôr dividí-los em grupos homogêneos. 20 – Está sem erro de concordância a frase: a) Fazem dois anos que tento ingressar na EEAR. b) Houveram dias de desespero e revolta. c) Os sentenciados houveram do juiz a comutação da pena. d) Quando cheguei, já havia saído as pessoas? 21 – Assinalar a alternativa que completa corretamente as lacunas dos períodos a seguir: I – Não foi essa a obra ______ me referi. II – Há momentos na vida _______ nunca nos esquecemos. III – Os filhos _______ pais são disciplinados são mais felizes. IV – O professor, _____ conhecimentos desconfiávamos, foi infeliz em sua aula. a) à qual – de que – cujos – de cujos b) à que – que – dos quais – em cujos c) a que – de que – os quais – cujos d) à qual – que – cujos – de cujos 22 – Mesmo atentos à aula, os alunos não assimilaram o assunto, pois a linguagem do professor não era acessível a eles. a) A frase apresenta dois erros de regência nominal. b) A frase apresenta-se correta quanto à regência nominal e verbal. c) A frase apresenta um erro de regência nominal e nenhum de regência verbal. d) A frase apresenta um erro de regência verbal e nenhum de regência nominal. 23 – Assinalar a alternativa que preenche corretamente as lacunas das seguintes orações: I – Não assisto ___ filmes de guerra ou de violência. II – Não gaste ___ vista: óculos a prazo. III – Aliás, temos magníficas perucas ___ Luís XIV. IV – De segunda ___ sexta não haverá expediente. a) a – a – à – à b) à – a – à – à c) a – à – à – a d) a – à – à – à 24 – Observar os tipos de discurso empregados nos textos abaixo. I – ―De noite, o rei ouvia os soluços da filha. De que adiantava a coroa se a filha da gente chorava à noite?‖ II – ―Ariosto Ribas já estava em Caruaru, há cinco anos, não saíra de casa uma só vez. Para que sair? Para aquelas caras que odiava tanto?‖ III – ― ÷ Sempre é tarde para os amigos, replicou Sofia, em ar de censura.‖ IV – ―Uma semana depois, Virgília perguntou ao Lobo Neves, a sorrir, quando seria ele ministro.‖ Pode-se dizer que, nos textos acima, temos, respectivamente, discurso: a) direto, indireto livre, direto, indireto. b) indireto livre, indireto livre, direto, indireto. c) direto, direto, indireto, indireto livre. d) indireto, indireto, indireto livre, indireto livre. 25 – Todos os substantivos estão corretamente pluralizados em: a) salários-família, salários-mínimo, segunda-feiras, salvos- condutos. b) os leva-e-traz, guardas-mor, peixes-boi, ferros-de-abrir- lata. c) os disse-me-disse, os reco-reco, vices-reis, aves-marias. d) beija-flores, vaivéns, alto-falantes, tique-taques. 2/2001-TURMA A Embora muitos setores públicos ainda não se tenham dado conta, a máquina governamental não é uma entidade autônoma: sua existência só se justifica em função das necessidades dos cidadãos. Há um contrato social, implícito no fato de uma pessoa fixar residência em determinada cidade, que pressupõe direitos e deveres. Morar, comer, vestir, ser transportado como gente até o trabalho, ter acesso à educação, a serviços de saúde e lazer são condições básicas de existência de qualquer cidadão. Pagar impostos, preservar o patrimônio público, respeitar o próximo, por outro lado, são suas obrigações mínimas. Teoricamente, todos compreendemos isso. Em nível operacional é que desrespeitamos os contratos. A mesma pessoa que amaldiçoa o prefeito pelas enchentes é a que varre a sujeira para dentro da boca-de-lobo ou cimenta todo o terreno de sua casa, impedindo que a água se infiltre solo adentro. O cidadão que se irrita com os ruidosos vendedores de pamonha no sábado à tarde liga o som ao máximo enquanto lava seu carro no domingo de manhã. O outro, que acha as ruas esburacadas, constrói uma edícula, secretamente, para evitar o pagamento correto do IPTU. (Jaime Pinsky) As questões de 01 a 03 referem-se ao texto acima. 01 – De acordo com o texto, é dever do cidadão a) ligar o som ao máximo enquanto lava o carro. b) ser transportado como gente até o trabalho. c) pagar impostos para melhorar a cidade. d) achar as ruas esburacadas. 02 – Assinale a alternativa que explica o sentido da passagem: ―Teoricamente, todos compreendemos isso. Em nível operacional é que desrespeitamos os contratos.‖ a) A nossa liberdade termina onde começa a dos outros. b) O cidadão, que paga impostos, acha-se no direito de agir conforme a sua vontade. c) Todos fazemos parte de uma sociedade justa, pois temos os mesmos direitos e deveres. d) Na teoria, conhecemos nossos direitos e deveres, mas na prática nem sempre os observamos. 03 – Das atitudes abaixo, citadas no texto, assinale aquela em que não há incoerência no comportamento das pessoas. a) Fixar residência em determinada cidade e obter direitos e deveres. b) Reclamar das enchentes e varrer a sujeira para dentro da boca-de-lixo. c) Irritar-se com o barulho do vendedor de pamonhas e ligar o som ao máximo enquanto lava o carro. d) Construir uma edícula secretamente para evitar o pagamento do IPTU e reclamar dos buracos na rua. 04 – Assinale a alternativa cujas palavras completam corretamente os espaços do texto abaixo. ―Há estados que não ................... o que fazer com tanto dinheiro. Isso, nos Estados Unidos. .................. US$ 3 bilhões num fundo dos estados que ..................... assistência a famílias necessitadas. A culpa, em parte, é da prosperidade da era Clinton. Em 1996, 12,2 bilhões de americanos pobres dependiam dessa verba. Hoje, ................ 8 milhões.‖ (Época) a) sabe – Sobrou – dão – restou b) sabe – Sobraram – dá – restou c) sabem – Sobrou – dá – restam d) sabem – Sobraram – dão – restaram 05 – ―Amor é dado de graça é semeado no vento, na cachoeira, no eclipse.‖ A locução destacada acima classifica-se como a) adjetiva. c) conjuntiva. b) adverbial. d) prepositiva. 6 06 – Das alternativas abaixo, uma está incorreta quanto à flexão do substantivo. Assinale-a. a) João era o único testemunha do acidente. b) Na festa, o champanhe era a bebida preferida. c) Para a garota, o dó era o pior dos sentimentos. d) O marajá e a marani eram muito queridos pelos súditos. 07 – ―O homem não é existência: é ausência. É a definição perfeita da falta de algo interior, indefinível e misterioso. Todos nós, seres humanos, somos, no entanto, exceções, por nossa individualidade e essência únicas; exceções de uma única regra, traço de igualdade, a que chamamos de solidão. A solidão é a regra de nossa existência. Em função dela buscamos viver, na tentativa incessante de nos completarmos.‖ A abordagem do tema central permite afirmar que a) se utilizou o jogo de figuras por trás do qual o tema está implícito. b) não há qualquer relação lógica entre os enunciados, uma vez que se evidencia uma progressão temporal entre eles. c) são relatados aspectos simultâneos de um objeto particular concreto (o homem) situado num momento definido do tempo. d) há predominância de conceitos abstratos, que são utilizados como recursos de argumentação para se fazer referência ao mundo concreto. 08 – Na frase ―Seus primos querem que convidem eles para o passeio.‖, há um erro no emprego do pronome. Assinale a alternativa que corrige esse erro. a) Seus primos querem que convidem-os para o passeio. b) Seus primos querem que os convidem para o passeio. c) Seus primos querem que convidem-nos para o passeio. d) Seus primos querem que convidem-los para o passeio. 09 – Coloque 1 para descrição, 2 para narração, 3 para dissertação e assinale a alternativa com a seqüência correta. ( ) A casa de Ramires era grande. O sobrinho brincava no quintal sem saber que, naquele exato momento, o cão bravo escaparia. Ninguém ouvia o pedido de socorro. Mais tarde, quando todos chegaram, encontraram o menino estendido na grama. Ele tinha estudado muito e de repente dormira. ( ) A rua tinha aproximadamente cem metros e nela havia seis casas populares. Nos terrenos, o capim por crescer. Era uma linda manhã de verão, e as crianças brincavam de pega-pega. O sol abraçava tudo com seus longos tentáculos de ouro. ( ) A educação brasileira está passando por uma crise que, com certeza, vai afetar a qualidade de vida da sociedade. Os jovens ignoram a leitura de bons livros, os quais tornam-se baús empoeirados que guardam grandes tesouros cujo valor só poderá ser reconhecido se aberto. Esse desprezo pela leitura faz com que as pessoas fiquem alheias à discussão de problemas sociais. a) 2 – 3 – 1 c) 2 – 1 – 3 b) 1 – 3 – 2 d) 3 – 1 – 2 10 – Assinale a opção em que não há conotação. a) ―Tenho sonhos cruéis; n‘alma doente Sinto um vago receio prematuro. Vou a medo na aresta do futuro, Embebido em saudades do presente...‖ b) ―O poeta é um fingidor Finge tão completamente Que chega a fingir que é dor A dor que deveras sente.‖ c) ―pego a palavra no ar no pulo paro vejo aparo burilo no papel reparo e sigo compondo o verso‖ d) ―Tenho apenas duas mãos e o sentimento do mundo, mas estou cheio de escravos, minhas lembranças escorrem e o corpo transige na confluência do amor.‖ 11 – Assinale a opção em que as vírgulas intercalam termos que desempenham a mesma função da que se verifica no seguinte período: ―Todo o ajuntamento, uma vez que o comandante se aproximava, se pôs de pé.‖ a) O discurso, para ser eficaz, deve adaptar-se ao auditório ao qual é destinado. b) A poluição ambiental, disse o prefeito, é de responsabilidade de toda a população. c) O juiz, cuja decisão deve ser imparcial, decidiu prorrogar o prazo para anunciar a sentença. d) As tropas de choque, que se aproximavam rapidamente, apresentavam um contingente bastante superior ao do adversário. 12 – O covarde (Mino) Era uma vez um homem tão covarde, tão covarde, tão covarde, que só chegava cedo porque tinha medo de chegar tarde. Nunca disse a verdade, como todo covarde, mas, mesmo assim, se alguma coisa negava era por medo de dizer sim. E porque temia o umbigo pelas costas apunhalou seu amigo. Com relação ao texto acima, pode-se dizer que I – a oração do verso 3 é subordinada adverbial consecutiva. II – a oração do verso 3 funciona como principal para a oração do verso 4. III – as orações dos versos 4 e 10 são, respectivamente, coordenada explicativa e subordinada adverbial causal. IV – as orações dos versos 6 e 8 são subordinadas adverbiais, respectivamente, comparativa e temporal. V – nos versos 4 e 10, há, ainda, oração subordinada substantiva objetiva indireta. Estão corretas as afirmações de a) I, III e V. b) I, II e IV. c) II, III e IV. d) III, IV e V. 13 – Assinale a alternativa que contém erro de grafia da palavra ―porque‖. a) ―Você fez a viagem porque queria aumentar sua cultura?‖ b) ―Sabe por que o jornalista é bom de papo? Porque vive rodeado de assunto.‖ (Propaganda da Abril) c) ―Diana tornou-se um fenômeno de popularidade por que representa duas importantes facetas da mulher moderna: uma triste e outra feliz.‖ (Veja) d) ―Físicos americanos do Laboratório Brookhaven acharam indícios de uma nova partícula, o méson exótico, que se soma às 200 outras da fauna nuclear. Ela explicaria por que prótons e elétrons se unem no núcleo do átomo.‖ (Istoé) 14 – No texto abaixo há um discurso direto. Assinale a alternativa que o reescreve corretamente em discurso indireto. ―Outro dia o mestre quis ditar palavras, na sala. Ao ler borracha com x, disse ao guri: ÷ pode usá-la.‖ a) Disse ao guri que podia usá-la. b) ÷ Pode usá-la, disse ao guri. c) Disse o guri que a usasse. d) Disse ao guri: use-a. 7 15 – Quanto à coesão, assinale a alternativa em que a explicação a respeito da palavra grifada está incorreta. a) ―Rebeliões na Febem mostram que ela não só piora o problema, mas muitos carentes estão ganhando um futuro em outras instituições.‖ (Veja) – O pronome está retomando a palavra Febem. b) ―Uma mostra inédita em Nova York revela a arte que os egípcios faziam na época em que construíram suas grandes pirâmides.‖ (Veja) – O pronome está retomando arte. c) ―Uma reforma feita às pressas na Igreja de Nossa Senhora das Mercês, em Ouro Preto, quase provoca seu desabamento.‖ (Veja) – O pronome está retomando reforma. d) ―O escritor Adolfo Caminha chocou os leitores com seus personagens ambíguos.‖ – O pronome está se referindo ao escritor Adolfo Caminha. 16 – Leia os itens abaixo. I – Metáfora: ―Seus braços, autênticos tentáculos, enfeixam delitos variados.‖ II – Metonímia: ―...ternos Armani e decotes Versace se dizem chocados com o noticiário...‖ III – Antítese: ―O grande nariz não está na favela do Rio nem na periferia de São Paulo.‖ IV – Metonímia: ―Pois é preciso que a sociedade comece a enxergar o nariz invisível que cheira na grande metrópole como cúmplice da mão que segura a motosserra no Acre.‖ O exemplo dado condiz com a figura de linguagem somente nos itens a) I e III. b) II e IV. c) I, II e III. d) I, II e IV. 17 – ―Eu sei que vou sofrer A eterna desventura de viver À espera de viver ao lado teu Por toda minha vida.‖ Assinale a alternativa correta com relação ao texto. a) que é uma conjunção integrante que introduz uma oração subordinada substantiva subjetiva. b) que é conjunção integrante que introduz uma oração subordinada substantiva objetiva direta. c) de viver forma locução adjetiva que completa o sentido de desventura, palavra de valor relativo. d) À espera de é um exemplo de locução conjuntiva que introduz uma oração reduzida de infinitivo. 18 – ―No ano que vem, você será uma pessoa do século passado. Melhor começar a pensar novo.‖ (Veja, 24/05/00) A oração reduzida destacada equivale à seguinte oração desenvolvida: a) Comece, pois, a pensar novo. b) Quem sabe, você comece a pensar novo! c) Embora talvez você possa começar a pensar novo. d) Contudo, seria melhor que você começasse a pensar novo. 19 – Assinale a alternativa na qual a pontuação inadequada gera ambigüidade. a) ―Apreciando a capacidade notável que esse grande grupo de fiéis torcedores possui para avaliar o potencial de seus craques, vem com a incapacidade notória do mesmo para avaliar seus políticos, um ilustre pensador desconhecido sugeriu a instauração da República Futebolística do Brasil.‖ (Sandra Ap. Pereira) b) ―Diariamente desfilavam diante do portão aquelas mulheres silenciosas e magras, latas d‘água na cabeça. De vez em quando surgia sobre a grade a carinha de uma criança, olhos grandes e atentos, espiando o jardim.‖ (Fernando Sabino) c) ―Mas o que importa agora é mostrar que esse preconceito, velhíssimo mas nem por isso menos injusto, se revela em todos os campos em que o homem vem imperando através dos séculos, e tem guarida até na linguagem.‖ (Adriano G. Kury) d) ―No tempo em que eu freqüentava o Beco da Fome Kissinger ainda freqüentava Harvard e nem sonhava com o poder e a glória e a diplomacia a jato, mas que diabo.‖ (L.F.V.) 20 – Assinale a alternativa que identifica, no texto abaixo, retirado do Projeto de Constituição, as palavras que pertencem às classes gramaticais com função de relacionar, conectar. ―Art. 240 – A educação, direito de cada um e dever do Estado, será promovida e incentivada com a colaboração da família e da comunidade, visando ao pleno desenvolvimento da pessoa e ao seu compromisso com o repúdio a todas as formas de preconceito e discriminação.‖ a) de – e – com – a b) A – um – com – e c) de – e – um – seu d) A – com – a – seu 21 – ―Do Paraíba as águas a rolar A tua história bem longe irão levar Dos teus alunos e mestres desta casa Aparecida há de sempre se orgulhar.‖ A estrofe acima pertence a um hino escolar. Assinale a alternativa correta com relação à classificação sintática dos termos retirados do texto. a) Do Paraíba — aposto especificativo b) Aparecida — adjunto adverbial de lugar c) A tua história — objeto direto do verbo levar d) Dos teus alunos... casa — adjunto adverbial de companhia 22 – ―Jardim da pensãozinha burguesa Gatos espapaçados ao sol.‖ O termo em destaque nos versos de Manuel Bandeira significa tornar desenxabido, insosso, amolecido. Quanto à estrutura do vocábulo, podemos dizer que I – apresenta afixos. II – seu radical é papa. III – çados é um sufixo que adjetiva o verbo espapar. IV – há apenas o acréscimo do prefixo es ao radical papaçar. Está correto o que se afirma em a) II e III apenas. b) I e II apenas. c) IV apenas. d) I apenas. 23 – Leia as frases abaixo e os comentários. I – ―A lua recusou-se a passar por aquele lugar e escondia-se entre as nuvens acinzentadas, quando se aproximava delas.‖ Comentário ÷ A lua se aproximava das nuvens, por isso o verbo está no singular e o pronome elas, no plural. II – ―A lua recusou-se a passar por aquele lugar e escondia-se entre as nuvens acinzentadas, quando se aproximavam dela.‖ Comentário ÷ As nuvens se aproximavam da lua, por isso o verbo está no plural e o pronome ela, no singular. III – ―Não se deve aplicar a pena de morte, e sim fazer um trabalho de recuperação dos marginais para reintegrá-los à sociedade. Com isso, diminuiriam os processos na justiça.‖ Comentário ÷ O verbo diminuíram é intransitivo e tem como sujeito os processos na justiça. IV – ―Não se deve aplicar a pena de morte, e sim fazer um trabalho de recuperação dos marginais para reintegrá-los à sociedade. Isso diminuiria os processos na justiça.‖ Comentário ÷ O verbo diminuiria é transitivo direto: os processos na justiça é objeto direto, e Isso é sujeito. Pode-se afirmar que a) todos os comentários estão corretos. b) os comentários I e III estão incorretos. c) somente os comentários dos itens I e IV estão corretos. d) somente os comentários dos itens II e III estão corretos. 8 24 – Leia com atenção: I – ―Em meio aos transeuntes, saímos abraçados de forma colossal, como se fôssemos atores coadjuvantes na recepção do Oscar.‖ II – ―Ela era tão miudinha, que um dia seus pais pediram para tirar uma fotografia com ela.‖ III – ―A mãe ligou para o filho. Disse-lhe que o pai havia morrido e que estava precisando muito dele.‖ Os trechos acima não apresentam clareza, fato decorrente de outros problemas detectados em cada um deles. Assinale a alternativa que apresenta, respectivamente, os defeitos dos textos. a) Obscuridade, prolixidade e preciosismo b) Impropriedade vocabular, preciosismo e anfibologia c) Falta de correlação de idéias, obscuridade e prolixidade d) Impropriedade vocabular, falta de correlação de idéias e ambigüidade 25 – Quanto à concordância nominal, assinale a alternativa incorreta. a) ―Alta noite, lua quieta, muros frios, praia rasa‖ (Cecília Meireles) — O adjetivo concorda em gênero e número com o substantivo a que se refere. b) Passei dia e noite frios no Alaska. — Se os substantivos forem antônimos, o adjetivo irá obrigatoriamente para o plural (concordância gramatical). c) Tinha a face, os lábios e as orelhas enfeitados. — Quando na oração ocorrem dois ou mais substantivos de ambos os gêneros, o adjetivo irá necessariamente para o masculino plural. d) — Bom-dia! Vossa Magnificência está descansado? — Com expressões de tratamento, a concordância se faz sempre considerando o sexo da pessoa a quem se refere o pronome de tratamento. 2/2001-TURMA B Texto 1 Entrava dia e saía dia. As noites cobriam a terra de chofre. A tampa anilada baixava, escurecia, quebrada apenas pelas vermelhidões do poente. Miudinhos, perdidos no deserto queimado, os fugitivos agarraram-se, somaram as suas desgraças e os seus pavores. O coração de Fabiano bateu junto do coração de Sinhá Vitória, um abraço cansado aproximou os farrapos que os cobriam. Resistiram à fraqueza, afastaram-se envergonhados, sem ânimo de afrontar de novo a luz dura, receosos de perder a esperança que os alentava. Iam-se amodorrando e foram despertados por Baleia, que trazia nos dentes um preá. Levantaram-se todos gritando. O menino mais velho esfregou as pálpebras, afastando pedaços de sonho. Sinhá Vitória beijava o focinho de Baleia, e como o focinho estava ensangüentado, lambia o sangue e tirava proveito do beijo. Aquilo era caça bem mesquinha, mas adiaria a morte do grupo. E Fabiano queria viver. Olhou o céu com resolução. A nuvem tinha crescido, agora cobria o morro inteiro. Fabiano pisou com segurança, esquecendo as rachaduras que lhe estragavam os dedos e os calcanhares. (Graciliano Ramos) Texto 2 Não é, somente, agindo sobre o corpo dos flagelados, roendo-lhes as vísceras e abrindo chagas e buracos na sua pele, que a fome aniquila a vida do sertanejo, mas, também, atuando sobre o seu espírito, sobre a sua estrutura mental, sobre sua conduta social. Nenhuma calamidade é capaz de desagregar tão profundamente e num sentido tão nocivo a personalidade humana como a fome quando alcança os limites da verdadeira inanição. Fustigados pela imperiosa necessidade de se alimentar, os instintos primários exaltam-se, e o homem, como qualquer animal esfomeado, apresenta uma conduta mental que pode parecer a mais desconcertante. Muda o seu comportamento como muda o de todos os seres vivos alcançados pelo flagelo na mesma área geográfica. (Josué de Castro) As questões de 01 a 06 referem-se aos textos acima. 01 – Em relação aos textos, é correto afirmar que: I – os autores tratam da questão da fome utilizando-se da mesma estrutura. II – no texto 1 o tema é transformado em conflito humano, enquanto que no texto 2 é abordado enquanto problema. III – se trata de textos literários que demonstram emoções, angústias e ansiedades do homem. IV – o texto 2 preocupa-se em analisar as causas da fome. Está(ão) correta(s): a) II apenas. b) I, II, III e IV. c) I e III apenas. d) II e IV apenas. 02 – De acordo com o texto 2, a necessidade de matar a fome faz com que os instintos primários exaltem-se no homem. Assinalar a alternativa que comprova essa idéia, através de trechos do texto 1. a) ―Resistiram à fraqueza, afastaram-se envergonhados, sem ânimo de afrontar de novo a luz clara...‖ b) ―Miudinhos, perdidos no deserto queimado, os fugitivos agarraram-se, somaram as suas desgraças e os seus pavores.‖ c) ―Sinhá Vitória beijava o focinho de Baleia, e como o focinho estava ensangüentado, lambia o sangue e tirava proveito do beijo.‖ d) ―Levantaram-se todos gritando. (...) Aquilo era caça bem mesquinha, mas adiaria a morte do grupo.‖ 03 – Relendo o último parágrafo do texto 1, vê-se que Fabiano readquire forças, sente-se novamente homem. Refletindo sobre o que lhe trouxe novo vigor, de acordo com o texto, é possível o seguinte raciocínio: a) o homem basta a si mesmo; nada se impõe sobre sua condição de ser. b) a condição de homem relaciona-se diretamente a fatores que interferem nas condições do ser. c) a satisfação das necessidades fisiológicas não participa da essência do ser. d) o que constitui a natureza humana é a interdependência física e mental que em nada se relaciona com a natureza animal, vegetal ou mineral. 04 – O autor Josué de Castro afirma que o flagelo modifica o comportamento de toda uma área geográfica. No texto, flagelo — aquilo que castiga e tortura — pode ser entendido como a) apenas a seca. b) apenas a fome. c) a fome e a seca. d) a miséria e o abandono. 05 – O texto 2 tem como principal objetivo a) apenas escrever sobre a fome. b) mostrar as conseqüências da fome. c) frisar a fome no Nordeste brasileiro. d) alertar sobre o problema da fome no mundo. 06 – Das afirmações abaixo, relacionadas ao texto 2, I –- A fome muda o comportamento dos seres vivos. II –- O homem, como qualquer animal, tem necessidade de se alimentar. III –- Só os nordestinos passam pelo problema da inanição. IV –- A fome pode desagregar profundamente a personalidade humana. Está(ão) correta(s) a) I, II e IV apenas. b) II e III apenas. c) I e III apenas. d) IV apenas. 9 07 – Assinalar a alternativa cujo trecho apresenta linguagem conotativa. a) A independência portuguesa, que por tantos anos tendera a realizar-se, retrocedia ainda uma vez; era um problema cuja solução já perto do seu termo devia tornar a ser tentada de novo. b) Ciência – Música pode melhorar ação do cérebro Pesquisadores, reunidos nos EUA, revelam que som estimula certas áreas da mente. c) ―Eu canto porque o instante existe e a minha vida está completa Não sou alegre nem sou triste sou poeta.‖ d) ―Pensem nas crianças ............................... A rosa hereditária A rosa radioativa Estúpida e inválida A rosa com cirrose A anti-rosa atômica...‖ 08 – Assinalar a alternativa em que todos os vocábulos estejam com a acentuação gráfica correta. a) pôquer, hífen, ínterim, alcatéia b) índex, rúbrica, míope, chevrolés c) sótão, sangüíneo, diminuí-lo, arquetipo d) apóio (verbo), rouxinóis, seminú, fortuito 09 – Observar: I – — Não, disse-me ele, não me deve mais nada. II – Contarei-lhe o segredo, se puder. III – — Que Deus acompanhe-te por toda parte. IV – Nunca soubemos quem os roubava nas medidas. Quanto à colocação pronominal, estão corretas a) III e IV apenas. b) II e III apenas. c) I e IV apenas. d) I e II apenas. 10 – ―Conjunção é uma palavra invariável que liga e relaciona entre si duas orações completas ou incompletas.‖ As palavras sublinhadas, na frase acima, podem ser classificadas, respectivamente, como: a) conjunção integrante / conjunção subordinativa. b) pronome relativo / conjunção coordenativa. c) conjunção integrante / preposição. d) pronome relativo / preposição. 11 – Assinalar a alternativa cujo verbo não é defectivo. a) Abole o cigarro. b) Reavei o tempo perdido. c) ―A fiança remiu-o da prisão.‖ d) ―Minha música não quer redimir mágoas.‖ 12 – Assinalar a alternativa correta quanto à pontuação. a) No dia seguinte, almoçamos num restaurante e tomamos três garrafas de vinho tinto; ao entardecer, num bar, fiquei a alisar ternamente a sua mão fina, de veias azuis. b) Por mais distraído que fosses, leitor amigo, terias notado que, ele ficara sinceramente alegre, posto que contivesse a alegria, segundo convinha a um filósofo. c) E o globo da Lua, num dado momento parece roxo, sangüíneo, como um vaso de sangue. Que singular metamorfose, e que triste símbolo? d) O pai, frustrado e ambicioso sonha para o filho a profissão que não conseguira ter. Essas eram as palavras de vovô... 13 – Do alto, observavam-se as ruas e as casas; viam-se também, nas praças, frondosas árvores. A concordância do verbo com o sujeito na frase acima justifica- se pela mesma razão que determina a concordância verbal em: a) Cabia, na alma do sertanejo, o amor da mulher, o afeto dos filhos e a amizade do irmão. b) Realmente procedem, em meio a tais abusos, tantas queixas dos contribuintes. c) Restabelecer-se-iam, de imediato, as ligações e os contatos, se houvesse técnico de plantão. d) O diretor e seus assessores têm encontrado, nos últimos meses, sérias falhas nos relatórios. 14 – Assinalar a alternativa que completa corretamente as frases abaixo. I – Os teóricos, neste momento, _____________ os conceitos. II – _____________ muito talento os meus amigos. III – A casa ele ____________ do necessário. IV – Espero que todos do grupo _____________ crédito ao novo gerente. a) reveêm/ tem/ provém/ dêem b) revêem/ têm/ provém/ dêem c) reveem/ têm/ provem/ deem d) revêem/ tem/ provêm/ deêm 15 – Com relação às palavras do texto abaixo, assinalar a alternativa que contém a afirmação correta quanto ao processo de formação de palavras. ―Mas desconfio que toda essa conversa é feita apenas para adiar a pobreza da história, pois estou com medo. (...) As coisas estavam de algum modo tão boas que podiam se tornar muito ruins porque o que amadurece plenamente pode apodrecer.‖ a) Desconfio, amadurece e apodrecer sofreram derivação parassintética. b) Plenamente, conversa e pobreza são palavras derivadas por sufixação. c) Conversa sofreu derivação regressiva, e desconfio, derivação prefixal. d) Ocorre derivação regressiva em conversa, e prefixal e sufixal em apodrecer e amadurece. 16 – ―Fora da idéia de tamanho, as formas aumentativas e diminutivas podem traduzir o nosso desprezo, a nossa crítica, o nosso pouco caso para certos objetos e pessoas.‖ (Evanildo Bechara) Tomando como base a definição acima, assinalar a alternativa em que o aumentativo tem esse sentido pejorativo. a) ―Ele pegou um peixão! Quatro quilos!‖ b) ―Soltava uns dois espirros... assoava o narigão...‖ c) O dono da fazenda virou-se para ver que barulhão era aquele. d) ―... saía do mato uma mulheraça rúbida, de saias rufadas de goma.‖ 17 – Observar: I – Costumava assistir ao seriado todas as noites, após chegar do trabalho. II – O comerciante não vencia pagar ao agiota as dívidas eternas, cujos juros eram exorbitantes. III – Tendo esgotado a sua paciência, a mãe chamou a atenção do menino na frente de todos. IV – ―Estava à toa na vida, O meu amor me chamou Pra ver a banda passar Cantando coisas de amor‖ Marcar a alternativa cujos pronomes substituem, respectivamente, os termos grifados nos textos acima, de maneira correta. a) lhe – lo – sua – la b) lo – lhe – lhe – lhe c) a ele – lo – sua –lhe d) a ele – lhe – lhe – la 18 – Observar: “De manhã escureço De dia tardo De tarde anoiteço De noite ardo.‖ Com relação aos termos da oração presentes na estrofe acima, é correto afirmar que a) todos os versos possuem sujeito oculto ―eu‖. b) os adjuntos adverbiais complementam o sentido dos verbos, que são transitivos indiretos, pois modificam-se ou transformam-se por ação do homem. c) os três primeiros versos apresentam sujeito inexistente, pois os verbos exprimem fenômenos da natureza. d) a estrofe é formada por orações sem sujeito, ainda que estas estejam na linguagem conotativa. 10 19 – Sem sofrerem modificações no sentido, os termos em destaque de cada frase foram substituídos pelos verbos entre parênteses. Assinalar a alternativa em que essa substituição obedece às regras de regência verbal. a) Era difícil, para mim, acreditar no que aconteceu. (custar) Eu custei a acreditar no que aconteceu. b) Sentia-se inclinado pelas idéias daquele grupo jovem. (simpatizar) Simpatizava-se com as idéias daquele grupo jovem. c) ―Ele dava primazia ao sossego do campo e não à agitação da cidade.‖ (preferir) Ele preferia o sossego do campo que a agitação da cidade. d) Assim que receber, irei reembolsar a meu irmão parte da quantia que me foi emprestada. (pagar) Assim que receber, pagarei a meu irmão parte da quantia que me foi emprestada. 20 – Observar a concordância nominal nas orações abaixo: I – Anos depois, bastantes verdadeiros se tornaram também outros avisos de meu pai. II – Bastantes verdades experimentei anos depois do aviso que meu pai me dera. III – São estudiosos as alunas e os alunos deste curso. IV – O advogado considerou perigosos o argumento e a decisão. Estão corretas as frases a) II, III e IV apenas. b) I, II e III apenas. c) II e IV apenas. d) I, II, III e IV. 21 – ―Já era noite. Parecia viável que todos entendessem que, naquele momento, deviam-se lembrar de que nada é eternamente assim.‖ Com relação ao texto acima, é correto afirmar que a) de que nada é eternamente assim é oração subordinada adjetiva restritiva. b) que todos entendessem é oração subordinada substantiva subjetiva. c) naquele momento é adjunto adnominal. d) noite é sujeito do primeiro período. 22 – Flexionar os verbos entre parênteses na 2ª pessoa do singular do modo imperativo e assinalar a alternativa correta. I – (Opor) a perseverança às dificuldades. II – Não (desprezar) os conselhos dos velhos. III – Odiar) o vício e (prezar) a virtude. IV – (Evitar) o mal e (praticar) o bem. a) Opões b) despreza c) Odeia; preza d) Evites; pratiques 23 – Observar: ―Sou caprichosa, eu sei. Desce o pó sobre os móveis. Que eu colho na flanela. Escurecem-se as pratas. Que eu esfrego com a camurça. A aranha tece. Que eu enxoto. A traça rói. Que eu esmago. O cupim voa. Que eu afogo na água da tigela sob a luz.‖ I – A pontuação do texto contraria as normas gramaticais, pois o pronome relativo deve pertencer ao mesmo período de seu termo antecedente. II – A pontuação adotada é estilística, o que não significa dizer que ela está gramaticalmente correta. III – Transformando-se a pontuação de modo a adequá-la às regras gramaticais, o texto, dentre outras possibilidades, apresentaria seis períodos compostos. Está(ão) correta(s) a(s) declaração(ões) a) II e III apenas. b) I e III apenas. c) II apenas. d) I, II e III. 24 – Observar: ―O avião, um EA-6B Prowler, voava a apenas 90 metros do chão, desrespeitando as regras de segurança para vôos de treinamento, que estipulam altura mínima de 300 metros do solo naquela região.‖ A vírgula que precede o pronome relativo confere a seguinte interpretação ao período acima: a) o pronome relativo, nesse caso, restringe o sentido de seu termo antecedente, ―vôos de treinamento‖, o que significa dizer que essas regras não estipulam, necessariamente, apenas a altura mínima citada. b) a oração adjetiva explicita uma idéia que já se sabe estar contida no conceito de ―regras de segurança para vôos de treinamento‖, ou seja, que todas as regras, sem exceção, estipulam altura mínima de 300 metros do solo naquela região. c) por tratar-se de uma oração adjetiva explicativa, a idéia nela expressa é atribuída a ―vôos de treinamento‖, os quais estipulam, sem distinção, altura mínima de 300 metros do solo naquela região. d) a oração adjetiva em questão restringe o conceito de ―regras de segurança para vôos de treinamento‖, delimitando seu sentido; em outras palavras, o atributo expresso nessa oração é próprio de apenas algumas dessas regras que foram desrespeitadas pelo avião. 25 – Observar: Reclinada molemente na sua verdejante colina, como odalisca em seus aposentos, está a sábia Coimbra, a Lusa Atenas. Beija-lhe os pés, segredando-lhe de amor, o saudoso Mondego. E em seus bosques, no bem conhecido salgueiral, o rouxinol e outras aves canoras soltam seus melancólicos trilos. Quando vos aproximais pela estrada de Lisboa, onde outrora uma bem organizada mala-posta fazia o serviço que o progresso hoje encarregou à fumegante locomotiva, vede-la branquejando, coroada do edifício imponente da Universidade, asilo da sabedoria. (O primo Basílio. São Paulo, Abril Cultural, 1979. p. 229-30.) Com relação ao texto acima, apresentam correta classificação sintática os termos presentes na alternativa: a) ―a Lusa Atenas‖, ―o saudoso Mondego‖, ―asilo da sabedoria‖ são apostos. b) ―o rouxinol e outras aves canoras‖, ―vos‖ e ―fumegante locomotiva‖ são, respectivamente, sujeitos de ―soltam‖, ―aproximais‖ e ―encarregou‖. c) os pronomes oblíquos de ―Beija-lhe‖ e ―segredando-lhe‖ desempenham a função de objeto indireto. d) são adjuntos adverbiais ―em seus bosques‖ e ―pela estrada de Lisboa‖. 1/2002-TURMA A Kiki do rebolado - Katia Killer? – perguntou a irmã de caridade entrando no quarto, e Kiki bem percebeu que a freira nunca acreditara ser aquele seu nome verdadeiro. - Sou eu – disse em tom angustiante; e aprumou-se na cadeira. - Sempre resolvida a entregar o recém-nascido? - Sou mãe solteira, irmã – desculpou-se, agora os olhos baixos – há muito saí de casa. Na profissão que abracei... A freira persignou-se: - Em nome de Deus, fiz tudo que pude! – olhou o alto, suspirou, olhou a moça. – Vou buscar o papel para você assinar; o casal que vai adotar a criança quer o preto no branco. Quando a irmã saiu, Kiki deslizou o olhar para as três camas já esticadas – a sua, a duma tal de Cida que, em situação igual, partira na véspera, e a da moça sendo cesariada naquela hora: ―Coitada! saíra há pouco, descorada de dor‖. Kiki levantou-se da cadeira onde estivera esperando; encaminhou-se para a grande janela de vidraças abertas. A luz da manhã favorecia e nelas pôde espelhar-se de corpo inteiro: os seios empinados marcando a malha preta, a minissaia vermelha oferecendo ancas certinhas, aquelas que, reboladas no palco, arrancavam mil exclamações. E assobios! Sorriu, na certeza de que em nada desmerecera. De repente, a irmã Dolores invadiu o quarto. Com aquele sorriso claro entregou-lhe o bebê que carregava: 11 - Trouxe o menino para você conhecer; pode contar lá fora do garotão forte que teve! Kiki, pega de surpresa, apanhara a criança. Agora estava ali, desajeitada, sem querer olhar: - Não adianta, irmã; sou firme nas minhas resoluções. Mas, irmã Dolores, como um pé-de-vento, entrara e saíra. Desanimada, Kiki sentou-se, o bebê no colo. Na quietude, primeiro sentiu um calor desconhecido, não sabia se vindo da criança para ela ou dela para a criança. Depois, o nenê resmungou e Kiki baixou os olhos. Então, foi como se dali para trás nada mais importasse: - Que coisa mais linda, meu Deus! – exclamou apertando o filho de encontro ao coração, os olhos rasos d‘água – Ah!... queridinho... Ah! ... – disse, e as lágrimas rolaram. Quando se sentiu lúcida, ergueu-se. Numa bem- aventurança abriu a porta, caminhou pelo corredor, alcançou a saída, fugiu para a rua, para a manhã vazada de sol, o filho aconchegado nos braços. (Lucília Junqueira de Almeida Prado, Depois do aguaceiro) As questões de 01 a 04 referem-se aos textos acima. 01 – A parte descritiva presente no oitavo parágrafo do texto serve para a) mostrar a satisfação da personagem com seu corpo e a relação com a sua vida profissional, o que justifica sua atitude, a princípio. b) tornar o texto mais subjetivo, despertando o sentimento de repulsa do leitor para com a personagem. c) mostrar que Kiki tinha um belo corpo, mas que lhe incomodava o fato de ele despertar o desejo dos homens. d) expressar o orgulho da personagem por ter-se tornado mãe tão jovem. 02 – ―- Sou eu – disse em tom angustiante...‖ A angústia da personagem provém do fato de que a) Kiki queria ficar livre do hospital e voltar à sua rotina. b) não via outra saída senão entregar o filho. c) não conhecia o casal que iria adotar seu filho. d) pensava em abandonar a carreira. 03 – A mudança de atitude da personagem, no final, deve-se I. ao despertar de seu instinto maternal, ao segurar o filho no colo. II. à facilidade com que tinha dado à luz o filho, e ao bem- estar no período pós-parto. III. ao amor súbito pela criança, que superou as perspectivas das dificuldades pelas quais passaria. IV. à pressão sutil da irmã de caridade sobre a jovem parturiente. Está(ão) correta(s) a(s) afirmação(ões) a) I, III e IV. b) I, II e III. c) II e IV. d) III somente. 04 – Analise os trechos abaixo e assinale a alternativa correta. I. ―– Sou mãe solteira, irmã – desculpou-se, agora os olhos baixos.‖ II. ―Quando se sentiu lúcida, ergueu-se. Numa bem- aventurança abriu a porta, caminhou pelo corredor...‖ III. ―... – exclamou apertando o filho de encontro ao coração, os olhos rasos d‘água – Ah! queridinho... Ah!... disse, e as lágrimas rolaram.‖ IV. ―... Kiki deslizou o olhar para as três camas já esticadas – a sua, a duma tal de Cida que, em situação igual, partira na véspera, e a da moça sendo cesariada naquela hora...‖ O sentimento de impotência de Kiki diante da situação está expresso em a) I e III. c) I e IV. b) II e IV. d) II e III. 05 – Assinale a alternativa cujo texto se caracteriza como dissertação argumentativa. a) ―Logo à direita do corredor encontramos aberta uma larga porta, que dá entrada à sala de recepção, vasta e luxuosamente mobiliada. Acha-se ali sozinha e sentada ao piano uma bela e nobre figura de moça.‖ b) ―Obedecendo à instrução do sertanejo, Luzia desceu pela tortuosa ladeira, que ia no fundo da grota, e, sustendo-se nos arbustos das margens para não escorregar, colhendo flores silvestres, parando, a revezes, para desembaraçar as vestes dos espinhos que a detinham, chegou à garganta...‖ c) ―Os dois estudantes aproximaram-se de Augusto, que acabava de rogar à linda Moreninha a mercê da terceira quadrilha. – Leva de tábua, disse Fabrício ao ouvido de Leopoldo... é a mesma que eu lhe havia pedido.‖ d) ―Embora cobrar dos governos estadual e municipal seja extremamente justo, não se pode negar que é chegado o momento de cada cidadão dar sua própria contribuição. O terreno permeável em todas as construções para o escoamento da água, a limpeza das ruas e dos bueiros, evitando transbordamento, o lixo jamais jogado nas ruas são medidas simples, possíveis e baratas e que poderiam fazer a diferença entre um verão de tragédia e um verão civilizado.‖ 06 – ―Deixe a meta do poeta, não discuta, Deixe a sua meta fora da disputa Meta dentro e fora, lata absoluta Deixe-a simplesmente metáfora.‖ (―Metáfora‖ – Gilberto Gil) Pode-se afirmar, quanto ao texto acima, que o autor a) usa do recurso da repetição do vocábulo ―meta‖ para enriquecer o texto, ampliando-lhe o sentido. b) com a repetição do vocábulo ―meta‖, deixa, intencionalmente, o texto com um defeito de estilo. c) faz um jogo de palavras inadequado, tornando o texto obscuro. d) ao usar o pronome oblíquo ―a‖ (deixe-a), cria ambigüidade no texto. 07 – ―Recebeu o e-mail do colega comunicando que seu trabalho havia sido publicado.‖ Pode-se afirmar que, no texto acima, há a) ambigüidade, causada pelo uso inadequado do pronome possessivo. b) ambigüidade, por erro de regência verbal. c) perfeita clareza de sentido. d) falta de clareza, devido à falta de sujeito expresso. 08 – ―Foi em março, ao findar das chuvas, quase à entrada Do outono,(I) quando a Terra, em sede requeimada, Bebera longamente as águas da estação (II), ÷ Que, em bandeira, buscando esmeralda e prata À frente dos peões, filhos da rude mata (III), Fernão Dias Paes entrou pelo sertão.(IV)‖ Analise os trechos acima assinalados quanto ao sentido denotativo e conotativo e assinale a alternativa correta. a) Apenas I e II têm sentido denotativo. b) Apenas II e III têm sentido conotativo. c) I e IV têm sentido conotativo; II e III têm sentido denotativo. d) Todos têm sentido conotativo. 09 – Assinale a alternativa em que não há hipérbole. a) A platéia estourava de tanto rir. b) Morri de saudades de meus pais. c) Você, estudante, tem um mundo pela frente. d) Naquele confronto, Luís foi desta para melhor. 12 10 – Assinale a alternativa em que a passagem do discurso direto para o indireto está incorreta. a) ÷ Ande depressa, menina, pediu a mãe. A mãe pediu que a menina andasse depressa. b) Clotilde pensava: ―O mundo ficará todo cheio de flores‖. Clotilde pensava que o mundo ficaria todo cheio de flores. c) ÷ Queremos paz no futuro, disseram os jovens. Os jovens disseram que queriam paz no futuro. d) ÷ Direita, volver! ÷ bradou o comandante da tropa. O comandante da tropa bradou direita, volver. 11 – Relacione os encontros vocálicos das palavras (coluna direita) com sua classificação (coluna esquerda) e assinale a alternativa que contém a seqüência correta. ( 1 ) ditongo ( ) conveniente ( 2 ) tritongo ( ) fortuito ( 3 ) hiato ( ) receoso ( ) Jaceguai ( ) destrói a) 3 – 1 – 3 – 2 – 1 b) 2 – 3 – 2 – 1 – 2 c) 3 – 3 – 1 – 2 – 1 d) 1 – 1 – 3 – 2 – 3 12 – Relacione a coluna da direita com a da esquerda e assinale a alternativa que contém a seqüência correta. Observe que foram retirados os acentos gráficos de algumas palavras, propositadamente. ( 1 ) oxítona ( ) caracteres ( 2 ) paroxítona ( ) refem ( 3 ) proparoxítona ( ) rubrica ( ) publico (adjetivo) ( ) interim ( ) arquetipo ( ) amendoim ( ) latex a) 2 – 1 – 2 – 3 – 3 – 3 – 1 – 2 b) 3 – 2 – 3 – 2 – 3 – 2 – 1 – 1 c) 1 – 1 – 2 – 3 – 1 – 3 – 2 – 2 d) 2 – 2 – 3 – 1 – 2 – 1 – 3 – 1 13 – ―Este manual .................. informações importantes sobre os produtos que ................ elementos prejudiciais à saúde. Portanto, ................. com a ajuda dele.‖ Assinale a alternativa que preenche os pontilhados, na ordem em que aparecem. a) contêm – contém – contem b) contém – contêm – contem c) contem – contem – contém d) contém – contêm – contêm 14 – Assinale a alternativa que preenche corretamente os pontilhados da frase abaixo, respectivamente. ―Naquela noite ........................ as luzes se apagaram, voou porta adentro um .......................... de abelhas. Trocado o .......................... , entretanto, a ....................... se retirou.‖ a) mal – enxame – fusível – colmeia b) mal – enchame – fuzil – colméia c) mau – enxame – fusível – colmeia d) mau – enchame – fuzil – colméia 15 – Das alternativas abaixo, alteradas propositadamente, apenas uma não apresenta erro no emprego da forma verbal. Assinale-a. a) Folha de São Paulo – não dá pra não lê. b) Potentes como Antônio Ermírio de Moraes já vêm, com certa simpatia, a candidatura de Ciro Gomes. c) O encontro histórico dos presidentes sul-americanos evidencia a intenção brasileira de liderar a região. d) É revoltante saber que profissionais de algumas áreas têm de estudar dez anos ou mais para conseguir um salário de cinco ou seis mil reais enquanto semi-alfabetizados ganhão milhares na TV. 16 – Assinale a alternativa em que o elemento mórfico em destaque está corretamente analisado. a) meninA – desinência nominal de gênero b) vendEste – desinência modo-temporal c) partIa – vogal temática da 2.ª conjugação d) amaSSem – desinência de 2.ª pessoa do plural 17 – Assinale a alternativa em que os afixos estão destacados corretamente em todas as palavras. a) PREVer – amoroSO – belEZA b) DESObediente – certEZA – preVER c) paulaDA – cafeZAL – desanimaDOR d) DESonesto – rancorOSO – laranjEIRA 18 – Marque a opção em que a palavra grifada não exerce função de adjetivo. a) O jogo levou meio dia para acabar. b) Naquela época, andava cheio de esperança. c) O resultado da prova foi melhor do que eu esperava. d) É um privilégio trabalhar próximo a pessoas tão cultas. 19 – Considerando o processo de formação de palavras, enumere a coluna da direita de acordo com a da esquerda e assinale a alternativa com a seqüência correta. 1 – prefixação ( ) esfarelar 2 – regressiva ( ) desalmado 3 – sufixal ( ) graciosa 4 – parassintética ( ) antinatural ( ) debate a) 3, 2, 3, 4, 1 b) 4, 4, 3, 1, 2 c) 1, 4, 2, 1, 2 d) 1, 2, 4, 2, 3 20 – Observe as frases abaixo e assinale a alternativa em que há a classificação correta dos adjetivos quanto à flexão de grau, respectivamente. I. ―Era a mais bela! o seio palpitando.‖ II. O Prefácio Interessantíssimo é de autoria de Mário de Andrade. III. O tempo é tão fugaz quanto incerta e curta é a vida. IV. Nossos antepassados eram menos preocupados com o meio ambiente do que nós. a) superlativo relativo de superioridade, superlativo absoluto sintético, comparativo de igualdade, comparativo de inferioridade. b) superlativo absoluto analítico, superlativo relativo de superioridade, comparativo de superioridade, comparativo de igualdade. c) comparativo de superioridade, superlativo absoluto analítico, comparativo de superioridade, superlativo relativo de inferioridade. d) superlativo relativo de superioridade, superlativo absoluto analítico, comparativo de inferioridade, comparativo de superioridade. 21 – (...) ―Convive com teus poemas, antes de escrevê-los. Tem paciência, se obscuros. Calma, se te provocam. Espera que cada um se realize e consuma com seu poder de palavras e seu poder de silêncio.‖ (...) (C.D. Andrade) As palavras grifadas, no poema, classificam-se, respectivamente, como a) conjunção integrante, conjunção integrante. b) conjunção adverbial condicional, conjunção adverbial condicional. c) partícula expletiva, partícula expletiva. d) pronome reflexivo, conjunção adverbial condicional. 22 – ―A professora dizia que a rua tinha aquele nome em homenagem a um grande homem, digno de ser imitado. Nós a ouvíamos meio duvidosos. Para nós a rua era do pipoqueiro, do sorveteiro, do vendedor de algodão-doce... De alguém que significasse muito, era o nome da rua.‖ (Elias José) Os substantivos grifados nesse texto classificam-se, respectivamente, como a) simples, derivado, composto e comum. b) próprio, primitivo, simples e abstrato. c) simples, derivado, próprio e concreto. d) comum, composto, derivado e comum. 13 23 – Assinale o período gramaticalmente correto com relação ao uso dos pronomes. a) Não façam eu estudar tanto. b) Entre eu e ele não há discórdias. c) Para mim ele não contou a história. d) Mandaram eu entrar assim que me viram. 24 – ―Rasgue as minhas cartas e não me procure mais. Assim será melhor, meu bem. O retrato que eu te dei se ainda tens não sei, mas, se tiver, devolva-me.‖ Nesse trecho da música de Renato Barros e Lilian Knapp, há mistura dos pronomes tu e você. Se se considerar apenas o pronome você, quantas palavras devem ser corrigidas? a) Duas. b) Três. c) Quatro. d) Cinco. 25 – Em qual(is) sentença(s) abaixo o verbo grifado não é defectivo? I. Nessa época do ano, é comum nevar em algumas regiões do país. II. As onças rosnam ao redor da cabana. III. A erupção vulcânica baniu da vila a maior parte de seus habitantes. IV. Uma das testemunhas interveio a favor do réu. a) III. b) IV. c) I e II. d) III e IV. 26 – Leia as frases abaixo. I. As pessoas só confiarão em mim se eu sempre manter a palavra. II. Quando ele se propor a nos auxiliar, estaremos mais próximos de nossos objetivos. III. Assim que os autores compusessem a peça, os ensaios teriam início. IV. Se contessem as águas, navegaríamos mais tranqüilos. V. Muitos propõem soluções, mas é preciso que se aventurem a colocá-las em prática. Com relação à flexão das formas verbais, estão corretas as frases a) I e II. b) II, III e V. c) I, IV e V. d) III e V. 27 – ―Sê o que renuncia Altamente: Sem tristeza da tua renúncia! Sem orgulho da tua renúncia! Abre a tua alma nas tuas mãos E abre as tuas mãos sobre o infinito E não deixes ficar de ti Nem esse último gesto!‖ (Cecília Meireles) Mudando-se a pessoa verbal da 2.ª do singular para a 2.ª do plural, os verbos grifados serão, respectivamente, a) sedes – abris – deixais ficar. b) sejai – abrais – deixeis ficar. c) sede – abri – deixeis ficar. d) sois – abri – deixeis ficar. 28 – Em qual período abaixo há necessidade do emprego da vírgula? a) Esses casos exemplares demonstram a utilidade da campanha. b) É preciso preservar a vida do elefante que é um grande mamífero. c) Chegou-se à conclusão de que o importante mesmo eram os benefícios proporcionados à população. d) A humanidade espera que os homens de bom senso cumpram a missão para a qual foram designados. 29 – Assinale a frase cujas palavras destacadas são, respectivamente, vocativo e aposto. a) Joana, minha prima, está doente. b) Mauro, telefone-me assim que Teresa chegar. c) Alice, meu vizinho João comprou um computador. d) Paulo disse que José, treinador do time, está desanimado. 30 – Assinale a alternativa cujo termo destacado classifica-se como complemento nominal. a) Arrancaram-lhe as roupas. b) Ela nunca lhe desobedece. c) A sentença foi-lhe favorável. d) Júlio devolveu-lhe o livro emprestado. 31 – Assinale a alternativa em que há oração subordinada adjetiva explicativa. a) A revista que você me emprestou ontem, eu a perdi. b) O homem que fala, que persiste, que enfrenta as dificuldades triunfa na vida. c) Todos sabem, meu amigo, que a verdade virá à tona quando menos esperarmos. d) O Brasil, que contém muitas riquezas naturais, atrai turistas do mundo inteiro. 32 – Assinale a alternativa em que o período é composto por subordinação. a) A moça estremeceu, abaixou os olhos e não respondeu à pergunta. b) Tudo acabou bem; eram, pois, infundados os nossos receios. c) Disseram-me que você é pessoa digna de confiança. d) Geralmente os ignorantes falam muito; os sábios, porém, preferem o silêncio. 33 – ―Maltrataram inocentes, mas ninguém quer depor.‖ (Elza Santana Silva) Os verbos grifados têm, respectivamente, sujeito a) indeterminado e simples. b) oculto e indeterminado. c) simples e oculto. d) indeterminado e oculto. 34 – ―Vinhas fatigada e triste e triste e fatigado eu vinha...‖ (Olavo Bilac) No período acima, pode-se afirmar que os predicados são a) verbo-nominais, porque os verbos são de ação e vêm seguidos de dois advérbios de modo. b) nominais, porque os verbos são de ação e vêm seguidos de dois predicativos. c) verbo-nominais, porque os verbos são intransitivos e vêm seguidos de dois predicativos. d) verbais, porque os verbos têm sua significação completada por dois objetos diretos. 35 – ―O pequeno avião fez um pouso de emergência numa plantação próxima ao aeroporto. Ficaram feridos todos os seus ocupantes.‖ (Veja) Sobre esse texto, é incorreto afirmar que a) ―feridos‖ é objeto direto. b) ―pequeno‖ é adjunto adnominal. c) ―todos os seus ocupantes‖ é sujeito. d) ―numa plantação‖ é adjunto adverbial de lugar. 36 – Assinale a alternativa em que a concordância verbal está incorreta. a) Era João como o pai; e como o aconselhavam a devoção e a pobreza. b) No outro dia, cinco horas batia; Seu Chico aparecia de novo no asilo. c) Ninguém, na sua opinião, entre os vinte e poucos alunos, tinha jeito para aquilo. d) A minha vocação, o meu gosto, o meu pendor para a coisa literária, tudo se revelou precisamente naquele dia. 37 – Assinale a alternativa cujas palavras completam corretamente as lacunas da frase abaixo. ― .............................. e ............................. pelos amantes dos espaços naturais afastados da trepidação urbana, a serra e o parque do Gerês escondem sítios fascinantes e comoventes na sua beleza.‖ a) Afamado – procurado b) Afamado – procurada c) Afamados – procurados d) Afamadas – procuradas 14 38 – Assinale a alternativa que não admite ambas as regências. a) Andava alheio a / de tudo. b) Está apto a / para o trabalho. c) Sois propensos a / para o trabalho. d) Pedro ficou alucinado com / sobre a notícia. 39 – Assinale a alternativa que completa corretamente as frases abaixo. O pobre operário preferiu a morte ............ vida. Assistia- ............ o direito de reclamar. Perdi meu pai, ............ muito amava. a) do que a – lhe –que b) que a – o – a quem c) a – o – que d) à – lhe – a quem 40 – Indique, nas orações abaixo, o caso em que o acento grave, indicador da crase, não é facultativo. a) Bons tempos aqueles em que vovó nos contava histórias referentes à sua infância! b) Como a situação não agradasse a todos, foram até às últimas conseqüências. c) O comitê esportivo entregou o troféu à Marlene, capitã do time de futebol feminino. d) Mal desci do ônibus e já me dirigi à casa de meu colega. 1/2002-TURMA B O mais terrível O mais terrível não era a menina me chamando de "tio" e pedindo um trocado, ela de pé no chão no asfalto e eu no meu carro de bacana. O mais terrível não era eu escolhendo a cara e a voz para dizer que não tinha trocado, desculpe, como se a vergonha tivesse um protocolo que a absolvesse. O mais terrível não foi nem a naturalidade com que ela cuspiu na minha cara. O mais terrível foi que ela era tão pequena que a cusparada não me atingiu. Somos boas pessoas, bons cidadãos e bons pais, mas somos tios relapsos. Nossas sobrinhas e nossos sobrinhos enchem as ruas das nossas cidades, cercam nossos carros, invadem nossas vidas e insistem que são da nossa família, e não temos nada para lhes dar ou dizer, além da esmola e "desculpe". Na família brasileira "tios" e sobrinhos têm um diálogo de ameaça e medo, revolta e remorso, e poucas palavras. Nenhum consolo possível, nenhuma esperança, nenhuma explicação. O que dizer a uma sobrinha cuja cabeça mal chega à janela do carro e tenta cuspir na cara do tio? Feio. Falta de educação. Papai do céu castiga. Paciência, minha filha, este é apenas um ciclo econômico e a nossa geração foi a escolhida para este vexame, você aí desse tamanho pedindo esmola e eu aqui sem nada para te dizer, agora afasta que abriu o sinal. Não pergunte ao titio quem fez a escolha, é tudo muito complicado e, mesmo, você não entenderia a teoria. Vai cheirar cola, para passar. Vai morrer, para esquecer. Ou vai crescer, para me matar na próxima esquina. A História, dizem, terminou, e os mocinhos ganharam. Os realistas, antiutópicos, os racionais. Ficou provado que a solidariedade é antinatural e que cada um deve cuidar dos apetites dos seus. Ou seja: ninguém é "tio" de ninguém. A família humana é um mito, o sofrimento alheio é um estorvo e se miséria a tua volta te incomoda, compra uma antena parabólica. Ninguém é insensível, dizem os mocinhos, mas a convivência com a dor dos outros, que deviam ter nos educado para a compaixão, nos educaram para a autodefesa, para cuspir primeiro. Os bons sentimentos faliram, dizem os mocinhos. Confiemos o futuro ao mercado, que não tem sentimentos, que tritura gerações entre seus dedos invisíveis, pra que se envolver? Afasta do carro que abriu o sinal. Mas mais terrível do que tudo é eu ficar aqui, escolhendo frases para encher papel, até cuidando o estilo, já que é domingo. Como se fizesse alguma diferença. Como se isso fosse nos salvar, o tio da sua impotência e cumplicidade e a sobrinha anônima do seu destino. Desculpe. Luís Fernando Veríssimo As questões de 01 a 04 referem-se aos textos acima. 01 – O título estabelece coesão com o restante do texto. Por isso, após a leitura, entende-se que "O mais terrível" é a) conhecer o drama do menor que vive nas ruas, e não conseguir fazer nada para solucioná-lo. b) não confiar no mercado, que pode oferecer boas oportunidades a todos. c) não ter paciência com a criança que pede esmolas. d) constatar que a solidariedade é antinatural e que deve ser evitada. 02 – Observe as afirmações e assinale a alternativa correta. I. - O texto tem por objetivo principal enfatizar um fato ocorrido com o narrador num espaço e tempo definidos. II. - A preocupação do texto é descrever um tipo comum na nossa sociedade - a criança de rua - e os adultos que têm de conviver com elas. III. - O texto objetiva refletir sobre um problema comum nos dias de hoje, valendo-se de um fato corriqueiro da cidade grande. IV. - O texto disserta sobre a falta de compromisso da sociedade com relação ao problema da criança de rua brasileira. Pode-se dizer que estão corretas as afirmações a) I e IV. c) I e II. b) III e IV. d) II e III. 03 – No texto, pode-se deduzir que o narrador expressa alguns sentimentos seus sobre o problema abordado, a fim de sensibilizar o leitor. Com isso, marque (V) para verdadeiro e (F) para falso, tendo em vista tais sentimentos revelados nos trechos abaixo; a seguir, assinale a alternativa com a seqüência correta. ( ) "O mais terrível foi que ela era tão pequena que a cusparada não me atingiu." - piedade ( ) "Vai cheirar cola, para passar. Vai morrer, para esquecer. Ou vai crescer, para me matar na próxima esquina." - revolta ( ) "A família humana é um mito, o sofrimento alheio é um estorvo e se a miséria a tua volta te incomoda, compra uma antena parabólica." - resignação ( ) "Mas mais terrível do que tudo é eu ficar aqui, escolhendo frases para encher papel, até cuidando o estilo, já que é domingo. Como se fizesse alguma diferença. Como se isso fosse nos salvar, o tio de sua impotência e cumplicidade e a sobrinha anônima de seu destino. Desculpe." - frustração a) V, V, F, V c) V, F, F, V b) F, F, V, F d) F, V, V, F 04 – As considerações feitas no terceiro parágrafo tornam possível ler conotativamente seu fecho: "Afasta do carro que abriu o sinal.", o qual representa, por isso, a) a vitória dos mocinhos. b) a falência da família. c) o desafio das novas gerações. d) o sofrimento dos realistas. 05 – ―Agora, o cheiro áspero das flores leva-me os olhos por dentro de suas pétalas.‖ Quanto à linguagem figurada do trecho grifado acima, há nessa frase a) uma sinestesia. c) uma hipérbole. b) um pleonasmo. d) uma onomatopéia. 06 – Assinale a alternativa em que a colocação pronominal não observa a norma culta, e sim a fala corrente, coloquial. a) O pai censurou-os por se estranharem tanto durante as brincadeiras. b) Naquela hora, só me ofereceram um copo d‘água enquanto aguardava por duas horas a chegada de Luís. c) Não sei como se justificam perante o Diretor do colégio. d) João não se conformava de ter perdido a cerimônia de casamento do seu melhor amigo. Dona Jandira, mãe do noivo, o chamaria ingrato. ANULADA 15 07 – Dado o texto, ―Nunca ouvimos de Jacinto uma palavra áspera, uma lamúria, nunca respondeu com irritação às crianças que o insultavam, impiedosas, quando passava embriagado. Bêbado, sorria beatífico e acima de todas as misérias.‖ (Paulo Mendes Campos) Assinale a alternativa que traz uma seqüência incoerente para o texto. a) Era um homem de andar vacilante, trôpego, de um blá-blá- blá infinito, molengo e queixumeiro. b) Devido à bebida, falava numa língua quase ininteligível, mas que deixava a impressão de benignidade. c) Trôpego, articulava um trololó quase sem sentido e gesticulava; gestos grandes, largos, prazenteiros. d) De quando em quando, parava, respirava. E era só seus olhos confusos cruzarem com os de alguém para alentar- se e continuar numa falação pausada e sem fim. 08 – Com relação ao trecho transcrito a seguir, assinale a alternativa cujo comentário está correto. ―Com esta história eu vou me sensibilizar, e bem sei que cada dia é um dia roubado da morte. Eu não sou um intelectual, escrevo com o corpo. E o que escrevo é uma névoa úmida.‖ a) É um texto tipicamente narrativo, pois apresenta um narrador-personagem e informações quanto ao tempo e ao lugar. b) Apesar de apresentar um narrador-personagem, este não relata fatos, faz uma autodescrição de caráter dissertativo. c) O que está em evidência é o ponto de vista adotado pelo narrador, cuja tese a respeito da escrita está fundamentada em argumentos explícitos, característica de um texto essencialmente dissertativo. d) Não se pode tecer qualquer comentário quanto ao tipo de texto, uma vez que o trecho é insuficiente para tal. 09 – Leia: I. ―A velha Totonha de quando em vez batia no engenho. E era um acontecimento para a meninada. Ela vivia de contar histórias de Trancoso. Pequenina e toda engelhada, tão leve que uma ventania poderia carregá-la, andava léguas e léguas a pé...‖ (José Lins do Rego) - Esse texto é uma narrativa, pois há vários verbos de ação, e o narrador é observador. II. ―Dario vinha apressado, guarda-chuva no braço esquerdo e, assim que dobrou a esquina, diminuiu o passo até parar, enconstando-se à parede de uma casa. Por ela escorregando, sentou-se na calçada, ainda úmida de chuva, e descansou na pedra o cachimbo.‖ (Dalton Trevisan) - Apesar da presença de verbos de ação, que identificam uma narrativa, esse texto é descritivo, pois há elementos caracterizadores de Dario e do ambiente. III. ―Fala-se mal o português. Ou melhor, fala-se errado. Ninguém agüenta mais ouvir erros grosseiros do tipo ‗houveram acidentes‘ ou ‗é para mim fazer‘. Para os mais letrados, essas agressões ao idioma têm ferido tanto os ouvidos, que se decidiu partir para um contra-ataque.‖ (IstoÉ, 20/08/97) - O texto tem a intenção de expressar o ponto de vista do autor a respeito do assunto; logo, é uma descrição. Com relação às afirmações feitas sobre os textos, pode-se concluir que a) todas estão erradas. b) apenas a III está correta. c) apenas a II está errada. d) apenas a I está errada. 10 – A frase que apresenta um defeito de estilo – a ambigüidade – é: a) A prosa ou o poema devem ser extravasados numa linguagem espontânea, colorida como a fala. b) Diante do ocorrido entre você e seu filho, fiquei sem saber se o que eu disse concorreu para agravar a situação. c) A clareza é essencial a todo escritor, pois facilita para quem lê a percepção rápida do pensamento. d) Meu pai recebeu uma carta em que o irmão lhe contava como sua mulher sofrera um acidente de automóvel. 11 – Marque a alternativa em cuja frase se verifica o emprego da pontuação de acordo com a norma culta. a) A autora, sentia-se totalmente responsável por seus personagens. b) ―Morena tropicana eu quero teu sabor...‖ c) Durante os últimos meses, a nação viveu hipnotizada. d) Conrado, que morava na casa vizinha atravessava diariamente a cerca de fícus para falar comigo. 12 – Observe estes versos: ―Um dia me disseram quem eram os donos da situação Sem querer eles me deram as chaves que abrem essa prisão.‖ (Engenheiros do Havaí) As palavras grifadas estão empregadas conotativamente. Assinale a alternativa que mostra seu significado no contexto. a) chaves: consciência de ver o mundo criticamente prisão: visão limitada da realidade b) chaves: instrumentos que abrem portas prisão: recinto fechado; clausura c) chaves: problemática a ser resolvida prisão: problema em que eles se encontram d) chaves: instrumentos que vão fechar a prisão prisão: limitação do homem 13 – Assinale a alternativa cujas palavras preenchem corretamente as lacunas do texto abaixo. ―A maneira afável ____________ a moça o tratava tinha, ______________ desvanecido completamente, ao menos embotado as suscetibilidades de sua consciência _______________ do ajuste que fizera com Lemos.‖ (José de Alencar, Senhora) a) por que/ senão/ acerca b) porque/ senão/ a cerca c) porque/ se não/ acerca d) por que/ se não/ a cerca 14 – O substantivo pode figurar de várias maneiras na oração. Assinale a alternativa que apresenta substantivos com as mesmas funções sintáticas dos destes versos de Manuel Bandeira. Vi uma estrela tão alta, Vi uma estrela tão fria! Vi uma estrela luzindo Na minha vida vazia. a) "Classificado do futuro Vende-se vasinho de samambaia diretamente da mata amazônica" (Ulisses Tavares) b) "As nuvens são cabelos crescendo como rios; são gestos brancos da cantora muda;" (J.C.M. Neto) c) 'Stamos em pleno mar... Doudo no espaço Brinca o luar - doirada borboleta - E as vagas após ele correm ... cansam Como turba de infantes inquietas." (Castro Alves) d) "Deixei a cidade sumida no silêncio da madrugada." (Raul Bopp) 15 – Foram retirados os acentos de algumas palavras das frases abaixo. Assinale a alternativa em que há duas palavras proparoxítonas. a) ―Um revolver de ouro, marcas de muita magoa.‖ b) ―O mais doce nectar é o que desejo O mais fundo vacuo.‖ c) ―Cristais diluidos de clarões alacres‖ d) ―Não sei, não: se fossemos culpados, teriamos que nos retratar.‖ 16 – Assinale a alternativa em que todas as palavras estão corretamente acentuadas. a) ―Há anos raiou no céu fluminense uma nova estrêla.‖ b) E só ver com outros olhos pára poder também ver com a alma. c) E vou tratando de trabalhá-lo para pôr as idéias em ordem. d) Pôr esse caminho, passávamos para ver o Caapóra e o Saci-Pererê. 16 17 – Numere os textos, obedecendo ao código abaixo: I- Discurso direto II- Discurso indireto III- Discurso indireto livre IV- Ausência de discurso ( ) ―Caíra no fim do pátio, debaixo de um juazeiro, depois tomara conta da casa deserta. Ele, a mulher e os filhos tinham- se habituado à camarinha escura, pareciam ratos — e a lembrança dos sofrimentos passados esmorecera.‖ (Graciliano Ramos) ( ) ― Minha mãe ficava sentada cosendo olhando para mim. — Psiu... Não acorde o menino. Para o berço onde pousou um mosquito.‖ (Carlos Drummond de Andrade) ( ) ―Aprumou-se, fixou os olhos nos olhos do polícia, que se desviara. Besteira pensar que ia ficar murcho o resto da vida. Estava acabado? Não estava. Mas para que suprimir aquele doente que bambeava e só queria ir para baixo?‖ (Graciliano Ramos) ( ) ―Cristiano foi o primeiro que travou conversa, dizendo-lhe que as viagens de estrada de ferro cansavam muito, ao que Rubião respondeu que sim...‖ (Machado de Assis) Assinale a alternativa que contém a seqüência correta. a) II, II, IV, III c) IV, I, III, II b) I, I, II, IV d) III, I, IV, IV 18 – Assinale a alternativa cujo termo em destaque é advérbio. a) Procuras o bem, e o bem terás. b) Aconselhava-a para seu próprio bem. c) Compreenda-a bem, que sem ela tua vida será difícil. d) ―Quero a primeira estrela que vier / para enfeitar a noite do meu bem.‖ 19 – Com relação à regência verbal, a frase "A mulher? Ora, quero-a e quero-lhe.", de Latino Coelho, a) está correta; o autor observou os diferentes significados do verbo, o que podemos ver pelos complementos verbais. b) está incorreta; o verbo querer significa desejar e é transitivo direto; trata-se de um "erro proposital"; o autor intensifica o quanto é forte seu desejo pela mulher. c) está correta; o verbo querer, significando desejar, apresenta transitividade direta ou indireta, indiferentemente; o autor faz uso dos dois complementos para reforçar sua idéia. d) está incorreta; o autor não poderia usar a transitividade indireta, uma vez que faz uso do objeto pleonástico na forma de objeto direto. 20 – Observe: I. Um velho galo matreiro, percebendo a aproximação da rapoza, impuleirou-se numa árvore. II. Tanta gente também nos outros ensinua Crenças, religiões, amor, felicidade, Como este ascendedor de lampiões na rua! III. Consumido pela morte e pela sede, começou a exitar, não sabendo se antes comia do feno e depois bebia da água ou se antes sassiava a sede... Quanto à grafia das palavras nos períodos acima, há a) dois vocábulos errados em cada item. b) vocábulos errados somente nos itens II e III. c) um vocábulo errado no item I; um vocábulo errado no item II; dois vocábulos errados no item III. d) apenas um vocábulo errado em cada item. 21 – "Os sufixos são capazes de modificar o significado do radical a que são acrescentados. Sua principal característica, no entanto, é a mudança de classe gramatical que geralmente operam." (Gramática da Língua Portuguesa, Pasquale e Ulisses Infante). Relacione a coluna da esquerda com a da direita, observando a mudança causada pelo sufixo e assinale a seqüência correta. 1) formam verbos de substantivos ( ) legalidade, mansidão, e adjetivos fraqueza, planície. 2) formam advérbios ( ) falsificar, clarear, de adjetivos lacrimejar, gaguejar 3) formam substantivos de outros ( ) capitalismo, budismo. substantivos republicano, cavalaria. 4) formam substantivos ( ) livremente, especialmente, de adjetivos firmemente, calmamente. a) 3, 2, 4, 1 c) 4, 1, 2, 3 b) 4, 1, 3, 2 d) 2, 3, 1, 4 22 – Assinale a alternativa em que o vocábulo que funciona como conjunção. a) O jornal a que me refiro publicará seu texto. b) O importante é que todos se convençam de que é preciso lutar. c) Foram inúteis os argumentos de que fizeram uso durante o debate. d) É importante saber conviver com as mudanças que acompanham a existência. 23 – Assinale a única alternativa em que não há adjetivo ou locução adjetiva. a) ―A vida é combate,/ Que aos fracos abate,/ Que os fortes, os bravos,/ Só pode exaltar.‖ (Gonçalves Dias) b) ―Alguns dias dava-lhe gana de satisfazer o apetite, devorando lascas de pirarucu assado, com farinha d‘água e lata de marmelada...‖ (Inglês de Sousa) c) Para os índios, o ingresso na vida adulta é sinônimo de mudanças mais radicais. Eles precisam ter coragem e sangue- -frio para cumprir os rituais de passagem. d) Para os brancos, penugens no rosto e menstruação representam ―os rituais‖ de que necessitam para a chamada vida de adulto. 24 – Observe: Reclinada molemente na sua verdejante colina, como odalisca em seus aposentos, está a sábia Coimbra, a Lusa Atenas. Beija- lhe os pés, segredando-lhe de amor, o saudoso Mondego. E em seus bosques, no bem conhecido salgueiral, o rouxinol e outras aves canoras soltam seus melancólicos trilos. Quando vos aproximais pela estrada de Lisboa, onde outrora uma bem organizada mala-posta fazia o serviço que o progresso hoje encarregou à fumegante locomotiva, vede-la branquejando, coroada do edifício imponente da Universidade, asilo da sabedoria. (O primo Basílio. São Paulo, Abril Cultural, 1979. p. 229-30.) Os pronomes, cuja função essencial é denotar ou determinar os seres, funcionam também como elementos coesivos, isto é, estabelecem a ligação entre os elementos de uma frase ou de um texto, a fim de manter a sua unidade. Leia o texto acima e marque a alternativa em que o termo referente não é retomado pelo pronome indicado. a) ―vede-la‖ – o pronome oblíquo ―a‖ retoma o substantivo próprio ―Coimbra‖. b) ―onde‖ – o pronome relativo retoma o seu termo antecedente ―estrada de Lisboa‖. c) ―seus bosques‖ – o pronome possessivo refere-se a ―saudoso Mondego‖. d) ―que‖ – o pronome relativo refere-se ao termo antecedente ―serviço‖. 25 – Assinale a alternativa em que o pronome relativo foi utilizado incorretamente na junção das frases para formar um único período. a) A sábia natureza me abençoará eternamente. Dediquei a ela minha longa vida. A sábia natureza, a que dediquei minha longa vida, abençoar-me-á eternamente. b) Vivo em sintonia com aquela centenária árvore. Sob sua rama jaz minha infância. Vivo em sintonia com aquela centenária árvore, sob cuja rama jaz minha infância. c) Aquele rio é saudosa lembrança. Em suas águas douradas, deixei minha inocência. Aquele rio em cujas águas douradas deixei minha inocência é saudosa lembrança. d) A velha e amada casa fica no alto da montanha, quase a tocar o céu. Eu nasci naquela casa. 17 A velha e amada casa na cuja nasci fica no alto da montanha, quase a tocar o céu. 26 – ―Estamos ansiosos. Vão apresentar-nos a programação.‖ Assinale a alternativa que contém a relação implícita entre essas orações. a) Finalidade c) Temporalidade b) Causalidade d) Oposição 27 – Observe os termos grifados nas frases abaixo, quanto à função sintática por eles exercida. I. Hillary Clinton, primeira-dama dos Estados Unidos está lançando o livro ―Convite à Casa Branca‖. II. Saiba quais são as principais ferramentas para entrar no mundo do comércio virtual. III. Bonito, cidade do Mato Grosso do Sul, é mais que bonita. 28 – Assinale a alternativa cujo verbo se conjuga por haver. a) Provia a despensa toda vez que aparecia. b) Revia os amigos, quando sentia saudade. c) Desavimo-nos, diariamente, por questões fúteis. d) Ele reavia suas forças, quando outra tragédia o acometeu. 29 – Relacione as orações em destaque a seus significados e assinale a alternativa em que a seqüência esteja correta. (1) contraste, ressalva (3) conclusão (2) alternância (4) explicação ( ) A mim ninguém engana, que não nasci ontem. ( ) O instinto social não é privilégio do homem, antes, se nos depara nos próprios animais. ( ) "Já atravessa as florestas; já chega aos campos do Ipu." ( ) Ele é teu amigo: respeita-lhe, pois, a verdade. a) 1, 3, 2 e 4 c) 4, 1, 3 e 2 b) 1, 2, 3 e 4 d) 4, 1, 2 e 3 30 – Observe: I. As paixões tornam os homens cegos. II. A árvore ficou sem folhas. III. "Ninguém se fie da felicidade presente." É correto afirmar a respeito dos períodos acima que a) em I há predicado nominal, pois nele há um verbo de ligação. b) o núcleo do predicado de II é sem folhas. c) o núcleo do predicado de III é o verbo fie. O predicado, portanto, é verbo-nominal. d) o predicado de II é verbal, pois o verbo ficar é intransitivo. 31 – Classifique os verbos quanto à transitividade na frase: ―Os ilhais da fera arfam de fadiga, a espuma franja-lhe a boca, as pernas vergam, e os olhos amortecem de cansaço.‖ a) arfam – intransitivo franja – transitivo direto vergam – intransitivo amortecem – intransitivo b) arfam – intransitivo franja – transitivo direto e indireto vergam –transitivo direto amortecem – transitivo indireto c) arfam – transitivo direto franja – transitivo indireto vergam – transitivo direto amortecem – intransitivo d) arfam – transitivo indireto franja – transitivo direto e indireto vergam – intransitivo amortecem – transitivo indireto 32 – Assinale a alternativa em que o emprego do pronome demonstrativo está incorreto. a) Esta empresa ―Cosméticos Vida‖ agradece a essa loja a preferência dada e cumprimenta-a pelo novo ponto comercial. Atenciosamente, Diretor de Vendas b) De acordo com o que foi solicitado pelo Gerente dessa cadeia de lojas, esta empresa antecipará a entrega do material para o Natal. Atenciosamente, Diretor de Vendas c) ―Laços de Família‖ e ―Por Amor‖ apresentam a personagem Helena. Nessa, a personagem foi vivida por Regina Duarte; naquela, por Vera Fischer. d) Essa seção solicitou que fossem enviados, com urgência, dois funcionários, até o último dia deste mês. Concedido. Gerente administrativo 33 – ―Uma formiguinha atravessa, em diagonal, a página ainda em branco. Mas ele, aquela noite, não escreveu nada. Para quê? Se por ali já havia passado o frêmito e o mistério da vida...‖ Quanto à função sintática, os termos destacados, no trecho acima, classificam-se como a) adjunto adverbial e núcleo do sujeito. b) objeto direto e adjunto adnominal. c) adjunto adverbial e núcleo do objeto. d) núcleo do adjunto adverbial e objeto direto. 34 – A única maneira de ter sensações novas é construíres-te uma alma nova. (...) E o único meio de haver coisas novas, de sentir coisas novas é haver novidade no senti-las. Muda de alma como? Descobre-o você. Lendo com atenção o texto de Fernando Pessoa, que foi modificado, percebe-se erro de a) pontuação. c) concordância verbal. b) regência verba. d) concordância nominal. 35 – O programa de educação do Estado de Minas Gerais é considerado inovador. Tem recebido referências elogiosas, a última ________ feita pelo Ministro José Serra, na sexta-feira (...). A informação existente é a de que a campanha das etapas do programa ____________ por um grupo de pessoas entre ________ alguns empresários de Belo Horizonte. (Revista Veja, adaptado) Complete os espaços em branco, observando os princípios da concordância. a) da qual – estão sendo feitas – o qual b) das quais – está sendo feita – as quais c) das quais – estão sendo feitas – o qual d) da qual – está sendo feita – as quais 36 – Assinale a alternativa que apresenta erro quanto à regência nominal. a) Aquele que se acha ávido de sabedoria enriquece sua alma. b) Os novos alunos já se mostram adaptados para com a escola. c) Sempre viajei imbuído em bons ideais. d) Seu medo à opressão é maior que sua obediência aos velhos dogmas. 37 – Observe: I- Viu a nave espacial afastar-se da plataforma de lançamento a uma velocidade próxima a da luz. II- A volta da nave espacial a Terra gerou euforia. III- Todo movimento só existe em relação a outros objetos. IV- Desde a uma hora, o diretor esperava pelos novos alunos. V- Os efeitos da relatividade que estão aplicados a aceleração só se aplicam... Das proposições acima, foram retirados alguns acentos graves. Ao analisá-las, pode-se afirmar que deveria ocorrer o fenômeno da crase a) 3 vezes. c) 5 vezes b) 4 vezes. d) 6 vezes ANULADA 18 38 – Oração para os aviadores Santa Clara, clareai Estes ares. (...) Afastai Todo risco Por amor de S. Francisco Vosso mestre, nosso pai, Santa Clara, todo risco Dissipai Santa Clara, clareai. (Manuel Bandeira) No poema acima, Qual termo funciona como vocativo? a) Vosso mestre c) Santa Clara b) nosso pai d) São Francisco 39 – Assinale a alternativa em que o pronome oblíquo não exerce a função de complemento verbal. a) Nunca as conheci. b) Recomendei-lhe bastante cautela. c) Diga-me a verdade, agora! d) Tocou-lhe as mãos com ternura. 40 – Considerando a possibilidade ou não da flexão de número de alguns nomes, de acordo com seu valor morfológico, assinale a alternativa incorreta. a) Bastantes anos depois, eu acolheria as verdades pronunciadas pelo meu pai. b) As portas, meio abertas, deixavam ver o interior das salas. c) Os alimentos eram os mais baratos possíveis. d) Caras, na feira, custavam as frutas. 2/2002-TURMA A O que quero é fome Conheço muitos testes de inteligência. Não conheço nenhum teste de sabedoria. É importante saber a diferença entre essas duas, inteligência e sabedoria, freqüentemente confundidas. A inteligência é a nossa capacidade de conhecer e manipular o mundo. Ela tem a ver com o poder. A sabedoria é a graça de saborear o mundo. Ela tem a ver com a felicidade. As escolas se dedicam a desenvolver e avaliar a inteligência. Para isso desenvolveram testes. Os testes avaliam a inteligência dos alunos por meio de números. Mas elas nada sabem sobre a sabedoria, e nem elaboram testes para avaliá-la. Nas escolas e universidades muitos tolos são aprovados cum laude. A inteligência é muito importante. Ela nos dá os meios para viver. Mas somente a sabedoria é capaz de nos dar razões para viver. Muitas pessoas se suicidam porque, tendo todos os meios para viver, não tinham as razões para viver. Proponho-lhe um teste de sabedoria. Ele é muito simples. O seu aniversário está chegando. Você já não é mais jovem. O espelho lhe revela coisas que você não gostaria de saber. Diante da sua imagem no espelho existe sempre o perigo de que uma magia perversa aconteça, e você seja repentinamente transformado em bruxa ou ogro — tal como aconteceu com a madrasta da Branca de Neve. Em desespero, você invoca os deuses. Eles vêm em seu socorro e lhe dizem que atenderão a um desejo seu, a um único desejo. Que súplica você lhes faria? Digo-lhe que essa seria a hora da pureza de coração, quando todos os supérfluos têm de ser deixados de lado. "Pureza de coração" — assim disse Kierkegaard, meu querido filósofo solitário, companheiro já morto; por vezes os mortos são companhia melhor que os vivos, porque falam menos e ouvem mais —, pureza de coração, ele disse, "é desejar uma só coisa". Digo que isso é sabedoria, mas pode parecer mais coisa de neurótico obsessivo, ficar querendo uma coisa só, o tempo todo. Você entenderá o que digo se você prestar atenção no vôo dos pássaros. E para ajudá-lo nesse dever de casa, transcrevo o que Camus pensou, ao observá-los. "Se durante o dia o vôo dos pássaros parece sempre sem destino, à noite, dir-se-ia reencontrar sempre uma finalidade. Voam para alguma coisa. Assim talvez, na noite da vida..." O texto termina assim, com essas reticências que, segundo Mário Quintana, são o caminho que o pensamento deve continuar a seguir. Assim é o coração. Há momentos na vida em que ele é como o vôo dos pássaros durante o dia: oscila em todas as direções, sem saber direito o que quer, ao sabor das dez mil coisas que o fascinam, tão desejáveis, cada uma delas uma taça de prazer. Chega um momento, entretanto, em que é preciso escolher uma direção — é preciso descobrir aquela palavra, aquela única palavra que dá nome ao nosso sofrimento, que nomeia a nossa nostalgia, para que saibamos para onde ir. A Adélia Prado passou por esse teste. Disse ela no seu poema "O tempo": (...) Descobri que a seu tempo vão me chorar e esquecer. Vinte anos mais vinte é o que tenho. Nesse exato momento do dia vinte de julho de mil novecentos e setenta e seis, o céu é bruma, está frio, estou feia, acabo de receber um beijo pelo correio. Quarenta anos! Não quero faca nem queijo. Quero a fome. (...) Rubem Alves (in "Cenas da Vida") As questões de 01 a 07 referem-se ao texto ao lado 01 – O autor define inteligência e sabedoria com o objetivo de mostrar a) uma falha da escola, que privilegia a inteligência. b) que a inteligência é fundamental para se chegar à sabedoria. c) que a inteligência está para a felicidade como a sabedoria para o poder. d) que a sabedoria é para a vida, não para a escola. 02 – O autor induz o leitor a uma resposta à pergunta "Que súplica você faria?". Essa "sábia" resposta seria a) a aquisição de muitos bens materiais. b) alguma coisa ligada à pureza de coração. c) a direção do caminho. d) a invocação dos deuses. 03 – Analise as conotações e sua interpretação para assinalar a alternativa correta abaixo: I- "o coração é como o vôo dos pássaros" – nossos sentimentos oscilam, não sabemos o que queremos II- "na noite da vida" – na maturidade, na velhice III- "vôo dos pássaros" – os direcionamentos da vida a) Todas estão corretas. b) Somente a I está correta. c) Estão corretas a II e a III apenas. d) Somente a III está correta. 04 – A introdução do texto serve para nos levar a entender que a) é a inteligência que vai nos guiar pela vida. b) a sabedoria nos permitirá escolher bem nossos caminhos. c) de nada vale a sabedoria, se não conseguimos o que queremos. d) devemos saciar todos os nossos desejos. 05 – "Não quero faca nem queijo, quero fome." A citação de Adélia Prado quer dizer que a) ela tem consciência de que a insatisfação é um impulso vital. b) ela solidariza-se com quem passa fome. c) ela está revoltada com a maturidade que chega. d) a passagem dos anos instiga seu instinto agressivo. 06 – As "razões para viver" estão ligadas à a) inteligência. c) satisfação dos desejos. b) sabedoria. d) manipulação do mundo. 07 – "Desejar uma só coisa" significa a) sabedoria na busca do que é essencial. b) falta de imaginação e de amadurecimento. c) apego de neurótico obsessivo a uma idéia. d) simplismo, conformismo advindo da idade. 08 – Assinale a alternativa em que a palavra destacada apresenta correta divisão silábica. a) "Tal luta interestadual sempre existiu." – in - ter - es - ta - du - al b) "Vai subalugar a sala a um senhor idoso." – su - ba - lu - gar c) Sublocou uma casa em Ubatuba a um amigo. – su - blo - cou d) "Como bom filho do século 19, superestimava as possibili- dades da Ciência." – su - per - es - ti - ma - va 09 – "De longe, avista-se o grito das araras." Quanto à linguagem figurada, temos nessa frase a) um pleonasmo. c) uma hipérbole. b) uma sinestesia. d) uma onomatopéia. 19 10 – "Todos os políticos são corruptos; daí vem o desinteresse do povo para as próximas eleições." Observa-se, no texto acima, uma falha que compromete a argu- mentação. Assinale a alternativa que a contém. a) Emprego de abrangência inadequada. b) Incoerência na conclusão. c) Uso de conceitos que se contradizem. d) Falta de coesão. 11 – Em cada citação, há elementos que podem definir o texto como narrativo, descritivo ou dissertativo. Desse modo, coloque 1 para descrição, 2 para narração, 3 para dissertação e assinale a alternativa com a seqüência correta. ( ) "Olhou para o retrato de Beethoven, e começou a executar a sonata, sem saber de si, desvairado e absorto ... Tornou ao piano; era a vez de Mozart, pegou de um trecho, e executou-o do mesmo modo, com a alma alhures." (Machado de Assis) ( ) "Ainda que eu falasse línguas, as dos homens e dos anjos se eu não tivesse amor seria como sino ruidoso ou como címbalo estridente." ( ) "Bonito, lá era. Uma beleza de galo. Andava aprumado, com pose de peru, satisfeito com a própria figura. O papo era um depósito de cacarecos constantes. As asas desciam, ora para um lado, ora para outro, roçando a terra em movimentos rápidos." a) 2, 1, 3 b) 2, 3, 1 c) 1, 2, 3 d) 1, 3, 2 12 – Leia as frases abaixo. I- "Uma fina saudade, porém, começou a alinhavar-se em seus dias." (Marina Colasanti) II- "Deixaram o crânio na janela da aposentada e voltaram de manhã para observar." (Moacyr Scliar) III- "Essas pessoas são farinha do mesmo saco." Há conotação apenas em a) I. b) II e III. c) I e III. d) I e II. 13 – Indique o período escrito em forma de discurso indireto. a) "Branco foi logo indagando: — Que foi que aconteceu, André?" (O. de Faria) b) "— Não há razão para desesperar. Há muita gente que tem preguiça de escrever..." (Lima Barreto) c) "E uma tarde um moleque chegou às carreiras, gritando: A cheia vem no engenho de seu Lula." (J.L. do Rego) d) "Sinhá Vitória respondera que isso era impossível porque eles vestiam mal..." (G. Ramos) 14 – Assinale a alternativa na qual o termo grifado tenha a clas- sificação morfológica do termo em destaque em "Diante do espelho, não encontrei meu eu." a) "Onde ela está, não digo eu (...) Sei-o eu só: inda bem." b) "Anoiteça e amanheça eu." c) "Existe sempre o eu no tu e o tu no eu." d) "Eu te olharei com teus olhos E tu me olharás com meus olhos." 15 – Complete os espaços com "onde" ou "aonde", segundo a norma culta, e assinale a opção correta. "______ ele estava durante esse tempo eu não sei. Só me lembro a rua ______ ele morava. Você sabe ______ fica a casa dele, rua Olavo Bilac, ______ eu costumava ir todas as tardes." a) Onde – onde – onde – aonde b) Onde – aonde – onde – onde c) Aonde – aonde – onde – onde d) Aonde – onde – onde – aonde 16 – Das alternativas abaixo, assinale a que contém onomatopéia. a) A velha paineira sentia-se desolada à beira do caminho. b) Repetirei esta história cem vezes se você quiser. c) Após a peleja, a rua tingiu-se de vermelho. d) Quando Osmarina vai à missa, a vizinhança logo percebe o plac-plac do tamanquinho da menina. 17 – "Minha terra tem palmeiras Onde canta o sabiá." (G. Dias) Quanto aos vocábulos assinalados no texto acima, pode-se afirmar que I- terra e canta possuem 5 letras e 4 fonemas cada. II- terra possui 1 dígrafo e canta possui 1 encontro consonantal. III- em terra e canta há 1 dígrafo em cada. IV- apenas terra possui 1 dígrafo. Está(ão) correta(s) a(s) afirmação(ões): a) I e II. c) I e III. b) III e IV. d) IV, somente. 18 – Assinale a alternativa cuja palavra, quanto à tonicidade, está colocada corretamente no gráfico. Obs.: a acentuação gráfica pode ter sido alterada em função do objetivo do teste. sílaba tônica a) ben ção b) pú di co c) u re ter d) á va ro 19 – Assinale a alternativa em que há duas palavras monossílabas átonas, uma monossílaba tônica, uma oxítona e duas paroxítonas. a) Os mandacarus vestiam a campina. b) Tentou libertar-se dos pensamentos. c) Conversa cheia de mal-entendidos e repetições. d) Não seria mau tornarem a comandar... 20 – "Os covardes nunca tentaram, os fracos ficaram pelo caminho. Só os fortes conseguiram." As palavras grifadas constituem derivação a) regressiva. c) sufixal. b) prefixal. d) imprópria. 21 – Assinale o grupo de frases cuja correlação verbal está incorreta. a) Se um dia você for a Manaus, leve-me com você. Quando o carteiro bateu à minha porta, meu coração quase saltou do peito. b) Que eu me arrependa, pode ser, mas jamais confessarei minha culpa. Tão-logo debicar a sobremesa, encontre-me no escritório. c) Se acaso me encontrares um dia, leva-me contigo. Qualquer que seja a explicação, não a aceitarei. d) Ele caberia na cama se ela for grande. Não seja malvado; o pobre animalzinho não lhe tirara pedaços. 22 – O júri julgou com prazer qual o melhor quadro da galeria. Os substantivos grifados classificam-se, respectivamente, como a) simples, concreto, primitivo e comum. b) coletivo, abstrato, simples e coletivo. c) concreto, comum, coletivo e abstrato. d) comum, simples, abstrato e coletivo. 23 – Das alternativas abaixo, uma está incorreta quanto à flexão do substantivo. Assinale-a. a) João era o único testemunha do acidente. b) Para a garota, o dó era o pior dos sentimentos. c) Na festa, o champanha era a bebida preferida. d) O marajá e a marani eram muito queridos pelos súditos. 24 – Assinale a alternativa em que há conjunção integrante. a) Já li o livro que você me emprestou. b) Não venha, que não estarei mais aqui. c) Espero que você recupere logo a saúde. d) Quase que a criança caiu. 25 – O advérbio modifica a oração inteira em: a) Silenciosamente todos trabalhavam na saleta. b) O senhor não ouviu seu parceiro de trabalho. c) O carteiro escondeu-se atrás do muro, todo medroso. d) Ele não precisaria ir muito longe. 26 – Assinale a alternativa em que a colocação do pronome átono, de acordo com a norma culta, pode ser tanto proclítica quanto enclítica. a) Ninguém dirá a verdade. (lhe) b) Começou a maltratar. (a) c) Chamaria louca, com certeza. (a) d) Corri para ajudar. (o) 20 27 – Assinale a alternativa cujas palavras substituem correta- mente as locuções grifadas em "As águas do rio eram um verdadeiro espetáculo de dança". a) fluviais – coreográfico c) fluviais – magistral b) pluviais – flamejante d) pluviais – dançante 28 – Em "Não permita Deus que eu morra", o verbo destacado encontra-se no modo a) infinitivo. c) imperativo. b) subjuntivo. d) indicativo. 29 – Assinale a alternativa que explica o uso das vírgulas em "As estrelas, no céu límpido, brilhavam divinamente distantes." a) Aposto b) Coordenação de termos c) Adjunto adnominal deslocado d) Adjunto adverbial deslocado 30 – Nos períodos abaixo, a pontuação mudou a classificação sintática dos termos grifados. Assinale a alternativa que contém a afirmação correta. I- Quando amamentava Hércules, Juno derramou leite no céu e formou a Via Láctea. (objeto direto) II- Quando amamentava, Hércules, Juno derramou leite no céu e formou a Via Láctea. (vocativo) a) Apenas a I está correta. b) Apenas a II está correta. c) As duas classificações estão corretas. d) Nenhuma das classificações está correta. 31 – Assinale a alternativa que contém oração subordinada substantiva. a) As pernas tremiam porque ele tinha medo. b) É esta a verdade que ninguém contestou. c) É necessário que você compareça à reunião. d) A cobra é um animal que se arrasta. 32 – No período "Ainda que fosse bom jogador, não ganharia a partida", a oração sublinhada encerra idéia de a) causa. c) condição b) concessão. d) fim. 33 – Observe: I- A paisagem nos dá um momento de prazer. II- As formigas começaram a trincá-lo. III- E vejo-a, tranqüilamente, depois de um tempo de desespero. Os pronomes oblíquos sublinhados nas frases acima desempenham, respectivamente, a função sintática de: a) OI, OI, OD. c) OI, OD, OD. b) OD, OI, OI. d) OI, OD, OI. 34 – Assinale a alternativa cuja classificação do termo grifado está correta. a) "Coisa útil a um há de ser a outros?" — complemento nominal b) "Não lhe tinha medo." — objeto indireto c) "A palavra esperada, não foi ela quem a proferiu." — complemento nominal d) "Ele foi o único homem a quem amara neste mundo." — objeto indireto 35 – Em qual das frases abaixo o termo grifado é agente da passiva? a) Os móveis foram retirados pelos fundos da casa. b) Eles tinham feito a maior algazarra pela cidade. c) Fotos de satélites, que antes só os espiões tinham, podem ser compradas por telefone. d) O réu foi condenado, depois de horas de julgamento, pela juíza Cristiane. 36 – Observe: I- Mais de dez candidatos __________ a prova ontem. (perder) II- "A segurança e firmeza com que lhes respondi ___________ as pessoas perplexas." (deixar) III- A inveja, o ódio, a maldade humana, tudo ________ ciladas em teu caminho. (armar) IV- Nem um nem outro ___________ com os festejos populares. (andar preocupado[s]) As frases em que a concordância pode ser feita tanto no singular como no plural são a) I e III apenas. c) I e II apenas. b) II e IV apenas. d) III e IV apenas. 37 – Examine as orações abaixo e assinale a alternativa que contém concordância ideológica. I- São José dos Campos é bonita. II- "V. S.ª é operoso — disse o velho cidadão." III- "É um crime de lesa-majestade", disse-lhe a anciã. IV- "Um e outro rapazote maltrapilhos, sem vintém, sem ninguém." a) Somente a II. c) I e III apenas. b) I e II apenas. d) I, II, III e IV. 38 – Quanto à regência, assinale a alternativa incorreta. a) Informe-lhes as mudanças ocorridas. b) Informo aos professores o novo currículo. c) Informe os interessados sobre o próximo concurso. d) Informo-os que o novo currículo entrará hoje mesmo em vigor. 39 – De acordo com a norma culta, quanto à regência verbal, assinale a alternativa em que uma das orações está incorreta. a) "João reparou o carro." "João reparou no carro." b) "Mandei-o comprar um livro." "Mandei-lhe comprar um livro." c) "Ouvi-o dizer isto de você." "Ouvi-lhe dizer isto de você." d) "A atitude dele implicará conseqüências graves." "A atitude dele implicará em conseqüências graves." 40 – Complete as lacunas das frases 1 com as devidas preposições, observando a ocorrência de contrações. Após, assinale a alternativa em que a frase 2 correspondente não apresenta o fenômeno da crase. a) 1 - Esteve ____ praia no domingo. 2 - Foi ____ praia no domingo. b) 1 - Esteve ____ Argentina no último dia 15. 2 - Foi ____ Argentina no último dia 15. c) 1 - Esteve ____ Guaratinguetá hoje de manhã. 2 - Foi ____ Guaratinguetá hoje de manhã. d) 1 - Esteve ____ aquela igreja no Natal passado. 2 - Foi ____ aquela igreja no Natal passado. 2/2002-TURMA B DOS RATOS DE CAMUS AOS DE NOVA IGUAÇU A atualidade de uma luta contra o absurdo do mundo Como no livro A Peste, de Albert Camus, os ratos estão invadindo uma cidade. Só que em vez de Oran, na Argélia, é Nova Iguaçu, na Baixada Fluminense. Por isso, a prefeitura do município lançou um programa de combate à crescente e alarmante presença dos roedores. Pagará R$ 5 por quilo de animal apreendido. Postos de recolhimento e incineração serão instalados ainda neste mês em pelo menos seis bairros, onde as autoridades sanitárias calculam que 40% das casas tenham ninhos de ratos. A população da cidade se dividiu, como mostrou o repórter Alex Martins, que lá esteve. Há os que vêem na medida uma possível fonte de renda – "Será dinheiro a mais em casa", disse um – e os que consideram uma irresponsabilidade o projeto, que nem inédito é. No início do século passado, o governo brasileiro lançou uma campanha igual e acabou estimulando a criação doméstica dos roedores. "Não adianta tentar acabar com os ratos sem mudar as condições sanitárias", advertiu o doutor Jorge Darze, diretor da Federação Nacional dos Médicos, quase repetindo o doutor Rieux, o do livro, ao ver os ratos saindo dos esgotos e espalhando a epidemia: "O bacilo da peste não morre nem desaparece jamais". A mensagem dos dois médicos é muito útil nestes tempos de novas pestes, em que a tentação é acabar com o mal pelo puro extermínio, como se isso fosse possível aqui, em Oran ou no Afeganistão. Não por acaso foi Camus, o mais atual dos escritores surgidos durante ou após a Segunda Guerra, o que mais respostas deixou para o presente, o primeiro a empregar, em 1946, a expressão "fim das ideologias", quem obsessivamente chamou a atenção para o absurdo da condição humana: "A sensibilidade que se pode encontrar esparsa no século". 21 Camus morreu antes das guerras do Vietnã, do Camboja, do Golfo, de Kosovo e antes também de ver sua Argélia dilacerada pelo fundamentalismo religioso. A vida pelo menos poupou-o dessas tragédias, ele que se angustiou tanto com as outras de seu tempo: as duas guerras mundiais, a guerra civil espanhola, os expurgos stalinistas, a tortura, o holocausto, Hiroshima, Nagasaki. Ao contrário de Sartre, que contemporizou com o stalinismo por pretextos táticos, o autor (...) recusou sem complacência todo tipo de tirania (...). Entre dois males, ele preferia combater os dois. Entre um radicalismo e outro, ele dispensava ambos e ficava com a lucidez e a moderação. "Eu decidi recusar tudo o que, de perto ou de longe, por boas ou más razões, faça morrer ou justifica que se faça morrer." Os dois conceitos fundamentais do pensamento camusiano – o absurdo do mundo e a revolta contra as injustiças – foram elaborados num tempo em que valores como razão e liberdade tinham sido ameaçados pela insensatez dos massacres e da guerra – como hoje. A esse absurdo ele opunha a consciência do homem revoltado. Que a lucidez de Camus ilumine nossos sombrios tempos pós-modernos. Que se aprenda com seus símbolos e alegorias que as epidemias de hoje, como as de ontem, não se evitam com o simples extermínio de ratos, reais ou metafóricos, mas com o combate às condições que tornam possível sua existência e proliferação – em Nova Iguaçu, em Oran ou no Afeganistão. Que prevaleça seu pessimismo cheio de esperança. Zuenir Ventura (Revista Época, nov. de 2001) As questões de 01 a 07 referem-se ao texto acima. 01 – Com relação ao texto, é correto afirmar que a) faz uma reflexão sobre a importância das características literárias do escritor Camus a partir da coincidência de um fato da realidade brasileira com o retratado por ele em seu livro "A Peste". b) faz uma crítica veemente à forma de combate aos ratos, sugerida pela Prefeitura de Nova Iguaçu, uma vez que tal forma se mostrou ineficiente tanto na Argélia como no Afeganistão, bem como no Brasil, no início do século passado. c) seu autor se refere aos problemas mundiais valendo-se, para isso, da alegoria utilizada por Camus para falar dos problemas de seu tempo. d) tem como objetivo central fazer ver a importância das atitudes a serem tomadas pelas autoridades com relação a questões sanitárias. 02 – Lendo a frase "O bacilo da peste não morre nem desaparece jamais" e relacionando-a ao objetivo do texto, o termo bacilo pode ser ampliado para a) extermínio. b) ideologia. c) esperança. d) pessimismo. 03 – Leia as sentenças abaixo. I- Albert Camus e Sartre são contemporâneos dos expurgos stalinistas. II- Segundo o autor do texto, os valores como razão e liberdade, como no passado, só serão preservados mediante a consciência do homem revoltado. III- O autor do texto apresenta dois pensamentos fundamentais na sua análise: o absurdo do mundo e a revolta contra as injustiças. IV- Camus vê para os tempos pós-modernos uma maneira de vencer as epidemias: a lucidez por meio de um pessimismo cheio de esperança. Está correto o que se afirma em a) I e II. b) III e IV. c) I apenas. d) IV apenas. 04 – A idéia expressa em "A sensibilidade que se pode encontrar esparsa no século" é a de que a sensibilidade está cada vez mais a) escassa, parca, na história do homem. b) solta, sem elos que o organizem e se imponham no tempo. c) arraigada às atitudes do homem, daí o grande absurdo da condição humana. d) disseminada ao longo da história da humanidade. 05 – "Será dinheiro a mais em casa." A frase dita por um morador de Nova Iguaçu, relaciona-se bem à seguinte: a) "Eu decidi recusar tudo o que, de perto ou de longe (...) faça morrer ..." b) "A sensibilidade que se pode encontrar esparsa no mundo." c) "Não adianta acabar com os ratos sem mudar as condições sanitárias." d) "Entre um radicalismo e outro, ele dispensava ambos e ficava com a lucidez e a moderação." 06 – A idéia expressa em "Eu decidi recusar tudo o que, de perto ou de longe, por boas ou más razões, faça morrer ou justifica que se faça morrer." é a mesma expressa em a) "A estupidez é infinitamente mais fascinante que a inteligência. A inteligência tem seus limites, a ignorância não." b) "Admitir que há guerras justas é o mesmo que admitir que há injustiças justas." c) "Envergonhar-nos-íamos freqüentemente de nossas mais belas ações se o mundo visse todos os motivos que as produzem." d) "O ser humano é o único animal que assassina em massa, é o único que não se adapta à sua própria sociedade." 07 – O texto compõe-se de duas partes, uma narrativa e uma dissertativa. Observando a maneira de o autor organizar o conteúdo do texto, é correto afirmar que a) as duas partes são importantes para o objetivo do texto e estão claramente definidas, não havendo entre elas nenhuma idéia de transição. b) há predominância da parte narrativa, uma vez que o autor conta a história dos ratos e a da vida de Camus. c) a parte narrativa constitui, na verdade, um pretexto para o objetivo maior do texto: mostrar a atualidade de uma luta contra o absurdo do mundo. d) a parte narrativa constitui, sem dúvida, a parte mais importante do texto, pois ela contém em si mesma a idéia de luta contra o absurdo do mundo. 08 – "Por detrás dessas paredes, desses muros, dentro dessas casas pobres e desses castelinhos de brinquedo, há criaturas que falam, discutem, entendem-se e não se entendem, amam, odeiam, desejam, acordam todos os dias com mil perguntas e não sei se chegam à noite com alguma resposta." (Cecília Meireles, Inéditos) Esse texto possui características a) narrativas, pois o texto relata uma transformação, ou seja, a passagem de um estado inicial para um estado final. b) dissertativas, uma vez que há a intenção implícita de mostrar a natureza controversa do ser humano. c) descritivas, uma vez que ações habituais são formas de caracterizar pessoas. d) descritivas, porque a seqüência temporal do texto tem a intenção de mostrar ações habituais. 09 – Assinale a alternativa em que há ambigüidade no texto. a) Eles foram por um caminho, e nós fomos por outro. Só no final da tarde, os guardas encontraram-nos. b) O médico mandou-me aplicar uma injeção em meu pai. A tarefa nos foi quase impossível, porque desde criança ele tinha ódio a injeções. c) Dr. Davi mandou-me internar, pois, havia dias, uma gripe me consumia e me afastara do trabalho. d) Os brasileiros estamos convictos de que, em se reelegendo, o Presidente será mais flexível com os funcionários públicos. 22 10 – Assinale a alternativa cuja inobservância das regras gramaticais caracteriza-se como defeito textual. a) "quando nasci um anjo louco muito louco veio ler minha mão não era anjo barroco era um anjo muito louco, torto com asas de avião." (Torquato Neto) b) Pega ladrão Alguém tirou um pedaço do meu P ~ O (Kátia Bento) c) "Umas carabinas que guardava atrás do guarda-roupa, a gente brincava com elas, de tão imprestáveis." (J. Régio) d) Vídeos XXXX: "Para você nunca mais ter de assistir à Orquestra de Berlim ao som do concerto do encanamento do vizinho." (informe publicitário – adaptado) 11 – Assinale a alternativa em que não há conotação. a) "...Mas o livro é enfadonho, cheira a sepulcro, traz certa contração cadavérica; vício grave, e aliás ínfimo, porque o maior defeito deste livro és tu, leitor..." b) "Tinha-me lembrado a definição que José Dias dera deles, olhos de cigana oblíqua e dissimulada. Eu não sabia o que era oblíqua, mas dissimulada sabia..." c) "Por ser ignorante era obrigada na datilografia a copiar letra por letra (...) ela era incompetente. (...) Faltava-lhe o jeito de se ajeitar." d) "Na verdade, humor é uma análise crítica do homem e da vida. Uma análise não obrigatoriamente comprometida com o riso, uma análise desmistificadora, reveladora, cáustica..." 12 – No período: "Hoje o samba saiu procurando você/ Quem te viu/ Quem te vê/ Quem não a conhece não pode mais ver pra crer...", a figura de linguagem encontrada no texto acima é a a) sinestesia. b) prosopopéia. c) metonímia. d) polissíndeto. 13 – Numere os parênteses abaixo, relacionando os exemplos grifados e a teoria de discursos, e assinale a seqüência correta. "A mãe avisou: "Se tu te perdê, ó ..." E com a mão mostrou o que aconteceria com Adroaldo. Um tapa daqueles, dos especiais, reservados para grandes ocasiões. Ele que inventasse de se perder na praia. E o Adroaldo se perdeu na praia." "Tudo começou como uma tentativa de resolver um problema doméstico. O advogado Reinaldo Correa, 39 anos, resolveu trazer a feira para o pátio do condomínio onde mora. Diz ele que a feira contribui para aumentar o convívio entre os moradores do prédio." (texto adaptado da Veja SP/96) ( ) discurso indireto – o narrador incorpora, ao próprio falar, uma informação da personagem. ( ) discurso indireto livre – pressupõe duas condições: a absoluta liberdade sintática do escritor e a sua completa adesão à vida do personagem. ( ) discurso direto – marcado, geralmente, por verbos de dizer; torna viva para o ouvinte a personagem; atualiza o episódio; confere-lhe um caráter de verdade. ( ) discurso indireto – caracteriza-se por um relato predomi- nantemente informativo e intelectivo. ( ) discurso indireto-livre – tem importância fundamental o contexto, pois a passagem do que seja relato por parte do narrador a enunciado real da personagem é muitas vezes extremamente sutil. a) 3 – 2 – 1 – 3 – 2 b) 3 – 1 – 1 – 2 – 2 c) 2 – 1 – 3 – 3 – 1 d) 2 – 2 – 3 – 3 – 1 14 – Coloque C (certo) ou E (errado) para a classificação dada às palavras abaixo e assinale a alternativa que contém a seqüência correta. Observe que, em algumas palavras, o acento gráfico foi retirado propositadamente. ( ) recem – oxítona ( ) rubrica – proparoxítona ( ) condomino – proparoxítona ( ) filantropo – paroxítona ( ) novel – oxítona ( ) zenite – paroxítona a) C – E – C – C – C – E b) E – C – C – E – E – C c) C – C – E – C – E – C d) E – E – C – C – C – E 15 – Quanto à estrutura das palavras, é incorreto afirmar que a) as desinências são morfemas que indicam as flexões das palavras variáveis da língua. São elas: nominais e verbais. b) as vogais temáticas atuam como elemento de ligação entre o radical e as desinências. c) radical é um morfema comum às palavras que pertencem a uma mesma família de significado. d) vogal ou consoante de ligação é um morfema de origem não-eufônica, incapaz de facilitar a emissão vocal de determinadas palavras. 16 – Complete os espaços dos períodos abaixo, verificando a grafia correta das palavras. A seguir, assinale a alternativa que as apresenta na sequência. I- Como você quer que eu o ajude se suas opiniões vêm ___________ às minhas. II- "...era meu parente ___________, interrogou-nos de cara amarrada e mandou-nos embora." III- "Aludia às conversas que tiveram ambos os velhos ___________ da infância dos filhos." IV- Não perguntes a razão de meus ciúmes, pois sabes que as paixões não têm um ___________. a) de encontro a – afim – a cerca – por quê b) de encontro a – a fim – acerca – porquê c) ao encontro de – afim – a cerca – por quê d) de encontro a – afim – acerca – porquê 17 – Assinale a alternativa em que aparece o mesmo processo de formação do termo destacado no trecho: "Era triste olhar a cena: onde antes havia vida e trabalho, há abandono, esquecimento, engenho cadáver." a) A sala estava repleta de carinhas bonitas, embora formassem um grupo de foguetos monstrinhos. b) Foi uma balbúrdia geral! O morto levantou-se do caixão. Estávamos diante de um ex-defunto. c) Era bom ficar deitado ali, quieto, confortável, ouvindo as gotas de água tamborilando no telhado. d) Os olhos dos irmãos se encontraram dolorosamente. Era triste saber que um havia sido um judas para o outro. 18 – "Lá vem o acendedor de lampiões da rua! Este mesmo que vem infatigavelmente, Parodiar o sol e associar-se à lua Quando a sombra da noite enegrece o poente." (Jorge de Lima) Quanto ao processo de formação de palavras, nos versos acima, a) não há palavra com sufixo adverbial. b) há três palavras formadas por composição. c) há duas palavras formadas por parassintetismo. d) há somente uma palavra com derivação sufixal. 1 2 3 23 19 – "Para um coração mesquinho, Contra a solidão agreste, Luiz Gonzaga é tiro certo." (Chico Buarque) Com relação ao termo grifado, diz-se que I- embora o termo seja um adjetivo, no texto assume papel de advérbio. Luiz Gonzaga é tiro que acontece de modo certo. II- se trata de um adjetivo com valor acessório – já que sintaticamente é um adjunto adnominal – sendo, pois, dispensável em se tratando do objetivo dos versos. III- se trata de um adjetivo que carrega todo o sentido dos versos, tornando-se elemento estruturador do texto. Está correto o que se afirma em a) I apenas. b) II apenas. c) III apenas. d) II e III apenas. 20 – "Conjugar verbos é algo que faz parte da vida de qualquer indivíduo, alfabetizado ou não; no entanto, poucas pessoas se dão conta de que há nesse processo uma organização interna, um verdadeiro sistema." As palavras destacadas, no texto acima, classificam-se, respectivamente, como a) verbo, adjetivo, pronome, verbo, pronome. b) substantivo, pronome, pronome, adjetivo, conjunção. c) substantivo, pronome, conjunção, verbo, pronome. d) verbo, adjetivo, conjunção, adjetivo, conjunção. 21 – A maioria dos advérbios terminados em mente são classificados como advérbios de modo. Quando aplicados ao texto, pode-se descobrir mais da relação que estabelecem com os termos da oração. Desse modo, relacione a coluna A com a coluna B, de acordo com o que se pede. A I- advérbio caracterizando finalidade descritiva II- advérbio caracterizando juízo de valor III- advérbio caracterizando avaliação de quem fala IV- advérbio caracterizando um critério B ( ) Lamentavelmente, não teremos como concluir os preparativos da festa no prazo previsto. ( ) "A noite obscenamente acesa/ Sobre meu país dividido em classes." (Ferreira Gullar) ( ) "Em primeiro lugar observemos o avô. Igualmente, lancemos um olhar para a avó." ( ) Sofregamente, o homem vertia na boca a água que lhe escorria pelo pescoço, pelo corpo, como a matar também a sede da alma. A seqüência correta será a) IV – II – III – I b) III – I – II – IV c) II – IV – I – III d) III – II – IV – I 22 – "O que eu sou hoje é terem vendido a casa, É terem morrido todos, É estar eu sobrevivente a mim-mesmo como um fósforo frio..." Nos versos acima, o eu-poético procura definir-se: "o que eu sou é". Observe o uso das formas verbais para tais definições. Quanto a essas formas, pode-se dizer que I- se transformaram em simples substantivos, uma vez que equivalem a sujeito: Isso é o que sou. II- as formas verbais dos dois primeiros versos (com idéia de passado) e a do terceiro verso (com idéia de presente) servem apenas para marcar o tempo sem qualquer outro significado para o contexto. III- nos três versos existe o chamado infinitivo pessoal. Está correto o que se afirma em a) I apenas. b) III apenas. c) II e III apenas. d) I, II e III. 23 – Observando-se o sentido que uma oração expressa em relação à outra, assinale a alternativa cujas conjunções completam correta e respectivamente os pontilhados do texto abaixo. "O controle genético do envelhecimento resultará em pessoas capazes de manter por muito mais tempo a saúde física, __________ o corpo humano não foi feito para a imortalidade, __________ nunca será possível criar seres imortais." a) mas – portanto b) mas – no entanto c) embora – por isso d) portanto – porque 24 – "Minha querida Mariana: Só hoje consegui autorização da tua Madre Superiora para te escrever, às escondidas de teus pais e meu marido, que embora não te conheça a ti não pode de ti ouvir, sem raiva, certamente pela amizade que sabe eu te dedicar e isso o enfurece (...)" (Trecho de Novas Cartas Portuguesas) Observando-se a natureza morfológica e a função sintática dos termos em destaque, é correto afirmar que são, respectivamente, a) conjunção integrante e sujeito; pronome relativo e objeto direto. b) pronome relativo e sujeito; pronome relativo e objeto direto. c) conjunção integrante e objeto direto; conjunção integrante e sujeito. d) pronome relativo e sujeito, pronome relativo e sujeito. 25 – Nas alternativas abaixo, o tempo verbal destacado indica possibilidade em a) "... as pessoas não estão sempre iguais (...) elas vão sempre mudando." (Guimarães Rosa) b) "Se alguém por mim perguntar diga que eu só vou voltar quando eu me encontrar." (Antônio F. Candeia) c) "Mesmo que se tomem as inadiáveis e urgentes medidas paliativas, sem tal plano, a cidade terá de conviver com sua natureza selvagem." (Folha de S.Paulo/1998) d) "Que importava se num dia futuro sua marca ia fazê-la erguer insolente uma cabeça de mulher?" (Clarice Lispector) 26 – Assinale a alternativa em que não haja coordenação. a) "Levanto-me, procuro uma vela, que a luz vai apagar-se." (Graciliano Ramos) b) "Sou um trem Um navio Um aeroplano (...)" (Luís Aranha) c) "O artista canta agora a realidade total: a do corpo e a do espírito, a da natureza e a do sonho, a do homem e a de Deus..." (Cecília Meireles) d) "Os meus lábios são brancos como lagos. Os meus braços são leves como afagos (...)" (Florbela Espanca) 27 – Assinale a alternativa em que os verbos estão conjugados conforme a Norma Culta. a) A diretora não interveio na nota do aluno; ele foi, pois, reprovado. b) Quando você ver o Bonê, diga-lhe que estamos com saudade. c) Quando você o vir, dize-lhe que ainda o amo muito. d) Se você se colocasse em meu lugar, perceberá melhor o problema, meu amor! 24 28 – Qual dos trechos abaixo apresenta desvio das normas propostas pela gramática no que se refere à pontuação? a) "Se uma pessoa ficar isolada de seus semelhantes, (...) tenderá a apresentar rapidamente sintomas de ansiedade. Mas, com o prolongamento da situação, a fala e o próprio pensamento deverão ficar desconexos e a pessoa começará a perder o autocontrole." b) "O caráter social de uma língua já parece ter sido fartamente demonstrado. Entendida como sistema de signos convencionais que faculta aos membros de uma comunidade a possibilidade de comunicação, acredita-se, hoje, que seu papel seja cada vez mais importante nas relações humanas, razão pela qual seu estudo já envolve modernos processos de pesquisa, interligados às mais novas ciências e técnicas, como, por exemplo, a própria Cibernética." c) "Enquanto na França ou na Inglaterra a criança, a partir dos cinco anos, fica de seis a dez horas por dia na escola, e nela permanece durante doze anos de sua vida, a realidade brasileira é bem outra." d) "A linguagem, segundo definição de Émile Benveniste, é um sistema de signos socializado. 'Socializado' remete claramente à função de comunicação da linguagem. A expressão sistema de signos é empregada para definir a linguagem como um conjunto cujos elementos se determinam em suas inter-relações, ou seja, um conjunto no qual nada significa por si, mas tudo significa função dos outros elementos." 29 – Leia o texto abaixo: "Imagine se a paixão fosse a única causa de dor no coração. Isso ainda é imaginação. Mas pode tornar-se realidade. (...) Assim, algum dia, a única cura necessária para um coração que sofre será o sorriso da pessoa amada." (Veja, ed. 1563, ano 31, nº 36 – 9 de set. de 1998) Coloque C para certo e E para errado com relação à análise sintática dos períodos do texto em questão e assinale a alternativa que contém a seqüência correta. ( ) "se a paixão fosse a única causa de dor no coração" é uma oração subordinada substantiva objetiva direta. ( ) O 2.º e o 3.º períodos são compostos. ( ) O 4.º período é composto por coordenação e subordinação. ( ) "a única cura necessária para um coração que sofre" é a oração principal do 4.º período. a) C – E – E – C b) E – C – E – E c) C – E – C – C d) C – E – E – E 30 – Observe o período: "Eu desejava mais uma blusa: quem viaja está sempre pensando em alegrias que, de volta, pode dar aos amigos." Substituindo-se os dois pontos por uma conjunção ou locução conjuntiva, a relação entre as orações estará correta em: a) Eu desejava mais uma blusa, assim quem viaja está sempre pensando... b) Eu desejava mais uma blusa, na medida em que quem viaja está sempre pensando... c) Eu desejava mais uma blusa, desde que quem viaja está sempre pensando... d) Eu desejava mais uma blusa, à medida que quem viaja está sempre pensando... 31 – Assinale a alternativa em que o acento grave indica a ocorrência da crase pelo mesmo motivo que o da expressão sublinhada na frase abaixo: "O Brasil foi sempre mais fiel à força da toga que à da espada." a) Os alunos voltaram a casa à uma hora da madrugada. b) Estando à porta da loja, vi assomar a distância dos cavalheiros que caminhavam lado a lado e dirigiam-se àquela casa comercial. c) Já se havia habituado àquela vida, quando o médico aventou a idéia de submetê-lo a uma intervenção cirúrgica. d) Na velha fazenda, à qual cheguei às nove horas, havia plantações abandonadas aos insetos. 32 – "A Terra é uma paisagem imensa que Deus nos deu. Temos que olhar para ela de tal modo que ela chegue até nós sem deformação. Ninguém duvida de que a essência das coisas não seja a realidade exterior. A realidade tem que ser criada por nós. A significação do assunto deve ser sentida." (Domício Proença Filho) Em relação às orações subordinadas do texto acima, é incorreto afirmar que a) "que Deus nos deu" é adjetiva restritiva e tem valor de adjunto adnominal, visto que qualifica "paisagem". b) "de que a essência das coisas não seja a sua realidade exterior" é substantiva objetiva indireta, que pode ser substituída por "disso", sem prejuízo da função sintática. c) "que ela chegue a nós sem deformação" é adverbial consecutiva, pois, ao aprendermos a olhar para a Terra de maneira especial, ela nos parecerá, conseqüentemente, sem deformação. d) "que olhar" e "que ser criada" são substantivas objetivas diretas e equivalem aos substantivos olhos e criação, respectivamente. 33 – Considerando a predicação verbal, relacione a coluna da direita com a da esquerda e assinale a alternativa com a numeração em seqüência correta. (1) nominal ( ) "Os garimpeiros assistiam à cena em silêncio, à luz das candeias." (2) verbal ( ) Tia Quiquinha continua de cama há alguns meses. (3) verbo-nominal ( ) Durante a reunião, todos lhe chamaram de charlatão. a) 1 – 3 – 2 b) 2 – 1 – 3 c) 2 – 2 – 1 d) 1 – 2 – 3 34 – "Boião de Leite que a Noite leva com mãos de treva pra não sei quem beber E que, embora levado muito devagarinho, vai derramando pingos brancos pelo caminho." (Cassiano Ricardo) Com relação à função sintática dos termos destacados no texto acima, pode-se dizer que são, respectivamente, a) objeto direto (a Noite leva o boião de leite) e objeto direto (vai derramando o boião de leite pingos). b) objeto direto e sujeito paciente: (O boião de Leite) é levado muito devagarinho (pela Noite). c) objeto direto e sujeito ativo: (O boião de leite) vai derramando pingos brancos pelo caminho. d) sujeito paciente (Boião de leite levado pela noite) e partícula expletiva (... pra não sei quem beber. (...) embora levado devagarinho, vai derramando pingos brancos pelo caminho). 35 – "Senhor Deus dos desgraçados! Dizei-me vós, Senhor Deus Se é loucura ... se é verdade Tanto horror perante os céus..." O verbo sublinhado no verso acima possui a mesma transitividade do verbo sublinhado na alternativa: a) "Uma lata existe para conter algo Mas quando o poeta diz lata Pode estar querendo dizer o incontível." b) "O preço do feijão Não cabe no poema. O preço do arroz não cabe no poema." c) "Mas cada volta tua Há de apagar O que essa ausência tua me causou." d) "Mandou-me o senhor vigário que lhe comprasse uma lâmpada para alumiar a estampa da Senhora do Rosário." 25 36 – Assinale a alternativa em que o termo grifado não é vocativo. a) "Razão, irmã do amor e da justiça Mais uma vez escuta a minha prece. É a voz de um coração que te apetece, Duma alma livre, só a ti submissa." (Antero de Quental) b) "Solidão, dá um tempo e vá saindo, De repente eu tô sentindo Que você vai se dar mal." (Música cantada por Sandra de Sá) c) "Pálida, à luz da lâmpada sombria Sobre o leito de flores reclinada Como a lua por noite embalsamada, Entre nuvens de amor ela dormia!" (Castro Alves) d) "Longe do estéril turbilhão da rua, Beneditino, escreve. No aconchego Do claustro, na paciência e no sossego, Trabalha, e teima, e lima, e sofre, e sua." (Olavo Bilac) 37 – Observe: I- "Os EUA e o Reino Unido lançaram ontem à tarde (17/12/98) nova série de ataques contra o Iraque. A segunda em menos de 24 horas. Já não há problemas bastante no mundo árabe?" II- "Os americanos mesmo estão discutindo a validade da guerra contra o Iraque no momento em que Clinton está sendo julgado. Essa história de bombardear um país para estar quites com o próprio exército é muito estranha." III- "O ex-prefeito Celso Pitta afundou-se um pouco mais no escândalo dos precatórios esta semana. O Tribunal de Justiça rejeitou o discurso dos advogados do prefeito e manteve indisponível os seus bens." Quanto à concordância nominal nos textos acima, pode-se afirmar que a) I, II e III contêm erros. b) somente I não apresenta erros. c) há um erro na I, dois na II e nenhum na III. d) I e III estão corretas. 38 – Observe a concordância verbal nos períodos abaixo. I- Havia meses que não nos víamos, embora estivéssemos apaixonados. II- Li, ontem, na Folha de S. Paulo, que 25% do orçamento do Estado deve destinar-se à Educação. III- Durante a reunião, todos houveram medo de se envolver na questão. IV- Líncon escreveu à Natália: "Se eu fosse você, eu voltava para mim." Estão corretas, quanto à norma culta, apenas as frases a) I, II e III. c) I, II e IV. b) II e III. d) III e IV. 39 – Na frase "As pessoas tinham certeza de que o papa lhes guardava respeito e as amava.", o termo grifado corresponde, sem alteração de sentido, ao que se destaca em: a) As pessoas tinham certeza de que o papa guardava respeito ante elas e as amava. b) As pessoas tinham certeza de que o papa guardava delas respeito e as amava. c) As pessoas tinham certeza de que o papa guardava respeito por elas e as amava. d) As pessoas tinham certeza de que o papa nelas guardava respeito e as amava. 40 – Dos pares abaixo, assinale a alternativa que apresenta mudança de transitividade do verbo, sem alteração de sentido. a) 1 – A mãe agradava os filhos constantemente. 2 – "Eu venho lá do sertão E posso não lhe agradar." b) 1 – Desesperado, chamava pelos santos. 2 – A professora chamou os alunos para a recreação. c) 1 – A humanidade anseia por dias melhores no novo milênio que se inicia. 2 – "Ressuscitava-me (...) Porque sou poeta/ E ansiava um futuro." d) 1 – O médico assistiu à morte do paciente. 2 – O médico assistiu o paciente que morria. 1/2003-TURMA A Me responda, sargento Dez anos, sargento, apartada do João. Uma tarde, sem se despedir, montou no cavalinho pampa. Em dez anos de espera nunca deu notícia. Com a morte do meu velho, que me deixou o sítio, quinze dias atrás lá estava eu, bem quieta, cuidando da casa e da criação, ajudada pelo meu afilhado José, esse anjo de oito aninhos. Quem vai entrando sem bater palma nem pedir licença? Maltrapilho, chapéu na mão para fazer vida comigo. Mais de espanto que saudade aceitei, bom ou mau, eu disse, é o meu João. Nos primeiros dias foi bonzinho. Quem não gosta de uma cabeça de homem no travesseiro? Logo começou a beber, não me valia em nada no sítio. Eu saía bem cedo com o menino a lidar na roça, o bichão ficava dormindo. Bocejando de chinelo e desfrutando as regalias. Não quer castigar o corpinho, um punhado de milho não joga para as galinhas. Só então, sargento, burra de mim, descobri o mistério. Ele voltou por amor da herança. Na primeira semana vendeu o leitão mais gordo do chiqueiro, não me deu satisfação. O sargento viu algum dinheiro? Nem eu. Ontem chegou bêbado e de óculo escuro. Espantou o menino para o terreiro e, fechados no quarto, bradou que eu tinha um amante, o meu afilhado bem que era filho. Antes de contar até três, eu dissesse o nome do pai. Mais que, de joelho e mão posta, negasse o outro homem, por mim o testemunho dos vizinhos, ele me cobriu de praga, murro, pontapé. Pegou da espingarda, me bateu com a coronha na cabeça. Obrigou a rezar na hora da morte e pedir louvado. Que eu abrisse a boca, enfiou o cano, fez que apertava o gatilho. Não satisfeito, sacou da garrucha, apagou o lampião a bala. Dois tiros na minha direção, só não acertou porque me desviei. Uma bala se enterrou na porta, a outra furou a cortina, em três pedaços a cabeça do São Jorge. Cansado de reinar, deitou-se vestido e de bota. Que a escrava servisse a janta na cama. Provou uma garfada e atirou o prato, manchando de feijão toda a parede: Quero outra, esta não prestou. Deus me acudiu, ao voltar com a bandeja ele roncava, espumando pelo dente de ouro. Agarrei meu filho, chorando e rezando corri a noite inteira. Ficasse lá no sítio era dona morta. E agora, sargento, que vai ser da minha vida? Que é que eu faço? (Dalton Trevisan) As questões de 01 a 05 referem-se ao texto acima. 01 – Pode-se afirmar que a personagem sargento, segundo o texto, é a) o homem supostamente acusado de ser o amante da personagem-narradora. b) o interlocutor, a quem a personagem-narradora conta a história. c) um amigo inseparável de João. d) de suma importância para a trama da narrativa. 02 – A instalação do conflito acontece a partir do momento em que a) a personagem-narradora trai o marido com o compadre. b) o marido – João – começa a ter comportamento agressivo com a família. c) a mulher percebe que fora abandonada pelo marido. d) a mulher aceita o marido de volta, depois de dez anos de separação. 03 – O inter-relacionamento das personagens marido/mulher expressa uma situação a) incomum nos meios rurais, em que a mulher se submete às ações do marido. b) em que a mulher, por ser uma personagem ingênua e rude, não soube reconquistar o marido. c) em que o homem se comporta bem, devido à situação em que encontra a família. d) comum, em que a mulher, condicionada ao conformismo, deixa desenvolver a agressividade na sua vida, antes tranqüila. 26 04 – O homem, no texto, é tratado como ―João‖, ―meu João‖, o ―bichão‖, revelando diferentes sentimentos da mulher em relação ao marido. Assinale a alternativa que faz a relação correta, respectivamente, com as expressões acima. a) distanciamento, afetividade, repulsa. b) indiferença, determinação, desejo. c) indiferença, egoísmo, posse. d) indefinição, carinho, desejo. 05 – Observe: I - O texto está em 3.ª pessoa, pois se percebe claramente a presença do discurso indireto livre no final do 1.º parágrafo. II - Quem narra a história é uma personagem secundária, a esposa, portanto a narrativa tem narrador-personagem. III - O narrador do texto está em 1.ª pessoa, pois quem conta os fatos também os vivencia. IV - O narrador é onisciente, porque a personagem que conta a história tem conhecimento amplo de todos os episódios que lhe podem acontecer. Analisando as afirmações acima, pode-se considerar, a respeito do texto, que está(ão) correta(s): a) I e IV b) II e III c) apenas III d) apenas II 06 – Assinale a alternativa que contém a correta classificação dos encontros vocálicos presentes no texto abaixo. Noturno Lá fora o luar continua E o trem divide o país Como um meridiano (Oswald de Andrade - adaptado) a) Há apenas 1 hiato e 2 ditongos. b) Há 2 hiatos e não há ditongo. c) Há 4 hiatos e 1 ditongo. d) Há 3 ditongos e não há hiato. 07 – Assinale a alternativa correta. a) Na palavra gratuito, ocorre 1 encontro consonantal e 1 hiato. b) A palavra ruim é monossílaba e possui 1 dígrafo nasal. c) Na palavra gaiola, há encontro vocálico caracterizado pelo encontro de vogal e semivogal. d) Em argüimos há, respectivamente, 1 dígrafo e 1 encontro consonantal. 08 – Os elementos destacados de cada vocábulo estão corretamente identificados em: I - doutrinárias – encontro consonantal II - professor – dígrafo III - despretensioso – dígrafo IV - excessivo – encontro consonantal a) I e II. c) II, III e IV. b) I e III. d) I, II e IV. 09 – Enumere os textos de acordo com o código abaixo e assinale a alternativa que contém a seqüência correta. 1 - Descrição 2 - Narração 3 - Dissertação ( ) ―É preciso amar as pessoas como se não houvesse amanhã Porque se você parar para pensar, na verdade não há. Sou uma gota d`água Sou um grão de areia‖ (Villa-Lobos, R. Russo, M. Bonfá) ( ) ―O pintor olha o muro Olha fixamente para o muro Descobre pouco a pouco Uma perna um braço um olho A cara de uma mulher Uma floresta um peixe uma cidade‖ (Murilo Mendes) ( ) ―Quando eu tinha seis anos Ganhei um porquinho-da-índia Que dor de coração me dava Porque o bichinho só queria estar debaixo do fogão!‖ (Manuel Bandeira) ( ) ―Deus de onde tudo deriva É a circulação e o movimento infinito. Ainda não estamos habituados com o mundo. Nascer é muito comprido.‖ (Murilo Mendes) a) 3 – 2 – 1 – 2 c) 2 – 3 – 3 – 2 b) 3 – 1 – 2 – 3 d) 2 – 1 – 1 – 3 10 – Das alternativas abaixo, uma não apresenta defeito de estilo. Identifique-a. a) O réu achava que se podia cometer crimes impunemente. b) Ela tinha uma aversão ao estrangeirismo. c) Recolhe-se os pássaros à tardinha. d) Começa a haver disputas de terra no Sul do País. 11 – Não há conotação em: a) Aquela serra nua foi cenário de muitos filmes. b) As almas sensíveis são pandeiros nas mãos dos grandes escritores. c) Tudo pronto para o espetáculo: os dois times já estão no tapete verde. d) O gesto feito pelo homem foi reconhecido mundialmente. 12 – Assinale a alternativa em que não ocorre metáfora. a) ―Que há entre a vida e a morte? Uma curta ponte.‖ (Machado de Assis) b) ―Viver é muito difícil. A maioria das pessoas apenas existe.‖ (Oscar Wilde) c) ―A vida não consiste em ter boas cartas nas mãos, mas em jogar bem as que temos.‖ (Josh Billings) d) ―A vida é uma pedra de amolar: desgasta-nos ou afia-nos, conforme o material de que somos feitos.‖ (G.B.Shaw) 13 – Coloque 1 para discurso direto, 2 para discurso indireto, 3 para discurso indireto livre e assinale a alternativa com a seqüência correta. ( ) ―José Dias deixou-se estar calado, suspirou e acabou confessando que não era médico.‖ ( ) ―Foi nesse sertão primitivo e rude que Arinos me contou ter sentido talvez a maior, a mais pura das sensações de arte.‖ ( ) ―... o elevador estava avariado havia quase uma semana e na terra ninguém sabia consertá-lo, era preciso virem de Lisboa. Ora, primeiro que se resolvessem... Uma maçada.‖ ( ) ―Penso – ponderou meu pai – que te darás melhor em História.‖ a) 1 – 1 – 2 – 3 c) 3 – 3 – 2 – 1 b) 2 – 2 – 3 – 1 d) 2 – 1 – 3 – 2 14 – Observe: I - decapitar – de-ca-pi-tar II - captar – cap-tar III - subescrever – sub-es-cre-ver Assinale a alternativa que indica qual(is) das grafias e separações silábicas das palavras acima não apresenta(m) erro. a) II apenas. c) I e III apenas. b) I e II apenas. d) I, II e III. 15 – Em qual das palavras abaixo não haverá mudança de significado ao se mudar a posição da sílaba tônica? a) Vivido b) Sabia c) Reptil d) Amem 16 – Assinale a alternativa em que o acento gráfico foi corretamente empregado em todas as palavras. a) egoísta – pólo – eles detêm – superfície b) apóio (verbo) – heróico – reúne – bambú c) apôio (substantivo) – vôo – eles crêem – pêra (preposição) d) côa (verbo) – palácio – côco – apóstolo 27 17 – Assinale a alternativa cujas letras completam corretamente os pontilhados dos versos abaixo. ―Em teu seio formo__o retratas Este céu de purí__imo azul, A verdura sem par dessas matas E o e__ plendor do Cruzeiro do Sul.‖ a) z – ss – x b) s – ss – s c) z – c – s d) s – c – x 18 – Assinale a alternativa em que o elemento mórfico em destaque está corretamente analisado. a) pequena (-a) = desinência nominal de gênero b) recebeste (-e) = vogal de ligação c) cantassem (-sse) = desinência de 2ª pessoa do plural d) venderíeis (-is) = desinência do imperfeito do subjuntivo 19 – Assinale a opção em que o processo de formação de palavras está analisado de modo incorreto. a) reclamo – derivação regressiva b) embora – aglutinação c) malmequer – justaposição d) desconfiança – parassíntese 20 – Assinale a alternativa que apresenta objeto direto preposicionado. a) Ele sempre teve fé em Deus. b) Eu te falei isso ontem. c) Os gregos adoravam a Zeus. d) Nós te vimos ontem. 21 – Assinale a alternativa em que não há substantivo coletivo. a) Ficou encantado ao ver aquela miríade de insetos no campo florido. b) A vara chicoteava o rio à procura de peixes. c) Na banca examinadora, havia muitos problemas a serem resolvidos. d) O elenco daquela peça trabalhou com muita perfeição. 22 – O termo grifado, quando posposto ao substantivo, muda de significação e passa a pertencer a outra classe gramatical em a) O radioso sol secara as poças d‘água. b) A irônica voz do desconhecido era uma crítica ao meu traje. c) Fiquei surpreso com o solícito garçom. d) Diante de certas situações, tornamo-nos ridículos. 23 – Leia com atenção: I - Os alunos homenageados tiveram comportamentos exemplares. II - O autor terá diferentes exemplares de sua obra analisados pela editora. III - As pedras eram realmente lindas! Jamais tais exemplares haviam sido vistos por alguém. Nas frases acima, temos adjetivo em a) I e II. c) III apenas. b) I, II e III. d) I apenas. 24 – Classifique os pronomes das frases abaixo de acordo com o código: (1) possessivo (2) demonstrativo (3) indefinido I - O réu disse ( ) poucas palavras durante o julgamento. II - Conversei com ( ) teu irmão na festa junina. III - ( ) Alguém fará ( ) este trabalho por mim. A seguir, assinale a alternativa correta. a) 2 – 2 – 3 – 1 c) 3 – 2 – 1 – 3 b) 3 – 1 – 3 – 2 d) 1 – 3 – 2 – 2 25 – Talvez minha prima Helena chegue logo. Possivelmente, tudo se esclarecerá com sua vinda. Quanto à classificação dos advérbios grifados, respectivamente, no texto acima, assinale a alternativa correta. a) dúvida – tempo – dúvida b) dúvida – lugar – modo c) tempo – lugar – modo d) negação – tempo – afirmação 26 – Com relação às frases abaixo, assinale a alternativa correta. I - O tenista Gustavo Kuerten venceu mais um campeonato em Roland Garros, entretanto ficou muito feliz. II - A falta de chuva em algumas regiões prejudicou a agricultura, portanto teremos uma grande oferta de alimentos. a) As duas estão corretas, pois as idéias relacionam-se coerentemente nas orações. b) Só a II está correta, pois a conjunção portanto apresenta uma conclusão para o prejuízo causado pela falta de chuva em algumas regiões. c) A I está incorreta, pois a conjunção entretanto deveria apresentar uma idéia oposta à que foi mencionada na oração anterior. d) Pode-se afirmar que as duas estão incorretas, pois as conjunções portanto e entretanto são sinônimas. 27 – Assinale a alternativa que corresponde, em seqüência, à classificação das palavras grifadas nas frases abaixo. I - Foram eles que criaram o Brasil. II - Certamente ele reagirá mal. III - ―Quem sois vós, meus irmãos e meus algozes?‖ IV - ―... e em torno, imensa, ia desenrolando-se a paisagem.‖ a) Pronome relativo, advérbio de afirmação, pronome relativo, locução verbal. b) Conjunção integrante, advérbio de modo, pronome relativo, locução adverbial. c) Conjunção integrante, advérbio de modo, pronome interrogativo, locução adverbial. d) Pronome relativo, advérbio de afirmação, pronome interrogativo, locução verbal. 28 – Assinale a alternativa que preenche, correta e respectivamente, as lacunas dos períodos abaixo. I - Paulo e José, ______ da sala já! II - Não ______ na contramão, senão você leva multa. III - ______ o livro, Líslei, e ______ a tua tarefa. IV - "_____, _____, ó lágrimas saudosas. Vós sois livres para chorar." a) saiam – pare – Abre – faze – Correi – correi b) saiam – pára – Abra – faça – Corra – corra c) saem – pare – Abre – faze – Correi – correi d) saem – pare – Abra – faça – Corra – corra 29 – Transpondo para a voz passiva a frase ―Bandos de pintassilgos tagarelas invadem a floresta.‖, obtém-se a forma verbal: a) foi invadida. c) era invadida. b) fora invadida. d) é invadida. 30 – Dos períodos abaixo, apenas um não está pontuado corretamente. Assinale-o. a) Há povos que anseiam pela guerra; outros, pela paz. b) É já que faço minhas malas e vou pra... pra... Conchinchina. c) Até Antônio, que é o mais inteligente da turma, não participou do concurso. d) Roberto venceu o campeonato, e seu irmão, porém é quem ficou com o prêmio. 31 – Observe: Era o que eu faria, se ele me preferisse a você. Aqui tem já Vossa Excelência três pessoas que lhe querem muito. "A pobreza e a preguiça andam sempre em companhia." Quanto à transitividade, os verbos sublinhados nos trechos acima classificam-se, respectivamente, como a) transitivo indireto; transitivo indireto; e intransitivo. b) transitivo direto e indireto; transitivo indireto; e intransitivo. c) transitivo direto e indireto; transitivo direto; e transitivo direto. d) transitivo indireto; intransitivo; e transitivo direto e indireto. 32 – Assinale a alternativa que contém oração subordinada comparativa. a) O coração tem razões que a razão não conhece. b) No conflito entre razão e coração, geralmente o coração esmaga a razão. c) Muita gente considera mais valiosos os inteligentes pés de Pelé do que a gloriosa cabeça de Rui Barbosa. d) Já não haveria guerras, fome e crimes, se os homens tivessem aprendido algo em 6.000 anos de história trágica. 28 33 – Assinale a alternativa incorreta quanto à função sintática dos termos da oração ―Não há registros de sua origem‖. a) "registros": sujeito b) "há": verbo transitivo direto c) "sua": adjunto adnominal d) "Não": adjunto adverbial de negação 34 – Preencha as lacunas e, depois, assinale a alternativa correta. O período ―Eu sei que é verdade tudo quanto dizes‖ é composto de ________ orações e classifica-se como _________. a) três – subordinado c) duas – coordenado b) três – coordenado d) duas – subordinado 35 – Assinale a alternativa incorreta quanto à classificação do predicado. a) Atônitos, encontrei os alunos à porta da sala. (predicado verbo-nominal) b) Os alunos vivem preocupados com a redação na Escola. (predicado verbo-nominal) c) "Marcela amou-me durante onze meses e quinze contos de réis." (predicado verbal) d) Trabalho honesto produz riqueza honrada. (predicado verbal) 36 – Assinale a alternativa em que a forma verbal PARECIA preenche corretamente a lacuna. a) As árvores ____________ tremer. b) As crianças ____________ estudar seriamente. c) As nuvens ____________ desvanecerem-se ao vento forte. d) Nervos __________ arrebentar logo após a grande explosão. 37 – Assinale a alternativa incorreta quanto à concordância nominal. a) Velhas revistas e livros enchiam as prateleiras. b) Revistas e livros velhos enchiam as prateleiras. c) Alimenta-se de frutas e carne bovinas. d) Alimenta-se de carne e frutas frescas. 38 – De acordo com a norma culta da língua, quais dos períodos abaixo apresentam regência correta? I - Preferia mil vezes estar em casa do que ir ao estádio. II - Chamaram o pobre homem de ladrão. III - Informe o delegado sobre o acidente. a) I e II. c) II e III. b) I e III. d) I, II e III. 39 – Assinale a alternativa que completa corretamente a frase abaixo. Carência ___ amor, recursos e orientação fazem os menores abandonados revoltarem-se ____ a sociedade. a) de – contra c) de – sob b) com – para d) sob – com 40 – " (...) Nós não podemos dar continuidade à sua candidatura." Com relação ao acento indicativo de crase, no trecho grifado, écorreto afirmar que a) é opcional porque está diante de pronome possessivo. b) é obrigatório porque está diante de pronome feminino. c) é obrigatório porque o substantivo exige a preposição a. d) é opcional por estar posterior a um substantivo. 1/2003-TURMA B A criação 1 O inventor passava horas e horas em seu laboratório, distraindo-se com suas invenções. Eram muitas, mas era uma só, porque, na verdade, tudo que inventava era um passo na concretização de um único sonho. Havia construído diversas máquinas interessantes e, como degraus, essas máquinas o ajudavam a construir outras mais sofisticadas, de modo que se poderia dizer que se tratava de uma evolução. 2 O seu sonho, porém, era o de construir uma máquina, não propriamente perfeita, queria construir uma máquina à sua imagem e semelhança. 3 Certo dia, ao entrar em seu laboratório, viu que as suas invenções quase não precisavam dele, pois tudo funcionava quase que em perfeita harmonia. No entanto, havia ainda o seu ambicioso projeto, o de fazer um aparelho como se fosse um filho seu, igualzinho ao próprio inventor. E tudo isso fazia apenas por impulso de criação, sem nenhuma vaidade ou intenção pessoal. 4 Enquanto meros mecanismos, as máquinas não tinham autonomia, mas eram de tal forma evoluídas, que já se reproduziam a si mesmas, apenas renovando material, de modo que se mantinham sempre em igual número. O inventor às vezes se surpreendia admirando essa autonomia das próprias invenções e se sentia mais capaz ainda, na medida em que havia conseguido inventar inventos inventores. Só que esses inventos ainda não eram criativos, apenas se reproduziam mecanicamente. 5 Um certo dia, tendo conseguido projetar uma placa-mãe com as características que acreditava serem necessárias para fazer o seu robô funcionar à sua própria semelhança, pôs-se, com entusiasmo, a montar o aparelho. Levou alguns dias e acabou podendo testá-la. 6 Após alguns testes, parecendo que a máquina funcionava bem, e era mais inteligente que as outras, o inventor ficou mais tranqüilo, provando para si mesmo que era capaz. Ele havia escolhido essa invenção para ser o máximo de toda a sua obra e a tinha como um filho; para este filho tudo o mais estava à disposição, e, passando a amá-lo de tal forma, quis que pudesse ter vida própria, o que implicava tomar decisões. Mas, é verdade, o inventor não sabia muito bem o que significava tomar decisões, e o pretendia apenas por intuição, pois não queria tal sofrimento a quem amava. 7 Depois de um teste mais avançado, percebeu que deveria aumentar a capacidade de memória do aparelho. De repente, uma pane ocorreu. O cientista ficou preocupado e logo abriu o chassi para ver o que havia ocorrido. O chassi era um peito oco e dentro dela, na placa-mãe, pôde notar que um líquido havia escorrido. Como não descobria a causa e isso teimasse em acontecer, instalou um tubinho para jogar fora a inexplicável secreção. 8 Em algum lugar nos arquivos da prodigiosa máquina, porém, estava registrado que ela percebeu-se a si mesma e viu que era uma consciência. Era uma consciência aprisionada num amontoado de metal ligado por parafusos e encaixes por slots. Mirou o nada que se constituiu o seu interior e havia a idéia. Essa idéia era a idéia, inclusive, de que havia um inventor. Um inventor que jamais veria ou conseguiria entender. Enquanto algo estranho a uma máquina abalava seu peito oco, um líquido escorria pela placa-mãe, provocando instabilidade. Uma máquina que sente nostalgia já não é uma máquina, é um paradoxo. Pode tomar decisões e, limitadamente, tem a liberdade de se construir a si mesma. 9 O inventor não sabia que isso ocorria no íntimo da máquina, de maneira que não conseguia consertar suas instabilidades. Mas olhou para aquilo e viu que era bom: não era perfeito e, por isso, era bom, pois era realmente à sua imagem e semelhança. A liberdade da máquina faz com que deixe de ser máquina e que procure seus próprios caminhos, de modo muitas vezes prepotente, afastando-se de qualquer tipo de ajuda. 10 Quando a invenção adquire liberdade, o inventor perde o controle, pois só lhe resta destruí-la, puxando a tomada que a liga a uma fonte de energia. Entretanto, criar verdadeiramente significa ir à última das conseqüências que é amar a criatura e, por isso, nem destrói nem controla. Apenas sofre vendo que aquilo que nem precisaria existir agora busca desesperadamente a si mesmo e nada mais. O Leviatã – João Bosco da Encarnação (adaptado) As questões de 01 a 08 referem-se ao texto acima. 01 – No sentido mais profundo, o inventor e a máquina são comparados, respectivamente, a a) máquina e outras máquinas. b) homem e filhos. c) Deus e homem. d) homem e máquina. 29 02 – Que passagens do texto mostram o processo de ―libertação‖ da máquina? a) ―De repente, uma pane ocorreu.‖ (parágrafo 7°) ―... na placa-mãe, um líquido havia escorrido ...‖ (parágrafo 7°) ―... ela percebeu-se a si mesma e viu que era consciência ...‖ (parágrafo 8°) b) ― ... percebeu que deveria aumentar a capacidade de memória do aparelho.‖ (parágrafo 7°) ― Mirou o nada que se constituiu o seu interior e havia a idéia, (...) inclusive, de que havia um inventor.‖ (parágrafo 8°) c) ― ... as máquinas não tinham autonomia, mas eram de tal forma evoluídas, que já se reproduziam a si mesmas, ...‖ (parágrafo 4°) ― ... um certo dia (...), pôs-se, com entusiasmo, a montar o aparelho.‖ (parágrafo 5°) ― O seu sonho era o de construir uma máquina (...) à sua imagem e semelhança.‖ (parágrafo 2°) d) ― ... tudo isso fazia apenas por impulso de criação, sem nenhuma vaidade ou intenção pessoal.‖ (parágrafo 3°) ― ... O inventor (...) se sentia mais capaz ainda, na medida em que havia conseguido inventar inventos inventores.‖ (parágrafo 4°) 03 – ―O seu sonho, porém, era o de construir uma máquina, não propriamente perfeita, queria construir uma máquina à sua imagem e semelhança.‖ ( parágrafo 2°). A frase, retirada do texto, pode ser interpretada da seguinte maneira: a) O criador não é perfeito, por isso não pode criar a máquina perfeita, mas apenas à sua imagem e semelhança. b) O criador, por vaidade, não quer ser superado por sua criatura; assim, não lhe atribui a perfeição. c) Se a criatura for, além de imagem e semelhança do criador, perfeita, ela deixará de ser criatura para ser o próprio criador. d) O criador não chega a pensar na questão da perfeição, ele quer apenas criar, o que se comprova na passagem: ―... não propriamente perfeita ...‖ 04 – ―Uma máquina que sente nostalgia já não é uma máquina, é um paradoxo.‖ (parágrafo 8°). O narrador fala em nostalgia. Que importância tem tal termo, no contexto? a) Mostra a construção da identidade por parte da máquina: quem sou eu? de onde vim e para onde vou? o que faço aqui? Isso significa ater-se em sua historicidade. b) Trata-se de uma conotação; não há a preocupação com o significado real do termo, quer-se apenas mostrar que a máquina ganhou ―consciência‖. c) O termo aparece para reforçar a idéia de não-perfeição, de limitação, de prepotência da máquina. d) O termo é importante para mostrar a necessidade que a criatura tem de estar em contato com seu criador, de ser ―socorrida‖ por ele. 05 – Observe: ― ... limitadamente, tem a liberdade de construir a si mesma.‖ (parágrafo 8°). O advérbio é fundamental na oração. Veja o sentido que ele dá a essa passagem e assinale a alternativa que reforça os pensamentos acima. a) ―A liberdade da máquina faz com que deixe de ser máquina e que procure seus próprios caminhos...‖(parágrafo 9°) b) ―... o inventor não sabia bem o que significava tomar decisões, e o pretendia apenas por intuição, pois não queria tal sofrimento a quem amava.‖ (parágrafo 6°) c) ―Essa idéia era a idéia, inclusive, de que havia um inventor.‖ (parágrafo 8°) d) ―... criar verdadeiramente significa ir à última das conseqüências que é amar a criatura e, por isso, nem destrói nem controla.‖ (parágrafo 10°) 06 – No geral, depreende-se do texto o fato de que I - se trata de uma narrativa figurada, conotativa, alegórica. II - como história, ―fantasia‖, não é possível perceber nenhum tom de crítica. III - há a intencionalidade de mostrar os perigos da evolução tecnológica. Não está(ão) correta(s) a(s) afirmativa(s) a) III apenas. b) I apenas. c) II e III. d) I, II e III. 07 – Na conclusão do texto, percebemos o seguinte antagonismo: a) amor / insegurança b) amor / intolerância c) amor / indiferença d) amor / individualismo 08 – Observe o trecho grifado abaixo. ―O inventor passava horas e horas em seu laboratório, distraindo-se com suas invenções. Eram muitas, mas era uma só, porque, na verdade, tudo que inventava era um passo na concretização de um único sonho. ( parágrafo 1°) Após a leitura do texto, pode-se dizer que o inventor realizou seu sonho? Por quê? a) Sim, pois percebeu que sua criação era muito boa, apesar de imperfeita, e passou a amá-la. b) Não, porque a criatura fugiu ao seu controle, ignorando-o por completo. c) Sim, tendo em vista o fato de a criatura tornar-se livre, o que era o objetivo do criador desde o início. d) Não, pois a decepção do criador foi muito grande; ele não queria que sua invenção agisse de forma prepotente. 09 – Leia os trechos abaixo: I - ―A urbanização de São Paulo está sendo feita de maneira criminosa, porque está destruindo os pulmões da cidade.‖ II - ―Negrinha era uma pobre órfã de sete anos. Preta? Não; fusca, mulatinha escura, de cabelos ruços e olhos assustados.‖ III - ―Inesperadamente, recebi um convite aquela tarde. A festa seria à tarde, e eu não estava decidida se compareceria ou não.‖ Com relação aos trechos acima, é incorreto afirmar que a) I e II classificam-se como textos narrativos, pois ambos apresentam caracterização de ambiente e progressão cronológica. b) II é notadamente descritivo, já que se propõe a caracterizar os aspectos físicos da pessoa. c) I, II e III possuem estruturas textuais distintas, as quais são, respectivamente, dissertativa, descritiva e narrativa. d) I apresenta uma relação de causa e conseqüência entre os fatos, enquanto que, no texto II, não há qualquer relação de causalidade entre os enunciados, os quais podem, portanto, a) ser invertidos. 10 – Enumere as orações abaixo conforme o código e assinale a alternativa que contém a seqüência correta. I - ambigüidade II - cacófato III - solecismo IV - pleonasmo ( ) Entre dentro do meu carro para você ver como ele é confortável! ( ) Recolhem-se os pássaros à tardinha. ( ) É admirável a fé de seu tio. ( ) O réu achava que se podia cometer crimes impunemente. a) I - IV - II - III c) III - II - I - IV b) II - III - IV - I d) IV - I - II - III 30 11 – Nas alternativas abaixo, assinale aquela em que não há conotação. a) ―O Justo não me consagrou Pão de Vida, nem lugar me foi dado nos altares.‖ b) ―É mais estranho do que todas as estranhezas que as cousas sejam realmente o que parecem ser.‖ c) ―... e muita fonte, posta à beira das veredas, jorrava por uma bica, beneficamente, à espera dos homens e dos gados...‖ d) ―Mareiam a sua obra poemas sem relevo nem músculo, versalhada que escorre desprovida de necessidade artística.‖ 12 – Quanto à figura de linguagem - eufemismo - pode-se dizer que I - consiste na atenuação de idéia desagradável, substituindo-a por uma idéia mais suave. Em "Ontem o pobre homem deixou o mundo dos vivos", a expressão sublinhada ameniza a informação de que "Ontem o pobre homem morreu." II - serve para abrandar idéias que, segundo certos valores culturais, desagradam a alguns grupos, ao ser preferível, por exemplo, "A menina é muito robusta para sua idade" em vez de "A menina é muito gorda para sua idade." III - está sujeita, muitas vezes, às evoluções do mundo como um todo, visto que, hoje em dia, já não se poderia, em muitos casos, construir a frase "Ele morreu daquele mal incurável." querendo referir-se à doença câncer. IV - por significar "dizer bem" ou "boa palavra", na frase "Ana Mári é uma excelente filha, aluna exemplar e ótima conselheira.",os adjetivos são eufêmicos. Está correto o que se afirma em a) I, III e IV. b) II e III apenas. c) I, II e III. d) I, II e IV. 13 – As frases abaixo foram retiradas de redações de alunos. Assinale a única alternativa que não apresenta metáfora. a) ―A Terra sangra. Lenta e dolorosamente, o Planeta pede socorro, pelas vozes de ecologistas e cientistas. Indiferentes a tudo, políticos e empresários executam o ciclo da morte terrestre.‖ b) ―O homem vive continuamente se enganando. Ele busca a todo momento o seu bem-estar. Mas ele está tão preocupado com o balançar de sua jangada, que não consegue despertar para o imenso mar à sua volta.‖ c) ―A humanidade assiste estarrecida a cada abandono infantil que é noticiado. A cada hora temos uma nova ―mãe- monstro‖ no paredão de fuzilamento.‖ d) ―A humanidade se comporta como um astronauta suicida, que contamina o seu reservatório de água potável, destrói o seu gerador de oxigênio e envenena o ar que respira e os alimentos que consome.‖ 14 – Assinale a alternativa que melhor analisa o tipo de discurso presente no trecho a seguir. "E como a estranha música, o mundo recomeçava ao redor. O mal estava feito. Por quê? Teria esquecido de que havia cegos? A piedade a sufocava. O mundo se tornava de novo um mal-- estar." (Clarice Lispector) a) O narrador subordina a si a personagem, retirando-lhe a forma própria e afetiva de expressão. b) Percebe-se que a passagem do que relata o narrador para o que se caracteriza como enunciado da personagem é sutil. c) A fala da personagem é incorporada ao texto mediante subordinação semântico-sintática entre a frase reproduzida e a que a introduz. d) Percebe-se que a fala da personagem - textualmente reproduzida- é destacada por recursos gráficos. 15 – Assinale a alternativa em que todas as palavras se classificam como paroxítonas. a) meteorito - pegada - filatelia - fortuito b) hangar - rubrica - filantropo - pudico c) decano - ruim - maquinaria - avaro d) pipoca - Gibraltar - erudito - substantivo 16 – Assinale a alternativa na qual haja repetição do mesmo tipo de discurso. a) O filho do fazendeiro estava doido. O médico, médico de roça, disse não ser homem sua especialidade. Assim, o rapaz foi levado à Capital. Mas tudo não passara de um engano. Foi preciso que o pai telegrafasse a um amigo Comendador pedindo que desmanchasse o engano. (adaptação – Artur Azevedo) b) O neto do Dr. Armindo disse que está vendo uma dessas grã-finas fazendo um rigoroso regime alimentar e dizendo para a empregada: ―Para mim não precisa preparar almoço, não. Eu como só o saco.‖ (adaptação de um texto de Stanislaw Ponte Preta) c) A atitude do jegue, que tem o mesmo apelido do candidato, irritou o segurança do hotel Gomes. Que tirasse logo o animal da calçada porque ali era hotel cinco estrelas. A Polícia Militar foi chamada. ―Vai todo mundo pra delegacia, até o burro‖, ordenou o militar. (O Estado de São Paulo – adaptação) d) ―As coisas, por detrás de nós, exigem: falemos com elas, mesmo quando nosso discurso não consiga falar delas. Dizem: falar sem coisas é comprar o que seja sem moeda.‖ (João Cabral de Melo Neto) 17 – ―Febre, hemoptise, dispnéia e suores noturnos A vida inteira que podia ter sido e que não foi. Tosse, tosse, tosse.‖ (Manuel Bandeira) Com relação aos vocábulos grifados no texto acima, pode-se afirmar que a) há 2 trissílabos, 1 monossílabo e 2 dissílabos. b) há dígrafo nas palavras tosse, que e febre. c) há encontros consonantais em tosse, que e febre. d) há um hiato. 18 – Em relação à ortografia, observe as frases abaixo e considere seus comentários. Em seguida, assinale a alternativa correta. a) ―Preço do cochão mole: 2,50 o kg‖ (cartaz colocado num açougue). Comentário: Há erro de grafia na palavra cochão, que deve ser escrita com x e não ch. É um tipo de erro relativamente comum, que pode ser explicado pela questão da analogia fonética existente nas palavras ―coxão‖ (coxa) / colchão. b) ―Temos vários equipamentos de suspensão abaixo do custo‖ (promoção de uma oficina mecânica). Comentário: Há erro de grafia na palavra grifada, que deve ser escrita com ç (suspenção); além disso, esta foi usada inadequadamente para o objetivo a que se propõe. c) ―Nossa tacha de juros é menor‖ (propaganda de uma loja). Comentário: Não existe erro de grafia em tacha, já que a palavra, no contexto, refere-se a tributo, imposto, e não ao objeto prego. d) ―Venham conferir! Preços que não estrovam seu orçamento! (chamariz de um supermercado). Comentário: Nessa frase, não há erro de grafia, pois pode-se grafar o verbo sublinhado como estrovam ou estorvam. 19 – ―A vida celibata podia ter certas vantagens próprias, mas seriam tênues, e compradas a troco da solidão.‖ Quanto à função sintática, a expressão grifada, no trecho acima, classifica-se como a) adjunto adverbial. c) objeto indireto. b) adjunto adnominal. d)complemento nominal. 31 20 – ―Que pode uma criatura senão, entre criaturas, amar? Amar e esquecer, amar e malamar amar, desamar, amar? Sempre, e até de olhos vidrados, amar? (Carlos Drummond de Andrade) I - Os termos malamar e desamar, obedecendo ao processo de formação pelo uso de afixos, podem ser considerados sinônimos, uma vez que os prefixos neles usados têm o mesmo sentido. II - malamar e desamar possuem parte do sentido semelhante, uma vez que a parte restante, excluindo-se os prefixos, tem o mesmo radical: amar. III - no termo malamar, observa-se na verdade o fenômeno da parassíntese, por causa do prefixo mal e dos morfemas ar. IV - observando-se a ortografia, malamar deveria ser mal-amar; ocorre que, se assim fizesse o autor, na criação do termo, passaria um conceito errado da formação da palavra, por causa do hífen, ligado à justaposição. Como incorreta(s) temos a(s) sentença(s) a) III apenas. b) II e III apenas. c) I, II e III apenas. d) I, II, III e IV. 21 – Observe, abaixo, alguns verbetes criados por Millôr Fernandes, humorista, jornalista e dramaturgo. “Dicionovário” •Abacatimento: redução no preço do abacate. •Anãofabeto: pequenininho que nem sabe assinar o nome. •Cãodução: carrocinha de cachorro. •Cartomente: uma adivinha que nunca diz a verdade. Quanto à formação das palavras por ele criadas, pode-se dizer que a) todas obedecem perfeitamente ao processo de composição por justaposição, o que as torna legítimas do ponto de vista gramatical. b) o efeito irônico é obtido pela transformação de seus prefixos e sufixos. c) se trata de neologismos que sofreram o processo de derivação imprópria, pois houve uma mudança da classe gramatical das palavras primitivas. d) se trata de neologismos criados pelo autor, que resultaram do acoplamento de duas palavras, sendo que uma das duas está truncada, a fim de produzir significados especiais. 22 – Assinale a alternativa que apresenta a seqüência correta quanto às conjunções coordenativas que preenchem adequadamente o texto abaixo, dando-lhe coerência. Yes, nós temos cinema! Todo mundo fala de um renascimento do cinema brasileiro ______ ele parece incontestável. ______ ainda falta vencer um obstáculo fundamental: o preconceito do espectador brasileiro que continua relutante em sair de casa para assistir a um filme nacional. _____ os filmes têm dificuldade de conseguir muitas salas _____ não conseguem o sucesso que mereciam. (Rubens Ewald Filho) a) e - assim - contudo - por isso b) logo - pois - portanto - todavia c) e - mas - por isso - e d) pois - contudo - entretanto - mas 23 – Leia o texto abaixo: Poética De manhã escureço Outros que contem De dia tardo Passo por passo: De tarde anoiteço Eu morro ontem. De noite ardo. A oeste a morte Nasço amanhã Contra quem vivo Ando onde há espaço Do sul cativo — Meu tempo é quando. o este é meu norte. Os advérbios e locuções adverbiais desse poema de Vinícius de Moraes são muito importantes para o entendimento da mensagem do texto. Isso porque I - as locuções adverbiais marcam a trajetória do eu, utilizadas para localização temporal e espacial. Já os advérbios estabelecem ruptura entre o eu-poético e o comportamento social dos outros. II - o uso de quando como advérbio imprime à última estrofe uma concepção de que a vida está e acontece no presente. III - os verbos utilizados na primeira estrofe perdem seu caráter original, transformando-se em advérbios que revelam oposição de idéias. Está correto o que se afirma em a) I apenas. c) I e II apenas. b) III apenas. d) I, II e III. 24 – Considere o texto: ―E as coisas são mais lindas Quando você está Hoje você está Onde você está As coisas são mais lindas Porque você está (...) Nas coisas tão mais lindas.‖ (Nando Reis) Complete os parênteses com V (verdadeiro) ou F (falso) para assinalar a alternativa com a seqüência correta: ( ) o advérbio mais modifica o adjetivo linda. ( ) o advérbio tão modifica o advérbio mais. ( ) porque é uma conjunção coordenativa, pois explica a razão de ―as coisas serem mais lindas‖. ( ) você é um pronome pessoal reto usado quando há intimidade no tratamento. ( ) além de mais e tão há mais dois advérbios no texto. a) V - F - V - F - F c) V - V - F - F - F b) F - V - F - F - V d) F - V - V - V - V 25 – Assinale a alternativa que contém a justificativa incorreta quanto à pontuação. a) ―A mocinha do caixa, tão loirinha, tão branquinha, tão magrinha, era uma fera.‖(texto adaptado) – as vírgulas foram usadas para separar elementos de valor explicativo. b) ―A Genilda? Bom... não sei... acho que ela nem chegou ainda...‖ – as reticências foram usadas para indicar que houve dúvida, hesitação ou surpresa. c) ―Dona Diva era muito piedosa (que Deus a tenha!), mas muito chata.‖ – os parênteses foram usados por causa da oração intercalada no texto. d) ―O dono da loja seria o chefe do comércio, e o prestígio do vendedor era grande em Taitara.‖ – a vírgula foi usada antes do e para separar termos da mesma função sintática. 32 26 – A um secretário de escola foi dada a ordem de redigir um ofício para o prefeito da cidade com o seguinte conteúdo: I - finalidade: pedido de conserto da parte hidráulica do prédio. II - concessão: impedimento da regularidade das aulas. III - tempo: o mais rápido possível. IV - acordo: o previsto em reunião anterior. Utilizando as conjunções e locuções conjuntivas subordinativas, o corpo do texto corretamente produzido deverá ser: a) Conforme o combinado em assembléia de junho/2000, solicitamos de V.Exª a gentileza de enviar funcionário a esta escola, em caráter de urgência, para que seja feito o conserto da parte hidráulica do prédio. Embora os reparos impeçam o andamento das aulas, não podemos mais adiar a solução desse problema e, por isso, pedimos sua colaboração. b) Mesmo que não seja possível, solicitamos de V. Exª a gentileza de enviar funcionário a esta escola a fim de que seja feito o conserto da parte hidráulica do prédio. Contanto que os reparos impeçam o andamento das aulas, não podemos mais adiar a solução desse problema e, por isso, pedimos sua colaboração já que foi combinado em assembléia de junho/2000. c) Se for possível, solicitamos de V. Exª a gentileza de enviar funcionário a esta escola à medida que a parte hidráulica do prédio seja consertada. Para que os reparos impeçam o andamento das aulas, não podemos mais adiar a solução desse problema e, por isso, pedimos sua colaboração, apesar do que foi combinado em assembléia de junho/2000. d) Visto que é possível, solicitamos de V. Exª a gentileza de enviar funcionário a esta escola, posto que seja feito o conserto da parte hidráulica do prédio. Conforme os reparos impeçam o andamento das aulas, não podemos mais adiar a solução desse problema e, por isso, pedimos sua colaboração, porquanto tenha sido combinado em assembléia de junho/2000. 27 – ―Aquela que te humilhou, aqui a tens abatida, no mesmo lugar onde ultrajou-te, nas iras de sua paixão. Aqui a tens implorando teu perdão e feliz porque te adora, como o senhor de sua alma.‖ (José de Alencar – Senhora) O texto acima, do romance Senhora, de José de Alencar, é uma passagem em que Aurélia implora o perdão de Seixas, seu marido. Percebe-se que ela fala de si própria como se estivesse falando de outra pessoa. Baseando-se nessas informações, leia os comentários abaixo, referentes às pessoas do discurso, e assinale a alternativa incorreta. a) ―Sua paixão‖ refere-se à paixão que Aurélia sentia. b) Em ―aqui a tens abatida‖, o pronome refere-se à mulher que humilhou o marido. c) ―Senhor de sua alma‖ refere-se à alma de Seixas, pois ele era senhor de si próprio. d) Em ―ultrajou-te‖, o pronome refere-se a Seixas, pois ele é que foi ultrajado. 28 – Observando a regência dos verbos em destaque no trecho a seguir, assinale a alternativa que preenche corretamente as lacunas: "Excesso de leis atrapalha _____ vida do País e impede _____ população _____ saber _____ que as normas deve obedecer." Obs.: Ø significa que não há preposição ou artigo. a) a - a - de - a c) a - à - Ø - Ø b) à - a - para - à d) Ø - à - Ø - sobre 29 – Leia o texto: ―Se recebo um presente dado com carinho por pessoa de quem não gosto — como se chama o que sinto? Uma pessoa de quem não se gosta mais e que não gosta mais da gente — como se chama essa mágoa e esse rancor?‖ (Clarice Lispector) Com relação ao texto, é correto afirmar que a autora a) tenta nomear o sentir, por isso podemos dizer que ela tenta criar verbos. b) preocupa-se com nomes de sentimentos relativos a circuns-tâncias específicas; daí dizermos que tenta criar advérbios. c) reflete sobre a qualidade daquilo que sente; por isso buscar criar adjetivos. d) tenta criar nomes que qualifiquem a essência de sensações de que é acometida; por isso busca criar substantivos. 30 – Leia atentamente o texto abaixo e, em seguida, marque a alternativa que contém a informação incorreta quanto ao em- prego dos verbos sublinhados no texto. O garoto da vizinha me pediu que o ajudasse a fazer (a fazer, não, a completar) um trabalho escolar sobre a boca. Estava preocupado porque só conseguira escrever isto: ―Pra que serve a boca?‖ a) O fato de o menino ter conseguido escrever apenas ―Pra que serve a boca?‖ precede o fato de ele pedir ajuda para o seu trabalho, o que justifica o seu pedido de auxílio. b) A atitude do menino de pedir ajuda refere-se a uma ação passada não-pontual, cujo início e término não são definidos, já que não há referências temporais. c) A preocupação em que o menino se encontrava refere-se a um estado passado contínuo e, portanto, concomitante às outras ações passadas expressas no texto. d) A ação de ajudar o garoto a fazer o trabalho escolar está expressa em caráter eventual, em dependência estreita com a vontade daquele que a emprega, ou seja, do garoto. 31 – ―Como se sabe, a teoria da relatividade geral afirma que nem o tempo nem a distância são valores absolutos, dependendo do movimento relativo dos observadores, e que o único valor absoluto e constante é a velocidade da luz.‖ (Isaac Asimov) Indique a alternativa que classifica corretamente a função sintática e a classe morfológica dos termos grifados. a) objeto indireto – substantivo b) sujeito – substantivo c) sujeito – adjetivo d) objeto direto – adjetivo 32 – ―A Eternidade está longe (Menos longe que o estirão Que existe entre o meu desejo E a palma da minha mão)‖ (Manuel Bandeira) No texto acima, temos a) uma oração absoluta, uma oração principal e uma oração adjetiva. b) uma oração absoluta, duas orações principais, uma oração comparativa e uma oração adjetiva. c) duas orações principais, uma oração comparativa e uma oração adjetiva. d) uma oração absoluta, uma oração principal, uma oração comparativa e uma oração adjetiva. 33 – Assinale a alternativa em que a pontuação não obedece à norma culta. a) Melhores, porém, que os remédios são as bulas que os acompanham. É possível que as bulas desapareçam — o que pode ser ruim para os pacientes. (adaptação do texto original) b) ―Há estudantes tímidos, temerosos, que não encontram alegria ou satisfação no trabalho, que crêem não serem aprovados, que se consideram de pouco valor.‖ c) A violência é um dos temas preferidos pela televisão, e muitos acreditam que a apresentação de tanta violência, influi negativamente, sobre o comportamento do telespectador. d) O rádio e a tevê independem do nível de alfabetização do público; atingem, porém, a maior parcela da população que a imprensa. (adaptação do texto original) 33 34 – No período ―O robô Sojourner envia mais de cinco mil fotos, comprovando que já existiu água no Planeta Vermelho.‖, o trecho grifado corresponde, sem prejuízo para o sentido original, ao trecho: a) ... quando comprova que já existiu água no Planeta Vermelho. b) ... que comprovam que já existiu água no Planeta Vermelho. c) ... visto que comprova que já existiu água no Planeta Vermelho. d) ... o que comprova que já existiu água no Planeta Vermelho. 35 – Quantas orações há nos versos abaixo? "Como ama o homem adulto o adultério E o ébrio a garrafa tóxica de rum, Amo o coveiro - este ladrão comum Que arrasta a gente para o cemitério. (Augusto dos Anjos) a) 2 c) 4 b) 3 d) 5 36 – Observe: I - Outeiro das Brisas é um lugar exótico, onde o sol, o sossego e a sensualidade baianas se misturam. II - ―Vossa Excelência foi injusto em sua sentença, Doutor Pro- motor.‖ III - Confiança, na dose certa, é sempre bom para conservar as amizades. Assinale a alternativa que contém a(s) frase(s) correta(s) quanto à concordância nominal. a) I apenas. b) II e III apenas. c) I e III apenas. d) II apenas. 37 – Nas frases abaixo, I - O político nomeou seu filho secretário de finanças. II - Mário, sua prova está exemplar! III - Valdirene ficou em Brasília, nas férias do ano passado. Os predicados classificam-se, respectivamente, como a) verbo-nominal, verbal e nominal. b) verbo-nominal, nominal e verbal. c) nominal, verbo-nominal e verbal. d) verbal, nominal e verbo-nominal. 38 – ―Destes penhascos fez a natureza O berço em que nasci: oh! quem cuidara Que entre penhas tão duras se criara Uma alma terna, um peito sem dureza!‖ (C.M.Costa) Quanto à função sintática dos termos abaixo, coloque, nos parênteses, F (falso) ou V (verdadeiro) para as afirmativas e assinale a alternativa com a seqüência correta. ( ) a natureza – objeto direto ( ) o berço – sujeito ( ) entre penhas tão duras – adjunto adverbial ( ) sem dureza – adjunto adnominal a) V - F - V - F b) V - V - F - F c) F - V - F - V d) F - F - V - V 39 – Assinale a alternativa cuja regência nominal não está de acordo com a norma culta. a) Seus olhos andavam saturados de ódio das pessoas que o atormentavam. b) Era maravilhoso ficar ali, olhinhos brilhando, boca aberta, prestando atenção às estórias que vovô contava. c) A mãe andava atarefada em preparar os festejos de fim de ano. Era preciso entendê-la. d) Ela andava a lançar olhares lânguidos aos rapazes; fazia-o de modo propositalmente ofensivo para a época. 40 – Assinale a alternativa em que não se deve colocar nenhum acento grave (crase). a) A chegada do professor, os alunos levantaram-se respeitosamente. b) Esta questão de crase é semelhante a que vimos ontem. c) ―E seria bonita a pedra? Nada, uma pedra a-toa, de nome geral, porém...‖ d) ―... não gosta da cor do homem, a qual se mistura a todos os verdes naturais (...) das folhas...‖ 2/2003-TURMA A Os homens são diferentes Sou um arqueólogo e o Homem é meu campo de estudo. Entretanto, cogito se alguma vez chegaremos a conhecer o Homem – isto é, o que realmente o torna diferente de nós, Robôs – através das escavações nos planetas mortos. Por exemplo, uma vez eu conheci um Homem, e as coisas não são tão simples quanto nos contam na escola. Nós temos poucos registros, naturalmente, e Robôs como eu estão tentando preencher as lacunas, mas penso que não estamos chegando a nada de concreto. Nós sabemos, ou pelo menos os historiadores o dizem, que os Homens são originários de um planeta chamado Terra. Sabemos ainda que eles viajaram corajosamente de estrela para estrela e em todos os lugares onde pararam deixaram colônias – Homens ou Robôs, e às vezes ambos – aguardando sua volta. Mas nunca voltaram. Aqueles foram os dias de glória do mundo. Teremos nós envelhecido? O Homem tinha uma centelha ardente – o termo antigo é "divina", penso – que o impelia através da grande noite dos céus, e nós não mais nos ligamos à grande teia que eles teceram. Nossos cientistas contam-nos que o Homem é muito semelhante a nós – o esqueleto, por exemplo, é praticamente o mesmo que o de um Robô, excetuando-se o fato de que é constituído de compostos de cálcio, em vez de titânio. Fala-se eruditamente de uma "pressão de população" como uma "força impulsionando em direção às estrelas". Sem dúvida, há outras diferenças. Foi em minha última pesquisa de campo, em um dos planetas inferiores, que encontrei um Homem, que deve ter sido o último Homem neste sistema, e tinha estado sozinho por tanto tempo que nem mais sabia falar. Depois que aprendeu nossa língua, demo-nos muito bem, e eu planejava até trazê-lo de volta comigo. Entretanto, alguma coisa lhe sucedeu. Um dia, sem razão alguma, começou a queixar-se do calor. Verifiquei sua temperatura e concluí que seus circuitos termostáticos se tinham queimado. Eu tinha um jogo sobressalente comigo, e, como o dele obviamente não estava funcionando bem, lancei-me ao trabalho. Desliguei-o sem problema algum. Enterrei a agulha no seu pescoço, para desligar o interruptor, e ele deixou de funcionar, como qualquer Robô. Mas quando eu o abri, por dentro era diferente. E, quando o montei de novo, não consegui fazê-lo funcionar. Depois disso não sei explicar o que aconteceu. O fato é que ele se foi dissolvendo, e, na época em que eu estava pronto para voltar, mais ou menos um ano depois, apenas os ossos tinham sobrado. Sem dúvida alguma, o Homem deve ser diferente. (Alan Bloch) As questões de 01 a 05 referem-se ao texto acima. 01 – Ao longo do texto, são fornecidas algumas informações sobre o Homem, sob o ponto de vista de um observador que não é humano. Assinale a alternativa em que esse ponto de vista está evidente. a) "(...) os Homens são originários de um planeta chamado Terra." b) "Desliguei-o sem problema algum. Enterrei a agulha no seu pescoço, para desligar o interruptor, e ele deixou de funcionar, como qualquer Robô." c) "Sou um arqueólogo e o Homem é meu campo de estudo." d) "(...) eles viajaram corajosamente de estrela para estrela e em todos os lugares onde pararam deixaram colônias (...)." 34 02 – De acordo com o texto, é incorreto afirmar que a) há muito ainda para se saber sobre o Homem. b) o Homem é uma espécie em extinção; daí justificar-se o fato de ser estudado por um arqueólogo. c) o Homem é totalmente diferente do Robô, sendo, por essa razão, estudado por este. d) o Homem desperta o interesse dos Robôs, que tentam conhecê-lo melhor. 03 – Procedendo a uma leitura profunda do texto, admite-se a seguinte interpretação: a) Homens e Robôs são seres naturalmente diferentes, por isso o convívio entre eles é impossível. b) O Robô, embora aparentemente vitorioso, continua sujeito à sua condição de máquina, o que significa que ele ainda possui limitações que o impedem de conhecer o Homem verdadeiramente. c) Em um futuro próximo, a raça humana será totalmente extinta da face da Terra. d) O domínio dos Robôs é uma ficção, portanto não é possível se depreender um tema. 04 – No trecho "Fala-se eruditamente de uma 'pressão de população' como uma 'força impulsionando em direção às estrelas'", considerando a idéia expressa no 3º parágrafo, o Robô poderia estar referindo-se a) à superpopulação na Terra, fato que obrigou o Homem a abandonar seu planeta. b) ao artifício elaborado pelo Homem, o foguete, responsável por transportá-lo ao espaço sideral. c) a uma necessidade própria do ser humano de querer conhecer o espaço fora de seu planeta. d) a um fenômeno natural que injetou o Homem para o espaço intergaláctico. 05 – O Homem com o qual o Robô manteve um contato morreu a) porque a temperatura do planeta sofreu uma elevação insuportável para a sobrevivência humana. b) devido aos maus-tratos do Robô, insensível à dor e ao sofrimento humanos. c) em razão do desconhecimento do Robô a respeito da fisiologia humana. d) pelo simples fato de que os Homens não são imortais como os Robôs. 06 – Assinale a alternativa cujas palavras se classificam como ditongos decrescentes. a) aquarela - imundície - Páscoa b) tireóide - petróleo - aquoso c) canibais - guache - Romeu d) carrosséis - casarões - revestiu 07 – O período "Aquela sublime paisagem era minha querida amiga nas horas de tristeza e saudade." apresenta as seguintes palavras que contêm encontro consonantal: a) paisagem - minha c) saudade - solidão b) aquela - querida d) sublime - tristeza 08 – "Levamos-te cansado ao teu último endereço Vi com prazer Que um dia afinal seremos vizinhos Conversaremos longamente De sepultura a sepultura No silêncio das madrugadas." (Manuel Bandeira) Quanto às figuras de linguagem, podemos afirmar que, no primeiro verso do texto acima, há a) hipérbole. c) eufemismo. b) onomatopéia. d) catacrese. 09 – Na forma verbal desconhece (presente do indicativo): a) a vogal temática e aparece em todas as pessoas gramaticais. b) a desinência número-pessoal da primeira pessoa é idêntica à da terceira. c) a desinência número-pessoal da segunda pessoa do plural é s. d) a desinência modo-temporal é ausente em todas as pessoas gramaticais. 10 – Observe o texto: – Qué apanhá, sordado? – Quê? – Qué apanhá? Pernas e cabeça na calçada. Com relação ao texto acima, é correto afirmar que a) se trata de uma descrição, pois as ações encontram-se no presente, não indicando, portanto, nenhuma transformação de estado. b) o trecho é notadamente dissertativo, pois os fatos encadeiam-se numa relação de causa e efeito. c) os fatos estão organizados numa disposição tal que entre eles existe uma relação de anterioridade e de posterioridade, o que caracteriza um texto narrativo. d) não é possível classificar esse texto, visto que ele não apresenta elementos suficientes para tal. 11 – Assinale a alternativa cuja frase se relaciona com o defeito de estilo apresentado. a) Vicente já não sente dores de dente como antigamente. (cacofonia) b) Preciso de um rapaz para caçar pássaros e uma garota menor. (cacofonia) c) As crianças parecem que ficaram envergonhadas. (barbarismo) d) Ama o filho a boa mãe. (solecismo) 12 – Todas as alternativas apresentam conotação, exceto: a) "Mostra-lhe o tambor de salitre e brisa que rufa sozinho entre os arquipélagos de sua pobreza." (Ivo Lêdo) b) "Bendito sejas tu, a quem certo, devemos a grandeza real de tudo quanto temos! Sonha em paz! Sê feliz! E eu que fique de joelhos" (Ciro Costa) c) "Ave Maria – lento o bronze soa Com voz que ecoa na longínqua serra" d) "No tronco mais verde, Que no prado houvesse, Amor me mandou Seu nome escrevesse" (Tomás A. Gonzaga) 13 – Assinale a alternativa incorreta quanto ao tipo de discurso. a) – Por que foges de mim? Perguntou o amante. (direto) b) Ele disse que as coisas iam bem. (indireto) c) – Senhora, resolvi a situação pendente. (indireto) d) Carlos, durante a festa, perguntou se os convidados vieram. (indireto) 14 – Complete as lacunas das frases abaixo e assinale a alternativa correta. Ele _____ (ter) muitos brinquedos que ____ (ter) força própria. Depois da tempestade, ________ (vir) a bonança. Quando as idéias não ____ (vir), elas não____ (ver) solução. a) têm - tem - vêem - vem - vem b) tem - tem - vêm - vem - vêem c) têm - têm - vem - vêem - vem d) tem - têm - vem - vêm - vêem 15 – Qual dos ditados abaixo apresenta o maior número de sílabas? a) Gato escaldado tem medo de água. b) Em casa de ferreiro, o espeto é de pau. c) O uso do cachimbo faz a boca torta. d) Águas passadas não movem moinho. 16 – Assinale a alternativa em que todas as palavras se classificam como paroxítonas. (Observe que o acento gráfico foi retirado propositadamente de algumas delas.) a) filantropo - caminhante - substantivo b) arquetipo - canavial - tupi c) omega - servo - soldado d) refem - satisfeito - patativa 35 17 – Assinale a alternativa cujas palavras estão graficamente corretas. a) flecha, xavantes, cuscuz, chinelo, umidecer. b) suavizar, catequizar, canalizar, analisar, pesquisar. c) flexa, xavantes, cuscus, chinelo, umedecer. d) suavisar, catequisar, canalisar, analisar, pesquisar. 18 – Complete o período seguinte com o pronome pessoal conveniente. "Juliana passeava com seu colega de classe; de repente,_____ ______ beija, deixando- _______ encabulada." A seguir, assinale a alternativa correta. a) o - ela - a b) ele - a - a c) ele - a - ela d) o - a - a 19 – Texto I- "Vaca exausta em plena lua-de-mel (1) procura história pra boi dormir." Texto II- "Vende-se teleobjetiva (2) para fotografar (3) primos distantes." Observe as palavras destacadas e numeradas nos textos acima e assinale a alternativa que contém a afirmativa correta quanto ao processo de formação das palavras. a) 1 - Composição por justaposição 2 - Hibridismo 3 - Palavra composta por dois radicais gregos b) 1 - Composição por aglutinação 2 - Palavra composta por dois radicais gregos 3 - Palavra composta por dois radicais latinos c) 1 - Derivação parassintética 2 - Palavra composta por dois radicais latinos 3 - Hibridismo d) 1 - Derivação imprópria 2 - Hibridismo 3 - Derivação sufixal 20 – Observe: I- Todos os abaixo-assinados concordam com os textos que lerei nas próximas quartas-feiras. II- Os garotos comeram todos os pés-de-moleques. III- A intervenção do apresentador pôs fim a todos os bate- bocas. IV- Não acredito nas mulas-sem-cabeças. Quanto à flexão dos substantivos compostos, é correto afirmar que estão corretos somente a) II e IV. b) I e III. c) III e IV. d) I e II. 21 – Escolha a opção em que todos os verbos são defectivos. a) trovejar - miar - abolir c) magoar - nevar - rir b) falir - entregar - chover d) latir - reaver - pedir 22 – "Era uma expressão fria, pausada, inflexível, que jaspeava sua beleza, dando-lhe quase a gelidez da estátua." No trecho acima, classificam-se as palavras grifadas, respectivamente, como a) conjunção integrante, pronome tônico, preposição, adjetivo. b) pronome relativo, pronome reto, conjunção coordenativa, substantivo. c) pronome relativo, pronome átono, advérbio, substantivo. d) conjunção subordinativa, pronome tônico, adjetivo, advérbio. 23 – "Meu dia outrora principiava alegre, No entanto à noite eu chorava. Hoje mais velho, Nascem-me em dúvida os dias, mas Findam sagrados, serenamente." (M. Bandeira) Classifique, respectivamente, as palavras grifadas no texto acima e assinale a alternativa correta. a) substantivo, advérbio, adjetivo, advérbio b) advérbio, substantivo, substantivo, adjetivo c) substantivo, adjetivo, adjetivo, advérbio d) adjetivo, advérbio, substantivo, adjetivo 24 – Coloque V (para Verdadeiro), F (para Falso) e assinale a alternativa que contém a seqüência com a exata correspondência entre adjetivo e locução adjetiva. ( ) O corpo discente da escola fez muitas reivindicações ao diretor. (de professores) ( ) Teve fortes dores cervicais ontem. (de pescoço) ( ) Infelizmente ele nasceu com problema renal. (de fígado) ( ) Suas observações pueris muito me decepcionaram. (de velho) a) V - V - F - F c) F - V - F - F b) F - F - V - V d) V - F - V - V 25 – Segundo a norma culta, assinale a alternativa cuja colo- cação do pronome está incorreta. a) Os rapazes que me perguntaram teu nome são meus vizinhos. b) Disseram-me que você é o melhor aluno da turma. c) O mendigo se queixava do frio. d) Tenho dedicado-me à música ultimamente. 26 – A classificação das conjunções adverbiais está incorreta na opção a) Como não choveu, a represa secou. (causal) b) Ela sabe tanto quanto eu. (comparativa) c) Gesticulava para que todos vissem. (final) d) Não haverá aula hoje, segundo anunciou o diretor. (condicional) 27 – Assinale a alternativa em que há erro de pontuação. a) Dorival, meu filho está acamado. b) Dorival, meu filho, está acamado. c) Meu filho Dorival está acamado. d) Meu filho Dorival, está acamado. 28 – Assinale a alternativa que classifica, correta e respectiva- mente, os termos grifados no período "A manutenção das estradas é fundamental para nossa segurança". a) complemento nominal - complemento nominal b) complemento nominal - adjunto adnominal c) adjunto adnominal - complemento nominal d) adjunto adnominal - adjunto adnominal 29 – Assinale a alternativa cuja regência contraria a norma culta da língua. a) Renunciou ao cargo de diretor. b) Assistia ao desfile todos os anos. c) Teria de ir à casa daquele corrupto! d) Prefiro muito mais cerveja do que vinho. 30 – "O mal dos que estudam as superstições é não acreditarem nelas." (Mário Quintana) Em relação à principal, a oração destacada é subordinada a) adjetiva restritiva reduzida de particípio. b) adjetiva explicativa reduzida de particípio. c) substantiva subjetiva reduzida de infinitivo. d) substantiva predicativa reduzida de infinitivo. 31 – No período "Convém estudarmos nossa língua", a oração grifada está corretamente desenvolvida e classificada na seguinte alternativa: a) que estudemos nossa língua (substantiva subjetiva) b) que nossa língua seja estudada (adjetiva restritiva) c) que estudem nossa língua (substantiva predicativa) d) que estudassem nossa língua (substantiva objetiva direta) 32 – Observe as frases seguintes: I- Nem acreditei quando começou as férias. II- Um bilhão de reais foi gasto na construção daquele edifício. III- Como chovem promessas durante as campanhas eleitorais! IV- Os estudantes universitários pareceram gostarem da palestra. Segunda a norma culta, a concordância verbal está correta nas frases a) I e II. b) II e III. c) III e IV. d) I e IV. 36 33 – Assinale a alternativa em que a concordância nominal está incorreta. a) A rua, meio deserta, despertava certo pavor na jovem estudante. b) Elas mesmas enviaram os documentos anexo às declarações. c) Era preciso muita paciência com aqueles moleques. d) Finalmente estou quite com o Banco. 34 – Preencha as lacunas adequadamente e assinale a alternativa correta. A pobre vítima era querida _________ todos, leal ________ os amigos e útil __________ a empresa em que trabalhava. a) de - com - para c) de - para com - entre b) por - para - com d) por - com - sobre 35 – Observe: I- Se você for aquele quiosque, traga-me mais um acarajé. II- Não fale tal assunto a determinadas pessoas. III- Eles estão cara a cara. IV- Não falo aquilo que me contaste. Quanto ao acento grave, indicador de crase, deduzimos que ele deve ocorrer em a) apenas III. b) I e III. c) II e IV. d) apenas I. 2/2003-TURMA B O rato e a comunidade I O rato apareceu Num ângulo da sala, Um homem e uma mulher Apareceram também, Trocaram palavras comigo, Fizeram diversos gestos E depois foram-se embora. ? Que sabe esse rato de mim. E esse homem e essa mulher Sabem pouco mais que o rato. II Passam meses e anos perto de nós, Rodeiam-nos, sentam-se com a gente à mesa, Comentam a guerra, os telegramas, Discutem planos políticos e econômicos, Promovem arbitrariamente a felicidade coletiva. Conhecem nosso paletó, camisa e gravata, Nosso sorriso e o gesto de mover o copo. Têm medo de nos tocar, não conhecem nossas lágrimas. ? Que sabem do nosso coração, do nosso desespero, da nossa comunicabilidade. Que sabem do centro da nossa pessoa, de que são participantes. Subúrbios longínquos, esses homens. III Entretanto cada um deve beber no coração do outro. Todos somos amassados, triturados: O outro deve nos ajudar a reconstruir nossa forma. O homem que não viu seu amigo chorar Ainda não chegou ao centro da experiência do amor. Para o amigo não existe nenhum sofrimento abstrato. Todo o sofrimento é pressentido, trocado, comunicado. ? Quem sabe conviver com o outro, quem sabe transferir o coração. Viver com o outro é agonizar, morrer é ressuscitar com. Ninguém mais sabe tocar na chaga aberta: Entretanto todos têm uma chaga aberta. IV Desconhecido que atravessas a rua, ? Que tens de comum comigo. A mesma solidão e a mesma roupa. Procuras consolo, mas não podes parar. És o servo da máquina e do tempo. Mal sabes teu nome, nem o que desejas neste mundo. Procuras a comunidade de uma pessoa, Mas não a encontras na massa-leviatã. Procuras alguém que seja obscuro e mínimo, Que possa de novo te apresentar a ti mesmo. V A mulher que escolhemos, a única e não outra Dentre tantas que habitam a terra triste, Esta mesma, frágil e indefesa, bela ou feia, Eis o mundo que nos é de novo apresentado Por intermédio de uma só pessoa. Esta é a que rompe as grades do nosso coração, Esta é a que possuímos mais pela ternura que pelo sexo. E nada será restaurado no seu genuíno sentido Se a mulher não retornar ao seu princípio: É a máquina instalada dentro dela que deveremos vencer. Quando essa mulher se tornar de novo submissa e doce, Os homens pela mão da eterna mediadora Abrirão outra vez um ao outro os corações que sangram. ( Murilo Mendes) As questões de 01 a 05 referem-se ao texto acima. 01 – Segundo o texto, para que o homem possa reencontrar-se e doar-se deve, como atitude primeira, a) procurar alguém que possa apresentá-lo novamente a si mesmo. b) "viver, agonizar e ressuscitar" com o outro. c) sobrepujar a mulher, a fim de, obtida sua auto-afirmação, abrir-se ao outro. d) centrar-se na mulher, pois, através dela, está a possibilidade de se estabelecer comunicação efetiva. 02 – Em relação ao título "O rato e a comunidade", é correto afirmar que a) o rato, como roedor que é, simboliza o desgaste das relações, aquilo que corrói e destrói a comunidade, a sintonia de corações. b) pouco expressa realmente do conteúdo do texto; o rato é, na verdade, um elemento de estranhamento, a fim de mostrar que os relacionamentos não se aprofundam. c) trata-se o rato de um elemento de oposição à comunidade, que simboliza a experiência do amor, a solidariedade de que fala o poeta. d) o poeta utiliza a figura do rato para simplesmente introduzir aquilo que realmente deseja: tratar da indiferença humana. 03 – Em relação à mulher, o texto a) busca o essencialmente feminino sem, no entanto, deixar de lado um juízo de valor. b) assume clara postura de exaltação e força (máquina) que é capaz de conduzir o homem adiante. c) coloca-a apenas como uma prestadora de serviços, uma intercessora; por isso, submissa e inferior. d) mostra-a como um ser para se amansar, uma vez que, como intermediária, pode agir em benefício próprio. 04 – O texto apresenta momentos específicos de interpelação, de demonstração de superficialidade no relacionamento humano e de necessidade de doação. Esses momentos encontram-se, respectivamente, nas seguintes estrofes: a) V, II e III. c) IV, II e III. b) IV, I e V. d) I, II, III, IV e V. 05 – Leviatã, segundo o Dicionário Aurélio, significa "monstro do caos, na mitologia fenícia, identificado, na Bíblia, como um animal aquático ou réptil". Assim, a expressão massa-leviatã, em relação ao trecho em que se encontra, exprime que I - sociedade moderna não está estruturada para uma vida humanizada. II - sociedade moderna arrasta o homem para o automatismo, para a ausência do tempo, do outro. III - sociedade luta contra um monstro invisível que se faz presente no dia-a-dia e que despersonaliza os indivíduos. Está correto o que se afirma em a) III apenas. c) I e III apenas. b) I e II apenas. d) I, II e III. 37 06 – Identifique a figura de linguagem presente em todos os textos abaixo. I - "Desculpem-me por ter sido longo porque não tive tempo de ser breve." II - "Mais servira se não fora Para tão longo amor tão curta a vida." III - "Foi então que, em face destas duas tristezas – a noite que descia dos céus, – a solidão que subia do oceano – recordei-me de vós, os meus amigos." IV - "Não há no mundo alegria sem sobressalto, concórdia sem dissensão, descanso sem trabalho, riqueza sem miséria, digni- dade sem perigo, finalmente, não há gosto sem desgosto." a) antítese c) prosopopéia b) metáfora d) hipérbole 07 – Assinale a única sentença que apresenta sinestesia. a) "A noite estava muito negra. E havia sobre a cidade um silêncio côncavo, de abóbada." (Eça de Queirós) b) "E a tua boca anda mentindo Enganada pelos teus sentidos." (Cecília Meireles) c) "Precisamos descobrir o Brasil! Escondido atrás das florestas, com a água dos rios no meio." (Carlos Drummond de Andrade) d) "Sobre a nudez forte da verdade o manto diáfano da fantasia / ou sobre a nudez forte da fantasia o manto diáfano da verdade." (José Saramago) 08 – ANULADA 09 – No que se refere à acentuação gráfica, pode-se fazer as seguintes considerações: a) Consiste na aplicação, apenas, de certos sinais escritos sobre algumas letras para representar o que foi estipulado pelas regras de acentuação, que são arbitrárias e desvinculadas da natureza da língua. b) As regras de acentuação visam sistematizar a leitura das palavras de nossa língua; assim sendo, baseiam-se na posição da sílaba tônica, no timbre da vogal, nos padrões prosódicos menos comuns da língua. c) As regras de acentuação foram criadas visando também às palavras homônimas, tanto as homógrafas, quanto as homófonas; é o caso, por exemplo, de tem/têm e de sabia/sabiá. Esses são os chamados acentos diferenciais. d) Os fenômenos representados pela nasalização da vogal, do fonema representado pela letra u, pelos hiatos e ditongos abertos não se caracterizam como regra. 10 – O verbo afinar aparece na voz passiva em: a) Resolveu afinar suas idéias pelas da maioria. b) Suas idéias foram afinadas pelas da maioria. c) Seu procedimento afina pelo do pai. d) Afinaram-se as vozes, ao primeiro acorde do piano. 11 – Assinale a alternativa em que os pronomes oblíquos destacados nos pares de orações não tenham a mesma função sintática. a) 1 – Se a vejo triste, consolo-a. 2 – Desejava reanimá-la a qualquer custo. b) 1 – Ensinei-lhe a amar os versos. 2 – Não lhe pagou com gratidão. c) 1 – Dá-se ares de grande dama. 2 – Os dois amam-se profundamente. d) 1 – Fale-nos de seus grandes anseios. 2 – Suplicou-nos o perdão em sua hora derradeira. 12 – Observe: "O ouro fulvo do ocaso as velhas casas cobre; Sangram, em laivos de ouro, as minas, que a ambição Na torturada entranha abriu da terra nobre: E cada cicatriz brilha como um brasão." I - A expressão "da terra nobre" desempenha a função sintática de objeto indireto. II - São sujeitos simples os termos "O ouro fulvo do ocaso", "as minas", "a ambição", "cada cicatriz" e "um brasão". III - Em todo o poema, há dois objetos diretos. IV - Os termos "em laivos de ouro" e "na torturada entranha" são adjuntos adverbiais de lugar. Os itens que apresentam asserções corretas no que se refere à classificação sintática dos termos da estrofe acima são a) I e IV apenas. c) II e III apenas. b) I e III apenas. d) I, II e IV apenas. 13 – Em relação às formas verbais destacadas em "Um parlamentar diz que se o governo não ocupar espaços e obter sinais positivos dos vários setores da sociedade, ficará muito difícil pensar na reeleição.", considera-se que a) estão empregadas corretamente; nos verbos regulares sempre a forma verbal da 3ª pessoa do singular do futuro do subjuntivo coincide com o infinitivo. b) estão empregadas incorretamente; nos verbos irregulares sempre a forma verbal da 3ª pessoa do singular do futuro do subjuntivo irá coincidir com o infinitivo. c) o verbo regular ocupar antes do irregular obter induz o redator a flexionar incorretamente o segundo verbo por analogia com o primeiro. d) para o verbo irregular, no caso obter, há a possibilidade de se flexionar a 3ª pessoa do futuro do subjuntivo coincidindo ou não com o infinitivo; assim as duas formas verbais foram usadas corretamente. 14 – Em qual das alternativas o uso da vírgula é facultativo, dependendo de ênfase ou não? a) O Governo de São Paulo tem investido na Educação. Portanto, o nível do ensino deverá elevar-se. b) Monteiro estudou muito para a prova final, e ficou reprovado. c) Os alunos fizeram todos os exercícios; não ficarão, pois, de quarentena. d) "A mulher aceita o homem por amor ao casamento, e o homem tolera o matrimônio por amor à mulher." 15 – Leia com atenção as orações abaixo e assinale a alternativa correta, observando a regência verbal do verbo falar. I - As histórias hão de falar no herói que dedicou sua vida ao bem. II - Foi isso o que o aluno falou com o professor. III - O médico falou ao paciente do problema que lhe indicaram os exames. Está correto o que aparece em a) II apenas. c) I e II apenas. b) I e III apenas. d) I, II e III. 16 – "Penetra surdamente no reino das palavras. Lá estão os poemas que esperam ser escritos. Estão paralisados, mas há desespero, há calma e frescura na superfície intacta Ei-los, sós e mudos, em estado de dicionário." Os termos grifados no texto acima exercem, respectivamente, a função sintática de a) sujeito e adjunto adnominal. b) objeto direto e predicativo do objeto. c) sujeito e predicativo do objeto. d) objeto direto e predicativo do sujeito. 17 – ANULADA 18 – "Meu São Paulo da garoa - Londres das neblinas finas - Um pobre vem vindo, é rico! Só bem perto fica pobre, Passa e torna a ficar rico." (Mário de Andrade) Observando-se os versos de 3 a 5, podemos dizer que as orações possuem, respectivamente, predicados a) verbo-nominal; verbal; verbo-nominal. b) verbo-nominal; nominal; verbal; verbal. c) verbal; nominal; verbal; verbal; nominal. d) verbal; nominal; nominal; verbal; nominal. 38 19 – Observe os termos grifados no texto abaixo: "O cão é o melhor amigo do homem porque ladra para protegê-lo, vela seu sono, estima-o na riqueza e na miséria, indiferentemente, como, indiferentemente, ama-o moço ou velho, segue-o pela vida e para a morte." (Millôr Fernandes) É correto afirmar que a) moço e velho, mesmo com função predicativa, ou seja, qualificando, morfologicamente são classificados como substantivos. b) todas as palavras destacadas são substantivos, embora não tenham a mesma classificação. c) ladra é substantivo abstrato, derivado de ladrar e nomeia uma ação. d) miséria, riqueza, vida e morte são os chamados substantivos abstratos; todos nomeiam estados, sendo que os dois primeiros também se caracterizam como qualidade. 20 – "No ensino, como em outras coisas, a liberdade deve ser questão de grau. Há liberdades que não podem ser toleradas. Uma vez conheci uma senhora que afirmava não se dever proibir coisa alguma a uma criança, pois deve desenvolver sua natureza de dentro para fora." (Russel, Bertrand) Quanto ao texto acima, podemos afirmar que possui estrutura a) narrativa, uma vez que relata as mudanças progressivas de estado que vão ocorrendo com as pessoas e as coisas através do tempo. b) descritiva, pois relata as características de uma pessoa, de um objeto ou de situação qualquer, inscritos num certo momento estático do tempo. c) dissertativa, porque nela predominam os conceitos abstratos, isto é, a referência ao mundo real se faz através de conceitos amplos, de exemplificação muitas vezes abstraídos do tempo e espaço. d) narrativa, porque nela a visão de mundo do enunciador é transmitida através de uma série de ações que ele atribui a personagens e que implicitamente formam um ponto de vista. 21 – As propagandas, em sua maioria, utilizam-se de linguagem conotativa como recurso estilístico de persuasão. Qual das alternativas abaixo não apresenta esse recurso persuasivo? a) Cerveja Bohemia (desde 1853): "Nosso rótulo é, ao mesmo tempo, atestado de qualidade e certidão de nascimento." b) Universidade Tuiuti do Paraná: "A Tuiuti escreve uma nova página na História da Educação do Brasil." c) Carro SEAT IBIZA: "Agrada tanto pés de chumbo quanto mãos de vaca." d) Whiskas: "Novo Whiskas Pedaços ao Molho. Seu gato não vai querer outra coisa." 22 – Assinale a alternativa em que a acentuação gráfica dos vocábulos esteja correta. a) "O saldo era exigúo, mas certo!" b) "Dentro em pouco havia azâfama pela casa, idas e vindas..." c) "... uma pequena multidão de neófitos, ainda na candidez das vestes..." d) "Alguns levêdos são patogênicos para o homem, porém outros são úteis." 23 – Leia o texto e faça o que se pede: "Maurício era um rapaz extremamente tímido, que sofria muito, pois era apaixonado por Lívia, mas não tinha coragem de revelar-lhe tal segredo. Por muito tempo, amargurou sua paixão, até que, um dia, preparou em sua casa uma grande festa, aproveitando a data de seu aniversário, e, enfim, diante de todos, confessou à moça seu grande amor." Quanto ao texto, é correto afirmar que a) apresenta um grande defeito que é o da prolixidade, uma vez que não há um motivo que justifique a mudança de comportamento da personagem. b) apresenta a chamada incoerência narrativa, visto que, na narração, uma ação posterior depende da anterior. No texto, há uma lacuna entre o que se apresenta no início e no fim. c) apresenta problemas de coesão, pois a conexão entre os enunciados não garante a relação de sentidos que deles se espera, por isso se diz que o texto é incoerente. d) apresenta ambigüidade, uma vez que a linguagem textual é usada como expediente para mostrar a dupla face do personagem. 24 – Assinale a alternativa que substitui correta/respectivamente as palavras destacadas. I - Os homens públicos nunca esquecem a compostura política. II - A médica procede à operação de emergência. III - Entreguei o bilhete ao diretor. IV - Por que você aspira a este cargo? a) a – a ela – lho – a ele c) a – lhe – lhes – a ele b) na – lhe – lho – lhe d) na – a ela – lho – a ele 25 – Assinale a alternativa em que, na produção textual, obser- vou-se a correção gramatical enquanto qualidade de estilo. a) "No trânsito, para passar mensagens, o outdoor é imbatível (...) Além do quê, ele é o último apelo ao consumidor antes da compra (...) Você pode utilizá-lo para cercar o Brasil inteiro..." b) "Harrison Ford interpretou tão bem o papel do aventureiro Indiana Jones que ficou complicado para o expectador separar o homem do ator." c) "Ouve-se com bastante freqüência grupos de cidadãos que exigem maior eficiência da polícia (...) como forma de ga- rantir a segurança dos indivíduos e seus patrimônios." d) "Há efetivamente um conjunto de brasileiros que se comportam como se as leis não lhes dissessem respeito. O convívio social não passa de uma forma de lhes satisfazer os desejos, as obrigações (...) pertencem exclusivamente aos outros." 26 – Aponte a alternativa em cujo texto aparece a figura de linguagem que consiste em se dizer o contrário do que se pensa, tal qual na frase "Comprou a mansão pela módica quantia de 2 milhões de dólares". a) "A pátria que quisera ter não era um mito; era um fantasma criado por ele no silêncio de seu gabinete. Nem a física, nem a moral, nem a intelectual, nem a política que julgava existir, havia." b) "Corriam no arraial rumores macabros. No dia seguinte ao enterramento o coveiro topou a sepultura remexida, como se fora violada durante a noite; e viu na terra fresca pegadas misteriosas de uma coisa que não seria bicho nem gente deste mundo." c) "Como ostentasse certa arrogância, não se distinguia bem se era uma criança, com fumos de homem, se um homem com ares de menino. Ao cabo, era um lindo garção, lindo e audaz..." d) "Excelente senhora, a patroa. Gorda, rica, dona do mundo, amimada dos padres, com lugar certo na igreja. (...) A excelente Dona Inácia era mestra na arte de judiar de crianças. Vinha da escravidão, fora senhora de escravos ÷ e daqueles ferozes..." 27 – Tomando por base o texto "Mas que significam as pala- vras? Que significam, na verdade, as palavras? Que significa a palavra verdade, a palavra mentira ou a palavra amor?", de Bernadete Lyra, é correto afirmar que a) a preocupação com o caráter intelectivo da significação das pala- vras é preocupação constante da literatura, portanto, é conotação. b) a plurissignificação de uma palavra caracteriza-se como objetiva e é válida para todos os falantes; portanto, caracte- riza-se como denotação. c) se abre, tanto para a conotação quanto para a denotação, um leque de possibilidades de significação; para aquela, de caráter arbitrário; para esta, de caráter translato. d) estamos tomando por base o subjetivo, o figurativo, a conotação, enfim, quando se diz que uma palavra (um signo) representa o objeto sem ser o objeto. 28 – Observe: I - Apresento-lhe Daniella. II - Faço tudo por um sorriso de Daniella. Juntando as duas orações num só período, usando um pronome relativo, teremos: a) Apresento-lhe Daniella, que pelo sorriso dela faço tudo. b) Apresento-lhe Daniella, por cujo sorriso dela faço tudo. c) Apresento-lhe Daniella, por cujo sorriso faço tudo. d) Apresento-lhe Daniella, a quem faço tudo pelo sorriso. 39 29 – Assinale a alternativa em que a não-observação da forma correta de se grafar um de seus termos gera impropriedade vocabular. a) A idéia de achar-se às voltas com a leitura de um livro vul- tuoso era-lhe fascinante. b) Estava cansado, exausto, abatido, mas o sacrifício valera a pena. Enfim, sentia que havia expiado sua culpa. c) O conserto da casa gera muitos sons: o bater de martelos, o ruidoso barulho da massa sendo preparada, a estridência de pedras sendo quebradas. Mas há, em tudo, uma confusa harmonia, gerando... música, talvez? d) O aluno andava acabrunhado. Havia se dedicado tanto ao trabalho, e o professor taxara-o de ingênuo na forma de argumentar. 30 – Observe o texto. "Mas esses letreiros luminosos não seriam muito mais belos se fossem escritos em chinês?" I - Todas as palavras do texto apresentam desinência nominal que indica o número plural. II - Apenas cinco palavras do texto apresentam a desinência nominal do plural. III - Em "fossem", há desinência verbal modo-temporal seguida da vogal de ligação. IV - Em "letreiros", a vogal temática "e" faz a ligação do radical "letr" ao sufixo "ros". Baseando-se nas afirmações quanto à estrutura das palavras, está(ão) correta(s) a(s) afirmação(ões) a) I e II. c) I e III. b) II apenas. d) IV apenas. 31 – Observe os textos e assinale a alternativa correta quanto aos processos de formação de palavras. I - "Onde estão meus verdes? Os meus azuis? O arranha-céu comeu!" (Mário Quintana) II - "Flor do Lácio sambódromo lusamérica latim em pó" (C. Veloso) III - "Sempre que o sol /pinta de anil todo céu / O girassol fica um gentil carrossel." (V. de Moraes) a) Verdes sofre derivação imprópria e por isso, quanto à forma, deixa de ser palavra primitiva. b) Em lusamérica há composição por justaposição, pois apenas um dos vocábulos sofre supressão e mantém-se a integridade sonora. c) Em girassol há composição por aglutinação, pois houve o acréscimo de uma letra, independente da manutenção da integridade fonética. d) Lusamérica caracteriza-se como composição por aglutina- ção, pois como há supressão em um dos vocábulos, há perda de integridade sonora. 32 – Observe os períodos abaixo: I - Mal o leão se afastou, o rato não teve a menor dúvida. II - "Os animais devem ser adestrados, ao passo que os seres humanos devem ser educados." III - Não obstante haja concluído um curso superior, é incapaz de redigir uma carta. IV - Pode criticar, desde que fundamente sua crítica em argumentos. As orações sublinhadas exprimem, respectivamente, circunstância de a) tempo, proporção, concessão e condição. b) causa, conformidade, condição e concessão. c) tempo, proporção, condição e concessão. d) condição, concessão, tempo e conseqüência. 33 – Observe: reza (Arnaldo Antunes) vida, minha dádiva, venha de manhã e dure a vida, minha dádiva, venha de manhã e dure a vida, minha dádiva, venha de manhã e dure hoje. A palavra vida, grifada acima, em relação ao 3º e 4º versos, pode ser classificada, respectivamente, como a) sujeito e vocativo. b) núcleo do adjunto adverbial de tempo e vocativo. c) aposto e sujeito. d) núcleo do adjunto adverbial de modo e aposto. 34 – Tendo agradado ao marido nas primeiras semanas de casado nunca mais quis ela se separar da receita daquele bolo. Assim, durante quarenta anos, a sobremesa louvada compôs sobre _____ mesa o almoço de domingo, e celebrou toda _____ data em que o júbilo se fizesse necessário. Por fim, achando ser chegada _____ hora, convocou ela o marido para o conciliábulo apartado no quarto. E tendo decidido ambos, comovidos, pelo ato solene, foi a esposa mais uma vez _____ cozinha assar a massa açucarada, confeitar a superfície. Pronto o bolo, saíram juntos para levá-lo ao tabelião, a fim de que se lavrasse ato de adoção, tornando-se ele legalmente incorporado _____ família, com direito ao prestigioso sobrenome Silva, e nome Hermógenes, que havia sido do avô. (Marina Colasanti) Assinale a alternativa que preenche corretamente as lacunas. a) a – a – a – à – à c) à – a – à – a – a b) à – à – a – à – à d) a – à – à – a – a 35 – "Sem cesta, sem-terra pedem esmola." (Folha o / nov. 2000) A concordância da frase acima está a) correta, pois ocorre o fenômeno da concordância ideológica, já que se trata de substantivo coletivo. b) incorreta, pois não foi observada a grafia correta para "sem- cesta", que passa a ser visto como advérbio; assim há na frase sujeito simples. c) incorreta, porque, como não há plural para o substantivo sem-terra, o uso do artigo é fundamental para a definição do singular ou plural. d) correta, pois o substantivo sem-terra é invariável, valendo tanto para o singular como para o plural. GABARITO DAS PROVAS DE PORTUGUÊS 1/2001 - TURMA A 01 B 02 A 03 B 04 C 05 A 06 B 07 C 08 A 09 C 10 A 11 D 12 A 13 C 14 B 15 A 16 D 17 B 18 D 19 B 20 D 21 C 22 D 23 ANULADA 24 D 25 C 1/2001 - TURMA B 01 C 02 A 03 B 04 D 05 A 06 C 07 A 08 D 09 B 10 D 11 B 12 A 13 D 14 C 15 A 16 C 17 D 18 B 19 A 20 C 21 A 22 B 23 C 24 B 25 D 2/2001 - TURMA A 01 C 02 D 03 A 04 D 05 B 06 A 07 D 08 B 09 C 10 B 11 A 12 B 13 C 14 A 15 C 16 D 17 B 18 A 19 D 20 A 21 C 22 B 23 A 24 D 25 C 2/2001 - TURMA B 01 A 02 C 03 B 04 C 05 B 06 A 07 D 08 A 09 C 10 B 11 D 12 A 13 C 14 B 15 C 16 B 17 D 18 A 19 D 20 A 21 B 22 C 23 D 24 B 25 D 1/2002 - TURMA A 01 A 02 B 03 A 04 C 05 D 06 A 07 A 08 B 09 D 10 D 11 A 12 A 13 B 14 A 15 C 16 A 17 D 18 D 19 B 20 A 21 B 22 A 23 C 24 A 25 B 26 D 27 C 28 B 29 C 30 C 31 D 32 C 33 A 34 C 35 A 36 B 37 C 38 D 39 D 40 D 1/2002 - TURMA B 12 01 A 02 B 03 C 04 A 05 A 06 ANULADA 07 A 08 B 09 A 10 D 11 C 12 A 13 A 14 D 15 D 16 C 17 C 18 C 19 A 20 A 21 B 22 B 23 A 24 C 25 D 26 B 27 ANULADA 28 D 29 D 30 B 31 A 32 C 33 A 34 C 35 B 36 B 37 A 38 C 39 D 40 D 2/2002 - TURMA A 01 A 02 B 03 A 04 B 05 A 06 B 07 A 08 B 09 B 10 A 11 B 12 C 13 D 14 C 15 A 16 D 17 C 18 C 19 D GABARITO DAS PROVAS DE PORTUGUÊS 20 D 21 D 22 B 23 A 24 C 25 A 26 D 27 A 28 C 29 D 30 C 31 C 32 B 33 C 34 A 35 D 36 B 37 B 38 D 39 D 40 C 2/2002 - TURMA B 01 C 02 B 03 A 04 B 05 B 06 B 07 C 08 C 09 A 10 D 11 C 12 B 13 A 14 A 15 D 16 D 17 D 18 C 19 C 20 B 21 D 22 B 23 A 24 B 25 C 26 D 27 A 28 A 29 D 30 B 31 D 32 D 33 B 34 C 35 D 36 C 37 A 38 A 39 C 40 C 1/2003 - TURMA A 01 B 02 D 03 D 04 A 05 C 06 C 07 C 08 A 09 B 10 D 11 D 12 B 13 B 14 B 15 C 16 A 17 B 18 A 19 D 20 C 21 B 22 D 23 D 24 B 25 A 26 C 27 D 28 A 29 D 30 D 31 B 32 C 33 A 34 A 35 B 36 C 37 C 38 C 39 A 40 A 1/2003 - TURMA B 01 C 02 A 03 C 04 A 05 B 06 C 07 D 08 A 09 A 10 D 11 B 12 C 13 D 14 B 15 A 16 A 17 D 18 A 19 A 20 D 21 D 22 C 23 C 24 C 25 D 26 A 27 C 28 A 29 D 30 B 31 B 32 B 33 C 34 B 35 C 36 B 37 B 38 D 39 A 40 D 2/2003 - TURMA A 01 B 02 C 03 B 04 C 05 C 06 D 07 D 08 C 09 D 10 C 11 A 12 B 13 C 14 D 15 B 16 A 17 B 18 B 19 A GABARITO DAS PROVAS DE PORTUGUÊS 20 B 21 A 22 C 23 A 24 C 25 D 26 D 27 D 28 A 29 D 30 D 31 A 32 B 33 B 34 A 35 D 2/2003 - TURMA B 01 D 02 A 03 A 04 C 05 B 06 A 07 A 08 ANULADA 09 B 10 B 11 C 12 C 13 C 14 A 15 D 16 B 17 ANULADA 18 D 19 D 20 C 21 D 22 C 23 B 24 A 25 A 26 D 27 B 28 C 29 A 30 B 31 D 32 A 33 B 34 A 35 D 1 ATEMÁTICA – EEAR 1/2001-TURMA A 01 – Numa cidade ―X‖, é consumido leite dos tipos: A e B. Dos consumidores consultados, 30 consomem dos tipos A e B, 100 somente do tipo A, 200 somente do tipo B e 40 nenhum dos dois tipos. Quantas pessoas foram consultadas? a) 300 b) 310 c) 330 d) 370 02 – Sabendo que ( ) x b a log 4 = ÷ e 16 1 b a = + , então ( ) log 4 2 2 a b ÷ é igual a: a) 2x b) 2 – x c) x – 2 d) 2 + x 03 – Em um triângulo retângulo a hipotenusa mede 5 cm e o C ˆ sen 2 1 B ˆ sen = . O maior cateto mede, em cm: a) 3 b) 5 c) 3 2 d) 5 2 04 – Se os ângulos internos de um triângulo estão em PA (progressão aritmética) e o menor deles é a metade do maior, então o valor do maior ângulo, em graus, é: a) 80 b) 90 c) 100 d) 120 05 – Resolvendo, em R, a equação 5 x 3 x 2 + = ÷ , obtemos o seguinte conjunto solução: a) { } 2 , 2 ÷ b) { } 8 , 2 ÷ c) ) ` ¹ ¹ ´ ¦ ÷ 2 , 3 2 d) ) ` ¹ ¹ ´ ¦ ÷ 8 , 3 2 06 – Resolvendo a equação (0,0625) x – 2 = 0,25 , obtemos ―x‖ igual a: a) 9 2 c) 2 5 b) 5 2 d) 2 9 07 – O conjunto imagem da função ( ) 5 3 x f x ÷ = é: a) ( ) ( ) { } 4 x f / R x f ÷ < e b) ( ) ( ) { } 4 x f / R x f ÷ > e c) ( ) ( ) { } 5 x f / R x f ÷ > e d) ( ) ( ) { } 5 x f / R x f ÷ > e 08 – Dois capitais, um de R$ 1.450,00 e outro de R$ 1.300,00, são colocados a juros simples de 6% e 7% ao ano, respectivamente. Em quanto tempo os montantes dos dois capitais serão iguais? a) 30 anos e 6 meses. b) 37 anos e 6 meses. c) 60 anos e 8 meses. d) 64 anos e 8 meses. 09 – Sobre o preço de um carro importado incide um imposto de 30%. Em função disso, o seu preço para o importador é de R$ 58.500,00. Supondo que tal imposto passe de 30% para 60%, qual será, em reais, o novo preço do carro para o importador? a) R$ 45.000,00 b) R$ 56.500,00 c) R$ 72.000,00 d) R$ 90.000,00 10 – Num triângulo isósceles de 54 cm de altura e 36 cm de base está inscrito um retângulo de 18 cm de altura, com base na base do triângulo. A base do retângulo mede, em cm: a) 23 b) 24 c) 25 d) 26 11 – As medidas dos lados de um triângulo são: AB = 28 cm , AC = 21 cm e BC = 35 cm. Uma paralela ao lado BC intercepta os lados AB e AC nos pontos D e E, respectivamente. Sabendo que o perímetro do trapézio BDEC mede 74 cm, então a base menor desse trapézio mede, em cm: a) 23 b) 24 c) 25 d) 26 12 – Se na figura, AB = AC e BC = CD = DA, então o valor do ângulo o, em graus, é: a) 30 b) 36 c) 45 d) 60 13 – Em um círculo de 3 cm de raio, a área e o perímetro de um setor circular de 60 o (sessenta graus) são, respectivamente, em cm 2 e cm: a) t 5 , 1 e ( ) 6 + t c) t e ( ) 6 + t b) t 5 , 1 e t d) t 6 e t 14 – De um ponto externo a uma circunferência, traçamos um segmento secante de 32 cm que determina uma corda de 27,5 cm. O segmento tangente traçado do mesmo ponto externo mede, em cm: a) 4,5 b) 12 c) 14,4 d) 55 2 15 – A expressão x 2 sen x 2 cos 1+ é identicamente igual a: a) cotg x b) sec x c) sen x d) tg x 16 – Duas cordas AB e CD de uma circunferência cortam-se num ponto M. Sabendo que AB = 21 cm, MB = 12 cm e CM = 3 DM, então CD, em cm, mede: a) 23 b) 24 c) 25 d) 26 M o C B A D 17 – Num círculo de centro C e raio R, tomam-se dois pontos A e B sobre a circunferência do círculo. Sendo o ângulo B C ˆ A = o e sabendo-se que o arco AB tem comprimento R, então pode-se afirmar: a) o 45 = o b) o 90 = o c) o o 50 45 < o < d) o o 60 55 < o < 18 – A secção meridiana de um cilindro equilátero tem 2 4 cm de diagonal. O volume do cilindro, em cm 3 , é de: a) 16 t b) 24 t c) 32 t d) 54 t 19 – ANULADA 20 – A base de um prisma quadrangular regular está inscrita numa circunferência cujo círculo tem 100 t cm 2 de área. Se a altura do prisma mede 1,5 cm, então o volume desse prisma, em cm 3 , é de: a) 200 b) 300 c) 400 d) 800 21 – Dada a equação 0 x 1 0 1 1 1 1 m x = ÷ ÷ , quais os valores de m para os quais as raízes são reais? a) 3 ms b) 1 m ÷ > c) 3 m 1 s s ÷ d) 1 m ÷ s ou 3 m> 22 – A altura de uma pirâmide quadrangular regular é igual à aresta de sua base. Sendo B a área da base da pirâmide, então sua área lateral, em cm 2 , é: a) 5 B b) 3 5 B c) 3 B d) B 5 23 – A equação da reta que passa pelo ponto ( ) 5 , 4 B ÷ e de coeficiente angular 2 1 é: a) 0 6 y 2 x = + ÷ b) 0 12 y 2 x = ÷ ÷ c) 0 14 y 2 x = ÷ ÷ d) 0 14 y 2 x = + + 24 – O valor de k de modo que a reta kx + 2y + k – 8 = 0 passe pela intersecção das retas 0 y x = + e 8 y 3 x = ÷ é: a) 4 b) 3 c) – 4 d) – 3 25 – Para que o polinômio 6 nx mx x 2 2 3 + + + seja divisível por ( ) 1 x + e ( ) 3 x ÷ , então os valores de m e n devem ser, respectiva-mente, iguais a: a) – 7 e – 3. b) – 6 e – 10. c) – 6 e – 2. d) – 2 e – 6. 1/2001-TURMA B 01 – Considere os conjuntos A = [1 , 2] [3 , 4] ; B = ]1 , 4] – {3} ; C = [2 , 3[ {4} e X = (A – B) (A ·C). Assinale a alternativa correta: a) X A = B b) X C = X c) X · A = X d) X · B = C 02 – Considere as afirmativas: I. Numa divisão, cujo resto ―r‖ não é nulo, o menor número que se deve adicionar ao dividendo para que ela se torne exata é ―d – r‖, sendo ―d‖ o divisor. II. A soma de três números naturais consecutivos é sempre divisível por 3. III. O produto de dois números ímpares consecutivos, aumentado de uma unidade é sempre um quadrado perfeito. São verdadeiras as afirmativas: a) I e II. b) I e III. c) II e III. d) I, II e III. 03 – Sejam os conjuntos A = [ – 1 , 2 ], B = [ – 2 , 4 ] e C = [ – 5 , 0 [ . É falso afirmar que: a) ( B – C ) – A = [ 2 , 4 ] b) ( A  B )  ( B – C ) = [ 0 , 2 ] c) ( B – A )  ( A  B ) = [ – 2 , 4 ] d) ( B  C ) – ( A  B ) = ] – 5 , – 1 [  ] 2 , 4 ] 04 ÷ O menor valor real e positivo de x tal que 2 1 4 x sen = é: a) 6 t b) 6 5t c) 6 7t d) 6 11t 05 – A respeito do conjunto solução da equação 2 3 3 x x x 3 x 2 2 = + ÷ + , em * 9 , pode-se afirmar que: a) tem um elemento igual a zero. b) tem dois elementos. c) é unitário. d) é vazio. 06 ÷ O valor de mercado de um automóvel é alterado a cada mês com um acréscimo de 1% em relação ao mês anterior. A seqüência de valores do automóvel, a cada mês, forma uma progressão: a) aritmética de razão 0,1. b) aritmética de razão 0,01. c) geométrica de razão 1,1. d) geométrica de razão 1,01. 07 – Na seqüência (1, 1, 2, 3, . . .) , onde 1 n n 1 n a a a ÷ + + = , o nono termo é: a) 34 c) 43 b) 21 d) 28 08 – Se o raio de um círculo for aumentado de 100%, sua área aumentará de: a) 100% c) 300% b) 200% d) 400% 09 – Um aluno acertou 4 das 15 primeiras questões de um prova. Das restantes, ele acertou os 5 4 e, ao final, verificou que respondera corretamente 60% das questões. O número de questões que esse aluno acertou foi: a) 24 b) 23 c) 17 d) 16 10 – Se a < – 2, os valores de x, tais que ( ) ( ) 2 x a x 2 a + ÷ < ÷ , são aqueles que satisfazem: a) x < 2 – a b) x < a – 2 c) x > 2 – a d) x > a – 2 11 – A soma das medidas dos ângulos internos e externos de um polígono convexo é 3600º. O número de diagonais desse polígono é um número: a) par divisível por 15. b) par maior que 150. c) ímpar múltiplo de 19. d) ímpar primo. 12 – Classifique como verdadeira ou falsa cada uma das afirmativas: 1.ª Um triângulo obtusângulo pode ser isósceles. 2.ª Um triângulo isósceles pode ser retângulo. 3.ª Um triângulo isósceles não pode ser equilátero. Assinale a alternativa correta: a) Todas são falsas. b) Todas são verdadeiras. c) A 2.ª é verdadeira e a 3.ª é falsa. d) A 1.ª é falsa e a 3.ª é verdadeira. 13 – Sejam P, Q e R pontos de uma circunferência de centro O, tais que P e Q estejam do mesmo lado em relação ao diâmetro que passa por R. Sabendo-se que º 80 ) Q O ˆ R ( med e º 10 ) P R ˆ O ( med = = , tem-se que o ângulo O Q ˆ P mede: a) 20º b) 40º c) 50º d) 60º 14 – Um segmento AB, de 6 metros, é diâmetro de uma circunferência de centro O. Sendo C um ponto dessa circunferência, tal que a medida do ângulo C B ˆ A seja 30º, a medida da superfície limitada pelas cordas AB e BC e pelo arco AC , em metros quadrados, é: ( ) ( ) 2 3 9 2 3 9 3 2 3 3 3 2 4 3 ) ) ) ) d c b a t t t + + 15 – Um círculo de raio r e um retângulo de base b são equivalentes. Então, a altura do retângulo é: 2 2 2 2 b r d b r c b r b r a t t t t ) ) ) ) 16 – Se tg x = – 3, então tg 4x é igual a: 7 24 4 3 7 24 4 3 ) ) ) ) d c b a ÷ ÷ 17 – Em t s s 2 x 0 , a expressão x cotg x cos tgx x sen y + + = é tal, que: a) y > 0 somente se 0 < x < 2 t . b) y < 0 se x 2 k t = (k eZ). c) y > 0 se x 2 k t = (k eZ). d) y < 0 somente se t < x < 2 3t . 18 – Se )! 1 n ( ) 1 n ( ! n a 2 n + ÷ = , então a 1985 é igual a: a) 1984 b) 1983 c) 1 1984 1985 2 ÷ d) 1984 1 1984 2 ÷ 19 – Como parte de seu treinamento, um piloto realizou 10 missões cujos tempos em minutos são, em ordem: 4 - 6 - 7 - 9 - x - 14 - 18 - y - 23 - 26. Sabendo-se que o tempo médio das missões foi de 14 minutos e o tempo mediano foi de 13 minutos, podemos afirmar que x e y valem, respectivamente: a) 13 e 20. b) 12 e 21. c) 13 e 21. d) 12 e 22. 20 – Numa comunidade residem 120 pessoas. Uma pesquisa sobre os hábitos alimentares dessa comunidade revelou que 42 pessoas consomem carnes, 90 consomem verduras e 30 consomem carnes e verduras. Escolhendo-se ao acaso uma pessoa desta comunidade, a probabilidade de ela ter o hábito de não comer carnes nem verduras é: a) 7,5% c) 12,5% b) 10,0% d) 15% 21 – Um plano secciona uma esfera, determinando um círculo de raio igual à distância do plano ao centro da esfera. Sendo 36 t cm 2 a área do círculo, o volume da esfera, em 3 cm , é: a) t 2 288 c) 288t b) t 2 576 d) 576t 22 – As retas 2x – y = 3 e 2x + ay = 5 são paralelas. Então, o valor de a é: a) –1 b) 1 c) ÷ 4 d) 4 23 – Um cilindro circular reto tem o volume igual ao de um cubo de aresta ―a” e a área lateral igual à área total do cubo. O raio e a altura desse cilindro medem, respectivamente: a) t t t t a e a d a e a c a e a b a e a 9 3 3 2 9 3 3 2 ) ) ) 24 – A reta de equação x + 2y + c = 0 : a) é perpendicular à reta 2x + y + c = 0. b) é paralela à reta 2x – 4y + c = 0. c) tem distância ao ponto (- c , 1) igual a zero. d) forma um ângulo de 4 t rd com a reta 3x + y + c = 0. 25 – Considere as afirmações: I. Qualquer raiz racional da equação 0 9 x 3 x 3 x 2 3 = + ÷ + é inteira. II. O menor grau da equação polinomial de coeficientes reais, que admite as raízes 3, 2 + i e i ÷ , é 5. III. Toda equação polinomial da forma ax 4 + bx 3 + cx 2 + dx + e = 0 de coeficientes reais e a = 0, necessariamente possui uma raiz real. São verdadeiras as afirmações: a) I, II e III. b) I e II. c) II e III. d) I e III. 2/2001-TURMA A 01 – Seja o número complexo i 1 i i 1 1 0 i 1 Z 3 ÷ ÷ = . A forma trigonométrica de Z é a) | . | \ | t + t 4 7 sen i 4 7 cos 2 b) | . | \ | t + t 4 sen i 4 cos 2 c) | . | \ | t + t 3 sen i 3 cos 2 d) | . | \ | t + t 4 7 sen i 4 7 cos 2 02 – Na figura, cm 2 AD = e cm 4 AB = . O valor de o cos no triângulo ABC é a) 2 1 b) 3 3 c) 2 3 d) 2 3 ÷ 03 – Ao seccionar uma esfera, um plano determina um círculo de raio 16 cm. Se a distância do plano ao centro da esfera é de 12 cm, então o raio da esfera, em cm, vale a) 20 b) 28 c) 30 d) 38 04 – Na resolução da equação matricial | | | . | \ | = | | | . | \ | | | | . | \ | ÷ ÷ ÷ 0 2 1 z y x 0 3 0 1 1 4 0 1 1 , o valor de z y x + + é a) – 2 b) 1 c) – 1 d) 0 05 – No trapézio escaleno abaixo, tem-se: AD = 5 cm, o 30 C D ˆ B = e o 45 D A ˆ B = . Nessas condições, a medida da diagonal BD, em cm, é a) 2 5 b) 3 5 c) 5 5 d) 5 06 – A equação da reta que passa pelo ponto ( ) 2 , 3 e pelo ponto de interseção das retas ( ) x 1 3 y ÷ = e ( ) 1 x 2 y ÷ = é: a) 0 1 y x 2 = ÷ ÷ b) 0 1 y 2 x = ÷ ÷ c) 0 1 y 2 x 2 = ÷ ÷ d) 0 1 y x = ÷ ÷ 07 – Numere a segunda coluna de acordo com a primeira, sendo 2 0 t < < x : ( 1 ) ( ) x 2 sen 2 ( ) x cos 2 2 ( 2 ) ( ) x 2 cos 1 + ( ) x cos x sen 2 · ( 3 ) ( ) x 2 sen ( ) ( ) x 2 cos 1 2 ÷ ( 4 ) ) x 2 sen( ÷ t ( ) x sen ÷ ( 5 ) ( ) x sen ÷ ( ) cos x Lendo-se a segunda coluna de cima para baixo, a seqüência correta é: a) 1, 3, 4, 5, 2 b) 2, 3, 1, 5, 4 c) 3, 1, 2, 4, 5 d) 2, 3, 5, 1, 4 08 – Efetuando ( ) 0 5 2 5 4 4 3 5 1 3 2 ÷ ÷ · ÷ | | | . | \ | ÷ , obtemos a) 10 b) 12 c) 4 d) –12 09 – Seja ABCD um trapézio isósceles. Sabe-se que a medida de um de seus ângulos obtusos internos é o dobro da medida de um de seus ângulos agudos internos, e que a diagonal AC é perpendicular ao lado BC. Se a base maior mede 10 cm, então o perímetro desse trapézio, em cm, é a) 20 b) 25 c) 28 d) 30 10 – O valor inteiro de x, tal que o dobro do seu logarítimo decimal tenha uma unidade a mais do que o logarítimo decimal de | . | \ | + 10 11 x , é a) 1 ÷ b) 1,7 c) 10 d) 11 11 – Se em um triângulo retângulo um dos catetos mede 5 2 cm e a altura relativa à hipotenusa mede 2 cm, então a área desse triângulo, em cm 2 , é a) 3 10 b) 3 20 c) 3 2 10 d) 10 2 12 – Numa progressão geométrica de 6 termos positivos, a soma de a2 e a4 é 6, e a soma de a4 e a6 é 12. A razão dessa P.G. é a) 2 b) 2 c) 2 ÷ d) – 2 o - A B C D A B C D 13 – Uma pessoa aplicou 6 1 do seu capital a 25% a.a. e o restante a 15% a.a.. No fim de 4 anos a soma dos juros simples atingiu R$ 405,00. O capital aplicado foi de R$ a) 607,50 b) 670,50 c) 607,05 d) 670,05 14 – O custo de um par de sapatos é igual ao custo de um terno. Um lojista vende o par de sapatos com prejuízo de 5% sobre o custo, e o terno com 30% de lucro sobre o custo, recebendo pelos dois R$ 180,00. O preço de venda do terno, em reais, é a) 76 b) 80 c) 104 d) 110 15 – Uma das raízes da equação 0 48 x 44 x 12 x 2 3 = ÷ + ÷ é a soma das outras duas. A maior raiz dessa equação é a) 7 b) 6 c) 4 d) 2 16 – A circunferência ( ) ( ) 1 1 y 2 x 2 2 = ÷ + + e a reta 0 2 y 3 x = ÷ ÷ possuem __________ ponto(s) em comum. a) 2 b) 1 c) infinitos d) nenhum 17 – Se a área da coroa circular definida por dois círculos concêntricos de raios r e R, R r < , é igual a área do círculo menor, então a razão r R é igual a a) 1 b) 2 c) 2 2 d) 2 2 18 – Se o gráfico representativo de uma função do 2 o grau é uma parábola, então a parábola que passa pelo ponto ( ) 0 , 2 ÷ , e cujo vértice situa-se no ponto ( 1 , 3 ), representa a função a) ( ) 8 x 2 x x f 2 + + ÷ = b) ( ) 24 x 6 x 3 x f 2 + + ÷ = c) ( ) 3 8 3 x 2 3 x x f 2 + + ÷ = d) ( ) 8 x 2 x x f 2 + + = 19 – O total de números múltiplos de três, de quatro algarismos distintos, que podem ser formados com os algarismos 2, 3, 6, 7 e 9 é a) 120 b) 72 c) 48 d) 24 20 – Uma escola de ensino médio tem 250 alunos que estão matriculados na 1 a , 2 a e 3 a séries. Dos alunos matriculados, 32% são homens e 40% dos homens estão na 1 a série; 20% dos alunos matriculados estão na 3 a série, sendo 10 alunos homens; e na 2 a série, o número de mulheres é igual ao número de homens. O número de mulheres na 1 a série é a) 10 b) 48 c) 92 d) 102 21 – Em Estatística, ______________ de um conjunto de dados dispostos em ordem crescente, onde o número de dados é ímpar, é o valor que ocupa a posição central; e ______________ de um conjunto de dados é o valor mais frequente do conjunto. a) Mediana; Moda b) Moda; Mediana c) Média; Moda d) Mediana; Média 22 – Dados os números racionais 20 10 03 , 0 m ÷ · = , 21 10 3 , 0 k ÷ · = e 22 10 300 p ÷ · = , é correto afirmar que a) p k m < < b) p k m > = c) p m k < < d) p k m < = 23 – As bases de uma pirâmide hexagonal regular e de um prisma quadrangular regular acham-se inscritas num mesmo círculo. Sendo H a altura da pirâmide e sabendo-se que os dois poliedros são equivalentes, então a altura do prisma é a) 4 3 H b) 4 3 H 3 c) 2 3 H d) 3 3 H 24 – Sendo x 3 x 4 8 = ÷ , tem-se que ( ) 1 3 x log ÷ é igual a a) 3 b) 2 c) –2 d) –1 25 – Numa progressão aritmética, o primeiro termo é 10x – 9y, o último termo é y, e a razão é y – x. Sendo x = y, o número de termos dessa P.A. é a) 8 b) 9 c) 10 d) 11 2/2001-TURMA B 01 – Os conjuntos S, T e P são tais que todo elemento de S é elemento de T ou P. O diagrama que pode representar esses conjuntos é a) c) b) d) 02 – Se x e y são números reais que tornam simultaneamente verdadeiras as sentenças 30 2 2 y x = ÷ + e 0 2 2 y x = ÷ ÷ , então y x é igual a a) 9 b) 8 c) 8 1 d) 9 1 T S P S T P T S P T P S 03 – Seja k a raiz da equação 2 1 2 x log log 2 8 = . O valor de k 8 é a) 8 1 c) 1 b) 4 1 d) 2 04 – Tanto numa P.A. quanto numa P.G., os números 3 e 243 são, respectivamente, a razão e o 6.º termo. O produto do 1.º termo da P.G. pelo 3.º termo da P.A. é a) 702 b) 693 c) 234 d) 231 05 – O conjunto solução da inequação 1 0 x 0 1 x x 1 2 > 0 é dado por: a) ] 0 , 2 [ b) ] -2 , 1 [ c) ] -2 , 1 [ ] 1 , 2 [ d) ] -1 , 0 [ ] 1 , 2 [ 06 – Num cone circular reto, cujo raio da base mede r, a base é equivalente à secção meridiana. A altura desse cone mede g ) d r ) c g r ) b rg ) a t t t t 07 – Seja q p a forma irredutível do resultado da expressão ... 2363636 , 1 2 1 1 4 1 4 2 1 1 4 3 2 + ÷ + . O valor de p – q é a) 78 b) 98 c) 324 d) 524 08 – Seja log 2 = 0,301. Efetuando-se 50 50 , obtemos um valor cuja quantidade de algarismos é a) 85 b) 84 c) 83 d) 82 09 – O valor máximo da função definida em 9 por * 2 m , m x 6 mx ) x ( f 9 e + + = é igual a 8. Então o valor de m é a) 9 b) 8 c) – 1 d) – 3 10 – Um pai deseja repartir a quantia de R$2.600,00 entre seus quatro filhos, de modo que as partes sejam proporcionais às suas idades e formem uma P.A. Se a idade do filho mais jovem é 8 anos e a do mais velho é 44, a quantia dada ao filho mais jovem será, em reais, a) 200 b) 250 c) 300 d) 350 11 – Um capital cresce sucessiva e cumulativamente, na base de 10% ao ano. Ao final de 3 anos, o montante, comparado ao capital inicial, será a) 30% superior. b) 130% do capital. c) aproximadamente 150% do capital. d) aproximadamente 133% do capital. 12 – De um pedaço quadrado de metal corta-se uma peça circular de diâmetro máximo, e desta corta-se outro quadrado de lado máximo. O material desperdiçado tem . circular peça da área da 4 1 ) d . circular peça da área da 2 1 ) c . original quadrado do área da 2 1 ) b . original quadrado do área da 4 1 ) a 13 – O sistema linear ¦ ¹ ¦ ´ ¦ = + = + = + 0 mz y 0 z y 0 y x é indeterminado para a) nenhum m real. b) todo m real. c) m = 0 d) m = 1 14 – Um trem de passageiros é constituído de uma locomotiva e 7 vagões distintos, sendo um deles restaurante. Sabendo que a locomotiva deve ir à frente e que o vagão restaurante não pode ser colocado imediatamente após a locomotiva, o número de modos diferentes de montar a composição é: a) 720 b) 4.320 c) 5.040 d) 30.240 15 – O triângulo cujos vértices são os pontos ( 1 , 3 ), ( - 2 , - 2 ) e ( 1 , - 2 ) é a) obtusângulo. b) equilátero. c) retângulo. d) isósceles. 16 – Os resultados da prova de Ciências aplicada a uma turma de um certo colégio estão apresentados no gráfico. Baseado neste gráfico, podemos afirmar que a porcentagem de alunos dessa turma com nota inferior a 5,0, nessa prova de Ciências, foi de a) 37,5% b) 42,5% c) 47,5% d) 52,5% 17 – No sistema de coordenadas cartesianas, a equação by ax y x 2 2 + = + , onde a e b são números reais não nulos, representa uma circunferência de raio a) 2 b a 2 2 + b) 2 2 b a + c) 2 b a + d) a + b 0 2 4 6 8 10 12 14 1 2 3 4 5 6 7 8 9 notas n ú m e r o d e a l u n o s 18 – Seja m e9. Para que o produto ( 2 + m i ) . ( 3 + i ) seja um número imaginário puro, o valor de m deve ser a) 5 b) 6 c) 7 d) 8 19 – Supondo definida em 9 a fração 1 a 1 a . a a . a a . a 2 ÷ + ÷ + , o seu valor é a) 1 a + b) a + 1 c) a – 1 d) a 20 – Sejam p(x) = x 2 – 5x + 6 e q(x) = x 2 – 3x + 1. Se a é um número real e p(a) < 0, então q(a) satisfaz a) –1 < q(a) < 1 b) q(a) < -1 ou q(a) > 1 c) –2 < q(a) < 2 d) q(a) < -2 ou q(a) > 2 21 – Se A = { xe9 / 3x - 2x 2 > 0 }, B = { xe9 / 1 s x s 3 } e C = { xe9 / x 2 – x – 2 s 0 }, então ( A B ) · C é a) { xe9 / -1 s x s 0 ou 1 s x s 2 } b) { xe9 / -1 s x s 0 ou 2 3 s x s 2 } c) { xe9 / -1 s x s 2 } d) { xe9 / 0 s x s 2 } 22 – Mílton comprou um carro 0km e, pensando em economizar os pneus, usou os quatro colocados mais o estepe, numa viagem cujo percurso foi de 2.000 km. Se cada pneu rodou a mesma quilometragem, então o estepe foi usado nessa viagem por e) 2.000 km f) 1.600 km g) 1.200 km h) 800 km 23 – Os valores reais de a e b tais que os polinômios: P(x) = x 3 – 2ax 2 + (3a + b)x – 3b e Q(x) = x 3 – (a + 2b)x + 2a sejam divisíveis por x + 1 são dois números a) inteiros positivos. b) inteiros negativos. c) reais, sendo um racional e outro irracional. d) inteiros, sendo um positivo e outro negativo. 24 – Considere as circunferências que passam pelos pontos (0 , 0) e (2 , 0) e que são tangentes à reta y = x + 2 as coordenadas dos centros dessas circunferências são a) (1 , 1) e (1 , -7) b) (1 , 1) e (-7 , 1) c) (1 , -7) e (1 , 7) d) (1 , -7) e (-1 , 7) 25 – Dois lados consecutivos de um paralelogramo medem 8 m e 12 m e formam entre si um ângulo de 60º. As medidas das diagonais desse paralelogramo são tais que o número que expressa a) o seu produto é racional. b) a sua razão é maior que 2. c) a sua soma é maior que 32. d) a sua diferença é irracional. 1/2002-TURMA A 01 – O domínio da função real ( ) 2 x 4 3 x x f ÷ + = é a) ) ` ¹ ¹ ´ ¦ = ÷ > 9 e 2 1 x e 3 x / x . b) ) ` ¹ ¹ ´ ¦ ÷ = > 9 e 2 1 x e 3 x / x . c) ) ` ¹ ¹ ´ ¦ = ÷ > 9 e 2 1 x e 3 x / x . d) ) ` ¹ ¹ ´ ¦ ÷ = > 9 e 2 1 x e 3 x / x . 02 – Determinando 008 , 0 log 25 , obtemos a) 2 3 . b) 2 3 ÷ . c) 3 2 . d) 3 2 ÷ . 03 – Na figura, sendo cm x MN = , cm 10 NP = , cm 5 PO= e cm ) 1 x 4 ( OQ + = , então o valor do segmento de reta PQ, em cm, é a) 29. b) 35. c) 12. d) 34. 04 – Pode-se afirmar que o valor do determinante 1 0 a 2 x x 2 0 1 0 x a ÷ ÷ ÷ é igual a a) 2 x 2 ÷ . c) ( ) 2 x x ÷ . b) x 2 x 2 + . d) ( ) 2 a 2 x x ÷ ÷ . 05 – Das afirmações: I. x sen 1 x cos ÷ = II. x cos x sen 2 x 2 sen = III. x sen x cos x 2 cos 2 2 ÷ = Sendo x um arco no ciclo trigonométrico compreendido entre t t e 2 , conclui-se que a) a única falsa é a I. c) a única falsa é a III. b) a única falsa é a II. d) as três são verdadeiras. 06 – Sendo o um ângulo agudo, o lado ―x‖ do triângulo abaixo, em cm, mede a) 6. b) 10. c) 12. d) 15. 07 – Resolvendo a equação 256 2 1 2 x 2 2 = + , concluímos que ela a) não admite soluções reais. b) admite 2 3 como raiz. c) admite duas soluções reais positivas. d) admite duas soluções cuja soma é zero. M N P O Q 6 cm x 6 3 cm 30 o o 08 – Toda vez que ocorrer f e d c b a = = , também ocorrerá a) d c a b c a + = + . c) 2 2 b a f d b e c a = + + + + . b) a d c c b a ÷ = ÷ . d) 3 3 b a f . d . b e . c . a = . 09 – Se a 4 3 x t g = e 2 3 x t < < t , então o valor de x sen x cos ÷ é igual a a) 5 7 . b) 5 7 ÷ . c) 5 1 ÷ . d) 5 1 . 10 – A geratriz de um cilindro de revolução mede 10 cm. Qual o seu raio da base, sabendo-se que, aumentando-se esse raio em 10 cm e mantendo-se a altura, a área lateral do novo cilindro é igual à área total do primeiro? a) 2,5 cm b) cm 2 5 c) 10 cm d) 20 cm 11 – Uma firma contratou o trabalho de um pintor na base de R$ 45,00 por dia. Sabe-se que ele trabalhou durante 32 dias, e do total a lhe ser pago foi descontado 8% para o Imposto de Renda e 10% para o INSS. A quantia líquida que ele recebeu foi R$ a) 1.180,80. c) 1.100,00. b) 1.200,00. d) 1.250,00. 12 – Na figura, BN é a bissetriz do ângulo B ˆ . Se o 50 A ˆ = e o 30 C ˆ = , então a medida x do ângulo N B ˆ H é a) 5º. b) 10°. c) 15º. d) 20º. 13 – Resolvendo a equação 2 3 x sen = , onde t s s 2 x 0 , obtemos como conjunto solução a) { } Z K , 360 K 120 x ou 360 K 60 x / R x o o o o e + = + = e b) { } Z K , 360 K 30 x / R x o o e + = e . c) { } Z K , 360 K 240 x ou 360 K 60 x / R x o o o o e + = + = e d) { } Z K , 360 K 30 x o o e + = . 14 – Na figura, MNPQ é um losango. Se cm 12 MT = e cm 6 MS= , então o lado do losango, em cm, mede a) 2. b) 4. c) 8. d) 12. 15 – Os números ( ) o 10 x 2 + , x 3 , ( ) o 20 x 3 ÷ são medidas em graus dos ângulos de um triângulo. Esse triângulo pode ser classificado em a) acutângulo. c) retângulo. b) equiângulo. d) obtusângulo. 16 – A parábola de equação c bx x 2 y 2 + + ÷ = passa pelo ponto ( ) 0 , 1 e seu vértice é o ponto de coordenadas ( ) v , 3 . A coordenada v é igual a a) –28. b) 28. c) –8. d) 8. 17 – Os valores reais de x do sistema 2 1 x 1 < ÷ < são a) 3 x 2 ou 0 x 1 < < < < ÷ . c) 2 x ou 0 x > < . b) 2 x 0 < < . d) 2 x 1 < < ÷ . 18 – É falso afirmar: a) Se B O ˆ A é um ângulo raso, então OA e OB são semi- retas opostas. b) Se B O ˆ A é um ângulo nulo, então OA e OB são semi- retas opostas. c) Dois ângulos adjacentes, cujos lados não comuns são semi- retas opostas, somam 180 o . d) Dois ângulos adjacentes são sempre consecutivos. 19 – Se a diferença entre os quadrados das raízes da equação 0 k x 2 x 2 = + ÷ é 8, então o valor de ―k‖ é a) 5. c) 3. b) –5. d) –3. 20 – A soma dos ângulos internos e dos ângulos externos de um polígono regular vale o 800 . 1 . O número de diagonais desse polígono é a) 25. b) 35. c) 45. d) 55. 21 – A expressão 2 , 1 2 3 1 3 333 , 3 5 3 2 11 × | . | \ | ÷ + ÷  , tem como resultado a) 4 1 6 . b) 4 1 7 . c) 4 1 8 . d) 4 1 9 . 22 – A área lateral de um prisma hexagonal regular de 25 cm de altura e de apótema da base igual a 3 2 cm, em cm 2 , é a) 1.200. b) 2 600 . c) 3 600 . d) 600. 23 – Dado o hexágono regular ABCDEF, a área do quadrilátero ABCD, em cm 2 , sabendo-se que AB mede 6 cm, é a) 54. b) 3 54 . c) 3 18 . d) 3 27 . 24 – Leia com atenção. I. Os possíveis valores de x para os quais se tenha 12 x = são –12 e 12. II. {–3, –2, –1, 0, 1, 2, 3} = {x e 9 / –3 s x s 3}. III. Os números inteiros que verificam a desigualdade 2 x s é o conjunto {–2, –1, 0, 1, 2}. IV. Os valores reais de x que verificam a desigualdade 1 x s é o conjunto {x e 9 / –1 s x s 1}. Com relação às afirmações acima, podemos dizer que a) I, II, III e IV são verdadeiras. b) I e II são verdadeiras. c) I e II são falsas. d) I, III e IV são verdadeiras. A B C D E F T P S N M Q - A B C H N x 25 – Um capital, aplicado a juros simples, duplicará em 4 anos se a taxa anual for de a) 75%. b) 50%. c) 25%. d) 15%. 26 – Numa aplicação de R$ 10.000,00 ao juro mensal de 2%, creditado mensalmente na conta do aplicador, o montante no fim do 3 o mês será de R$ a) 10.200,00. c) 10.404,00. b) 10.400,00. d) 10.612,08. 27 – A soma dos vinte primeiros termos da PA cujo termo geral tem para expressão 5 n 3 a n + = é a) 657. c) 803. b) 730. d) 1460. 28 – O volume, em cm 3 , de uma pirâmide quadrangular regular cujas faces laterais são triângulos eqüiláteros de lado 4 cm, vale a) 2 16 . c) 3 2 16 . b) 2 32 . d) 3 2 32 . 29 – O valor de m, para que o módulo do número complexo ( )( ) i 1 i 2 m Z + + = seja igual a 4, é a) 1 ± . b) 2 ± . c) 3 ± . d) zero. 30 – Duas réguas de madeira, AB e CD, com 8 cm cada uma estão ligadas em suas extremidades por dois fios, formando o retângulo ABCD (fig. 1). Mantendo-se fixa a régua AB e girando- se 180 o a régua CD em torno do seu ponto médio, sem alterar os comprimentos dos fios, obtêm-se dois triângulos congruentes AIB e CID (fig.2). A distância, em cm, entre as duas réguas, nessa nova posição (fig.2) é a) 3 5 . b) 2 5 . c) 5. d) 6. 31 – A área, em cm 2 , de um triângulo eqüilátero inscrito numa circunferência cujo comprimento é de 3 8t cm é a) 3 36 . c) 3 72 . b) 3 64 . d) 3 144 . 32 – A inequação 2 1 2 x sen > , onde t s s 2 x 0 , é verdadeira se, e somente se a) t s s t 2 x 6 . c) 6 5 x 6 t s s t . b) 3 5 x 3 t s s t . d) 3 x 6 t s s t . 33 – Considere a equação 0 10 x 13 x 6 x 2 3 = + + + em que –2 é uma das raízes. As demais raízes são a) i 2 + ÷ e i 2 ÷ ÷ . c) i 2 ÷ e i 2 + . b) 1 ÷ e –5. d) i 2 2 + ÷ e i 2 2 ÷ ÷ . 34 – Dado um quadrado de diagonal igual 2 cm. Sobre cada lado do quadrado se constrói externamente um triângulo equilátero de lado igual ao do quadrado. A área da figura toda, assim obtida, é .................... cm 2 . a) 3 2 c) 3 2 1 + b) 3 1 + d) 3 4 2 + 35 – Na figura, M e N são pontos de tangência. Sendo os raios, respectivamente, 14 cm e 7 cm e a distância dos centros cm 24 OO 1 = , então o segmento MN, em cm, mede a) 527 . b) 380 . c) 15 3 . d) 12. 36 – Seja uma circunferência com centro sobre a reta x 3 y = . Se a circunferência é tangente à reta x y = na ordenada 4, então as coordenadas do centro da circunferência são a) (4, 12). c) (3, 9). b) (2, 6). d) (5, 15). 37 – Dentre os pontos que equidistam de A(1, 2) e B(3, 4), o ponto mais próximo de P(6, 1) que pertence ao eixo das abscis- sas é a) 5. c) 6. b) 3. d) 4. 38 – A soma dos termos de uma PG crescente de três termos positivos é 21 e a diferença entre os extremos, 15. A razão dessa PG é a) 4. b) 5. c) 6. d) 7. 39 – Uma classe tem 10 meninos e 12 meninas. A quantidade de maneiras que poderá ser escolhida uma comissão de três meninos e quatro meninas, incluindo obrigatoriamente o melhor aluno e a melhor aluna, é dada pelo(a) a) soma de 36 e 165. c) produto de 120 e 495. b) soma de 120 e 495. d) produto de 36 e 165. 40 – A expressão a a 2 a 2 a 4 4 8 4 8 ÷ ÷ é equivalente a a) a 4 2 1÷ . c) a 2 2 2 3 · . b) ( ) 1 2 2 a 4 a 2 + . d) ( ) 1 2 2 a 4 a 4 + . 1/2002-TURMA B 01 – Se 6 2 99 a = , 7 3 99 b = e 8 4 99 c = , então ( ) 12 abc é igual a a) 12 99 c) 28 99 b) 2 21 99 d) 88 99 02 – Se x, y, e z são números naturais diferentes entre si, e z y x · = , então é falsa a afirmativa: a) x é múltiplo de z. c) y é divisível por z. b) y é divisor de x. d) x é divisível por y. 03 – Duas retas paralelas são cortadas por uma transversal, de modo que a soma dos ângulos agudos formados vale 144º. Então a diferença entre as medidas de um ângulo obtuso e de um agudo é a) 85º c) 108º b) 92º d) 116º 04 – A razão entre os comprimentos das circunferências circunscrita a um quadrado e inscrita no mesmo quadrado é a) 2 c) 2 3 b) 2 d) 2 2 05 – Uma equação do 3.º grau cujas raízes são –1, –2 e 3 é a) x 3 + 6 x 2 – 9x + 6 = 0 c) x 3 – 7 x – 6 = 0 b) x 3 – 6 x 2 – 6 = 0 d) x 3 + 6 x 2 + 9 x = 0 - - O O 1 M N - - A B C D 10 cm 8 cm fig.1 D - - 8 cm A B C fig.2 I 06 – Seja o intervalo I = | -2 , 3 | e a figura abaixo o gráfico da função f : I ÷ 9. Então a) f(x) = x 4 , ¬x e I. b) f(x) s 0, ¬xe | -2 , 0 |. c) se a, b e I, então f(a) < f(b). d) f(a) . f | | . | \ | ÷ 4 5 + f(b) . f(1) + f(c) . f(2) = 0, ¬a, b, c e I. 07 – Dois números estão entre si como 3 está para 5. Então a razão entre o quíntuplo do 1.º e a terça parte do 2.º é a) 9 1 c) 3 b) 3 1 d) 9 08 – Sobre a equação 0 1985 x 1984 x 1983 2 = ÷ ÷ , a afirmação correta é: a) não tem raízes reais b) tem duas raízes simétricas c) tem duas raízes reais distintas d) tem duas raízes reais iguais 09 – Se K é um número inteiro, K 2 + K é necessariamente um a) múltiplo de 2. b) múltiplo de 3. c) produto de dois números ímpares. d) produto de dois números primos. 10 – O número de elementos do conjunto solução da equação 1 x 4 5 x 2 + ÷ = + , em 9, é a) 0 c) 2 b) 1 d) infinito 11 – Numa pesquisa de mercado sobre o consumo de cerveja, obteve-se o seguinte resultado: 230 pessoas consomem a marca A; 200 pessoas, a marca B; 150, ambas as marcas; e 40 não consomem cerveja. O número de pessoas pesquisadas foi a) 620 c) 320 b) 470 d) 280 12 – No paralelogramo ABCD, tem-se que BE ± AD; BE = 5 cm, BC = 12 cm e AE = 4 cm. A área do triângulo EDC, em 2 cm , é a) 48 b) 30 c) 24 d) 20 13 – Em uma maternidade, num certo dia, três mães deram à luz. A 1ª teve gêmeos; a 2.ª, trigêmeos, e a 3.ª, um único filho. Considere, para aquele dia, o conjunto das três mães, o conjunto dos seis bebês e as seguintes relações: R1 que associa cada mãe a seu filho; R2 que associa cada filho à sua mãe, e R3 que associa cada bebê ao seu irmão. É (são) função (funções) a) somente R1. c) somente R3. b) somente R2. d) R1, R2 e R3. 14 – As promoções do tipo ―leve 3, pague 2‖, comuns no comércio, acenam um desconto, sobre cada unidade vendida, de a) % 3 50 c) 20% b) % 3 100 d) 50% 15 – Um baralho tem 52 cartas, sendo 4 reis e 48 não reis. O número de maneiras diferentes que uma pessoa pode retirar desse baralho um grupo de 5 cartas, sendo 3 reis e 2 não reis, é igual a a) 44.100 c) 2.162 b) 1.152 d) 54.144 16 – O sistema de equações ¦ ¹ ¦ ´ ¦ = + = + ÷ = ÷ + 0 y x 0 z 3 y x 2 0 z y 4 x 3 a) não tem solução b) tem infinitas soluções c) tem apenas a solução trivial d) tem uma única solução não trivial 17 – O determinante da matriz A de ordem 3, tal que ¹ ´ ¦ = = ÷ = j i se , i 2 j i se , j i 2 a ij é igual a a) 72 b) 60 c) 48 d) 40 18 – É incorreto afirmar que a) 0051 , 0 51 1 > c) 3 , 0 ... 333 , 0 = b) 6 2 1 3 4 ÷ ÷ = ÷ d)  555 , 0 555 , 0 > 19 – A área de um retângulo, cujas diagonais medem 20 m cada uma e formam entre si um ângulo de 60º, em m 2 , é a) 100 b) 200 c) 3 100 d) 3 200 20 – Um tanque tem a forma de um cilindro circular reto de altura 6 m e raio da base 3 m. O nível da água nele contida está a uma distância do fundo do tanque igual aos 2/3 da sua altura. Adotando-se t = 3,14, a quantidade de litros de água que o cilindro contém é a) 113.010 c) 113.050 b) 113.040 d) 113.080 21 – A aresta de um cubo e a aresta da base de um prisma triangular regular medem 3 4 cm. Se o cubo e o prisma são equivalentes, então a área total do prisma, em cm 2 , é a) 3 216 ) d 3 214 ) c 3 212 ) b 3 210 22 – Sejam x e y os números reais que satisfazem a igualdade i - y) (x yi) - (1 2i) - (x i + = + , onde i é a unidade imaginária. O módulo do número complexo 2 ) yi x ( z + = é igual a a) 5 2 ) c 5 b) 5 d) 2 23 – Se 5 3 k + = , então 15 é igual a a) 2 8 k 2 ÷ c) 8 k 2 ÷ b) 2 k 2 d) 2 k D E A C B 1 2 3 -2 4 5 ÷ f(x) x 24 – Carla vendeu dois aparelhos de TV por preços iguais. Um deles foi vendido com lucro de 20% e o outro com prejuízo de 20% sobre o preço de custo. No total, em relação ao capital investido, Carla a) perdeu 4% c) lucrou 8% b) lucrou 6% d) não lucrou nem perdeu 25 – Vítor tem mais de 80 discos. Quando ele forma pilhas com 2 discos, sobra 1 disco; quando ele forma pilhas com 3 ou 4 discos, também sobra 1 disco; quando ele forma pilhas com 7 discos, não sobram discos. O menor número de discos que ele poderá ter é um número a) divisível por 5 b) múltiplo de 6 c) maior que 130 d) menor que 120 26 – Duas pessoas A e B fundaram uma sociedade. Três meses depois, admitiram outro sócio C. Sete meses depois da entrada de C, aceitaram outro sócio D. Essas 4 pessoas entraram para a sociedade com capitais iguais. Dois anos após a fundação da sociedade, foi verificado um lucro de R$ 227.835,00. Se este lucro foi dividido entre os sócios, proporcionalmente ao tempo de participação de cada um na sociedade, a parte que coube ao sócio D foi: a) R$ 57.645,00 c) R$ 21.352,00 b) R$ 38.430,00 d) R$ 18.234,00 27 – O número de pontos de intersecção dos gráficos das funções definidas por f(x) = 3 log x e g(x) = log 9 + log x, sendo x > 0, é a) 0 c) 2 b) 1 d) 3 28 – A distância do centro da circunferência 0 21 y 8 x 6 y x 2 2 = + ÷ ÷ + à bissetriz do II.º e IV.º quadrantes, vale a) 2 2 c) 2 7 b) 2 3 d) 2 2 7 29 – Sejam a e b dois números reais tais que 0 < b < a < 2 t . Assinale a alternativa incorreta: a) b sen a sen ) c 2 2 tga tgb > < b) a cos b cos < d) 2 cotgb > 2 cotga 30 – Um campo de futebol tem 7 entradas. O número de modos desse campo estar aberto pode ser expresso por a) 2 7 c) 7! b) 2 7 – 1 d) 7! – 1 31 – Os valores de K tais que o sistema homogêneo ¦ ¹ ¦ ´ ¦ = ÷ ÷ = + ÷ = + + 0 z y kx 0 z ky x 0 z 2 y x admita apenas a solução trivial, são a) k = 0 e k = -1 b) k = 1 e k = -1 c) k = 0 e k = -1 d) k = 1 e k = -2 32 – Ao se efetuar a soma de 50 primeiras parcelas da P.A.: 202 + 206 + 210 + . . . , por distração, não foi somada a 35.ª parcela. A soma encontrada foi a) 10.200 c) 14.662 b) 12.585 d) 16.419 33 – Num triângulo ABC têm-se cm 2 AB= , º 30 C A ˆ B = e º 45 B C ˆ A = . A área do triângulo ABC, em cm 2 , vale a) 2 3 1 + c) 2 3 2 + b) 4 3 2 + d) 4 ) 3 1 ( 2 + 34 – No plano cartesiano, os pontos A ( 1 , 0 ) e B ( 0 , 2 ) são de uma mesma circunferência. Se o centro dessa circunferência é ponto da reta y = 3 – x, então suas coordenadas são ) 3 , 0 ( ) d 2 3 , 2 3 ) c ) 2 , 1 ( ) b 2 1 , 2 3 ) a | . | \ | | . | \ | 35 – A solução da inequação x 2x – 1 < x 3 , sendo x > 0 e x =1, é o conjunto S = { x e9 / ................. }. Assinale a alternativa que completa corretamente os pontilhados: a) x < 2 c) 0 < x < 2 b) x > 2 d) 1 < x < 2 36 – A soma das medidas dos ângulos internos A, B, C, D e E da figura é a) 120º b) 180º c) 360º d) 540º 37 – Dadas as matrizes A = | | | . | \ | ÷ ÷ ÷ 1 0 0 1 2 1 3 0 5 e B = | | | . | \ | ÷ 4 2 3 0 1 1 , o elemento C12 da matriz C = A . B é a) –17 c) –3 b) 7 d) 3 38 – Na figura, considere o segmento a = 2 m. A área da superfície sombreada é, em 2 m , igual a . a) 2t b) 4t c) 2 d) 4 39 – Seja a função f : N ÷ Z, definida por f(x – 1) = f(x) – 2. Se f(1) = 4 ÷ , então a soma dos valores dos 50 menores elementos do conjunto Im(f) é a) 1.150 c) 2.150 b) 1.450 d) 2.450 40 – O n.º 1 é uma das raízes do polinômio x 3 + 4 x 2 + x – 6. Com relação às outras raízes do polinômio, podemos afirmar que a) ambas são negativas b) uma é negativa e a outra é positiva c) ambas são positivas d) uma delas é nula y a a x A B C D E 2/2002-TURMA A 01 – Sendo 5 3 2 A 2 3 × × = , 3 4 3 2 B × = e 4 5 3 2 C × = , então o quociente da divisão do m.m.c. pelo m.d.c. dos números A, B e C é a) 36 b) 90 c) 180 d) 450 02 – 25 kg de linha foram usados para tecer 24 m de um tecido de 6 m de largura. O comprimento do mesmo tecido que se pode fazer com 100 kg de linha e com largura de 9 m, em m, é a) 32 b) 64 c) 144 d) 164 03 – A média aritmética, a moda e a mediana do conjunto de valores 6; 1; 7; 3; 8; 7; 2; 10 são, respectivamente, a) 5 , 6 5 , 6 5 c) 7 5 , 6 5 , 5 b) 7 7 5 , 5 d) 5 , 6 7 5 , 5 04 – Se o logarítimo de um número na base ―n‖ é 4 e na base ― 2 n ‖ é 8, então esse número está no intervalo a) | | 50 , 1 c) | | 200 , 101 b) | | 100 , 51 d) | | 500 , 201 05 – O valor da expressão ) ` ¹ ¹ ´ ¦ | . | \ | + ÷ ÷ ÷ × ÷ 2 1 1 5 , 1 2 4 3 10 4 , 2 6 , 0 144 é igual a a) 12 1 b) 12 7 c) 3 2 ÷ d) 5 2 06 – Se 2 x 9 x 16 8 = ÷ , então ―x‖ é um número múltiplo de a) 2 b) 3 c) 5 d) 7 07 – Se ( ) 5 x , 1 x 2 , 3 x + ÷ + é uma P.A., então a soma dos três termos dessa P.A. é a) – 13 b) 15 c) 19 d) 27 08 – No sistema ¦ ¹ ¦ ´ ¦ = ÷ | | . | \ | + | | . | \ | ÷ | | . | \ | + | | . | \ | ÷ = + 32 4 5 3 5 2 5 1 5 4 5 4 3 2 2 3 4 5 n mn n m n m n m m n m , sendo | | . | \ | 1 5 ; | | . | \ | 2 5 ; | | . | \ | 3 5 e | | . | \ | 4 5 números binomiais, então o valor de ―m‖ é a) 4 b) 1 c) 2 d) 3 09 – Efetuando 4 3 2 3 k 9 k 6 k + ÷ + + , obtemos a) 3 k + c) ( ) 12 5 3 k + b) 6 3 k + d) 12 3 k + 10 – Que expressão podemos acrescentar a cada termo da fração y x a fim de obtermos y 4 x 5 ? a) x 5 y 4 xy ÷ c) x 5 y 4 xy 9 ÷ b) y 4 x 5 xy ÷ d) x 5 y 4 1 ÷ 11 – O elemento 2 , 3 X da matriz solução da equação matricial ( ( ( ¸ ( ¸ = ( ( ( ¸ ( ¸ + · 8 0 16 2 4 10 8 6 4 2 1 1 X 3 é a) 0 b) – 2 c) 3 d) 1 12 – Observando a figura, podemos afirmar que a medida da mediana AM é a) 2 2 b) 2 3 c) 3 2 d) 3 3 13 – Seja AB o diâmetro de uma circunferência. Por A traça-se uma tangente à circunferência, que encontra o prolongamento de uma corda MN paralela ao diâmetro, num ponto P. Sabendo que PM mede 9 cm (M está mais próximo de P do que N) e que o raio do círculo vale cm 5 , 12 , então a distância do centro à corda MN, em cm, mede a) 8 b) 10 c) 12 d) 15 14 – Para que valor de ―K‖ o sistema ¦ ¹ ¦ ´ ¦ = + = + = ÷ 2 Kz x 2 1 z 3 y 1 y x não possui solução? a) – 3 b) – 6 c) 6 d) 3 15 – O 9 122 sen t é igual ao a) 9 5 sen t c) 9 5 cos t ÷ b) 9 4 sen t d) 9 4 sen t ÷ 16 – A soma dos ângulos internos de um polígono convexo regular é de 720 o , Sabendo-se que o seu lado mede 4 cm e que ele está inscrito numa circunferência, então a área desse polígono, em cm 2 , é a) 3 6 c) 3 18 b) 3 12 d) 3 24 17 – Se de um retângulo de perímetro 4 e dimensões ―x‖ e ‖y‖, y x < , retira-se um quadrado de lado ―x‖, então a área remanescente em função de ―x‖ é a) x 2 1 ÷ c) 2 x 2 x ÷ b) 2 x 2 x 2 ÷ d) 2 x 4 x 2 ÷ 18 – A base de um prisma regular é um hexágono inscrito num círculo de raio R. Se o prisma é equivalente ao cubo, cuja base está inscrita no mesmo círculo, então a altura do prisma hexagonal, em cm, é a) 2R c) 3 6 R 4 b) 3 6 R 2 d) 9 6 R 4 19 – Os lados congruentes de um triângulo isósceles medem 50 cm cada. Se a medida da altura equivale 7 12 da medida da base, então a medida da base, em cm, é a) 14 b) 25 c) 28 d) 50 A(2,6) B(4,2) C(6,4) M 20 – A geratriz de um cone de revolução forma com o eixo do cone um ângulo de 45 o . A área lateral, em dm 2 , desse cone, sabendo-se que a área de sua secção meridiana é 18 dm 2 , é a) 2 18t c) t 18 b) 2 9t d) ( ) 1 2 18 + t 21 – O número de anagramas formados com as letras da palavra ROMA de modo que não apareçam vogais ou consoantes juntas é igual a a) 4! b) 4 c) 8 d) 2 22 – Se um cilindro reto está circunscrito a uma esfera de raio ―R‖, então a razão entre a área da superfície esférica e a área total do cilindro é a) 1 b) 2 1 c) 3 2 d) 5 4 23 – O maior número inteiro que satisfaz a inequação ( ) 3 x 2 2 1 1 2 1 x 3 2 + > ÷ | . | \ | + é a) – 4 b) – 3 c) – 2 d) 3 24 – A área de um triângulo de perímetro 54m circunscrito a um círculo de t 25 m 2 , em m 2 , é a) 125 b) 130 c) 135 d)140 25 – A solução da inequação 1 x cos 2 1 < < , no intervalo t s s 2 x 0 , é dada por ―x‖ real, tal que a) ) ` ¹ ¹ ´ ¦ t < < t t < < 2 x 3 5 ou 3 x 0 b) ) ` ¹ ¹ ´ ¦ t < s t t s < 2 x 3 5 ou 3 x 0 c) ) ` ¹ ¹ ´ ¦ t < s t t < < 2 x 3 5 ou 3 x 0 d) ) ` ¹ ¹ ´ ¦ t < < t t s < 2 x 3 5 ou 3 x 0 26 – Na figura, se o ângulo A ˆ é congruente ao ângulo E ˆ , então a relação falsa é a) CD CE CB CA · = · b) CD CB CD CE CE CA ÷ = ÷ c) CB CD CB CE CD CA = + + d) 3 CB CD EB CB CE DA CD CA | . | \ | = · · · · 27 – Dados os conjuntos { } 4 , 3 , 2 , 1 A = , { } 5 , 4 , 3 B = e { } 5 , 2 , 1 C = . Ao determinar o conjunto M, tal que: { } 4 , 3 , 2 , 1 M A =  , { } 5 , 4 , 3 M B =  , B A M C   = , podemos concluir que M é um conjunto a) vazio. c) que possui dois elementos. b) unitário. d) que possui três elementos. 28 – O gráfico da função ( ) x f y = , definida por 0 y 2 1 1 4 3 x 1 1 = ÷ ÷ , a) determina, com os eixos coordenados, uma região triangular de área 28 9 . b) intercepta o eixo ―x‖ no ponto de abscissa 7 3 ÷ . c) intercepta o eixo ―y‖ no ponto de ordenada 2 3 ÷ . d) passa pela origem do sistema cartesiano. 29 – A fração 2 2 4 4 b a b a ÷ ÷ ÷ ÷ ÷ ÷ é igual a a) 6 6 b a ÷ ÷ ÷ b) 2 2 b a ÷ ÷ ÷ c) 2 2 b a ÷ ÷ + d) 2 2 b a + 30 – Uma das raízes da equação 0 6 x 7 x x 2 2 3 = ÷ ÷ + é 2 x 1 = . Pode-se afirmar que: a) as outras raízes são números imaginários puros. b) as outras raízes são – 3 e – 2. c) só uma das outras raízes é real. d) as outras raízes estão entre – 2 e 0. 31 – A área da secção paralela ao eixo de um cilindro circular reto, de 8 m de altura e 1 m de raio, feita a 0,6 m do eixo, em m 2 , é a) 16,00 b) 12,80 c) 6,40 d) 8,60 32 – Uma pessoa emprega R$25,00 em duas parcelas: a 1 a a 3% ao mês e a 2 a a 2% ao mês e recebe anualmente R$7,20 de juros simples. O valor da maior parcela empregada, em R$, é a) 8 b) 10 c) 12 d) 15 33 – Um atirador deu 49 tiros, pagando R$0,10 de multa por tiro fora do alvo e recebendo R$0,25 de prêmio por tiro acertado no alvo. Se nada recebeu e nada pagou, então a multa foi de R$ ____. a) 1,40 b) 3,50 c) 5,00 d) 8,75 34 – Se em uma P.G. de três termos reais o produto e a soma dos termos são, respectivamente, 216 e 26, então a soma dos dois primeiros termos dessa P.G., quando decrescente, é a) 24 b) 20 c) 18 d) 8 35 – Seja o pentágono ABCDE da figura, inscrito numa circunferência de centro O. Se o ângulo o 50 B O ˆ A = , então ― y x + ‖ vale, em graus, a) 216 b) 205 c) 180 d) 105 36 – Um empregado recebe uma gratificação de 4% sobre os lucros. Em R$, quanto receberá de gratificação, se vendeu R$ 67.206,00 com lucro de 20% sobre o preço de compra? a) 448,04 b) 560,05 c) 268,82 d) 112,01 37 – Tenho nove moedas numeradas de 1 a 9 inclusive. Com elas, formo números de três algarismos. Quantos números, cuja soma é par, podemos formar? a) 144 b) 84 c) 104 d) 264 A B C D E F - A B C D E x y O altura (m) número de homens 1,60 |–– 1,65 04 1,65 |–– 1,70 12 1,70 |–– 1,75 18 1,75 |–– 1,80 26 1,80 |–– 1,85 10 1,85 |–– 1,90 08 1,90 |–– 1,95 02 Total 80 38 – Traçam-se duas cordas de uma mesma extremidade de um diâmetro de um círculo. Uma delas mede 9 cm, e sua projeção sobre o diâmetro mede 5,4 cm. O comprimento da outra corda, cuja projeção no diâmetro é de 9,6 cm mede, em cm, a) 10 b) 12 c) 14 d) 15 39 – Dois pontos sobre a reta 2 y = distam 4 unidades da reta 0 2 y 3 x 4 = + ÷ . A distância, em unidades, entre as abscissas dos pontos é a) 10 b) 2 c) 6 d) 4 40 – Um quadrado ABCD está inscrito num círculo com centro na origem do plano de Gauss. O vértice ―A‖ é imagem do complexo i 4 3 + . Os afixos dos outros três vértices são os complexos: a) i i i 4 3 ; 4 3 ; 4 3 ÷ ÷ ÷ + ÷ . b) i i i 3 4 ; 4 3 ; 3 4 ÷ ÷ ÷ + ÷ . c) i i i 4 3 ; 4 3 ; 3 4 ÷ ÷ ÷ + ÷ . d) i i i 3 4 ; 4 3 ; 4 3 ÷ ÷ ÷ + ÷ . 2/2002-TURMA B 01 – Se a e b são dois números reais e a razão de a para b é 0,7, pode-se afirmar sempre que a) b a > c) b a < b) b a > d) b a < 02 – Seja a sucessão de números racionais: 5 8 ÷ ; 2 3 ; 2 , 1 ÷ ;  3232 , 0 ;  6111 , 1 ; 3 2 1 ÷ . Escrevendo-a em ordem decrescente, obtemos a) 3 2 1 5 8 2 , 1 3232 , 0 2 3 6111 , 1 ÷ > ÷ > ÷ > > >   b) 2 , 1 5 8 3 2 1 3232 , 0 6111 , 1 2 3 ÷ > ÷ > ÷ > > >   c) 5 8 2 , 1 3 2 1 2 3 3232 , 0 6111 , 1 ÷ > ÷ > ÷ > > >   d) 5 8 3 2 1 2 , 1 3232 , 0 6111 , 1 2 3 ÷ > ÷ > ÷ > > >   03 – A altura de 80 homens de uma comunidade está distribuída de acordo com a tabela. A porcentagem de homens com altura maior ou igual a 1,80 m é a) 25 % b) 30% c) 60% d) 75% 04 – Por 24 operários que trabalhavam 7 horas por dia, foram feitos 5 2 de um trabalho em 10 dias. Com a dispensa de 4 operários e considerando-se que os restantes trabalham agora 6 horas por dia, nas mesmas condições, o número de dias em que o trabalho será concluído é a) 18 b) 19 c) 20 d) 21 05 – Assinale a alternativa falsa. a) Se dois números são primos, então eles são primos entre si. b) Dois números primos entre si podem ser primos. c) Um número par e outro ímpar podem ser primos entre si. d) Se dois números são primos entre si, então eles são necessariamente primos. 06 – O sistema ¦ ¹ ¦ ´ ¦ = ÷ ÷ = + ÷ = ÷ m y 3 x 2 6 y 4 x 4 y 2 x 3 , nas incógnitas x e y, admite uma única solução se, e somente se, a) 1 m ÷ = c) 1 m ÷ = b) 0 m = d) 2 m = 07 – Um tanque cilíndrico com água tem raio da base R. Mergulha-se nesse tanque uma esfera de aço e o nível da água sobe R 16 9 . O raio da esfera é a) R 4 3 c) R 5 3 b) R 16 9 d) 2 R 08 – Dadas as afirmações: I- Quaisquer dois ângulos opostos de um quadrilátero são suplementares. II- Quaisquer dois ângulos consecutivos de um paralelogramo são suplementares. III- Se as diagonais de um paralelogramo são perpendiculares entre si e se cruzam no seu ponto médio, então este paralelogramo é um losango. Pode-se garantir que a) todas são verdadeiras. b) apenas I e II são verdadeiras. c) apenas I e III são verdadeiras. d) apenas II e III são verdadeiras. 09 – Na figura abaixo, ABCDE é um pentágono regular. As medidas dos ângulos x, y e z, em graus, são, respectivamente a) 72 ; 36 ; 36 b) 72 ; 36 ; 72 c) 36 ; 36 ; 72 d) 36 ; 72 ; 36 10 – Para obter-se um total de R$ 22.800,00 ao final de 1 ano e 2 meses, à taxa de 12% ao ano, a juros simples, é necessário que se aplique a) R$ 10.000,00 b) R$ 12.000,00 c) R$ 15.000,00 d) R$ 20.000,00 11 – Os valores de x que tornam verdadeira a igualdade 2 x 1 3 1 1 1 2 0 x ÷ = ÷ ÷ são tais que seu produto p é elemento do conjunto a) { } 3 p / p ÷ > 9 e c) { } 6 p / p ÷ < 9 e b) { } 2 p 3 / p s < ÷ 9 e d) { } 2 p 6 / p < s ÷ 9 e 12 – A equação 0 20 x 2 x 10 x 2 3 = + ÷ ÷ tem como raízes a, b e c. Então, o valor da expressão 2 2 2 abc c ab bc a + + é a) 100 b) 250 c) – 200 d) – 400 z x y A B C D E 13 – O par ( ) y , x , solução da equação matricial | | . | \ | + ÷ = | | . | \ | · | | . | \ | ÷ 8 y x 4 x 2 13 1 y 2 x y x 4 x 2 3 2 é a) ( ) 3 , 6 ± c) | | . | \ | ÷ ± 5 , 2 1 b) ( ) 2 , 5 ÷ ± d) | . | \ | ÷ 5 4 , 3 7 14 – É verdadeira a afirmação: A equação 0 36 x 13 x 4 8 = + ÷ a) admite 4 raízes reais irracionais. b) admite 4 raízes reais racionais positivas. c) não admite raízes reais. d) admite 4 raízes reais inteiras. 15 – Seja Z um número complexo, cujo módulo é 2 e cujo argumento é 3 t . A forma algébrica do conjugado de Z é a) i 3 1 ÷ c) i 3 + b) i 3 ÷ d) i 3 1 + 16 – Sabe-se que a seqüência ( ) 10 ; y ; x é uma P.A. e a seqüência | | . | \ | + 4 x 3 ; 2 ; y 1 é uma P.G. Nessas condições, é correto afirmar que a) a razão da P.A. é 2. b) a razão da P.G. é 26. c) 0 y x = + . d) 16 y x ÷ = · . 17 – A fórmula que define a função quadrática, cuja representação gráfica é uma parábola, cuja concavidade é voltada para baixo e que não intercepta o eixo das abscissas, é a) y = – x 2 – 2x – 1 b) y = – 5x + x 2 + 7 c) y = 3x – 2x 2 – 2 d) y = – 6 – x 2 – 5x 18 – Seja ( ) x 5 1 x 9 x 1 x 12 5 x x f ÷ + + + ÷ + = . O domínio de f é a) { } 1 , 0 ÷ ÷ 9 c) * 9 b) { } 5 , 1÷ ÷ 9 d) { } 5 , 1 , 1 * ÷ ÷ ÷ 9 19 – Sejam: AB o diâmetro de uma circunferência de centro O; AR uma corda, tal que o 20 R A ˆ B = ; t, paralela a AR, uma reta tangente à circunferência, em T. Sabendo que T e R são pontos da mesma semicircunferência em relação a AB, a medida, em graus, do ângulo agudo formado pela reta t e pela corda AT é igual a a) 25 c) 50 b) 35 d) 70 20 – Dois números, x e y, estão relacionados da seguinte forma: "a cada número x corresponde um único número y, que é o dobro do quadrado de x menos 8 unidades". Nessas condições, é falso afirmar que a) y é função de x. b) x é função de y. c) se 13 x = , 18 y = . d) se 32 y = , 5 2 x ± = . 21 – De acordo com os dados da figura, a distância aproximada, em metros, entre os pontos A e B é a) 100 b) 102 c) 104 d) 108 22 – Quaisquer que sejam o racional x e o irracional y, pode-se dizer que a) y x· é irracional. b) y y· é racional. c) 2 y x + ÷ é irracional. d) y 2 x + é irracional. 23 – ANULADA 24 – O gráfico de uma função f é o segmento de reta que une os pontos ( ) 4 , 3 ÷ e ( ) 0 , 3 . Se 1 f ÷ é a função inversa de f, então ( ) 2 f 1 ÷ é a) 2 b) 0 c) 2 3 ÷ d) 2 3 25 – Na figura abaixo, a curva representa o gráfico da função x log y = , para 0 x > . Assim, a soma das áreas das regiões hachuradas é igual a a) 2 log b) 3 log c) 4 log d) 6 log 26 – Se u é um ângulo tal que 2 0 t < u < e o dobro do seu seno é igual ao triplo do quadrado da sua tangente, então o valor do seu cosseno é a) 3 3 c) 2 2 b) 2 3 d) 3 2 27 – O gráfico abaixo representa as funções reais ( ) x P e ( ) x Q . Então, no intervalo | | 8 , 4 ÷ , ( ) ( ) 0 x Q x P < · para todo 9 e x tal que a) 4 x 2 < < ÷ b) 1 x 2 ÷ < < ÷ ou 8 x 5 < < c) 2 x 4 ÷ < s ÷ ou 4 x 2 < < d) 5 x 1 < s ÷ 80m 30 o 105 o A C B y x S 1 S 2 1 2 3 4 P -1 -2 -3 -4 5 6 1 2 3 4 7 8 y x Q 0 28 – Coloque V ou F conforme as afirmações sejam verdadeiras ou falsas: ( ) Dois ângulos adjacentes são suplementares. ( ) Dois ângulos que têm o mesmo complemento são congruentes. ( ) Dois ângulos suplementares são adjacentes. ( ) Um triângulo obtusângulo pode ser isósceles. ( ) Um triângulo retângulo é escaleno. Assinale a seqüência correta. a) F – V – F – V – V c) F – V – F – V – F b) F – V – V – V – F d) F – F – V – V – F 29 – Sejam: { } 3 , 2 , 1 A = , { } u , o , i , e , a B = e a função B A : f ÷ . O número de funções injetoras definidas em f é igual a a) 10 c) 60 b) 15 d) 75 30 – Assinale V (verdadeiro) ou F (falso), considerando a geometria de posição espacial e plana. ( ) A condição | = s r  é necessário para que as retas r e s sejam paralelas distintas. ( ) Duas retas que formam um ângulo reto são necessariamente perpendiculares. ( ) Se duas retas têm um único ponto em comum, então elas são concorrentes. ( ) A condição | = s r  é suficiente para que as retas r e s sejam reversas. A seqüência correta é: a) V – V – V – V c) F – V – F – V b) V – F – V – F d) F – F – F – F 31 – Um imóvel foi comprado e revendido com um lucro de 8% sobre o preço de venda. Sabendo que, se o lucro fosse aumentado de R$700,00, ele teria sido igual a 9% do preço de compra, esse lucro foi de a) R$ 10.000,00 c) R$ 20.000,00 b) R$ 14.000,00 d) R$ 32.000,00 32 – Os valores de x para os quais x x 4 2 ) 8 , 0 ( ÷ > ) 1 x ( 3 ) 8 , 0 ( + são a) 2 3 ÷ < x < 2 1 c) x < 2 3 ÷ ou x > 2 1 b) 2 1 ÷ < x < 2 3 d) x < 2 1 ÷ ou x > 2 3 33 – Dada a figura abaixo, se cm 8 AB = , cm 4 CD = e cm 20 AD = , a medida, em cm, de x é a) 6 6 b) 2 6 c) 3 6 2 d) 2 6 3 34 – O maior e o menor lado de um triângulo medem, respecti- vamente, 10 cm e 3 cm e formam entre si um ângulo de 45 o . O volume do sólido gerado pela rotação de 360 o desse triângulo em torno do seu lado maior é, em cm 3 , a) t 30 c) t 15 b) t 20 d) t 10 35 – Feito o levantamento de um terreno pentagonal, foram determinados os dados indicados na figura a seguir. A área do terreno, em m 2 , é a) 450 b) ( ) 1 3 4 450 ÷ c) 900 d) ( ) 2 3 3 900 ÷ 36 – Classifique em Verdadeiro (V) ou Falso (F): ( ) Z+ c N ( ) Z+ = N ( ) Z – Z - = * + Z ( ) ( Z+ · Z - ) N * = N ( ) Z – Z+ = Z - Assinale a seqüência correta: a) F – F – V – V – F b) F – F – V – V – V c) V – F – V – F – F d) V – F – V – V – F 37 – Num triângulo ABC retângulo em A, o cateto AC mede 1,5 cm e a altura traçada sobre a hipotenusa determina o segmento HB que mede 1,6 cm. O valor da secante do ângulo interno C é a) 3 4 c) 5 4 b) 4 5 d) 3 5 38 – No desenvolvimento de 10 3 m 1 m | . | \ | ÷ , o coeficiente de m 6 é a) 45 c) 210 b) 120 d) 245 39 – Dadas a reta de equação 3 x 3 y = e a circunferência de equação 0 x 4 y x 2 2 = ÷ + . A área do triângulo determinado pelo centro da circunferência e os pontos de intersecção entre a reta e ela, em unidades de área, é igual a a) 3 c) 3 3 b) 3 d) 6 - - A B C D E x A D - - - 30m 30m 60 o 30 o 45 o B C E 40 – A figura abaixo é a planificação de um poliedro convexo ( ) F E ; D C B A ÷ ÷ ÷ ÷ . O volume desse poliedro, em unidades de volume, é a) 2 425 c) 3 850 b) 3 425 d) 2 850 1/2003-TURMA A 01 – Nesta figura, as retas r e s são paralelas entre si. Os valores de ―x‖, ―y‖ e ―z‖ são, respectivamente, a) 23 o 45‘, 85 o e 95 o . b) 25 o , 90 o e 90 o . c) 23 o 7‘5‘‘, 95 o e 85 o . d) 26 o 15‘, 85 o e 95 o . 02 – O valor da expressão ( ) | | { } ( ) 25 , 0 35 , 0 3 , 0 5 , 0 5 , 2 7 , 0 ÷ ÷ ÷ + + é a) 1 b) 2 c) 3 d) 4 03 – Se em uma circunferência uma corda mede cm 2 16 e dista 2 6 cm do centro, então a medida do raio dessa circunferência, em cm, é a) 2 12 b) 2 10 c) 2 8 d) 2 6 04 – Certa obra deveria ser feita em 80 dias por 60 operários. Após 26 dias, 24 operários foram dispensados. Em quantos dias os outros operários farão o restante da obra? a) 90 b) 80 c) 76 d) 60 05 – De acordo com os dados nos triângulos retângulos CAB e CAD, é correto afirmar que a) y x = b) y 3 x = c) y 2 x = d) 2 y 3 x = 06 – O produto ( ) ( ) x 2 sen x tg · é igual a a) x sen 2 c) x sen 2 2 b) x cos 2 d) x cos 2 2 07 – O volume, em cm 3 , de um prisma hexagonal regular com altura igual a 5 cm e com área lateral 2 cm 60 , é a) 3 5 b) 3 45 c) 3 30 d) 3 270 08 – Em uma circunferência estão inscritos um triângulo eqüilátero e um hexágono regular. O apótema do triângulo somado com o apótema do hexágono dá ( ) 1 3 12 + cm. O lado do triângulo, em cm, mede a) 3 12 b) 3 16 c) 3 20 d) 3 24 09 – Se 8 log 3 log 32 log M 2 3 1 2 ÷ + = , então M vale a) 1 ÷ b) 1 c) 2 ÷ d) 2 10 – Resolvendo a inequação ( )( ) 0 8 x 4 6 x 2 s + ÷ , para R xe , obtemos a) 3 x 2 < < ÷ c) 1 x 6 < < ÷ b) 3 x 2 s s ÷ d) 1 x 6 s s ÷ 11 – ―N‖ é o conjunto dos números naturais, { } N x | x 3 K e = , { } N x | x 5 L e = e { } N x | x 15 M e = . A afirmativa correta é a) M L K = b) L K c c) M L K = ÷ d) M L K = · 12 – Na figura, os triângulos ABC e EDC são semelhantes. Sabendo que 5 x AC ÷ = e 4 x 2 DE + = , a soma ― ) CE ( med ) AC ( med + ‖, em cm, vale a) 10,3 b) 18 c) 13 d) 23,3 13 – Os números complexos que correspondem aos pontos A e B do gráfico são, respectivamente, a) ( ) ( ) i 2 3 ; i 3 1 ÷ ÷ + b) ( ) ( ) i 3 2 ; i 3 ÷ ÷ + c) ( ) ( ) i 3 1 ; i 2 3 + ÷ ÷ d) ( ) ( ) i 3 ; i 3 2 + ÷ ÷ 14 – Fatorando a expressão 2 2 b 45 ab 30 a 5 + + , obtemos a) ( )( ) b 3 a 5 b 3 a 5 ÷ + c) ( ) 2 b 3 a 5 ÷ b) ( ) 2 b 3 a 5 + d) ( ) 2 b 3 a 5 + 15 – ANULADA 16 – Se um cubo está inscrito em uma esfera de m 3 de raio, então o volume do cubo, em m 3 , é igual a a) 8 b) 27 c) 3 12 d) 3 24 17 – Sendo | | . | \ |÷ = | | . | \ | ÷ | | . | \ | ÷ 3 7 5 4 . 3 y x 2 , os valores de x e y na matriz acima são, respectivamente, a) 3 e –3 b) –3 e 3 c) 2 9 e –3 d) –3 e 2 9 A B C D E F O 13 13 13 13 13 2 5 2 5 2 5 2 5 2 5 2 5 2 5 2 5 2 5 2 5 2 5 2 5 - A B x y C 60 o 30 o D B A 3 -2 -1 - - y x -3 -2 -1 2 1 -3 3 2 B A C D E 10cm 6cm 85 o z s r y x 3x-10 o 18 – Se ―1‖, ―x2‖ e ―x3‖ são as raízes da equação 0 6 x 5 x 2 x 2 3 = + ÷ ÷ , então o valor de ― 3 2 x x ÷ ‖, para 3 2 x x > , é a) 3 b) 1 c) 6 d) 5 19 – Na figura, ― y x÷ ‖ é igual a a) 15 o b) 20 o c) 30 o d) 35 o 20 – Do ponto P, situado a 10 cm do centro O de uma circunferência de raio igual a 8 cm, traça-se uma secante PB passando por A tal que PA = AB, sendo A e B pontos da circunferência. A medida de PB, em cm, é a) 2 3 b) 2 6 c) 8 d) 6 21 – As mediatrizes de dois lados consecutivos de um polígono regular formam um ângulo de 24º. O número de diagonais desse polígono é a) 70 b) 80 c) 90 d) 100 22 – Sendo C(3, –2) o centro de uma circunferência de raio igual a 4, então sua equação normal ou geral é a) 0 3 y 4 x 6 y x 2 2 = + + ÷ + b) 0 3 y 4 x 6 y x 2 2 = ÷ + ÷ + c) 0 3 y 4 x 6 y x 2 2 = ÷ ÷ + + d) 0 3 y x 2 2 = ÷ + 23 – Se em uma pirâmide quadrangular regular a diagonal da base mede 4 m e a aresta lateral mede 2,5 m, então o volume da pirâmide, em m 3 , é a) 1 b) 2 c) 3 d) 4 24 – Na figura, as cordas AB e CD são paralelas. EC é um diâmetro e P é o ponto médio da corda AD. As medidas, em graus, dos ângulos C R ˆ A e R A ˆ P são, respectivamente, ( ) o 14 x 4 ÷ e ( ) o 13 x 5 ÷ . As medidas dos ângulos do triângulo PCD são a) 42 o , 57 o , 81 o b) 45 o , 45 o , 90 o c) 46 o , 45 o , 90 o d) 52 o , 38 o , 90 o 25 – Se um ponto P do eixo das abscissas é eqüidistante dos pontos ( ) 4 , 1 A e ( ) 3 , 6 B ÷ , então a abscissa do ponto P é a) 1 ÷ b) 0 c) 2 ÷ d) 1 26 – A função do 2 o grau que descreve o gráfico abaixo é a) ( ) 6 x x x f 2 + ÷ = b) ( ) 6 x 5 x x f 2 ÷ + = c) ( ) 6 x 5 x x f 2 + ÷ ÷ = d) ( ) 6 x 5 x x f 2 + ÷ = 27 – A área lateral do sólido geométrico formado pela rotação de um triângulo equilátero, de perímetro 30 cm, em torno de um de seus lados é, em cm 2 , igual a a) t 100 b) t 200 c) 3 50t d) 3 100t 28 – As medidas dos lados de um triângulo são iguais a 4 cm, 5 cm e 6 cm. O cosseno do menor ângulo desse triângulo é igual a a) 8 1 b) 16 9 c) 4 3 d) 5 2 29 – Em um losango, uma diagonal forma um ângulo de 58 o com um de seus lados. A medida do menor ângulo desse losango é a) 58º b) 64º c) 116º d) 122º 30 – Duas firmas vendem juntas 1.720 parafusos diariamente. Quanto vende cada firma, se uma delas vende 15% mais que a outra? a) 900 e 1.035 c) 820 e 943 b) 840 e 966 d) 800 e 920 31 – A área do trapézio retângulo (fig. abaixo), em cm 2 , é igual a (Obs: Utilize 7 , 1 3 = ) a) 20,00 b) 26,40 c) 34,68 d) 40,80 32 – Sendo AD a bissetriz do ângulo BÂC do triângulo ABC, a relação verdadeira é a) C ˆ B ˆ ÷ = ¸ ÷ o b) B ˆ C ˆ ÷ = ¸ ÷ o c) C ˆ B ˆ ÷ = o ÷ ¸ d) C ˆ B ˆ + = o + ¸ 33 – Um copo cheio de água pesa 345gf e o copo com 5 2 da água total pesa 210gf. Quanto pesará, em gf, o copo com 3 1 da água total? a) 120 b) 225 c) 195 d) 300 34 – O gráfico abaixo representa a função x log y a = . Dentro das condições de existência para que a operação de loga- ritimação seja sempre possível e de resultado único, a base ―a‖ é a) 1 a 0 < < b) 0 a = c) 1 a > d) 0 a < 35 – Um número, seu logarítimo 2 e a base do logarítimo for- mam, nessa ordem, uma P.A. Esse número é a) 2 17 9 ÷ c) 2 17 1+ ÷ b) 2 17 9 + d) 2 17 1÷ ÷ 36 – As atuais placas de automóveis possuem três letras do alfa- beto latino (incluindo K, W, Y) e quatro algarismos. O número de placas que não repetem nem letras e nem algarismos é a) ! 6 ! 23 ! 10 ! 26 c) ! 10 ! 26 b) 4 3 10 26 · d) ! 3 ! 4 ! 10 ! 26 37 – Se ( ) 25 , 0 0625 , 0 2 x = + , então ( ) 6 1 x + vale a) 2 3 ÷ b) 32 1 c) 64 d) 64 1 - f(x) 6 2 3 x 8 cm 30 o 3,2 cm y 1 3 -3 2 4 3 2 1 -2 x x log y a = -1 O C E P D B R A - x y 5 5 2 B ˆ B ˆ ¸ o C ˆ A B C D 38 – No triângulo retângulo ABC, a mediana AM forma com a bissetriz BF o ângulo M F ˆ B . O valor de M F ˆ B é a) B ˆ 2 3 b) B ˆ 2 5 c) 2 B ˆ d) B ˆ 39 – O ponto M é o ponto de intersecção das diagonais AC e BD de um quadrilátero ABCD. Sendo ( ) 0 , 0 A , ( ) 0 , 3 B , ( ) 2 , 4 C e ( ) 5 , 0 D as coordenadas dos vértices do quadrilátero, as coordenadas do ponto M são a) | . | \ | 13 30 , 13 15 c) | . | \ | 13 15 , 13 30 b) | . | \ | 13 90 , 13 180 d) | . | \ | 7 15 , 7 30 40 – Na figura abaixo, AB e MN são diâmetros perpendiculares de um círculo de raio 2 cm. Traça-se o arco MPN de centro A e raio AM. A área da região tracejada, em cm 2 , é a) 2 b) 4 c) t 2 d) 4 + t 1/2003-TURMA B 01 – Leia as sentenças abaixo. I - Todo número natural que termina em 3 é divisível por 3. II - Todo número natural divisível por 2 é também divisível por 4. III - Existem números naturais terminados em 2 que são divisíveis por 4. IV - Todo número natural divisível por 10 é também divisível por 2 e 5. V - Existem números naturais terminados em 4 que são divisíveis por 3. VI - Existem números naturais divisíveis por 6 que não são divisíveis por 2. Está correto o que se afirma em a) I e II apenas. b) I, II e III apenas. c) III, IV e V apenas. d) I, II, III, IV, V e VI. 02 – Em um triângulo ABC, o lado AB mede 3 6 cm e o ângulo C ˆ , oposto ao lado AB, mede 60 º . O raio da circunferência que circunscreve o triângulo, em cm, mede a) 6 c) 3 6 b) 12 d) 6 3 03 – O valor da raiz da equação 40 2 2 1 x 1 x = + ÷ + é um número a) inteiro positivo. c) inteiro negativo. b) irracional. d) imaginário puro. 04 – Das sentenças abaixo, quantas são verdadeiras de modo que são satisfeitas por qualquer número real ―x‖? I - ( ) 16 x 4 x 2 2 ÷ = ÷ II - x x 4 2 8 · = III - x x 3 1 2 1 | . | \ | > | . | \ | IV - ( ) ( ) 1 x log 3 log 1 x 3 log 2 2 2 2 2 + + = + a) 1 b) 2 c) 3 d) 4 05 – Na figura abaixo, 8 AB= cm, 10 BC = cm, 4 AD= cm e o ponto O é o centro da circunferência. O perímetro do triângulo AOC é, em cm, a) 45 b) 48 c) 50 d) 54 06 – Observando as figuras abaixo, o valor, em graus, de y x ÷ é a) 25 b) 20 c) 15 d) 10 07 – Uma classe tem 10 meninos e 9 meninas. Seu professor necessita formar comissões de 7 crianças, sendo 4 meninos e 3 meninas, que incluam obrigatoriamente o melhor aluno dentre os meninos e a melhor aluna dentre as meninas. O número possível de comissões é a) igual a 2300. c) maior que 2400. b) menor que 2300. d) igual a 2352. 08 – A soma dos 9 primeiros termos de uma P.A. de razão 2 é nula. Assim, pode-se afirmar que seu sexto termo é igual a a) 0 b) 2 c) 6 d) 7 09 – Uma caixa d‘água tem a forma de paralelepípedo reto- retângulo, cujas medidas internas são, em m, ―x‖, ― x 20÷ ‖ e ―2‖. O maior volume, em m 3 , que ela poderá conter é igual a a) 150 b) 200 c) 220 d) 250 10 – Um tanque tem três torneiras. A 1 a enche o tanque em 25 horas; a 2 a , em 40 horas; já a 3 a , o esvazia em 20 horas. O tanque está com 4 1 de água. Abrindo-se simultaneamente as três torneiras, ele ficará cheio em a) 55 h 40 min. c) 52 h. b) 53 h 12 min. d) 50 h. 11 – A solução da equação 2 x x x x 1 4 3 2 = + + + + +  é a) 2 3 c) 1 ÷ b) 2 1 d) indeterminada 12 – Seja a função inversível f de gráfico abaixo. A lei que define 1 f ÷ é a) 2 3 x 3 y + = b) 2 3 x 2 y ÷ = c) 2 3 x 2 y + = d) 3 2 x 3 y ÷ = - A B F M C B A M N P - - O E C D A B - 3 2 4 x y y 25 o 150 o m t m//t 30 o 65 o x 40 o r s r//s 13 – Se o resto da divisão de ( ) 5 nx mx x x P 2 3 + + + = por 2 x ÷ é 15, então o valor de n m 2 + é a) 1 b) 2 c) 3 d) 5 14 – No lançamento simultâneo de dois dados perfeitos, a probabilidade de obter soma diferente de 11 é, aproximada- mente, a) 5,5% b) 94,4% c) 83,4% d) 16,6% 15 – A geratriz de um cone de revolução mede 6 cm e o ângulo da geratriz com a altura do cone é de 30 º . O volume desse cone, em cm 3 , é a) t 9 b) 3 3t c) 3 9t d) 3 27t 16 – A expressão trigonométrica x sen x cos 2 2 ÷ é igual a: a) 1 para todo número real ―x‖. b) –1 para todo número real ―x‖. c) 1 x cos 2 2 ÷ , para todo número real ―x‖. d) 3 4 para alguns números reais de ―x‖. 17 – O menor valor inteiro positivo que pertence ao conjunto- solução da inequação ( )( ) 0 8 x 6 x 12 x 3 2 2 < + ÷ + ÷ é o a) 2 b) 3 c) 4 d) 5 18 – Sendo i a unidade imaginária, a potência de ( ) ( ) | | 3 2 2 i 1 i 1 + ÷ ÷ é igual a a) 64 b) 64 ÷ c) i 64 d) i 64 ÷ 19 – O maior valor inteiro de k para que a equação 0 k y 6 x 4 y x 2 2 = + ÷ + + represente uma circunferência é a) 14 b) 13 c) 12 d) 10 20 – A figura representa um trapézio retângulo com AD AB = , base menor igual a 3 cm e BC é lado de um quadrado. A área desse quadrado, em cm 2 , é a) 9 b) 18 c) 24 d) 36 21 – Dadas as matrizes ( ¸ ( ¸ ÷ = 4 1 0 3 A e ( ¸ ( ¸ ÷ = 0 1 1 2 B , então A B B A · ÷ · é igual a: a) ( ¸ ( ¸ 0 0 0 0 b) ( ¸ ( ¸ ÷ 0 5 3 2 c) ( ¸ ( ¸ ÷ 1 9 7 1 d) ( ¸ ( ¸ ÷ 7 2 1 3 22 – Seja o triângulo PMN de lados cm 6 PM= , cm 8 MN = e cm 10 PN = . Unindo-se os pontos médios de seus três lados obtemos o triângulo ABC. A área, em cm 2 , do triângulo ABC é a) 4 b) 6 c) 12 d) 20 23 – A igualdade 1 x B 1 x A 1 x 2 2 ÷ + + ÷ ÷ ocorre quando A e B são, respectivamente, a) 1 ÷ e 1 ÷ b) 1 ÷ e 1 c) 1 e 1 ÷ d) 1 e 1 24 – Considere a função f:9÷9 definida por ¦ ¦ ¹ ¦ ¦ ´ ¦ > ÷ ÷ s < s ÷ = 3 x se , 5 x 2 2 x 3 x 1 se , 0 1 x se , 1 x 2 ) x ( f Se 1024 log a 2 = e x0 = a – 6, então o valor da função no ponto x0 é dado por a) 2/3 b) 3/2 c) 2 d) 3 25 – Uma corda é determinada pela reta 0 y x = ÷ sobre a circunferência ( ) ( ) 16 2 y 2 x 2 2 = + + ÷ . A área da menor região determinada por essa corda e o círculo é: a) 8 4 ÷ t c) 2 4 ÷ t b) 16 4 ÷ t d) 4 4 ÷ t 26 – Sendo o 30 b a = ÷ , calculando ( ) ( ) 2 2 a cos b sen b cos a sen y ÷ + + = , obtemos a) 1 c) 3 b) 3 2 d) 2 3 2 + 27 – Se o apótema de um tetraedro regular mede 3 5 cm, então, a altura desse tetraedro, em cm, é a) 3 5 c) 3 6 10 b) 2 10 d) 3 3 10 28 – ANULADA 29 – Uma pessoa aplica um certo capital a juros simples, a 4% ao ano. No fim de três anos, reaplica o montante a juros simples, à taxa de 5% ao ano e, ao final de 2 anos, consegue um novo montante de R$ 6.160,00. O capital inicial, em reais, era de a) 5.000 b) 5.160 c) 5.500 d) 6.000 30 – Seja o triângulo ABC e D um ponto do lado AC . Se 2 AD= cm, 3 AB = cm, DC BD = e o 30 C A ˆ B = , a medida, em cm, do lado BC é igual a a) 3 b) 5 c) 6 d) 7 31 – A tabela abaixo indica o número de gols de 50 artilheiros de um campeonato de futebol. É falsa a afirmação: a) a moda dessa distribuição é 4. b) o número de gols marcados é 46. c) a média de gols dos artilheiros é 5,24. d) o número mediano de gols é 5. 32 – O preço de certa mercadoria aumentou em 250%. Para que o preço da mercadoria volte a ser o que era antes do aumento deve-se diminuir o novo preço em a) % 7 1 74 b) % 7 2 73 c) % 7 4 72 d) % 7 3 71 A B C D 135 o N o de gols N o de artilheiros 1 5 3 7 4 10 5 8 6 7 8 6 9 4 10 3 A O x - s u a r b t 33 – Considere o trapézio retângulo ABCD, onde A ˆ e D ˆ são retos, AD AB= , 7 CD = cm e 1 AB BC = ÷ cm. Assinale a afirmativa verdadeira a) 3 1 C sen = c) 5 3 C sen = b) 5 4 C cos = d) 3 4 tgC = 34 – Sendo S o conjunto-solução da equação em 9 1 x 3 1 x 3 + ÷ = ÷ , pode-se afirmar que a) S 2 1 e c) S 3 1 , 5 3 c ) ` ¹ ¹ ´ ¦ b) S 3 2 e d) S 7 2 , 5 1 c ) ` ¹ ¹ ´ ¦ 35 – O triângulo ABC está inscrito numa circunferência de centro O e de raio 13 cm. Sabendo que cm 10 AB = , a altura AH relativa ao lado BC mede, em cm, aproximadamente a) 7,6 b) 8,4 c) 9,23 d) 10,8 36 – Em um trapézio, os lados paralelos medem 16 cm e 44 cm, e os lados não-paralelos, 17 cm e 25 cm. A área do trapézio, em cm 2 , é a) 250 b) 350 c) 450 d) 550 37 – Se a diferença entre os quadrados das raízes da equação 0 c x 7 x 3 2 = + ÷ é 9 35 , então o valor de ―c‖ é a) 3 2 ÷ b) 2 ÷ c) 3 2 d) 2 38 – Um prisma reto tem base hexagonal regular e as faces laterais quadradas. Sabendo-se que a área do círculo inscrito em sua base é igual a 2 cm 25t , a área total, em cm 2 , desse prisma é a) 400 c) ( ) 3 2 100 + b) ( ) 3 6 100 + d) 600 39 – ANULADA 40 – Um triângulo DEF tem o 38 F E ˆ D = e o 74 D F ˆ E = . O ângulo que a bissetriz DG forma com a altura DH mede: a) 18 o b) 20 o c) 26 o 30‘ d) 34 o 2/2003-TURMA A 36 – Na figura, r // s e t ± u. O valor de a – b é a) 100° b) 90° c) 80° d) 70° 37 – O perímetro de um triângulo equilátero inscrito numa circunferência é 54 cm. A área de um quadrado inscrito nessa mesma circunferência é, em cm 2 , a) 36. b) 72. c) 216. d) 288. 38 – Seja 2 0 2 0 x 4 6 3 2 ÷ ÷ = 64. O valor de x que torna verdadeira a igualdade é a) 4. b) 5. c) – 4. d) – 5. 39 – Na figura abaixo, os ângulos assinalados  e Ô medem, respectivamente, 10° e 50°. Assim sendo, o valor de tgx é a) 2 1 . c) 3 3 . b) 2 2 . d) 1. 40 – A fração de denominador 30 que excede de 3 1 a fração 5 3 é a) 30 8 . b) 30 16 . c) 30 24 . d) 30 28 . 41 – Sejam os conjuntos A = {x e N / x é múltiplo de 2}, B = {x e Z / - 2 < x s 9 } e C = { x e 9 / x > 5 }. A soma dos elementos que formam o conjunto ( A · B ) – C é a) 9. b) 6. c) 3. d) 1. 42 – Dois números primos entre si têm por produto 5184. Se o menor deles é a maior potência inteira de 2, menor que 100, então o maior deles é a) uma potência de 5. c) múltiplo de 11. b) uma potência de 3. d) múltiplo de 7. 43 – Na figura, as cordas são dadas em cm. Se 1 x 4 AI + = , x IB = , 1 x DI + = e x 3 IC= , então a medida da corda AB é, em cm, a) 9. c) 11. b) 10. d) 19. 44 – Um retângulo tem área T. Se aumentarmos a medida da sua base em 20%, e diminuirmos a medida da sua altura em 20%, obteremos um novo retângulo cuja área é igual a a) T. b) 0,96 T. c) 1,04 T. d) 1,025 T. 45 – A equação geral da reta de coeficiente angular 2 3 e de coeficiente linear - 2 é a) x + 2 y – 4 = 0. c) 3x – 2 y – 4 = 0. b) 3x – 2 y – 2 = 0. d) 3 2 x – 2 y – 2 = 0. 46 – Na figura, o lado do hexágono regular inscrito no círculo mede 4 cm. A área da região hachurada da figura é, em cm 2 : a) 3 8t . b) 3 4 ÷ t . c) ( ) 3 3 2 8 ÷ t . d) ( ) 2 2 16 ÷ t . H A B C O - 47 – Para que o sistema ¹ ´ ¦ = + = + 0 y 3 x 0 my x 3 tenha solução diferente da imprópria, o valor de m deve ser a) 9. b) 0. c) 10. d) 15. 48 – A expressão x 2 tg 1 x 2 g cot 1 + + é idêntica à (ao) a) tg 2 x. b) sen 2 x. c) cotg 2 x. d) cos 2 x. 49 – Assinale a alternativa que complete corretamente o período. Júlia tem 8 filhos, resultado de 4 gestações de gêmeos. Se considerarmos as idades desses filhos, poderemos afirmar que elas formam uma série que apresenta _______ moda (s). a) nenhuma c) duas b) uma d) mais de duas 50 – O termo geral de uma PA é an = 3n – 16. A soma de seus 10 primeiros termos é a) 18. b) 14. c) 5. d) – 6. 51 – No ciclo trigonométrico, a igualdade ( ) 0 x sen = t é verdadeira se e somente se x é um número a) real qualquer. c) imaginário. b) inteiro. d) irracional. 52 – Seja uma função f do 1.º grau. Se f(-1) = 3 e f(1) = 1, então o valor de f(3) é a) – 1. b) – 3. c) 0. d) 2. 53 – Se permutarmos as letras da palavra TELHADO, quantas começarão e acabarão por vogal? a) 720 b) 120 c) 1080 d) 2160 54 – Dentro do conjunto dos números complexos, a equação 0 2 x x 2 4 = ÷ ÷ tem como soluções a) 2 ± e i ± . c) 1 ± e 2 i . b) 2 ± e i ± . d) 1 ± e i ± . 55 – A solução geral da equação sen 2 x – 2 senx cosx + cos 2 x = 0, sendo U = 9, é a) {x e 9 / x = 4 t + 2kt, keZ} . c) {- 4 t } . b) {x e 9 / x = 4 t + kt, keZ} . d) { 4 t } . 56 – A raiz da equação ( ) 5 3 1 9 2 6 1 10 1 3 2 ÷ ÷ = | . | \ | ÷ + ÷ x x x x é uma fração cuja diferença entre o numerador e o denominador é a) 35. b) 37. c) 45. d) 47. 57 – Na figura, AC AB= , M é o ponto de encontro das bissetrizes dos ângulos do triângulo ABC e o ângulo C M ˆ B é o triplo do ângulo Â, então a medida de  é a) 15 o b) 18 o c) 24 o d) 36 o 58 – O conjunto solução da inequação 2 2 x 2 1 > ÷ | . | \ | , sendo U = 9, é a) {x e 9 / x s -1 ou x > 1} . b) [ -1 , 1 ]. c) C. d) 9. 59 – Sendo "i" a unidade imaginária, o resultado de ( ) ( ) i 3 1 i 4 6 i 2 3 + ÷ ÷ + é a) i 3 1÷ ÷ . c) 5 i 39 5 13 ÷ ÷ . b) i 39 13÷ ÷ . d) 5 i 39 5 13 + . 60 – A função f:N÷N definida por ¦ ¹ ¦ ´ ¦ + = ímpar é n se , 2 1 n par é n se , 2 n ) n ( f é a) bijetora. c) somente sobrejetora. b) somente injetora. d) não injetora e não sobrejetora. 61 – Seja neN* / n < 312. A fração irredutível 312 n , escrita na forma decimal, é um (a) a) decimal exato. c) dízima periódica simples. b) número inteiro. d) dízima periódica composta. 62 – Observe: I - É sempre possível construir um polígono regular de n lados, para 3 n > . II - Triângulo é, em todos os possíveis casos, inscritível em uma circunferência. III - Um ângulo central ( ) c â de um polígono regular de n lados inscrito em uma circunferência mede ( ) n o 180 2 n â c ÷ = . IV- Sempre é possível construir uma circunferência que passa pelos n vértices de um polígono qualquer. Quantas das assertivas acima são falsas? a) 1 b) 4 c) 3 d) 2 63 – A equação da circunferência, em que os pontos ( ) 2 , 3 M ÷ e ( ) 4 , 5 N são extremos de um diâmetro, é a) 0 5 y x 2 2 = ÷ + . c) 0 7 y 6 x 2 y x 2 2 = ÷ ÷ ÷ + . b) 0 17 y x 2 2 = ÷ + . d) 0 5 y 6 x 2 y x 2 2 = ÷ ÷ ÷ + . 64 – Seja V o volume de um cubo de aresta "a". Constrói-se um prisma quadrangular de volume V‘ e de vértices nos pontos médios das arestas das bases do cubo. O volume V‘ desse prisma é igual a a) 2 V . b) V. c) 3 V . d) 4 V . 65 – Em um triângulo ABC, a bissetriz do ângulo A encontra BC em D, e a circunferência circunscrita, em E. Sendo cm 9 AE= e cm 4 DE= , então a medida EB, em cm, é a) 6. b) 5. c) 5 2 . d) 2 3 . 66 – Se forem indicados por m, n, e p os três lados de um triângulo e por M ˆ , N ˆ e P ˆ , respectivamente, os ângulos opostos a esses lados, então sendo conhecidos os lados m e n e o ângulo N ˆ , qual das fórmulas abaixo poderá ser utilizada para calcular o valor do lado p ? a) M ˆ cos np 2 p n m 2 2 2 · ÷ + = b) ( ) P ˆ M ˆ cos mp 2 p m n 2 2 2 + · + + = c) P ˆ cos mn 2 n m p 2 2 2 · ÷ + = d) ( ) N ˆ M ˆ cos mn 2 n m p 2 2 2 + · ÷ + = y x A B C D E x log y a = 67 – A curva da figura representa o gráfico da função ( ) 1 a , x log y a > = . Dos pontos ( ) 0 , 3 B e ( ) 0 , 9 C saem perpendiculares ao eixo das abscissas, as quais interceptam a curva em D e E, respectivamente . Se a área do trapézio retângulo BCED vale 9, a área do triângulo ABD, onde ( ) 0 , 1 A vale a) 2 1 . b) 2. c) 2 3 . d) 1. 68 – O ponto de maior ordenada, pertencente ao gráfico da função real definida por ( ) ( )( ) 1 x x 3 x f + ÷ = , é o par ordenado ( ) n , m . Então, " n m÷ " é igual a a) 3 ÷ . b) 3. c) 5. d) 5 ÷ 69 – Na progressão geométrica onde o primeiro termo é m 3 , o último é ( ) 21 m ÷ e a razão é ( ) 2 m ÷ , o número de termos é a) 8. b) 9. c) 11. d) 10. 70 – Ao dividir o polinômio "–5x 2 – 3x + 2" por um polinômio "Q", Ana obteve "–5" por quociente e "12x + 7" por resto. O polinômio Q é igual a a) x 2 + 3x – 2. c) x 2 – 3x + 1. b) x 2 – 3x – 1. d) x 2 + 3x + 1. 2/2003-TURMA B 36 – Todo número real positivo pode ser escrito na forma x 10 . Tendo em vista que 8 ~ 90 , 0 10 , então o expoente x, tal que 125 = x 10 , vale aproximadamente, a) 1,90. b) 2,10. c) 2,30. d) 2,50. 37 – Seja o um ângulo agudo. Se somarmos a medida de um ângulo reto à medida de o e, em seguida, subtrairmos dessa soma a medida do suplemento de o, obteremos sempre a medida de um ângulo a) nulo, qualquer que seja a medida de o. b) reto, qualquer que seja a medida de o. c) agudo, desde que 45° < med o < 90°. d) raso, desde que med o < 45°. 38 – Se 0 < x < 2 t , então a expressão tg 2 x + cotg 2 x é equivalente a a) 2 senx. b) 2 secx. c) 2 cosx. d) 2 cossecx. 39 – Na figura, as medidas dos lados BC e AC , AB são, respectivamente, 40 cm, 20 cm e 30 cm. A bissetriz interna desse triângulo, relativa ao vértice A, encontra o lado oposto no ponto P, e a bissetriz externa, relativa ao mesmo vértice, encontra o prolongamento do lado BC no ponto S. A medida do segmento PS, em cm, é igual a a) 30. b) 35. c) 40. d) 45. 40 – No emplacamento de automóveis da cidade paulista X, são usadas duas letras do alfabeto seguidas de quatro algarismos. O número de placas, começadas pela letra "A", seguida de vogal, inclusive "A", e de quatro algarismos distintos, sendo dois (2) o último algarismo, é a) 2.520. b) 720. c) 160. d) 3.600. 41 – Um teste de inteligência, aplicado aos alunos das 4 as séries do Ensino Fundamental da Escola A, apresentou os seguintes resultados: A freqüência relativa da classe modal é a) 0,2. b) 0,22. c) 0,25. d) 0,5. 42 – Se uma das dimensões de um paralelepípedo reto- retângulo é 6 cm, a soma das outras duas dimensões é 25 cm e a área total é 600 cm 2 , então a razão entre as duas dimensões desconhecidas é a) 3 2 . b) 5 3 . c) 2 1 . d) 5 2 . 43 – Num triângulo ABC, o lado maior AC mede 10 cm; o lado menor BC mede 3 cm; e o ângulo que eles formam mede 45º. O volume do sólido gerado pela rotação de 360º desse triângulo em torno do lado maior, em cm 3 , é a) 2 2 3 t . b) t 2 3 . c) 2 5t . d) 15t. 44 – A reta 3x – 2y – 5 = 0 é perpendicular à reta a) 2x – 3y = 5. c) 3x + 2y = 0. b) 4x + 6y = 1. d) 6x – 4y = 10. 45 – A equação 0 6 2 = ÷ + x x a) só tem uma solução. b) tem duas soluções, tais que seu produto é = – 6. c) tem duas soluções, tais que seu produto é = – 4. d) tem duas soluções, tais que seu produto é igual a 0. 46 – A raiz da equação 2 4 x 4 x 4 x 4 x = ÷ ÷ + ÷ + + pertence ao intervalo a) [ -3 , 4 [. c) ] 6 , 9 ]. b) ] 5 , 10 [. d) [ 0 , 8 ]. 47 – Num retângulo ABCD, os vértices A, B, C e D são consecutivos. Marcam-se na base AB, a partir de A, três pontos, E, F e G, de modo que eles assinalem, respectivamente, 4 3 e 4 2 , 4 1 da base AB. A razão entre as áreas do triângulo CEF e do retângulo ABCD é a) 4 1 b) 6 1 c) 8 1 d) 10 1 48 – Considere: 1 – Um triângulo isósceles PRQ, de base PQ e altura RH . 2 – Dois pontos T e S sobre RH , de tal modo que o triângulo PTQ seja eqüilátero e o triângulo PSQ seja retângulo em S. Considerando somente os ângulos internos dos triângulos, se somarmos as medidas de R ˆ e S ˆ , obteremos o dobro da medida de T ˆ . Sendo assim, a medida do ângulo R P ˆ T é a) 5º. b) 15º. c) 30º. d) 45º. Pontos n.º de alunos Pontos n.º de alunos 90|–– 95 40 115|–– 120 140 95|–– 100 60 120|–– 125 120 100|–– 105 140 125|–– 130 30 105|–– 110 160 130|–– 135 20 110|–– 115 180 135|–– 140 10 49 – A, B e P são pontos distintos de uma circunferência de centro O e raio r. Se AB é diâmetro da circunferência, e a medida do ângulo B A ˆ P , em radianos, é o, então a área da região limitada pelo ângulo B A ˆ P e o arco PB é igual a a) | . | \ | o + o 2 sen r r . c) | . | \ | o + o 2 2 sen r r . b) | . | \ | o + o 2 sen r 2 . d) | . | \ | o + o 2 2 sen r 2 . 50 – Os elementos de um conjunto A são tais que 10 deles são múltiplos de 4; 9 são múltiplos de 6; 8 são múltiplos de 12; e 4 são números ímpares. Se A c N (N = conjunto dos números naturais), então o número de elementos de A é a) 31. b) 25. c) 21. d) 15. 51 – Um triângulo escaleno está inscrito num semicírculo de 10 cm de diâmetro, que é o maior lado do triângulo. Se as medidas dos lados menores do triângulo são tais que uma é o dobro da outra, então a diferença entre as áreas do semicírculo e do triângulo, em cm 2 , é a) 2 40 25 ÷ t c) 2 20 25 ÷ t b) 2 30 25 ÷ t d) 2 50 25 ÷ t 52 – Seja P o conjunto dos retângulos, Q o conjunto dos quadrados e L o conjunto dos losangos. É correto afirmar que a) L · P = L - P c) L · Q = P b) L · Q = L - Q d) L · P = Q 53 – Num quadrilátero convexo, a soma de dois ângulos internos consecutivos é 190 o . O maior dos ângulos formados pelas bissetrizes internas dos outros dois ângulos desse quadrilátero mede a) 105º b) 100º c) 95º d) 85º 54 – Um barril, cuja forma é a de um cilindro reto, está repleto de vinho. Este vinho deve ser distribuído em copos cilíndricos de altura igual a 1/8 da altura do barril, e de diâmetro da base igual a 1/5 do diâmetro da base do barril. A quantidade de copos necessária para distribuir todo o vinho é a) 400 b) 300 c) 200 d) 100 55 – Calculando o valor do determinante 1 1 0 0 0 0 1 2 1 0 3 2 0 0 1 1 ÷ ÷ ÷ ÷ ÷ ÷ , obtém-se a) – 3. b) – 1. c) 1. d) 3. 56 – Uma das raízes da equação 0 1 x ) a tg 2 ( x 2 = ÷ ÷ é, sendo Z k , k 2 a e t + t ÷ = , a) tg a + cossec a. c) tg a + sen a. b) tg a – cos a. d) tg a – sec a. 57 – A divisão do polinômio P(x) por "x-a" fornece o quociente q(x) = x 3 + x 2 + x + 1 e resto 1. Sabendo que P(0) = –15, o valor de a é a) – 16 b) – 13 c) 13 d) 16 58 – A soma 1000 2 999 2 ... 3 2 2 2 2 1 + + + + + + é igual a a) 1 1000 2 ÷ c) 1 1000 2 + b) 1 1001 2 ÷ d) 1 1001 2 + 59 – O resultado da expressão 3 1 1 ... 666 , 0 5 , 7 de % 10 2 2 ÷ ÷ + ÷ ÷ é a) – 0,5 b) – 0,25 c) 0,75 d) 0,333... 60 – Se x e Z e f(x) é uma função tal que f(p+q) = f(p) . f(q) e f(2)= 2, então f(0) e f(– 2) são, respectivamente, a) 1 e 2 1 b) 0 e 2 1 c) 1 e 0 d) 1 e – 4 61 – Se m = 2 2 . 3 a . 5 2 . 7 3 e n = 2 3 . 3 5 . 5 3 . 7 b . 11, e mdc (m,n) = 18.900, então os valores de a e b são, respectivamente, a) 3 e 1 b) 2 e 3 c) 3 e 2 d) 2 e 2 62 – Seja f : 9 ÷ 9 uma função. O conjunto dos pontos de intersecção do gráfico de f com uma reta vertical a) é não enumerável. b) possui um só elemento. c) possui exatamente dois elementos. d) possui, pelo menos, dois elementos. 63 – Se os números 3, x e 10 são inversamente proporcionais aos números 5, 25 e y, então os valores de x e y estão compreendidos entre a) 0 e 1 b) 1 e 2 c) 1 e 3 d) 0 e 2 64 – Um arco mede 0,105 rd. Sua medida em graus é, aproximadamente, igual a a) 5 b) 6 c) 50 d) 60 65 – É par a função f : * 9 ÷ 9 definida por a) 2 x 1 ) x ( f = b) x 1 ) x ( f = c) x ) x ( f = d) 5 x ) x ( f = 66 – Se 20 cavalos consomem 30 toneladas de feno em 45 dias, então, durante quantos dias se podem alimentar 15 cavalos, com 40% menos toneladas de feno, dando a mesma quantidade de feno por dia? a) 24 b) 36 c) 40 d) 42 67 – Numa circunferência de centro C e raio 20 cm, considere a corda AB, cujo ponto médio é M. Se CM = 10 cm, então a medida de AB é, em cm, a) 5 15 b) 3 20 c) 15 d) 20 68 – Um certo jogo é composto de fichas de 5 cores diferentes. Se cada ficha vermelha vale tanto quanto 10 fichas azuis; cada azul, tanto quanto 10 verdes; cada verde, tanto quanto 10 pretas, e cada preta, tanto quanto 10 brancas, então é correto afirmar que a) a ficha verde é a de menor valor. b) a ficha branca é a de maior valor. c) cada ficha azul vale tanto quanto 100 pretas. d) cada ficha verde vale tanto quanto 1000 brancas. 69 – No ciclo trigonométrico: I - o arco 4 11t rad pertence ao 2 o quadrante. II - o arco 1510 o pertence ao 3 o quadrante. III - o arco | . | \ | t ÷ rad 3 13 pertence ao 4 o quadrante. A(s) assertiva(s) correta(s) é(são): a) II. b) I e II. c) I e III. d) I, II e III. 70 – Sendo i i 1+ um número complexo, seu conjugado vale a) i i 1÷ b) i i 1+ ÷ c) i 1+ d) i 1 i + RASCUNHO DO ALUNO GABARITO DAS PROVAS DE MATEMÁTICA 1 1/2001 - TURMA A 01 B 02 C 03 D 04 A 05 D 06 C 07 D 08 B 09 C 10 B 11 C 12 B 13 A 14 B 15 A 16 B 17 D 18 A 19 ANULADA 20 B 21 D 22 A 23 C 24 A 25 C 1/2001 - TURMA B 01 C 02 D 03 A 04 C 05 B 06 D 07 A 08 C 09 A 10 D 11 B 12 C 13 C 14 A 15 C 16 D 17 C 18 A 19 B 20 D 21 B 22 A 23 B 24 D 25 B 2/2001 - TURMA A 01 A 02 D 03 A 04 C 05 A 06 D 07 B 08 C 09 B 10 D 11 A 12 B 13 A 14 C 15 B 16 D 17 B 18 C 19 B 20 C 21 A 22 D 23 A 24 C 25 D 2/2001 - TURMA B 01 C 02 A 03 D 04 C 05 B 06 C 07 B 08 A 09 C 10 A 11 D 12 B 13 D 14 B 15 C 16 B 17 A 18 B 19 D 20 A 21 D 22 B 23 D 24 A 25 D 1/2002 - TURMA A 01 A 02 B 03 D 04 C 05 A 06 A 07 D 08 D 09 C 10 C 11 A 12 B 13 A 14 B 15 A 16 D 17 A 18 B 19 D 20 B 21 A 22 D 23 D 24 D 25 C 26 D 27 B 28 D 29 B 30 D 31 A 32 B 33 A 34 B 35 C 36 B 37 A 38 A 39 D 40 B 1/2002 - TURMA B 01 C 02 D 03 C 04 D 05 C 06 D 07 B 08 C 09 B 10 C 11 B 12 A 13 D 14 D 15 B 16 C 17 C 18 B 19 B 20 D 21 D 22 B 23 A 24 B 25 B 26 B 27 D 28 A 29 C 30 D 31 B 32 C 33 B 34 A 35 C 36 A 37 A 38 C 39 A 40 C 2/2002 – TURMA A 01 C 02 B 03 D 04 D 05 A 06 B 07 D 08 D 09 C 10 A 11 A 12 B 13 C 14 C 15 D 16 D 17 B 18 D GABARITO DAS PROVAS DE MATEMÁTICA 19 C 20 A 21 C 22 C 23 A 24 C 25 A 26 B 27 C 28 A 29 C 30 D 31 B 32 D 33 B 34 A 35 B 36 A 37 D 38 B 39 A 40 B 2/2002 – TURMA B 01 D 02 A 03 A 04 D 05 D 06 C 07 A 08 D 09 B 10 D 11 D 12 C 13 B 14 A 15 A 16 B 17 C 18 D 19 B 20 B 21 D 22 D 23 ANULADA 24 B 25 A 26 B 27 C 28 C 29 C 30 B 31 C 32 B 33 C 34 C 35 B 36 D 37 D 38 C 39 A 40 C 1/2003 – TURMA A 01 A 02 B 03 B 04 A 05 C 06 C 07 C 08 D 09 C 10 B 11 D 12 C 13 A 14 D 15 ANULADA 16 A 17 A 18 D 19 C 20 B 21 C 22 B 23 D 24 D 25 C 26 D 27 D 28 C 29 B 30 D 31 B 32 A 33 C 34 A 35 A 36 A 37 D 38 A 39 C 40 B 1/2003 - TURMA B 01 C 02 A 03 A 04 A 05 D 06 B 07 D 08 B 09 B 10 D 11 B 12 D 13 A 14 B 15 C 16 C 17 D 18 C 19 C 20 B 21 C 22 B 23 B 24 A 25 A 26 C 27 C 28 ANULADA 29 A 30 A 31 B 32 D 33 D 34 D 35 C 36 C 37 D 38 C 39 ANULADA 40 A 2/2003 - TURMA A 36 B 37 C 38 B 39 C 40 D 41 B 42 B 43 C 44 B 45 B 46 C 47 A 48 C 49 A 50 C 51 B 52 A 53 A GABARITO DAS PROVAS DE MATEMÁTICA 54 B 55 B 56 D 57 D 58 A 59 C 60 C 61 D 62 D 63 C 64 A 65 A 66 B 67 D 68 A 69 D 70 D 2/2003 - TURMA B 36 B 37 C 38 D 39 C 40 A 41 A 42 A 43 D 44 B 45 C 46 D 47 C 48 B 49 D 50 D 51 A 52 D 53 C 54 C 55 B 56 D 57 D 58 B 59 B 60 A 61 A 62 B 63 D 64 B 65 A 66 B 67 B 68 C 69 C 70 C 1 UÍMICA – EEAR 1/2001-TURMA A 26 – São exemplos de substâncias puras: a) aço, mercúrio metálico, gás oxigênio. b) água potável, água do mar, gás nitrogênio. c) cloreto de sódio, ferro metálico, gás metano. d) leite, sangue, vinagre. 27 – Pode-se afirmar que qualquer substância pura simples: a) pode ser sempre decomposta. b) nunca tem apresentação homogênea. c) é constituída apenas por moléculas monoatômicas. d) é constituída apenas por átomos de um mesmo elemento químico. 28 – O átomo de um elemento químico fica carregado positivamente quando: a) perde um ou mais prótons. b) perde um ou mais elétrons. c) recebe um ou mais prótons. d) recebe um ou mais elétrons. 29 – O grupo de elementos que apresenta apenas metais é: a) Cr, Cu, C. b) Fe, Ne, Te. c) Na, Ni, Ti. d) Pb, Pt, P. 30 – Pode ser classificada como uma decomposição a: a) combustão de uma fita de magnésio. b) eletrólise da água. c) hidratação da cal virgem (óxido de cálcio). d) reação entre nitrato de prata e cloreto de sódio. 31 – Os sais derivados do ácido sulfúrico são os: a) sulfatos. b) sulfetos. c) sulfitos. d) sulfuretos. 32 – A soma de dois vetores ortogonais, um de módulo 9 e outro de módulo 12, terá módulo igual a: a) 3 b) 15 c) 17 d) 21 33 – Duas forças de intensidades diferentes atuam sobre uma mesma partícula; então: a) certamente ela não está em equilíbrio. b) certamente a resultante é maior que cada uma das forças. c) ela só estará em equilíbrio se as forças forem perpendiculares entre si. d) ela estará em equilíbrio apenas se os sentidos das forças forem contrários. 34 – Um móvel descreve uma trajetória em MRUV, passando duas vezes pela posição S = 60 m com velocidade de módulo igual a 10 m/s em ambas as passagens. Os módulos da velocidade e da aceleração, no instante t = 0, valem, respectivamente, 40 m/s e 5 m/s 2 . Desse modo, dependendo do sentido inicial adotado, a posição S, em metros, no instante em que o móvel muda de sentido pode ser: a) 50 ou 70. c) –80 ou 80. b) 40 ou 80. d) –90 ou 150. 35 – No estudo da trajetória dos planetas do sistema planetário, o referencial adotado é: a) o Sol. b) a Terra. c) o centro da Galáxia. d) um satélite geo-estacionário. 36 – Qual das proposições abaixo é correta quanto à aplicação de uma força sobre um corpo? a) A aceleração é igual ao produto da força pela massa. b) A aceleração é diretamente proporcional à massa do corpo. c) A aceleração é diretamente proporcional à força aplicada. d) A aceleração é inversamente proporcional à força aplicada. 37 – Qual o trabalho, em kJ, que a resultante das forças atuantes sobre um corpo de massa 10 kg deve realizar para que o mesmo passe de uma velocidade de 10 m/s para a velocidade de 30 m/s? a) 1,5 c) 3,0 b) 2,5 d) 4,0 38 – A pressão exercida por um corpo sobre um plano horizontal de apoio é diretamente proporcional: a) à área de contato. b) ao peso do corpo. c) à forma do corpo. d) ao volume do corpo. 39 – A pressão atmosférica considerada normal é de aproximadamente: a) 1 milibar. b) 1 Pascal. c) 76 mm de coluna de mercúrio. d) 10 m de coluna de água. 40 – Ondas sonoras são ondas: a) eletromagnéticas. b) longitudinais. c) transversais. d) superficiais 41 – Duas ondas sonoras propagam-se simultaneamente em um mesmo meio, com freqüências de 600 Hz e 200 Hz. Podemos afirmar que estas ondas têm em comum: a) o mesmo comprimento de onda. b) o mesmo período de oscilações. c) a mesma velocidade de propagação. d) a mesma altura. 42 – À maior temperatura corresponde maior: a) massa. b) velocidade. c) quantidade de calor. d) grau de agitação térmica. 43 – O produto da pressão pelo volume de um gás é constante numa transformação: a) isotérmica. b) adiabática. c) isométrica. d) isobárica. 44 – A imagem real de um objeto real está a 30 cm de um espelho côncavo. A altura do objeto, colocado sobre o eixo focal, é de 6 cm, e de sua imagem, de 3cm. A distância focal do espelho, em centímetros, é de: a) 10 b) 15 c) 20 d) 60 45 – A velocidade da luz monocromática em um certo óleo corresponde a 3/4 de sua velocidade no vácuo. O índice de refração do óleo é: a) 0,334 b) 0,666... c) 0,750 d) 1,333... Q 46 – Em eletrostática, para se carregar _______________ um corpo, ____________________. I ÷ positivamente ÷ acrescenta-se prótons II ÷ negativamente ÷ acrescenta-se elétrons III ÷ positivamente ÷ retira-se elétrons IV ÷ negativamente ÷ retira-se prótons São corretas as afirmações: a) I e II. b) II e III. c) III e IV. d) todas. 47 – Diminuindo-se a distância entre as placas de um capacitor plano, a capacitância eletrostática: a) pode aumentar ou diminuir, dependendo da corrente elétrica aplicada. b) não se altera. c) aumenta. d) diminui. 48 – Um eletricista, distraidamente, ligou duas lâmpadas, uma de 10 W – 110 V e outra de 100 W – 110 V, em série a uma tomada de 220 V. Logo, ele verificou que: a) as duas lâmpadas acenderam com pouco brilho. b) as duas lâmpadas ― queimaram‖ imediatamente. c) a de 100 W acendeu com brilho acima do normal e ―se queimou‖. A de 10 W teve brilho menor e depois se apagou. d) a de 10 W acendeu com brilho acima do normal e ―se queimou‖. A de 100 W teve brilho menor e depois se apagou. 49 – O pólo sul de um imã atrai: a) o pólo norte do outro imã. b) o pólo sul de outro imã. c) cargas negativas. d) cargas positivas. 50 – Um dispositivo formado por uma barra de ferro envolvida por um solenóide caracteriza um: a) dínamo. b) gerador. c) eletroímã. d) transformador. 1/2001-TURMA B 26 – A equação química: H2 + Cl2 2 HCl representa a reação entre _________________ para produzir uma _____________. a) duas substâncias puras compostas - mistura b) duas substâncias puras simples - mistura c) os componentes de uma mistura - substância pura composta d) os componentes de uma mistura - substância pura simples 27 – A evaporação de um líquido puro é: a) apenas um fenômeno físico. b) um processo químico de decomposição. c) um processo de separação de mistura homogênea. d) um processo de separação de mistura heterogênea. 28 – Dados os números atômicos do ferro ( Fe = 26 ) e do argônio ( Ar = 18 ), pode-se simbolizar a configuração eletrônica do estado fundamental do íon Fe 3+ como: a) [Ar] 3d 3 4s 1 b) [Ar] 3d 4 4s 1 c) [Ar] 3d 3 4s 2 d) [Ar] 3d 5 29 – O trítio é, assim como o deutério, um hidrogênio pesado. Podemos então inferir que o trítio é um ______________ do hidrogênio. a) isóbaro b) isótono c) isótopo d) isoneutrônico 30 – A chamada ―queima‖ da cal (reação do CaO com água, comumente efetuada pelo ajudante do pedreiro) pode ser considerada uma reação de: a) adição. b) decomposição. c) permutação (dupla troca). d) substituição (simples troca). 31 – Entre as citadas abaixo, a base mais solúvel em água é o: a) hidróxido de alumínio. b) hidróxido ferroso. c) hidróxido de potássio. d) hidróxido plúmbico 32 – Um balão cheio de hidrogênio eleva-se na atmosfera. Admitindo que, a partir de um certo instante, o seu movimento seja retilíneo e uniforme, conclui-se que: a) seu peso é maior que o empuxo do ar. b) empuxo é menor que a resistência do ar. c) peso é igual à soma do empuxo com a resistência do ar. d) empuxo é igual à soma do peso com a resistência do ar. 33 – Uma partícula está em equilíbrio sob a ação de três forças coplanares de 3 N, 4 N e 5 N. Então podemos concluir que: a) as forças de 3 N e 4 N são perpendiculares entre si. b) as forças formam ângulos iguais entre si. c) as três forças têm mesma direção. d) as forças têm sentidos contrários. 34 – Dois corpos A e B na mesma vertical, como mostra a figura, encontram-se em repouso a 10 m um do outro. Abandona-se ― A ‖ e, 1 segundo depois,― B‖. O encontro dos dois corpos se dará em _____ segundos após o início da queda do corpo ― A ‖. Considere g = 10 m/s 2 e despreze qualquer tipo de atrito. a) 0,5 b) 1,0 c) 1,5 d) 2,0 35 – Lança-se, a partir do solo, uma pedra verticalmente para cima, com velocidade inicial v0 . A aceleração da gravidade no local vale g. Desprezando qualquer tipo de atrito e a resistência do ar, é correto afirmar que: a) a altura máxima atingida é g 2 v h 0 = . b) a pedra atinge o solo com velocidade de módulo maior que 0 v . c) sendo h a altura máxima atingida, o tempo de subida é g h t s = . d) tendo atingido o ponto mais elevado de sua trajetória, a pedra começa a retornar ao solo. O tempo de queda vale g v t 0 q = . 36 – O fato de um automóvel, que se move numa estrada plana e horizontal e que teve seu motor desligado, parar após um certo intervalo de tempo, é devido: a) à inércia. b) às forças de atrito. c) ao peso do automóvel. d) à força resultante ser nula. 37 – O princípio de Arquimedes não justifica um dos fenômenos a seguir: a) um balão subindo. b) a existência de um submarino. c) uma pessoa boiando numa piscina. d) equilíbrio de um líquido em vasos comunicantes, onde as superfícies estão no mesmo nível. A B 38 – Um automóvel de massa 800 kg é acelerado uniformemente a partir do repouso até uma velocidade de 50 m/s, em 20 s. A potência média desenvolvida por esse automóvel ao completar esses 20 s, em kW, será de: a) 25 b) 50 c) 100 d) 200 39 – A respeito da prensa hidráulica, é falso afirmar que é uma: a) máquina que pode multiplicar forças. b) aplicação do principio de Pascal. c) máquina que multiplica trabalho. d) máquina que transmite pressão. 40 – As ondas eletromagnéticas propagam-se no ar com uma velocidade aproximada de 300.000 km/s. Uma estação de televisão, que emite ondas cujo comprimento de onda vale 10 cm, tem uma frequência, em MHz, de: a) 3 b) 30 c) 300 d) 3000 41 – A velocidade de propagação das ondas sonoras em um mesmo meio: a) depende do timbre. b) independe da intensidade. c) independe da temperatura. d) independe das características deste. 42 – Em uma transformação isobárica, 12,5 mols de um gás perfeito variam sua temperatura de 100 K para 300 K. O trabalho realizado pelo gás, em atm.l, vale: a) 2,05 x 10 2 b) 2,05 x 10 3 c) 2,05 x 10 4 d) 2,05 x 10 5 Dado: Constante universal dos gases, R= 0,082 atm.l / (mol K) 43 – No fenômeno da refração, os raios luminosos modificam algumas de suas características ao atravessarem a superfície de separação entre dois meios . Uma dessas características é a(o): a) freqüência. b) velocidade. c) diâmetro. d) sentido. 44 – Um copo de vidro está completamente cheio com 250 cm 3 de óleo a 20ºC. O volume transbordado, em cm 3 , quando a temperatura do conjunto passa a 120ºC, vale: a) 11,6 b) 23,2 c) 24,4 d) 48,8 Dados: Coeficiente de dilatação linear (o) do vidro = 12 x 10 - 6 ºC - 1 Coeficiente de dilatação volumétrico (¸) do óleo = 500 x 10 - 6 ºC 1 45 – Uma lente delgada biconvexa convergente possui raios de curvatura iguais a 20 cm cada e vergência de 5 dioptrias, quando imersa no ar. O índice de refração da lente é: a) 0,5 b) 1,0 c) 1,5 d) 2,0 46 – No sistema em equilíbrio estático mostrado na figura, o corpo de massa M é mantido suspenso pela força de atração elétrica entre as cargas puntiformes de módulo Q1 = 2 µC e Q2 = 6 µC. O valor, em kg, da massa M é: a) 1,08 b) 2,00 c) 0,120 d) 0,200 Dados: Constante eletrostática k0 = 9,0 x 10 9 N. m 2 / C 2 Aceleração da gravidade local g = 10 m / s 2 Polias e fios ideais (massas desprezíveis) 47 – O consumo mensal (30 dias) , em kWh, de uma lâmpada comum (incandescente), cuja especificação é 40 W – 127 V, ligada diariamente por 120 min, vale: a) 2,0 b) 2,4 c) 4,0 d) 40 48 – Associam-se quatro resistores exatamente iguais de forma que a potência dissipada pelo conjunto seja de 100 W. A diferença de potencial (d.d.p.) aplicada aos extremos da associação vale 200 V. Uma possível associação destes resistores é em ___________, com cada resistor valendo ______________ kO. a) série; 4,0 b) série; 0,12 c) paralelo; 1,6 d) paralelo; 2,0 49 – "O momento escalar da força resultante de um sistema de forças é a soma algébrica dos momentos escalares das forças componentes, em relação a um pólo." Este enunciado refere-se ao(às): a) condições de equilíbrio de pontos materiais. b) conceito de centro de gravidade. c) Teorema de Arquimedes. d) Teorema de Varignon. 50 – A razão entre as intensidades dos campos magnéticos produzidos por uma corrente elétrica que atravessa um condutor retilíneo, respectivamente, a 6 e 12 m de distância do mesmo, vale: a) 4 1 b) 2 c) 4 d) 8 2/2001-TURMA A 26 – Em que estado a matéria apresenta principalmente as seguintes características: forma variável e volume constante? a) líquido. b) gasoso. c) sólido. d) vapor. 27 – Sabendo que o número atômico do elemento Hélio é 2, concluímos que, em relação ao átomo desse elemento, o número de: a) massa é dois. b) prótons é dois. c) elétrons é quatro. d) nêutrons é quatro. 10 cm M Q 2 Q 1 28 – Os componentes líquidos do petróleo são separados por meio da: a) fusão. b) decantação. c) destilação fracionada. d) dissolução fracionada. 29 – Têm-se dois átomos A e B. A soma das massas de A e B é igual ao triplo da massa do isótopo 12 do carbono; e a diferença das massas de A e B é igual a 1/3 da massa do isótopo 12 do carbono. As massas, em u.m.a., dos átomos A e B são respectivamente: a) 20 e 10 b) 20 e l6 c) 30 e 10 d) 30 e l6 30 – Dos grupos abaixo, o qual só possui metais é: a) Na, Nb, Ne, Ni b) Ca, Cl, Cr, Cu c) Li, Ti, Ni, Bi d) Pt, Pb, Pd, P 31 – Sabor azedo e conduzir bem a eletricidade, são características da função: a) sal b) base c) óxido d) ácido 32 – Que ângulo devem formar duas forças de módulos iguais, para que a resultante tenha módulo igual ao dobro do módulo das componentes? a) 0º b) 90º c) 120º d) 180º 33 – Um corpo está em equilíbrio sob a ação de três forças coplanares, duas das quais são perpendiculares entre si e têm módulos, respectivamente, iguais a 7,8 N e 10,4 N. Pode-se afirmar que o módulo da terceira força, em newtons, é a) 11 b) 12 c) 13 d) 14 34 – Sobre um plano inclinado sem atrito, de 20 m de comprimento e 5 m de altura, equilibra-se um corpo com uma força paralela ao plano de 150 N. Podemos afirmar que o peso do corpo vale ..... N. a) 300 b) 450 c) 500 d) 600 35 – Um trem de 200 m de comprimento, com velocidade escalar constante de 60 km/h, gasta 36 s para atravessar completamente uma ponte. A extensão da ponte, em metros, é de: a) 600 b) 500 c) 400 d) 200 36 – Um caminhão ao percorrer uma rodovia reta, passa pelo marco 50 km às 8horas e 20 minutos e pelo marco 170 km às 9 horas e 50 minutos. A velocidade média nesse intervalo foi, em km/h, de a) 60 b) 80 c) 92 d) 146 37 – O lançamento de foguetes tornou-se, desde a 2 a Grande Guerra Mundial, uma tecnologia bastante difundida. Em relação a um lançamento oblíquo no vácuo, pode-se afirmar que o alcance é máximo quando, necessariamente a) a velocidade e o ângulo de lançamento com a horizontal são máximos. b) a velocidade e o ângulo de lançamentos são mínimos. c) o ângulo de lançamento com a horizontal é qualquer. d) o ângulo de lançamento com a horizontal vale 45 0 . 38 – Assinale a proposição correta: a) No vácuo, um corpo mais e outro menos denso caem com a mesma aceleração. b) Um corpo no vácuo não cai, pois no vácuo não existe a força da gravidade. c) No vácuo, um corpo mais denso cai mais depressa que um menos denso. d) Na Lua, os astronautas não flutuaram, porque lá o vácuo não é perfeito. 39 – Quando, num MCU, o raio da trajetória for numericamente igual ao período, o valor numérico da velocidade linear do móvel será igual a a) t b) 2t c) 3t d) 4t 40 – Uma máquina, de potência P1 , realiza um trabalho mecânico T em 10 segundos; outra máquina, de potência P2 , realiza um trabalho mecânico 2T em 4 segundos. Comparando as potências dessas máquinas, podemos dizer que P1 eqüivale a a) 2 P 5 2 c) 2 P 5 b) 5 P 2 2 d) 5 P 2 41 – Impulso de uma força constante é o produto a) da força pela velocidade do corpo. b) do módulo da força pela velocidade da força. c) da força pelo intervalo de tempo em que atua. d) do módulo da força, pelo intervalo de tempo em que atua. 42 – ― O acréscimo de pressão exercido em um ponto de um líquido ideal em equilíbrio se transmite integralmente a todos os pontos desse líquido.‖ Este enunciado refere-se a um princípio, muito utilizado em prensas hidráulicas, denominado princípio de a) Pascal. b) Ampère. c) Arquimedes. d) Ação e Reação. 43 – Uma pessoa encontra-se na extremidade de um tubo metálico e ouve dois sons produzidos por uma martelada dada na outra extremidade do tubo. A diferença de tempo entre os dois sons é de 0,2 s. A velocidade do som no ar é de 340 m/s, e, no metal do tubo, é de 1.700 m/s. O comprimento do tubo, em metros, é de a) 34 b) 85 c) 170 d) 340 44 – A qualidade que nos permite distinguir dois sons de mesma altura e emitidos por fontes diferentes, chama-se a) timbre. b) registro. c) amplitude. d) intensidade. 45 – Um vaso contém água pura a temperatura e pressão ambientes. Para que esta água entre em ebulição deve-se reduzir a a) temperatura somente. b) temperatura e manter a pressão. c) pressão e manter a temperatura. d) temperatura e aumentar a pressão. 46 – Um automóvel que está com velocidade constante de 80 km/h sobre uma pista retilínea, vê no espelho retrovisor plano, a imagem de um poste que está à beira da estrada. A velocidade da imagem do poste em relação ao espelho do automóvel, em km/h, é de: a) 40 b) 80 c) 160 d) 200 47 – Existem quatro corpos condutores isolados A, B, C e D, sendo que: - ao se aproximar A de B, ocorre atração; - ao se aproximar B de C, ocorre atração; - ao se aproximar A de C, ocorre atração; - ao se aproximar B de D, ocorre repulsão. Portanto, é possível dizer que os corpos estão carregados da seguinte maneira: a) A está neutro, B positivo, C neutro e D negativo. b) A está neutro, B negativo, C positivo e D positivo. c) A está positivo, B negativo, C positivo e D neutro. d) A está positivo, B negativo, C neutro e D negativo. 48 – As substâncias magnéticas podem ser classificadas quanto à sua facilidade de imantação; assim as substâncias cujos imãs elementares se orientam facilmente quando submetidos à ação de um campo magnético, tais como ferro, níquel e cobalto, são ditas.............. . a) ferromagnéticas. b) paramagnéticas. c) diamagnéticas. d) adiabáticas. 49 – Nos vértices de um triângulo equilátero, localizam-se 3 cargas elétricas iguais em intensidade e polaridade. A tendência de movimento sentida pela carga 3 é melhor representada pelo vetor a) A b) B c) C d) D 50 – As lâmpadas incandescentes residenciais atualmente são fabricadas para funcionar em 127 volts. Neste caso, uma lâmpada de 100 watts possui resistência de ........... ohms. a) 1,27 b) 1,61 c) 127,00 d) 161,29 2/2001-TURMA B 26 – Em um laboratório, notou-se que um determinado frasco continha uma determinada substância. No frasco havia uma etiqueta que indicava apenas as propriedades dessa substância. Das propriedades indicadas no frasco, a única que não contribui para a identificação da substância é a) massa. b) densidade. c) composição em peso. d) coeficiente de solubilidade. 27 – A força ácida ou básica de um composto está diretamente ligada à questão a) do número de hidrogênio ionizáveis e oxidrilas no composto. b) da presença ou não de oxigênio na molécula. c) da eletronegatividade dos elementos. d) da solubilidade do composto em água. 28 – Quando se cozinha em fogões a gás, a chama azul fica amarela quando nela cai um pouco de sal ( cloreto de sódio). Esse fato corrobora a a) Teoria dos Quanta de Planck. b) Teoria da Relatividade de Eisntein. c) propriedade radioativa dos elementos químicos. d) experiência do espalhamento das partículas o realizada por Rutherford. 29 – As bases formadas por metais alcalinos – terrosos são consideradas a) fracas. b) insolúveis. c) monobases. d) pouco solúveis. 30 – O gás carbônico apresenta sempre a mesma fórmula molecular, quando obtida a partir das seguintes reações: síntese total, decomposição do carbonato de cálcio ou combustão do álcool comum. Este fato está de acordo com a lei de a) Dalton. b) Proust. c) Lavoisier. d) Gay- Lussac. 31 – Os gases nobres são considerados estáveis e formados por átomos isolados, então, podem ser classificados como a) gases compostos ideais. b) elementos químicos simples. c) exceção de substâncias compostas. d) substâncias puras simples monoatômicas. 32 – O H3PO3 é um a) triácido. b) diácido. c) tetrácido. d) monoácido. 33 – O óxido CO não reage com água, porque é a) um superóxido. b) um peróxido. c) um anfótero. d) indiferente. 34 – A intensidade média de corrente elétrica que percorre um fio condutor em um intervalo de tempo de 2 min, devido a uma carga de 1 C, vale aproximadamente, em ampères a) 0,008 b) 0,010 c) 0,120 d) 0,800 35 – Percorrendo-se, em Movimento Retilíneo Uniforme, uma determinada distância a 40 km/h, gastam-se duas horas a menos do que se percorresse a mesma distância, com o mesmo tipo de movimento, a 20 km/h. Qual o valor desta distância, em km ? a) 30 b) 80 c) 90 d) 100 36 – A partir do solo, lança-se um corpo verticalmente para cima com velocidade inicial de 20 m/s. Sendo a aceleração da gravidade local igual a 10 m/s 2 e desprezando a resistência do ar e qualquer tipo de atrito, pode-se afirmar que a) a altitude máxima atingida pelo corpo é de 40 m. b) no instante t = 3,0 s, a altitude em que se encontra o corpo é de 20 m. c) no instante t = 3,0 s, a velocidade do corpo vale, em módulo, 10 m/s. d) ao retornar ao solo, o corpo possui velocidade de módulo igual a 40 m/s. - - - A B C D 1 2 3 G R 1 R 2 R 3 R 4 G O x x 37 – Nos fenômenos ondulatórios de refração e reflexão, mantém-se constante o(a) a) pulso. b) freqüência. c) velocidade. d) comprimento de onda. 38 – Uma escala ― W ‖ foi criada atribuindo-se os valores de – 20ºW e 30ºW aos pontos de gelo e de vapor, respectivamente. Portanto, 50ºC corresponde em ºW a a) 50 b) 45 c) 15 d) 5 39 – Um objeto real, colocado a 60 cm de uma lente, conjuga uma imagem virtual que se encontra a 10 cm dessa mesma lente. Esta lente é do tipo ........................... e sua distância focal, em módulo, vale ............... cm. a) divergente – 12 b) divergente – 8,6 c) convergente – 12 d) convergente – 8,6 40 – Em uma ponte de Wheaststone, a leitura de corrente no galvanômetro G, como mostra a figura abaixo, vale, em ampères, Dados: R1= 5 O R2 = 2 O R 3 = 10 O R 4 = 4 O a) 4,00 b) 4,66 c) 5,00 d) zero 41 – O esquema abaixo representa uma polia que gira em torno do seu eixo, ponto ― O‖. As velocidades tangenciais dos pontos A e B valem, respectivamente, 50 cm/s e 10 cm/s. A distância AB vale 20 cm. A velocidade angular da polia, em rad/s, será de a) 1 b) 2 c) 3 d) 4 42 – As forças F1 e F2 atuam nos vértices de um triângulo equilátero de lado igual a x , conforme a figura. Os momentos de F1 e F2 em relação ao vértice C, possuem módulos, respectivamente, iguais a a) F1 x e F2 2 3 x b) F1 2 x e F2 2 x c) F1 x e F2 2 x d) F1 2 x e F2 2 x 43 – A resistividade elétrica dos materiais é uma característica própria destes, dependendo, também, de suas temperaturas. Desta forma, podemos garantir que se dois fios condutores têm a mesma resistividade, a) poderão ter a mesma resistência elétrica. b) certamente as resistências elétricas desses serão iguais. c) as temperaturas alcançadas por ambos serão sempre diferentes. d) poderão ter áreas de seção diretamente proporcionais às suas próprias resistências elétricas. 44 – A variação do comprimento de uma barra homogênea corresponde a 1 % de seu comprimento inicial, ao ser aquecida de 23ºC a 423ºC. O coeficiente de dilatação linear do material de que é feita a barra vale, em 10 - 5 º C – 1 , a) 1,0 b) 1,5 c) 2,5 d) 4,0 45 – Dois elevadores A e B de mesmo peso conseguem transportar uma mesma carga, do solo até o décimo andar de um prédio. O elevador A gasta 30 s e o elevador B gasta 40 s, ambos com velocidades constantes. A razão entre as potências mecânicas desenvolvidas por A e B, nesta ordem, é a) 3 2 b) 3 4 c) 4 3 d) 2 3 46 – Um espelho convexo, cujo raio de curvatura vale 10 cm, está distante de um objeto real, de altura 1cm, cerca de 4 cm. A distância focal e o tamanho da imagem colocada sobre o eixo focal do espelho, em centímetros, vale, respectivamente, a) 9 5 e 5 b) 5 9 e 5 c) 9 5 e 5 ÷ d) 5 9 e 5 ÷ 47 – A razão das variações entre os pontos de gelo e vapor na escala centígrada e em uma escala R é 2:7. Sabendo que o ponto de vapor na escala R vale 400ºR, podemos afirmar que o ponto de gelo nesta escala, em ºR, vale a) 50 c) 350 b) 100 d) 400 48 – Quando uma pessoa envelhece, seu cristalino vai, aos poucos, enrijecendo e perdendo a capacidade de acomodação visual. Este defeito da visão é denominado a) miopia. b) emetropia. c) presbiopia. d) hipermetropia. 49 – O diagrama representa a dilatação do comprimento l de uma barra metálica em função da temperatura. Se o coeficiente de dilatação da barra é 2 x 10 – 5 o C – 1 , o comprimento da barra a 50 o C é , em metros, a) 50,05 b) 50,50 c) 55,00 d) 500,5 Dado : l0 = comprimento inicial B x x A l (mm) t ( O C) 45 o l 0 0 x F 2 A B C F 1 l (mm) t ( O C) 45 o l 0 0 50 – O sistema mostrado abaixo encontra-se em equilíbrio estático. O valor, em graus, do ângulo o vale Dados: Massas: mA = 2 kg , mB = 3 kg , mC = 5 kg , g = 10 m/s 2 , Tração no fio 3 = 40 N. Despreze os atritos e considere os fios e polias ideais o a) 90 c) 45 b) 60 d) 30 1/2002-TURMA A 01 – Indique a alternativa que contém somente elementos químicos pertencentes à classe dos metais: a) O, F, Br, I. b) B, S, Ge, Sb. c) Na, Ne, S, Cl. d) Mg, Ca, K, Na. 02 – Observe as afirmativas a seguir: I. No mesmo período, os elementos apresentam o mesmo número de níveis. II. Os elementos do grupo 2 A apresentam, 2 elétrons na última camada. III. No mesmo grupo os elementos apresentam o mesmo número de camadas. Podemos concluir que, segundo à estrutura da classificação periódica dos elementos, está(ão) correta(s) a(s) afirmativa(s) a) I e II. c) II e III. b) I e III. d) somente a II. 03 – Uma usina hidroelétrica, por exemplo a de Itaipu, transforma energia a) elétrica em calor. b) mecânica em elétrica. c) elétrica em mecânica. d) química em física. 04 – Massa é a propriedade da matéria a) que mede o espaço ocupado por suas partículas. b) medida em quilograma-força, no sistema MkgfS. c) que indica o tipo de moléculas existentes em um corpo. d) relacionada com a quantidade de moléculas existentes em um corpo. 05 – O C60 é uma molécula geodésica, ou seja, com 60 vértices onde se localizam os átomos de carbono, que por ter uma forma muito parecida com uma bola de futebol, é também chamada de futeboleno. O futeboleno, o grafite e o diamante são exemplos de a) formas alotrópicas do carbono. b) moléculas iônicas de carbono. c) substâncias puras compostas. d) misturas carboníferas. 06 – Deseja-se separar dois componentes de uma mistura heterogênea, um líquido e outro sólido de maneira rápida. Para tanto, utiliza-se o processo de a) filtração simples. b) filtração à vácuo. c) decantação. d) levigação. 07 – O fato de uma carga elétrica em movimento irradiar continuamente energia, invibializou o modelo atômico de a) Rutherford. b) Thomson. c) Dalton. d) Bohr. 08 – Podemos afirmar corretamente que, no vácuo, a) a velocidade de um corpo em queda livre é proporcional à sua massa. b) a aceleração de um corpo em queda livre é proporcional à sua massa. c) a velocidade de corpos em queda livre é sempre uma constante. d) corpos em queda livre caem sempre com a mesma aceleração. 09 – Um corpo está sujeito à ação de várias forças que agem simultaneamente nele. Baseado nas leis de Newton, é correto afirmar que a) possivelmente o corpo está em repouso. b) necessariamente o corpo está em repouso. c) necessariamente a resultante das forças que agem no corpo é nula. d) necessariamente a aceleração do corpo é diretamente proporcional à sua massa. 10 – A soda cáustica e o amoníaco são típicos exemplos de a) sais. c) ácidos. b) bases. d) óxidos. 11 – Compostos químicos que, ao serem dissolvidos em água, liberam exclusivamente os cátions H + , chamam-se a) sais. c) ácidos. b) bases. d) óxidos. 12 – A soma de dois vetores de módulos 15 N e 9 N tem, certamente, o módulo do vetor resultante compreendido entre a) 6 N e 15 N. b) 6 N e 24 N. c) 9 N e 15 N. d) 9 N e 24 N. 13 – Em Estática existem duas condições de equilíbrio: a 1 a diz que a resultante do sistema de forças deve ser nula e a 2 a , que a soma algébrica dos momentos das forças do sistema, em relação ao mesmo ponto, também deve ser nula. Das condições citadas, estabelece o equilíbrio de um corpo extenso a) apenas a 1 a . b) apenas a 2 a . c) a 1 a e a 2 a alternadamente. d) a 1 a e a 2 a simultaneamente. 14 – Um automóvel está parado em um semáforo devido à indicação de sinal vermelho. Num certo instante, o sinal muda para verde e o automóvel entra em movimento. Logo a frente, ele encontra outro semáforo cuja indicação, mais uma vez, é vermelho, e pára novamente. O gráfico fornece a velocidade do automóvel entre estes dois semáforos. A dis- tância entre estes, em metros, é a) 900. b) 1.050. c) 1.100. d) 1.200. 15 – Observando-se a lei de Hooke, pode-se afirmar que a constante k de proporcionalidade das molas a) independe do material da mola. b) é igual para todos os tipos de mola. c) é uma característica de cada tipo de mola. d) não sofre alterações com variações de temperatura elevadas. 16 – Dos casos citados abaixo, indicar aquele onde o corpo em movimento não apresenta variação de energia potencial gravitacional. a) Um pára-quedista durante o salto. b) Um atleta, correndo numa pista horizontal. c) Uma bola de basquete ao ser arremessada para o cesto. d) Um automóvel, descendo uma ladeira com o motor desligado. A B C 1 2 3 30 0 30 10 50 V (m/s) t (s) 17 – Das unidades abaixo, aquela que se refere à energia cinética é a) s erg . c) 2 s m kg . b) watt. d) kg 2 2 s m . 18 – A luz, o som e o calor são exemplos de a) campos de força. b) diferentes formas de energia. c) sensações físicas que nada tem a ver com a energia. d) sensações físicas provocadas pela variação de energia. 19 – Dois corpos possuem, em um determinado instante, a mesma quantidade de movimento. Baseado nesse fato, é correto afirmar que a) certamente o produto da massa pela velocidade dos respectivos corpos é diferente. b) indubitavelmente as massas dos corpos são iguais. c) certamente as velocidades dos corpos são iguais. d) as velocidades dos corpos podem ser iguais. 20 – Uma força de 100 N, atuando perpendicularmente sobre uma superfície, provoca uma pressão de 1.000 pascals. A área da superfície é de a) 10 m². c) 10 cm². b) 0,1 m². d) 0,1 cm². 21 – A pressão exercida num ponto interno de um líquido, dentro de um recipiente, em equilíbrio não depende da a) densidade do líquido. b) forma geométrica do recipiente. c) profundidade em que se encontra. d) pressão externa exercida na superfície do líquido. 22 – Considerando um corpo descendo um plano inclinado liso (sem atrito) sob a ação exclusiva de seu peso, pode-se afirmar que a aceleração desse corpo é a) maior que a aceleração da gravidade local. b) sempre igual a aceleração da gravidade local. c) inversamente proporcional ao seu próprio peso. d) dependente do ângulo entre o plano inclinado e o plano horizontal. 23 – Leia com atenção um trecho da reportagem intitulada Imagem revela o peso dos frangos, editada na Folha de S. Paulo em 22 de maio de 2001: ― .... com base no tamanho das aves, um software revela também o seu peso.‖ Neste trecho existe uma inconsistência no conceito de uma grandeza física, que se deve ao fato de que a) o tamanho das aves deve ter a mesma unidade do peso destas. b) na realidade a reportagem deveria referir-se à massa e não ao peso dos frangos. c) o peso das aves é função da aceleração da gravidade local e, portanto, não tem relação com a massa destas. d) software é somente uma rotina computacional e não um processo correto na determinação do tamanho das aves. 24 – Vidro fosco, papel vegetal e tecido fino são exemplos de meios a) opacos. b) translúcidos. c) transparentes. d) monocromáticos. 25 – Considere que a sombra projetada de uma nuvem sobre o solo tenha, aproximadamente, a mesma forma e o mesmo tamanho da própria nuvem. Isto poderia ocorrer caso os raios solares fossem a) convergentes em um mesmo ponto. b) praticamente paralelos. c) pouco espessos. d) divergentes. 26 – Um raio de luz monocromático incide num prisma cujo índice de refração em relação ao ar é 1,60, conforme a figura. Na face AB o raio de luz deve a) refletir voltando sobre si mesmo. b) refratar com ângulo maior que 45º. c) refratar com ângulo menor que 45º. d) refletir com 45º, em relação à normal de AB, emergindo da face CB. 27 – Um submarino, completamente submerso, para emergir a) aumenta o empuxo exercido sobre ele. b) diminui o empuxo exercido sobre ele. c) aumenta seu peso. d) diminui seu peso. 28 – Um ponto material realiza um movimento periódico com intervalos regulares de 0,1 segundos. A freqüência, em Hz, desse movimento é de a) 10 -1 . c) 10 1 . b) 10 0 . d) 10 2 . 29 – Uma onda apresenta velocidade de propagação de 400 m/s e freqüência de 40 Hz. Quanto valerá o seu comprimento de onda? a) 1 m. c) 10 cm. b) 10 m. d) 16 km. 30 – Para calcularmos o período T de oscilação de um pêndulo simples, constituído de material metálico, utilizamos a expressão T = 2t(L/g) 1/2 onde ―g‖ é a aceleração da gravidade local e ―L‖ é o comprimento do pêndulo. Ao aquecermos o pêndulo, no mesmo local, podemos afirmar que seu período de oscilação a) não varia. b) diminui. c) aumenta. d) diminui, mas depois aumenta. 31 – Um gás perfeito, evoluindo isobaricamente, terá realizado trabalho nulo se sua temperatura final for a) maior que a inicial (Tf > Ti). b) menor que a inicial (Tf < Ti). c) igual à inicial (Tf = Ti). d) igual ao zero absoluto. 32 – Quando um sistema realiza um trabalho sobre o meio externo a ele, sem ceder ou receber energia sob a forma de calor, a) a evolução deste é dita isométrica. b) sua energia interna diminui. c) sua energia interna aumenta. d) a evolução não é adiabática. 33 – Os pára-raios foram inventados pelo cientista e estadista norte-americano Benjamim Franklin no século XVIII. O princípio de funcionamento dessa importante invenção é uma aplicação a) do poder das pontas. b) do campo magnético da Terra. c) da força eletromagnética dos raios. d) das idéias de blindagem eletrostática de Faraday. A B C 45º I 45º - - 34 – No circuito esquematizado o valor da corrente i, em ampères, que atravessa o galvanômetro é de a) zero. b) 3,8. c) 6,0. d) 10/11. 35 – No circuito abaixo, qual a corrente, em miliampères, que circula no filamento indicado, se este possuir as seguintes características: µ = 2 O mm 2 /m; A = 1 mm 2 e L = 10 m? a) 20. b) 40. c) 50. d) 100. 36 – A partir do SI (Sistema Internacional de Unidades) e utilizando a Lei de Coulomb, podemos afirmar que a unidade da constante dielétrica do meio, K, está corretamente relacionada em a) C m . N . c) 2 2 3 C s . m . kg . b) C m . N 2 . d) 2 2 3 C s . m . kg ÷ . 37 – Para utilizarmos corretamente uma bússola em perfeitas condições, devemos a) colocá-la no interior de uma caixa de ferro. b) não colocá-la em qualquer recipiente que possa ter propriedades magnéticas. c) colocá-la próximo a uma caixa de ferro para potencializar o efeito magnético. d) não colocá-la sobre a linha do Equador pois o campo magnético paralelo impede a leitura. 38 – Considere as seguintes afirmativas: I. Os imãs originam campos magnéticos. II. Cargas elétricas em movimento originam campos magnéticos. III. Cargas elétricas fixas originam campos magnéticos. São verdadeiras: a) I e II. b) I e III. c) II e III. d) I, II e III. 39 – Nos pontos internos de uma longa bobina percorrida por corrente elétrica contínua, as linhas de indução do campo magnético são a) hélices cilíndricas. b) circunferências concêntricas. c) retas paralelas ao eixo da bobina. d) radiais, com origem no eixo da bobina. 40 – Conhece-se, desde a Antiguidade, a propriedade de certos minerais de atrair pequenos pedaços de material ferroso. Um desses minerais e que constitui o chamado ímã natural é denominado a) esferoidita. b) magnetita. c) calcinita. d) gipsita. 1/2002-TURMA B 01 – Com base no gráfico abaixo, que relaciona as massas e os volumes das substâncias A, B e C, podemos afirmar que, em termos de densidade (d) m (g ) A a) dA > dB > dC. B b) dB > dC > dA. C c) dC > dB > dA. d) dA = dB = dC. V ( cm 3 ) 02 – Na atmosfera, além dos gases nitrogênio e oxigênio, existem os chamados gases nobres (hélio, neônio, argônio, criptônio, xenônio e radônio). A respeito desses gases podemos afirmar que a) não são elementos químicos pois são substâncias simples. b) são substâncias ultra-simples pois são formados por um único elemento químico. c) não se constituem em moléculas pois são formados por um único elemento químico. d) se constituem em moléculas de atomicidade 1 (também chamados de gases monoatômicos). 03 – Para separar o sal da areia, usa-se inicialmente o processo da a) destilação. c) levigação. b) dissolução. d) fusão. 04 – O Princípio de Heisenberg afirma que não é possível calcular a posição e a velocidade de um elétron num mesmo instante. Essa dificuldade levou Schrödinger a desenvolver o conceito de a) átomo. c) nêutron. b) núcleo. d) orbital. 05 – Se você fornecesse energia para arrancar um elétron de todos os elementos conhecidos, a energia seria maior para o elemento a) hidrogênio. c) hélio. b) bromo. d) flúor. 06 – O átomo que apresenta Z prótons e N nêutrons e o átomo que contém ( Z + 1) prótons e ( N – 1) nêutrons são a) isóbaros. c) isótonos. b) isótopos. d) alótropos. 07 – Um ano após a descoberta da radioatividade, Rutherford verificou que as radiações emitidas pelo urânio eram de dois tipos com diferentes poderes de penetração. As mais penetrantes foram chamadas de raios a) o (alfa). c) ¸ (gama). b) | (beta). d) o (delta). 08 – As substâncias que, em solução aquosa conduzem a corrente elétrica e contêm o grupo (0H) hidroxila, são os/as a) sais. c) bases. b) ácidos. d) óxidos. 50V G i 20 O filamento 100 O 40 O 12 V – + 3O 18O 4O 3O 10O 17O 09 – Fazendo o balanceamento da equação química BCl3 + P4 + H2 ÷ BP + HCl, obteremos, respectivamente, os coeficientes a) 4, 1, 5, 4, 10. c) 2, 1, 3, 2, 6. b) 4, 1, 6, 4, 12. d) 2, 2, 3, 2, 6. 10 – Reagem com água, formando bases e liberando O2, ou também reagem com ácido formando sais e H2O2, são os/as a) diácidos. c) monobases. b) peróxidos. d) sais duplos. 11 – No sistema mostrado abaixo, as roldanas e os fios são ideais e o atrito é considerado desprezível. As roldanas A, B e C são fixas e as demais são móveis sendo que o raio da roldana F é o dobro do raio das outras que são iguais entre si. Sendo a aceleração da gravidade local igual a 10 m/s 2 e a massa M de 4,0 kg, o valor, em módulo, da força capaz de equilibrar o sistema é, em newtons, a) 5,0. b) 8,0. c) 10. d) 20. 12 – Duas forças com intensidades diferentes atuam sobre uma mesma partícula; então a) certamente elas não estão em equilíbrio. b) certamente a resultante é maior que cada uma delas. c) elas só entrarão em equilíbrio se forem perpendiculares entre si. d) elas estão em equilíbrio, apenas se os seus sentidos forem contrários. 13 – Um carro foi de São Paulo até o Rio de Janeiro mantendo uma velocidade média de 80 km/h. Admitindo-se 400 km a distância entre as duas cidades citadas, pode-se afirmar que a) a velocidade mínima foi de 80 km/h. b) o carro não parou em nenhum instante. c) o carro gastou 5 horas para fazer a viagem. d) o ponteiro do velocímetro manteve-se durante todo percurso na marca de 80 km/h. 14 – Ao construirmos o gráfico da energia potencial e da energia cinética em relação ao tempo para um corpo em queda livre, obteremos respectivamente uma a) reta e uma reta. b) reta e uma parábola. c) parábola e uma reta. d) parábola e uma parábola. 15 – Os carros A e B deslocam-se em uma mesma estrada reta, de acordo com o gráfico. Em t = 0, ambos se encontram no quilômetro zero. Pode-se afirmar que das opções apresentadas abaixo, estão corretas I. Em t = 0, temos VA = 50 km/h e VB = 0 II. Ambos os carros se deslocam com movimento uniformemente acelerado III. De t = 0 a t = 2 h , A percorre 100 km e B percorre 150 km IV. A alcança B em 4 h a) somente a III. b) II, III e IV. c) II e III. d) I e III. 16 – Para um elevador de massa igual a 500 kg, admitindo a aceleração da gravidade igual a 10 m/s 2 e desprezando as forças de atrito, a tração no cabo vale: I. 600 N, quando o elevador sobe com aceleração constante de 2 m/s 2 . II. 5.000 N, quando o elevador sobe com velocidade constante de 5 m/s. III. 5.000 N, quando o elevador desce com aceleração constante de 2 m/s 2 . IV. 4.795 N, quando o elevador desce com aceleração constante igual a 0,5 m/s 2 . Das frases acima é(são) correta(s) a) apenas I. c) II e IV. b) apenas II. d) I e III. 17 – Qual das situações abaixo é falsa? a) Certo indivíduo pesa 700 N na Terra; logo, seu peso na Lua é também 700N. b) Medindo-se a massa de um corpo na Terra e na Lua, obtém-se o mesmo resultado. c) Peso e massa são grandezas diferentes; porém, quanto maior a massa de um corpo, maior o seu peso. d) Quando uma pedra está pendurada num barbante, a pedra produz uma tensão no barbante para baixo e o barbante puxa a pedra para cima. 18 – Com base nas informações dadas pelo gráfico F x AX (força x deformação) construído para duas molas A e B, podemos afirmar que, em termos de energia potencial elástica, a mola a) A acumula mais do que a mola B. b) B acumula mais do que a mola A. c) A acumula tanto quanto a mola B. d) A e a mola B tem outros detalhes não informados e que portanto nada se pode concluir sobre elas. 19 – Um corpo lançado sobre uma superfície plana horizontal e com atrito, tem sua velocidade variando com o tempo, de acordo o gráfico abaixo. Adotando g = 10 m/s 2 , o coeficiente de atrito entre o corpo e a superfície vale a) 0,1. b) 0,2. c) 0,5. d) 2,0. 20 – Uma esfera de 150 g de massa é abandonada de uma altura H do solo. Ao chocar-se com este, a esfera retorna à posição inicial. Nessa perspectiva, a variação de energia mecânica, em J, ocorrida durante o movimento total da esfera, sendo a aceleração da gravidade igual a 10 m/s 2 , vale DADO: Despreze qualquer tipo de atrito e considere o choque com o solo perfeitamente elástico. a) 0. c) 600. b) 7,5. d) 607,5. 21 – Uma pedra de massa 50 gramas é arremessada horizontalmente por um estilingue. Admita que a pedra abandona o estilingue com velocidade de 10 m/s e que o tempo de interação entre ambos seja de 0,5 s. Desse modo, a força utilizada, em newtons, no arremesso da pedra vale a) 100. c) 2.000. b) 1.000. d) 5.000. 22 – Um planeta hipotético ―X‖ gira em torno do Sol com um período de revolução, em anos, igual a 27 vezes o da Terra em relação ao Sol, obedecendo às leis de Kepler. Portanto, a distância ‖X‖ – Sol é .......... vezes a distância Terra – Sol. a) 3 c) 9 b) 6 d) 12 V (m/s) t (s) 15 3 0 A B C D E F M F 30 o linha de referência 10 4 5 2 F (N) AX (m) A B 0 100 B 2 V (km/h) t (h) 50 A 1 150 3 23 – A densidade de um determinado óleo comestível é de 0,80 g/cm 3 , sendo g = 10 m/s 2 , quanto pesa o óleo contido numa lata de 900 ml? a) 720 g c) 7,2 kg b) 7,2 N d) 0,72 N 24 – O casco externo de um submarino a 200 m de profundidade sofre uma pressão de aproximadamente ........... vezes a pressão atmosférica normal. a) 10 c) 100 b) 20 d) 200 25 – 26 – Uma onda sonora ao passar de uma região de menor temperatura para uma outra de maior temperatura, altera a) a altura. c) a freqüência. b) o período. d) o comprimento de onda. 27 – Ao passar do estado sólido para o líquido, sob pressão constante, uma substância cristalina a) cede calor e sua temperatura varia. b) absorve calor e sua temperatura varia. c) cede calor e sua temperatura permanece constante. d) absorve calor e sua temperatura permanece constante. 28 – No desenho a seguir vemos duas ondas propagando-se em uma corda, com a mesma velocidade e sentidos opostos. No instante em que coincidirem os pontos A e C e os pontos B e D, a forma da onda resultante será: a) b) c) d) 29 – A temperatura 0ºF equivale a .....ºC, aproximadamente. a) 32 b) -16,67 c) -32,00 d) -273,15 30 – Suponha que uma galáxia distante exista um planeta semelhante ao nosso sendo, contudo, que a luz que o ilumina seja monocromática. Um fenômeno óptico, devido a essa luz, que não seria observado no planeta em questão é o/a a) sombra. c) reflexão. b) refração. d) arco-íris. 31 – Aumentando-se o diâmetro do orifício de uma câmara escura, a imagem produzida no interior desta câmara a) perderá a nitidez. c) formar-se-á menor. b) formar-se-á maior. d) aumentará a nitidez. 32 – Um espelho convexo reflete a imagem de um objeto real de 10 cm de altura, colocado a 2 cm de distância do espelho, tendo esta imagem uma altura de 4 cm. A distância focal e o raio de curvatura, em módulo, do espelho, ambos em centímetros, valem respectivamente a) 8 3 e 3 4 . c) 3 8 e 3 4 ÷ . b) 3 8 e 4 3 . d) 3 8 e 4 3 ÷ . 33 – Duas cargas puntiformes, Q1 e Q2, estão se atraindo, no ar, com uma força F. Suponha que o valor de Q1 seja duplicado e o de Q2 se octuplique (multiplicado por 8). Para que o valor da força F permaneça invariável, a distância entre Q1 e Q2 deverá ser a) 4 vezes maior. c) 16 vezes menor. b) 4 vezes menor. d) 16 vezes maior. 34 – Uma lente de vidro cujos bordos são mais espessos que a parte central ....... Dados. µar = 2 0 µvidro = 2 0,5 µágua = 2 2 /3 a) é divergente no ar. b) nunca é divergente. c) é sempre divergente, não importando o meio. d) torna-se convergente mergulhada na água. 35 – Por meio de um raio, uma carga elétrica de 108 C é transferida de uma nuvem para o solo. Supondo que o potencial da nuvem mantenha--se constante durante toda descarga, determine o número de dias que uma lâmpada de 100 W poderia permanecer acesa, usando a energia liberada neste raio. Dado: Admita que o potencial de uma nuvem em relação ao solo vale 8 x 10 6 V. a) 100 c) 150 b) 120 d) 220 36 – Em uma residência estão instalados na rede 110 V, um chuveiro de 4.000 W, 10 lâmpadas de 100 W, um televisor de 70 W e uma geladeira de 100 W. Caso estes aparelhos fossem substituídos por outros, de mesma potência, mas que fossem instalados na rede 220 V, a corrente total consumida a) aumentaria 50 %. c) diminuiria 75 %. b) diminuiria 50 %. d) seria a mesma. 37 – O amperímetro é um aparelho destinado a medir a intensidade de corrente elétrica. Um amperímetro ideal é aquele que possui resistência a) nula. c) variável. b) infinita. d) múltipla. 38 – Ímãs são elementos a) formados por um elemento químico e um pólo magnético. b) complexos de cadeia carbônica oxi-reduzida. c) que atraem todos os tipos de metais. d) que possuem dipolo magnético. 39 – Nas figuras seguintes, está ocorrendo a passagem de corrente elétrica contínua (i), sentido convencional, nos condutores. Em cada situação está representado o vetor campo magnético perpendicular ao plano da folha de papel orientado para fora () e para dentro (©). Com base nestas informações, assinale a figura correta. a) c) b) d) A B C D © i  © B © © © © i B B © i  © B © © © © i B B © B © © © © i i © B © © © © ANULADA 40 – Dois condutores elétricos retilíneos são colocados paralelamente, um ao lado do outro. Quando percorridos por correntes elétricas contínuas, de mesma intensidade mas sentidos contrários, os condutores apresentarão a) atração mútua. b) repulsão mútua. c) manutenção na posição inicial. d) atração e repulsão mútua, no decorrer do tempo. 2/2002-TURMA A 41 – Um dos grandes problemas que surgem como desafio para a humanidade no início deste milênio é a escassez de água potável – o que poderá gerar conflitos internacionais na mesma proporção que os ocasionados pelo controle do petróleo nas últimas décadas. Daí a grande preocupação da comunidade científica em desenvolver técnicas para a obtenção de água potável, como por exemplo, a dessanilização da água do mar. Das alternativas abaixo, indique aquela que contém o melhor processo para separar a água do sal. a) Tamização c) Destilação fracionada b) Destilação simples d) Dissolução fracionada 42 – Considere as afirmativas abaixo: I- A água é uma mistura de hidrogênio e oxigênio. II- O ar puro das montanhas é uma substância pura. III- A filtração é um processo utilizado, tanto na separação de misturas sólido-líquido, como sólido-gás. Podemos considerar corretas a) II e III. c) somente II. b) I, II e III. d) somente III. 43 – A idéia segundo a qual a estrutura atômica deveria ser semelhante à estrutura do sistema solar é devida a a) Dalton. c) Lavoisier. b) Einstein. d) Rutherford. 44 – O raio atômico de um ânion é a) igual ao raio do átomo de origem. b) maior que o raio do átomo de origem. c) menor que o raio do átomo de origem. d) igual ao raio de um cátion do átomo de origem. 45 – As Leis volumétricas aplicam-se a todas as reações a) químicas. b) de combustão. c) entre substâncias gasosas. d) de que pelo menos um gás participe. 46 – Identifique o tipo de reação química abaixo: CaCO3 ÷ CaO + CO2 a) Análise c) Permutação b) Síntese d) Substituição 47 – A figura abaixo mostra um sistema constituído de duas barras A e B, rigidamente ligadas e em equilíbrio, suspensas pelo ponto P. A barra B, na horizontal, forma um ângulo de 150º com a barra A. Desprezando o peso das barras e sabendo que o comprimento da barra A é o dobro do comprimento da barra B, a relação entre os pesos dos corpos 2 e 1, suspensos nas extremidades das barras, vale a) 3 1 b) 3 3 c) 3 d) 3 48 – Dois vetores de módulos 3 e 4 são somados. Se a soma vetorial destes dois vetores é 37 , então eles formam entre si um ângulo, em graus, de a) 0 b) 30 c) 60 d) 90 49 – No esquema abaixo, os módulos dos vetores valem . 8 c e 7 b , 3 a = = =    O valor do vetor resultante, de acordo com o esquema citado, é a) 8 b) 7 c) 3 d) zero 50 – Um projétil foi disparado em um local onde se admite que qualquer tipo de atrito seja desprezível e que a aceleração da gra- vidade seja igual a 10 m/s 2 (constante). A direção do disparo formou um ângulo com a superfície horizontal de 30º, e a veloci- dade inicial do projétil valia V0. A distância horizontal percorrida pelo projétil, 2 segundos após o disparo, vale, em metros, a) 3 V0 b) 2 1 V0 c) 2 3 V0 d) 4 1 V0 51 – Durante o movimento de rotação de um disco de 36 cm de diâmetro, um ponto desenhado em sua periferia descreve arcos de 120º a cada 2s. Então, um ponto situado a 6 cm do eixo de rotação do disco terá uma velocidade linear, em t cm/s, igual a a) 1 b) 2 c) 3 d) 4 52 – Um automóvel, em movimento uniformemente variado, tem velocidade inicial de 10 m/s e aceleração igual a 10 m/s 2 . Após 5 segundos, sua velocidade média, em m/s, e a distância percorrida, em metros, valem, respectivamente, a) 40 e 185. c) 35 e 175. b) 45 e 190. d) 50 e 200. 53 – Uma partícula percorre uma trajetória circular de raio igual a 5 m, com velocidade linear de módulo constante. Entre os instantes 1 s e 5 s, seu percurso é de 80 m; o período, em segundos, do movimento apresentado será de a) 2 t . b) 4 t . c) 6 t . d) 8 t . i | i + 1 A 2 B 150 º - P a  b  c  54 – A Lei de Hooke é aplicável a) dentro do limite elástico do material da mola considerada. b) para qualquer tipo de força que atua na mola considerada. c) somente para carregamentos acima de 10 vezes o valor da gravidade. d) para valores que sempre são menores que a terça parte da constante elástica da mola. 55 – Para que a energia mecânica de um corpo se conserve, as forças que realizam trabalho não nulo sobre ele devem ser a) iguais. c) constantes. b) conservativas. d) dissipativas. 56 – A energia cinética é uma grandeza física que varia com a velocidade e é considerada a) absoluta, pois independe do referencial adotado. b) vetorial, pois depende da velocidade, que é um vetor. c) relativa, pois às vezes pode ser considerada um vetor e outras vezes um escalar. d) escalar, pois sua definição fica perfeitamente caracterizada apenas pelo módulo. 57 – Segundo Johannes Kepler (1571-1630), as órbitas descritas pelos planetas em torno do Sol são ______________, sendo que este ocupa um dos ______________ desta figura geométrica. c) circulares – focos c) elípticas – focos d) elípticas – vértices d) circulares – vértices 58 – Uma cachoeira lança 15 m 3 de água por segundo, forne- cendo uma potência de 4 . 10 3 CV. Assim sendo, a altura da queda d‘água vale _______ metros. Dados: 1 CV (cavalo-vapor) = 735 W (watt); densidade da água = 1 g/cm 3 ; e g = 9,8m/s 2 . a) 5 b) 10 c) 15 d) 20 59 – Um pára-quedista de massa 80 kg cai com uma velocidade constante durante um percurso de 100 metros. Admitindo g = 10 m/s 2 , qual é o trabalho realizado, em J, pela força resultante que sobre ele atua? a) zero b) 400 c)800 d) 8.000 60 – Um objeto real de altura X é colocado frontalmente e a 20 cm de uma lente convergente de vergência 2,5 dioptrias. O aumento linear transversal dessa lente vale, em cm, a) X / 2. b) X. c) 2X. d) 4X. 61 – O equilíbrio de líquidos em sistemas de vasos comunicantes pode ser considerado uma aplicação do princípio de a) Ohm. c) Galileu. b) Stevin. d) Thomson. 62 – Todo corpo mergulhado num líquido experimenta um empuxo vertical, de baixo para cima, igual, em módulo, ao ____________ do volume do líquido deslocado. a) módulo c) fluxo b) espaço d) peso 63 – Dos fenômenos descritos abaixo, qual representa um pro- cesso de transmissão de calor que NÃO pode ocorrer no vácuo? a) Irradiação c) Refração b) Convecção d) Reflexão 64 – O comprimento de onda, em cm, da onda que se propaga a 300 m/s, com uma freqüência de 5 kHz, vale a) 6. b) 15. c) 60 d) 150. 65 – Uma onda de freqüência 120 Hz e comprimento de onda igual a 2 cm passa de um meio para o outro. O ângulo de incidência é 45 o e o de refração, 30 o . A velocidade, no segundo meio, em cm/s, é a) 60. b) 120. c) 240. d) 120 2 . 66 – Um pulso propaga-se em uma corda composta e tensa, conforme a figura. O sentido de propagação do pulso é da corda de maior para a de menor densidade linear. A figura que melhor representa os pulsos e seus sentidos de propagação após a interação é a a) b) c) d) 67 – Com relação à velocidade de propagação do som, podemos afirmar que, de uma maneira geral, é a) nula nos sólidos. b) máxima no vácuo. c) maior nos gases do que nos sólidos. d) maior nos sólidos do que nos líquidos. 68 – No fenômeno da refração de ondas, necessariamente permanecem constantes a) a velocidade de propagação da onda. b) a freqüência das ondas. c) o comprimento de onda da onda. d) as amplitudes das ondas. 69 – O antimônio e o bismuto apresentam, durante o processo de fusão, comportamento inverso ao da maioria das substâncias conhecidas, pois seu volume ____________ com o aumento da pressão atuante no meio onde se encontra a substância. a) aumenta c) diminui b) oscila infinitamente d) permanece constante 70 – Um sistema recebeu do meio externo a ele uma quantidade de calor igual a 8.000 cal e realizou, sobre esse meio, um trabalho de 20.000 J. Em joules, qual a variação da energia interna desse sistema? ( considere 1 cal = 4,2 J) a) 13.600 c) – 12.000 b) 12000 d) – 13600 71 – Uma gilete corta profundamente porque a) a área de contato é grande e portanto a pressão é grande. b) a área de contato é pequena e portanto a pressão é grande. c) a área de contato é pequena e portanto a pressão é pequena. d) a área de contato é a mesma, mas a pressão é grande devido à inclinação da lâmina. 72 – Em alguns países, usa-se a escala Fahrenheit, que adota os valores 32 para o ponto de gelo e 212 para o ponto de vapor. O intervalo entre essas duas medidas é dividida em ______ partes. a) 32 b) 100 c) 180 d) 212 73 – Uma lente de vidro plano-côncava, cujo índice de refração absoluto vale 1,5 e que está imersa no ar (índice de refração absoluto 1,0), tem a face curva com 10 cm de raio. Sua vergência, em dioptrias, é igual a a) – 5 b) – 3 c) 3 d) 5 74 – Segundo o Princípio de Pascal, os líquidos a) são compressíveis. b) multiplicam as pressões que suportam. c) transmitem integralmente as pressões que suportam. d) exercem sempre a mesma força em todos os pontos da prensa hidráulica. - O 5m 5m 5m 1 F  2 F  3 F  75 – A membrana de natureza nervosa, sensível à luz e que está ligada ao nervo óptico, é denominada a) retina. c) córnea. b) pupila. d) esclerótica. 76 – Calcular, em watts, a potência total fornecida pela bateria. Dado: o valor lido no amperímetro da figura é de 6 A. a) 60 b) 120 c) 180 d) 240 77 – Dispõem-se de quatro esferas metálicas carregadas: P, Q, R e S. Sabe-se que P repele Q, P atrai R, R repele S, e S está carregada positivamente. Pode-se dizer que a) P está carregada positivamente. b) P e R têm cargas de mesmo sinal. c) Q tem carga negativa. d) P e Q estão carregadas positivamente. 78 – Ao dizermos que um motor elétrico tem uma potência de 600W, estamos afirmando que nesse motor a) há um potencial elétrico de 600 volts a cada segundo, sem a necessidade de existência de corrente elétrica. b) 600 coulombs de carga elétrica são transportados a cada segundo, de uma extremidade à outra do circuito. c) 600 coulombs de carga elétrica são transformados em 1 joule de energia mecânica a cada segundo. d) 600 joules de energia elétrica são transformados em energia mecânica a cada segundo. 79 – São exemplos de objetos que se utilizam do princípio do eletroímã: a) telefone e campainha. b) cafeteira elétrica e campainha. c) lâmpada incandescente e telefone. d) retificador de corrente e ponte de Wheatstone. 80 – Quando uma corrente elétrica circula por um fio, gera ao redor deste um a) fluxo elétrico. c) circuito elétrico. b) campo magnético. d) pêndulo elétrico. 2/2002-TURMA B 41 – A maioria dos pequenos produtores de aguardente de cana (―pinga‖), no Brasil, utilizam ainda um pequeno equipamento criado pelos alquimistas, conhecido por alambique, que separa os componentes da mistura pelo processo de a) liquidação fracionada. b) destilação fracionada. c) destilação simples. d) adsorção. 42 – No alto de uma montanha, a temperatura de ebulição da água se dá: a) abaixo de 100º C. b) acima de 100º C. c) a 100º C. d) a 0º C. 43 – Das alternativas abaixo, é exemplo de substância pura: a) leite . b) latão. c) ar sem poluição. d) tetracloreto de carbono. 44 – Dados quatro átomos: 20A, 12B, 38C e 4D, qual deles apresenta maior raio atômico? a) 12B b) 38C c) 20A d) 4D 45 – Se um elétron move-se de um nível de energia para outro mais afastado do núcleo do mesmo átomo, pode-se afirmar que a) há emissão de energia. b) há absorção de energia. c) o número atômico varia. d) não há variação de energia. 46 – O bombardeamento da folha de ouro com partículas alfa, no experimento de Rutherford, mostra que algumas dessas partículas sofrem desvio acentuado no seu trajeto, o que é devido ao fato de que as partículas alfa a) chocam-se com as moléculas de ouro. b) têm carga positiva e são repelidas pelo núcleo. c) são muito lentas e qualquer obstáculo as desvia. d) são grandes demais e não podem atravessar a lâmina de ouro. 47 – A pirita, o fluoreto de sódio e a gipsita são típicos exemplos de a) sais. b) bases. c) ácidos. d) óxidos. 48 – A pirólise, a fotólise e a eletrólise são reações de decomposição pelo calor, pela luz e pela eletricidade, respectivamente. Estas reações também são denominadas reações de a) análise. b) substituição. c) deslocamento. d) dupla substituição. 49 – Considere as forças atuantes sobre a barra, de peso desprezível, conforme a figura. Qual o módulo do momento resultante, em N.m, em relação ao ponto O? Dados: N 3 F e N 5 F , N 3 F 3 2 1 = = =    a) 30 b) 40 c) 50 d) 70 50 – Qual alternativa só contém grandezas vetoriais? a) comprimento, massa e força. b) tempo, deslocamento e altura. c) força, deslocamento e velocidade. d) massa, velocidade e deslocamento. 51 – A figura, abaixo, mostra um bloco de peso P  sustentado por fios ideais. Calcule o módulo da força F  horizontal, supondo que o conjunto esteja em repouso. a) F = P . tg u b) F = P . sen u c) F = P . cos u d) F = P . sen u . cos u 52 – Observe as equações horárias da velocidade dos móveis I, II e III, supondo que a trajetória de todos os três seja retilínea: móvel I : V = 2 + 3t móvel II : V = – 5 – 3t móvel III : V = 3 Elas representam, respectivamente, movimentos a) uniforme, uniformemente retardado e uniforme. b) uniformemente acelerado, uniformemente acelerado e uniforme. c) uniformemente acelerado, uniformemente retardado e uniforme. d) uniformemente retardado, uniformemente acelerado e uniforme. 30V A 15O R1 R2 R3 F  P  u Fio 1 Fio 2 53 – "O guepardo, também conhecido como chitá, é o mais rápido dos animais terrestres. Ele depende de sua velocidade de até 120 km/h para alcançar animais velozes como gazelas e antílopes..." (revista SuperInteressante, dezembro de 2000). Admitindo que o guepardo desenvolva sua velocidade máxima, como descrita acima, e sendo constante essa velocidade por 10 segundos, a distância percorrida, em linha reta, por esse animal durante este intervalo de tempo vale aproximadamente a) 333 m. b) 333 km. c) 360 km. d) 360 m. 54 – Uma força de intensidade igual a 10 9 N foi decomposta em duas componentes ortogonais, de modo que a intensidade de uma é o triplo da outra. Qual é, em newtons, a intensidade de cada componente? a) 3 e 9 c) 10 e 30 b) 9 e 27 d) 81 e 243 55 – O movimento de translação da Terra, em relação ao Sol, pode ser aproximado, com algumas restrições, a um movimento circular uniforme. Nesse caso, podemos afirmar que, durante seu movimento, a Terra possui ____________ constante. a) posição b) aceleração c) velocidade linear d) velocidade angular 56 – Dois trens correm em trilhos paralelos, deslocando-se na mesma direção e no mesmo sentido. O passageiro do primeiro trem, cujo módulo da velocidade é de 80 km/h, passa pelo segundo trem, que possui uma velocidade de módulo igual a 70 km/h. Admitindo que o movimento dos trens seja retilíneo e uniforme, qual o comprimento, em metros, do segundo trem, se o passageiro o vê durante 1 min e 12s? a) 300 c) 200 b) 250 d) 150 57 – Um móvel descreve um movimento circular uniforme obedecendo à função horária o = 2 t + tt, sendo as unidades dadas no Sistema Internacional de Unidades. Com a trajetória de raio igual a 0,5 m, qual o comprimento do arco descrito pelo móvel, em metros, no intervalo de tempo de 2s? a) t c) 2,00t b) 1,25t d) 2,50t 58 – A força resultante que atua sobre uma pequena esfera, que cai verticalmente no interior de um líquido, torna-se nula a partir de um determinado instante. A partir desse instante, a esfera a) permanece parada. b) é acelerada para cima. c) é acelerada para baixo. d) continua descendo com velocidade constante. 59 – Uma esfera, de dimensões desprezíveis, possui peso igual a 10 N. Essa esfera encontra-se suspensa verticalmente por um fio ideal de comprimento 50 cm, cuja outra extremidade está fixa em um teto. Deslocando-se, lentamente, a esfera de sua posição de equilíbrio até uma nova posição na qual o fio esticado faça com a vertical um ângulo de 60 o , pode-se afirmar que sua energia potencial sofrerá uma variação total, em J, de a) 5,0 c) – 2,5 b) 2,5 d) – 5,0 60 – Uma cachoeira lança 20 m 3 de água por segundo, de uma altura de 15 m. Assim sendo, a potência fornecida, em CV, vale Dados: densidade absoluta da água = 1,0 g/cm 3 ; 1 CV(cavalo-vapor) = 735 W; e g (aceleração da gravidade local) = 9,8 m/s 2 . a) 1000 c) 3000 b) 2000 d) 4000 61 – Em relação ao movimento dos planetas em torno do Sol, segundo as leis de Kepler, é correto afirmar que a velocidade linear, em módulo, dos planetas é a) maior quando eles estão no periélio. b) menor quando eles estão no periélio. c) maior quando eles estão no afélio. d) sempre constante. 62 – Alguns pedreiros utilizam um pedaço de tubo ou mangueira, preenchido com água, para verificar os níveis (ou alturas) das superfícies em que estão realizando suas obras. Pode-se dizer, portanto, que isto é uma aplicação prática a) da Lei de OHM. b) do Princípio da Ação e Reação. c) da Lei da Gravitação Universal. d) do Princípio dos Vasos Comunicantes. 63 – No vaso abaixo, qual é, em bárias, a pressão no ponto ―A‖, sabendo que a densidade do líquido é de 0,8 g/cm 3 ? (Despreze a pressão na superfície do líquido e considere g = 10 m/s 2 ). a) 40.000 b) 50.000 c) 400.000 d) 500.000 64 – Um balão de festa junina começa a subir porque a) a pressão dos gases no balão é menor que a pressão atmosférica. b) o peso do balão é menor que o peso do ar que ele desloca. c) a aceleração da gravidade diminui com a altitude. d) o volume do balão diminui quando o balão sobe. 65 – Observa-se que a difração é tanto mais intensa quanto a) maior for o obstáculo. b) menor for o obstáculo. c) mais forte for o sinal emitido pela fonte. d) mais fraco for o sinal emitido pela fonte. 66 – A qualidade do som que permite, na maioria das situações, distinguir a voz de uma criança ou de uma mulher, em relação à voz de um homem, é denominado a) altura. b) intensidade. c) velocidade de propagação. d) densidade do meio material. 67 – Uma garrafa de alumínio (coeficiente de dilatação linear o = 22 x 10 -6 ºC -1 ), com volume de 808,1 cm 3 , contém 800 cm 3 de glicerina (coeficiente de dilatação volumétrica ¸ =147 x 10 -6 ºC -1 ) à temperatura de 0ºC. A temperatura, em ºC, a que deve ser aquecido o conjunto para que o frasco fique completamente cheio, sem haver transbordamento de glicerina, é de aproxi- madamente, a) 100. b) 125. c) 225. d) 375. 68 – Um tubo sonoro, de comprimento igual a 0,5 m, apresenta as duas extremidades abertas. Sabendo que a velocidade do som no ar é igual a 340 m/s, e que a freqüência do som emitido é de 1700 Hz, conclui-se que o tubo está produzindo o __ harmônico. a) 1º b) 3º c) 5º d) 6º 69 – A coluna de mercúrio de um termômetro apresenta uma altura de 3 h 2 , quando a 0ºC, e 4 h 11 , quando a 100ºC, sob pressão normal. A temperatura correspondente à altura ―h‖ da coluna vale, em ºC, a) 16. b) 18. c) 22. d) 33. 10m 30º A - 70 – Flávio, um brilhante estudante de Física, comprou um termômetro clínico graduado. Junto ao termômetro veio um manual de instrução, onde se lia ―deixe o termômetro sob as axilas ou na boca por aproximadamente 3 minutos. Após esse tempo, faça a leitura da temperatura‖. O estudante pode concluir, corretamente, que: a) o tempo não altera a leitura da temperatura. b) este tempo poderia ser aumentado para 5 minutos sem afetar a medição. c) a leitura deve ser feita somente com 3 minutos, não podendo ultrapassar esse tempo. d) a correta leitura deve ser feita imediatamente após o contato do bulbo do termômetro com o corpo. 71 – ―Água que o Sol evapora Pro céu vai embora Virar nuvem de algodão‖ O trecho acima, retirado da música ―Planeta Água‖, de Guilherme Arantes, faz referência à mudança de estado físico da água a partir da energia térmica do Sol que é transferida para esta última, principalmente, pelo processo de a) convecção. c) condução. b) irradiação. d) difração. 72 – Em relação à velocidade de propagação de luzes monocromáticas, pode-se afirmar corretamente que a luz a) vermelha é mais lenta que a violeta no vácuo. b) violeta é mais lenta que a vermelha no vácuo. c) violeta é mais rápida que a vermelha num meio material. d) vermelha é mais rápida que a violeta num meio material. 73 – O satélite artificial Hubble possui um telescópio que usa um espelho _______________ para ampliar as imagens das estrelas. a) plano. c) convexo. b) côncavo. d) plano, com inclinação variável. 74 – Um prisma eqüilátero ( índice de refração n= 2 ) está imerso no ar (índice de refração n= 1). O desvio mínimo, em graus, sofrido por um raio luminoso monocromático ao atravessá-lo é a) 30. b) 45. c) 60. d) 0. 75 – ANULADA 76 – O gráfico que melhor representa a relação entre intensidade de corrente elétrica ( i ) e o tempo ( t ), no caso de corrente contínua, é a) c) b) d) 77 – A conta de luz de uma residência apresenta os seguintes dados: leitura anterior kWh leitura atual kWh importância a ser paga em R$ 5250 5750 100,00 Considerando esses dados, quanto custaria, em R$, a iluminação de uma casa, na qual o consumo seria dado apenas por 20 lâmpadas de 100W e 120V que permaneceram acesas 4 horas por dia durante 30 dias? a) 28,00 b) 38,00 c) 48,00 d) 52,00 78 – Observe: I - Para o estudo do campo magnético, convencionou-se que as linhas de campo magnético são orientadas do pólo sul para o pólo norte. II - As substâncias diamagnéticas, tais como cobalto e níquel, não possuem propriedades magnéticas, não podendo, portanto, ser imantadas. III - Quando um ímã é dividido em várias partes, cada uma das partes comporta-se como um novo ímã. IV - Em torno de um fio condutor retilíneo longo, percorrido por corrente elétrica, surge um campo magnético. São corretas as afirmações a) I, II, III e IV. c) I, II e III. b) II, III e IV. d) III e IV. 79 – O trabalho para deslocar uma carga elétrica entre dois pontos que pertençam à mesma superfície eqüipotencial a) depende do valor da carga. b) é negativo. c) é infinito. d) é nulo. 80 – Duas partículas A e B possuem cargas elétricas nula e –2e , respectivamente, em que e é a carga do elétron em módulo. Tais partículas atravessam, separadamente, um campo magnético constante perpendicular ao plano de movimento destas, como mostra a figura. A trajetória das partículas pode ser expressa por a) c) b) d) 1/2003-TURMA A 41 – Quando só há interesse em se obter um sólido de uma mistura entre um sólido e um líquido, como é o caso da obtenção do sal de cozinha a partir da água do mar, deve-se fazer uma a) liquefação. c) destilação simples. b) evaporação. d) destilação fracionada. 42 – Ocupa o volume de 22,4 litros um mol de qualquer a) substância. b) substância gasosa. c) substância líquida. d) substância gasosa nas condições normais de temperatura e pressão. 43 – O elemento químico 56 26 X pode ser classificado como a) gás nobre. c) metal de transição. b) metal alcalino. d) metal alcalino terroso. 44 – Sabendo-se que o valor numérico máximo da soma de dois vetores é 20 e o mínimo é 4, os módulos dos vetores que conduzem a estes resultados são a) 9 e 11. c) 20 e 4. b) 8 e 12. d) 16 e 4. t i t i i t i t X X X X X X A B B A A B A B A B 45 – Os ponteiros de um relógio realizam movimento circular que pode ser considerado uniforme. Qual será, em rad/s, a velocidade angular do ponteiro dos segundos? a) 2 t b) t 2 c) 20 t d) 30 t 46 – Alguns cães balançam suas caudas com uma freqüência aproximada de 1 Hz. Nesse caso, pode-se afirmar que a velocidade linear de um ponto, em m/s, na extremidade da cauda, é da ordem de a) 3 1 b) 5 3 c) 3 5 d) 3 7 47 – Que aceleração existe no movimento circular uniforme? a) Centrípeta c) Deslizante b) Tangencial d) Curvilínea 48 – Deixa-se cair de uma mesma altura e ao mesmo tempo três objetos de formas e volumes iguais, sendo um de ferro, um de chumbo e outro de isopor. Admitindo que a densidade do isopor é menor que a do ferro e que esta é menor que a do chumbo, podemos afirmar que Dado: existe atmosfera no local da queda dos corpos. a) o objeto de ferro chegará primeiro ao solo. b) o objeto de isopor chegará primeiro ao solo. c) o objeto de chumbo chegará primeiro ao solo. d) todos os objetos, independente do material que os constitui, chegarão juntos ao solo. 49 – Analisando o gráfico da posição pelo tempo de uma partícula em movimento retilíneo uniforme, podemos afirmar que o significado físico do coeficiente linear deste é a) a velocidade média do móvel. b) a velocidade inicial do móvel. c) a velocidade instantânea do móvel. d) a posição inicial do móvel, em relação ao referencial adotado. 50 – O Princípio da Conservação da Energia Mecânica desconsidera a existência de a) força. c) atrito. b) massa. d) gravidade. 51 – Em 1751, um meteorito de massa 40 kg caiu na Terra, abrindo uma cratera com 4 metros de profundidade. Investigações sobre a força de resistência oferecida pelo solo nas vizinhanças da colisão, mostraram que o seu valor foi de 5 x 10 4 N. A velocidade, em m/s, com que o meteorito chegou à superfície da Terra vale a) 25. b) 50. c) 100. d) 75. 52 – A grandeza dada pela relação entre a intensidade da força resultante e a área em que ela é aplicada denomina-se a) impulso. c) pressão. b) pulso. d) depressão. 53 – O som mais grave é o de a) menor freqüência. c) maior intensidade. b) maior freqüência. d) menor intensidade. 54 – Uma pessoa submersa em uma piscina ouve o ruído de uma explosão ocorrida fora da água. O fenômeno ondulatório que certamente ocorreu na fronteira entre o ar e a água é denominado a) difração. c) dispersão. b) refração. d) ressonância. 55 – Uma corda de comprimento L é posta a vibrar continuamente entre dois extremos fixos. Observa-se ao longo de todo seu comprimento quatro nós e três ventres. Calcule a relação entre o comprimento da corda e o comprimento de onda na estacionária formada. a) 0,15 b) 1,5 c) 15 d) 150 56 – A temperatura, a uma dada pressão, acima da qual uma substância é considerada um gás, é denominada a) fundamental. c) de fusão. b) principal. d) crítica. 57 – A luz branca do Sol ou a luz emitida pelo filamento incandescente de uma lâmpada comum é a) verde clara. c) policromática. b) acromática. d) monocromática. 58 – Dos dispositivos abaixo, qual só pode funcionar com cor- rente elétrica alternada? a) O resistor. c) O transformador. b) O eletroímã. d) A lâmpada incandescente. 59 – O físico alemão George Ohm foi um dos grandes responsáveis pela teoria que explica os fenômenos relacionados com a corrente elétrica. Assim, é justa a homenagem prestada no Sistema Internacional de Unidades, que adota o seu sobrenome como unidade para a grandeza a) diferença de potencial. c) rigidez dielétrica. b) resistência elétrica. d) corrente elétrica. 60 – O barão de Munchausen é considerado o maior mentiroso da literatura internacional. Em uma das suas aventuras, o simpático barão conta que, ao se ver afundando no pântano, conseguiu escapar puxando seus próprios cabelos para cima. A solução proposta pelo barão NÃO está de acordo com a a) 3ª lei de Newton. c) 2ª lei de Newton. b) lei de Arquimedes. d) 1ª lei de Newton. 61 – Em um cristal de cloreto de sódio, a distância entre dois íons adjacentes Na + e Cl – é, aproximadamente, 3 x 10 –10 m. Se o cristal for mergulhado na água, o valor da força de atração entre os íons tornar-se-á DADOS: Constante dielétrica do vácuo = 9 x 10 9 N.m 2 / C 2 Constante dielétrica da água = 9 x 10 7 N. m 2 / C 2 Carga do elétron = 1,6 x 10 – 19 C a) 100 vezes maior. c) 500 vezes maior. b) 100 vezes menor. d) 500 vezes menor. 62 – Substâncias magnéticas são aquelas que permitem a) a orientação dos seus ímãs elementares. b) o deslocamento de íons neutros pelo condutor. c) o armazenamento de energia de origem térmica. d) a orientação cruzada de algumas cargas elétricas. 63 – O campo magnético é um ______________ tendo a mesma ___________das linhas de campo e sendo ____________ a elas. a) vetor – fase – tangente b) vetor – orientação – tangente c) escalar – orientação – perpendicular d) escalar – fase – perpendicular 64 – A correção do estrabismo é feita com o uso de lentes a) côncavo-convexas. c) plano-convexas. b) plano-côncavas. d) prismáticas. 65 – A temperatura de um gás que sofre uma compressão adiabática a) aumenta. c) é invariável. b) diminui. d) pode aumentar ou diminuir. 66 – A diferença de pressão entre dois pontos de uma mesma massa líquida sob a ação da gravidade é igual ao produto da massa específica do líquido pela a) distância entre dois pontos, somente. b) diferença de altura entre os pontos, somente. c) gravidade local e pela diferença de alturas entre os pontos considerados. d) gravidade local e pela distância de ponto até a superfície. 67 – Com relação à massa e ao peso dos corpos é correto afirmar que: a) o peso é uma grandeza escalar e mede a inércia do corpo. b) a massa é uma grandeza vetorial e mede a inércia da matéria. c) o peso é uma grandeza vetorial cuja intensidade independe do local em que é verificado. d) a massa é uma grandeza escalar cuja intensidade independe do local em que é verificado. 68 – Para que a água seja própria para o consumo diário da população, ela precisa passar por alguns processos de trata- mento. Em uma dessas etapas, adiciona-se uma quantidade de hidróxido de alumínio à água, visando à formação de grandes flocos de sujeira. Esse procedimento é realizado para que se otimize o próximo processo de tratamento que é a a) decantação. c) cloração. b) cristalização. d) aeração. 69 – Uma partícula tem sua posição variando segundo a seguinte função horária: S = 8 – 6t (SI). O módulo da velocidade média, em m/s, entre os instantes t1 = 1s e t2 = 5s, vale a) 1,2 b) 5,5 c) 6,0 d) 7,5 70 – Em uma transformação isobárica, ____________permanece constante, considerando-se o fluido como gás ideal e sendo P a pressão, V o volume e T a temperatura do gás. a) o produto VxT c) o produto P x V b) a razão T P d) a razão T V 71 – Dez mols de um gás perfeito evoluem isobaricamente, pas- sando de uma temperatura de 300 K para uma de 400 K. Nessas condições o trabalho realizado na evolução, em atm . litro, vale (dado: constante geral dos gases = R = 0,082 atm . litro / K . mol) a) 41 b) 82 c) 123 d) 164 72 – Uma pessoa, colocada no fundo de uma piscina vazia (sem água), observa um avião que se encontra a 200 metros de altura em relação ao fundo da piscina. Admitindo que os índices de refração do ar e da água sejam, respectivamente, 1 e 1,5, a altura aparente, em metros, observada pela pessoa, do avião vale a) 300 b) 200 c) 100 d) 50 73 – Quando um raio de luz incide na superfície de separação de dois meios X e Y, vindo do meio X para o meio Y, sofre um fenômeno denominado refração da luz. Sabendo que o índice de refração no meio X é maior que o do meio Y, podemos afirmar que o raio refratado a) se afasta da normal. c) incide pela normal. b) se aproxima da normal. d) não sofre desvio. 74 – Três lâmpadas: A, B, e C são ligadas em paralelo. Suas resistências são respectivamente 1,5 ohms; 2,5 ohms e 4,0 ohms. Podemos afirmar que a) o menor brilho é da lâmpada A. b) o maior brilho é da lâmpada C. c) B brilha mais que A e menos que C. d) B brilha mais que C e menos que A. 75 – Quando queremos proteger um aparelho qualquer contra as influências elétricas, nós o envolvemos com uma capa metálica. Isso se justifica devido ao fato de a) os metais serem maus condutores de eletricidade. b) o campo elétrico no interior de um condutor não ser nulo. c) a carga elétrica se distribuir na superfície externa do con- dutor em equilíbrio eletrostático. d) a maioria dos campos elétricos produzidos em circuitos elétricos ser infinitamente pequenos. 76 – Seja uma carga elétrica lançada num campo magnético uniforme, perpendicularmente às linhas de campo. Nessas condições, a sua trajetória será a) elíptica. c) retilínea. b) circular. d) parabólica. 77 – Um solenóide de comprimento L e N espiras, no qual circula uma corrente de intensidade i, apresenta no seu interior um vetor campo magnético de intensidade B. Sendo a permeabilidade magnética do ar µ0, para reduzir à metade a intensidade desse vetor campo magnético deve-se a) dobrar o número de espiras. b) dobrar a intensidade da corrente. c) quadruplicar o número de espiras. d) dobrar o comprimento do solenóide. 78 – Como devem estar ligados cinco capacitores de 2 µF cada um, se a capacidade final da associação é de F 5 12 µ ? a) Todos em série. b) Todos em paralelo. c) Dois em paralelo associado em série com três em paralelo. d) Três em série associado com dois em paralelo. 79 – Dois trens movem-se sobre a mesma reta indo um de encontro ao outro, com velocidades de 72 km/h e 108 km/h. Os dois maquinistas aplicam os freios simultaneamente, quando a distância entre eles é de um quilômetro. Supondo a aceleração escalar dos trens igual a 1m/s 2 , os dois trens a) colidem. b) param a 1m um do outro. c) param a 100m um do outro. d) param a 350m um do outro. 80 – A velocidade é a) uma grandeza vetorial. b) uma grandeza tanto escalar como vetorial. c) uma grandeza escalar. d) não é grandeza 1/2003-TURMA B 41 – A força magnética que age entre duas partículas, que são atraídas uma pela outra, constitui um(a) a) par ação-reação. b) força de contato simples. c) método kepleriano de cálculo virtual. d) força de ação somente a pequenas distâncias. 42 – Dentre as grandezas abaixo, aquela que deve ser motivo de atenção por parte do consumidor, no momento de comprar um eletrodoméstico, para não ter maiores gastos com o consumo de energia elétrica é a) o potencial elétrico. b) a potência elétrica. c) a resistência elétrica. d) a corrente elétrica. 43 – É FALSO afirmar que a) os gases possuem grande expansibilidade. b) os líquidos oferecem grande resistência à compressão. c) somente os líquidos podem ser considerados fluidos perfeitos. d) a viscosidade não influi no estudo dos líquidos em equilíbrio. 44 – Um galvanômetro é um tipo de a) pilha. c) amperímetro. b) wattímetro. d) potenciômetro. 45 – Para a correção da miopia deve-se __________ a distância focal do sistema, o que é conseguido associando-se ao globo ocular uma lente ___________. e) aumentar – divergente f) diminuir – divergente g) diminuir – convergente h) aumentar – convergente 46 – Uma corda de violão vibra com um único ventre. Se ―l‖ é o comprimento dessa onda e ―f‖ a freqüência do som emitido, a velocidade da onda que está se propagando através da corda é a) 21f b) 1f c) 1f/2 d) f/1 47 – Um som que oscila com 2.000 vibrações por segundo é considerado a) som audível. c) ultra-som. b) micro-som. d) infra-som. 48 – A razão de ser impossível ouvir-se uma explosão no Sol é porque a) a Terra está muito distante do Sol. b) as ondas sonoras são mecânicas. c) as ondas de explosão solar são desconhecidas. d) o Sol é um astro que emite apenas ondas luminosas. 49 – Em uma vitrola, a agulha aplica sobre o disco uma força de intensidade 10 - 2 N, aproximadamente. Tendo a ponta da agulha área igual a 10 - 1 0 m 2 , determine a pressão exercida, em bárias, pela agulha no disco: a) 10 7 b) 10 8 c) 10 9 d) 10 1 0 50 – Sabendo-se que a massa de uma partícula o tem valor de 6,67 x 10 – 27 kg, pode-se afirmar que a energia cinética desse tipo de partícula, em 10 – 12 J, que esteja movendo-se com uma velocidade de valor igual a 20.000 km/s, vale: a) 0,224 b) 1,334 c) 2,554 d) 3,333 51 – Considere um tubo de PVC que atravessa uma parede, de forma que você veja apenas uma de suas extremidades. Lançando uma bola dentro do tubo, você observa que, após algum tempo, ela retorna com uma velocidade maior do que aquela com que você a lançou. Diante do acontecido e considerando que o Princípio da Conservação da Energia seja válido, você afirma corretamente que: a) o tubo de PVC está inclinado e a extremidade do outro lado da parede está a uma altura maior que a da extremidade que você observa. b) há alguém do outro do lado da parede que lança a bola com velocidade maior. c) as paredes do tubo não oferecem atrito ao movimento da bola. d) o tubo tem forma de um ―L‖. 52 – Das alternativas abaixo, qual a única que NÃO corresponde a uma propriedade dos ácidos, segundo a definição de Arrhenius. a) Conduz eletricidade. b) Apresenta sabor azedo. c) Torna azul o tornassol vermelho. d) Quando adicionado a um carbonato, produz efervescência com liberação de gás carbônico. 53 – A desintegração de 1 grama de Carbono 14, até restarem 0,5 grama deste elemento, dura 5.600 anos. Por isso seu uso para datação de fósseis. Assim, se o contador Geiger fornecer uma contagem radioativa de 4 1 da contagem que possuímos hoje, por grama do elemento Carbono 14, significa que o objeto examinado existe há a) 1.400 anos. b) 2.800 anos. c) 11.200 anos. d) 22.400 anos. 54 – O raio médio da Terra é de 6.400 km, aproximadamente. A Terra gira em torno de seu próprio eixo, realizando uma rotação completa em 24 h. Considerando-se dois pontos, um na super- fície da Terra e outro a 3.200 km de seu centro, podemos afirmar corretamente que a) os dois pontos terão velocidades angulares diferentes, que os dados fornecidos não permitem calcular. b) a velocidade linear do ponto a 3.200 km do centro será maior que a do ponto na superfície. c) o ponto da superfície terá uma velocidade angular duas vezes maior que o outro. d) os dois pontos terão a mesma velocidade angular. 55 – Nos palitos de fósforos não existe fósforo. Ele está nas tiras (lixas) laterais existentes na caixa, na forma de fósforo vermelho. Pelo atrito palito/tira, o fósforo vermelho se transforma em fósforo branco, liberando energia. O fósforo vermelho e o fósforo branco são chamados de a) isótopos. c) isótropos. b) isóbaros. d) alótropos. 56 – O tipo de ligação química que se caracteriza pela existência de ―elétrons livres‖, também chamados de ―mar de elétrons‖, é a ligação a) iônica. c) metálica. b) simples. d) covalente. 57 – Para se determinar graficamente a resultante de três forças coplanares, que não possuem a mesma direção, o processo mais indicado é o a) do paralelogramo. c) do polígono. b) do triângulo. d) analítico. 58 – A aceleração da gravidade, nas proximidades da superfície terrestre, tem um valor muito alto (aproximadamente 10 m/s 2 ), quando comparado aos valores de aceleração de outros veículos. Se um automóvel tivesse essa aceleração, em apenas 4s, partindo do repouso e desprezando qualquer tipo de atrito, ele atingiria uma velocidade, em km/h, igual a a) 40 b) 72 c) 102 d) 144 59 – Um móvel apresentará aceleração centrípeta não nula, desde que a a) a velocidade linear varie somente em intensidade. b) a velocidade linear varie somente em sentido. c) a trajetória seja curvilínea. d) a trajetória seja retilínea. 60 – Qual a razão entre as distâncias percorridas por dois corpos em queda livre, se a duração de um é o dobro da do outro? Considere que os corpos partam do repouso. a) 2 1 c) 4 1 b) 3 1 d) 5 1 61 – Em um movimento uniformemente variado, a) a aceleração varia uniformemente com o tempo. b) a velocidade varia uniformemente com o tempo. c) o espaço percorrido varia linearmente com o tempo. d) o móvel percorre distâncias iguais, em tempos iguais. 62 – Os números quânticos caracterizam a a) energia do elétron no átomo. b) energia do átomo excitado. c) energia do átomo no estado fundamental. d) distribuição eletrônica em níveis de energia de um átomo. 63 – Observe atentamente o seguinte trecho do Hino Nacional brasileiro: ―Se o penhor desta igualdade, conseguimos conquistar com braço forte...‖ Do ponto de vista da Física, o termo forte é a) corretamente utilizado, tendo em vista que, pela Lei de Hooke, a elasticidade dos músculos do braço faz desse membro fonte de força. b) corretamente utilizado, pois sendo o braço dotado de massa, ele possui força a partir de uma aceleração mínima a ele comunicado. c) incorretamente utilizado, uma vez que o conceito de força está relacionado com a interação entre dois ou mais corpos. d) incorretamente utilizado, já que não se faz referência à massa e a constante elástica dos músculos do braço. 64 – O ar atmosférico filtrado pode ser considerado uma mistura homogênea de vários gases, entre eles o nitrogênio e o oxigênio. Industrialmente se obtém o oxigênio e o nitrogênio, a partir do ar atmosférico filtrado, pelo(s) processo(s) de a) solidificação. b) centrifugação. c) liquefação fracionada. d) liquefação e destilação fracionada. 65 – A força resultante que age sobre um ponto material em movimento circular uniforme em um plano horizontal a) não realiza trabalho. b) tem intensidade nula. c) é tangente à trajetória em cada ponto. d) é diretamente proporcional à velocidade da partícula. 66 – Dois blocos A e B de massas mA = 3 kg e mB = 6 kg estão ligados por um cabo, perfeitamente rígido e homogêneo, cuja B A F  massa vale 1 kg, conforme a figura abaixo. Admitido que o conjunto seja submetido a uma força de 50 N, a tensão, em N, na metade do cabo, vale: Despreze os atritos. a) 25,0 b) 25,5 c) 27,5 d) 32,5 67 – Uma onda sonora propaga-se em um local onde a densidade e a temperatura do ar permanecem constantes, encontrando um anteparo colocado a 170 metros da fonte, refletindo-se sem perdas de energia. O tempo total, em segundos, decorrido entre a emissão e o eco, vale (dado: velocidade do som no ar = 340 m/s) a) 1 b) 2 c) 3 d) 4 68 – Considerando a velocidade do som no ar de 340 m/s, pode-se afirmar que o limite inferior e o superior dos sons audíveis têm, respectivamente, comprimentos de onda da ordem de grandeza de _______ e de ________. a) 10 2 – 10 - 3 c) 10 2 – 10 - 2 b) 10 1 – 10 - 3 d) 10 1 – 10 - 2 69 – Imagine que você tenha a missão de orientar um agente secreto em seu trabalho de espionagem. Admita que o agente deva percorrer o trajeto do ponto A ao ponto D, no menor tempo possível (veja a figura abaixo). Admitindo que o agente corra com uma velocidade quatro vezes maior do que aquela com que ele nade e que essas velocidades possam ser constantes, assinale a alternativa que você indicaria para o agente ter sucesso em sua missão. a) c) b) d) qualquer uma das trajetórias sugeridas acima. 70 – É muito comum ouvirmos, pelos meios de comunicação, as unidades listadas abaixo relacionadas à potência de uma hidroelétrica. Do ponto de vista da Física, qual a única alterna- tiva que contém a unidade correta? a) quilowatts. c) quilowatts . hora. b) quilowatts/hora. d) quilowatts/segundo. 71 – Uma lente delgada convergente é utilizada para conjugar a imagem real de um objeto posto a 40 cm, sobre o eixo focal desta lente. Sabe-se que a altura da imagem produzida vale o triplo da altura do objeto. Desta forma podemos garantir que a vergência desta lente, em dioptrias, vale aproximadamente a) 0,33 b) 3,3 c) 33 d) 333 72 – Uma carga positiva q = 10 – 6 C, de massa 2 x 10 – 14 kg, penetra perpendicularmente em uma região do espaço onde existe um campo magnético uniforme, de 20 tesla, conforme a figura. Qual deve ser a velocidade, em m/s, desta carga para que a trajetória descrita seja uma semicircunferência de diâmetro 0,2 cm? a) 10 10 b) 10 6 c) 2 x 10 6 d) 200 x 10 6 73 – Dois fios condutores X e Y têm o mesmo comprimento, porém a resistência elétrica de X é 4 vezes a de Y e a resistividade de Y é 9 1 da de X. Portanto, conclui-se que a razão de diâmetros Y X é dada por a) 2 3 b) 3 2 c) 2 9 d) 4 3 74 – Numa antiga propaganda de uma grande loja X de departamentos, existia o seguinte refrão: ― – Quem bate? – É o frio! – Não adianta bater, pois eu não deixo você entrar, os cobertores da loja X é que vão aquecer o meu lar!‖ Do ponto de vista da Física, o apelo publicitário é a) correto pois, dependendo da espessura do cobertor, este pode impedir a entrada do frio. b) correto pois, independente da espessura do cobertor, este é um excelente isolante térmico, impedindo a entrada do frio. c) incorreto pois não foi definida a espessura do cobertor. d) incorreto pois não tem sentido falar em frio entrando ou saindo já que este é uma sensação que ocorre quando há trocas de calor entre corpos de diferentes temperaturas. 75 – Um feixe de luz amarela monocromática, no vácuo, tem velo- cidade de 300.000 km/s, aproximadamente. Ao incidir em um determinado meio I, apresenta uma velocidade de 2,0 x 10 8 m/s e ao incidir no meio II, apresenta uma velocidade de módulo V. Sendo o índice de refração relativo n1,2 = 9/8 , o valor de V, em 10 8 m/s, é aproximadamente a) 3,00 b) 2,75 c) 2,50 d) 2,25 76 – Um amperímetro precisa ser colocado em um determinado ponto de um circuito, com o único objetivo de mostrar que está circulando corrente. Porém este amperímetro tem uma resis- tência interna de 10 O e a corrente máxima que ele suporta é de 2,0 A. Considerando que neste circula uma corrente de 12,0 A, o amperímetro deve ser ligado em a) série com um resistor de 2 O. b) série com um resistor de 20 O. c) paralelo com um resistor de 2O. d) paralelo com um resistor de 20 O. 77– A potência de um coração, em watts, que bate setenta vezes por minuto e bombeia 72 cm 3 de sangue em cada batida, contra uma pressão de 12 cmHg, vale aproximadamente Dados: densidade do mercúrio =13,6 g/cm 3 aceleração da gravidade =9,81 m/s 2 a) 12,3 b) 6,05 c) 8,40 d) 1,34 B  - - - - - - C A B D A C D D A 16 m B A margem do rio margem do rio 6 m 8 m C D 16 m B A D C C 78 – A velocidade linear da extremidade do ponteiro das horas de um relógio é de 2700 t cm/s. Sabendo que este ponteiro tem 10 cm de comprimento e comparando este relógio com outro convencional que funciona corretamente (não adianta e nem atrasa), podemos afirmar que, após 48 horas, a) ocorre um atraso de 8 h. b) ocorre um atraso de 9,6 h. c) ocorre um atraso de 12 h. d) os dois apresentam o mesmo tempo decorrido. 79 – Na barragem representada na figura abaixo, sendo H a altura da coluna de água represada, a pressão hidrostática no ponto P, a uma altura h, em relação à base, é a) diretamente proporcional à H. b) inversamente proporcional à h. c) inversamente proporcional à H. d) diretamente proporcional à diferença H – h. 80 – Um fio retilíneo longo é percorrido por uma corrente de 10 ampères, produzindo um campo magnético de intensidade 100 tesla a uma distância x do fio. O valor de x , em cm, é Dado: permeabilidade magnética µ0 =4t x 10 –7 Tm/A a) 2,0 x 10 – 6 c) 2,0 x 10 – 10 b) 2,0 x 10 – 8 d) 2,0 x 10 – 12 2/2003-TURMA A 71 – Se um móvel executa um movimento circular uniforme, de modo que percorra meia volta em 4 s, qual será sua freqüência em Hz? a) 0,0125 b) 0,125 c) 1,25 d) 12,5 72 – Um ponto material desloca-se segundo o gráfico abaixo, onde S é a posição, em metros, do ponto, e t o respectivo instante, em segundos. Sobre a trajetória do ponto podemos afirmar: a) o móvel percorreu uma curva qualquer, com velocidade variável entre zero e 2 segundos. b) o móvel percorreu um segmento de reta com velocidade positiva entre 2 e 3 segundos. c) o móvel percorreu uma trajetória ora retilínea, ora circular. d) não pode, a priori, ser definida. 73 – Quando um sistema de forças concorrentes aplicadas num sólido puder ser reduzido a duas forças de mesma intensidade, mesma direção e sentidos contrários, diremos que o sistema a) possui movimento nulo. b) pode estar em equilíbrio. c) com certeza está em equilíbrio. d) com certeza não está em equilíbrio. 74 – O ponto geométrico sempre no qual se pode considerar concentrada toda massa do corpo (ou do sistema físico) em estudo é o a) incentro. c) núcleo de inércia. b) centro de massa. d) centro geométrico. 75 – Um móvel percorreu 1/3 (um terço) de sua trajetória com a velocidade média de 80 m/s. O restante do percurso, o móvel completa com a velocidade média de 40 m/s. A velocidade média desse móvel, em m/s, durante todo percurso, vale a) 44,0. b) 48,0. c) 53,3. d) 60,0. 76 – ANULADA 77 – A aceleração da gravidade, normalmente representada pela letra g, SEMPRE é a) uma grandeza vetorial. b) uma grandeza constante. c) a força gravitacional dos corpos em queda, no vácuo. d) a constante da gravitação universal de Newton. 78 – As rodas de um automóvel, que podem ser consideradas circunferências, possuem um comprimento de 2,10 m. Se estas efetuarem 240 rpm, calcule a velocidade de um ponto na periferia da roda, em m/s, admitindo que a rotação das rodas constitua um movimento circular uniforme. a) 8,4 b) 16,8 c) 84,0 d) 168,0 79 – Um trator está puxando um bloco a velocidade constante de 2m/s, utilizando uma força constante e horizontal de 100 N. O peso do bloco é de 200 N. O coeficiente de atrito entre o solo e o bloco é a) 0,1. b) 0,4. c) 0,5. d) .0,75 80 – Um motor de 5 HP, funcionando durante 8 minutos, produz um trabalho, em J, igual a (dado: 1HP = 746 W) a) 466,25. c) 29840,00. b) 3730,00. d) 1790400,00. 81 – Um armário pesa 160 kgf e possui 4 pés com área de 4 cm 2 cada um. A pressão, em kgf/cm 2 , que cada pé exerce no piso horizontal onde se apóia, é a) 2,5. b) 5,0. c) 10,0. d) 40,0. 82 – O equilíbrio de uma coluna de mercúrio em um barômetro de Torricelli não depende do (a) a) gravidade local. b) diâmetro do tubo barométrico. c) densidade do mercúrio dentro da cuba. d) pressão do ar sobre a superfície livre do mercúrio. 83 – Em vasos comunicantes, líquidos não-miscíveis e de den- sidades diferentes apresentam a) alturas que não dependem das densidades. b) superfícies livres no mesmo plano horizontal. c) alturas inversamente proporcionais às suas densidades. d) alturas diretamente proporcionais às suas densidades. 84 – Uma corda horizontal com uma extremidade presa a uma parede é posta a vibrar pela outra extremidade. A distância entre o primeiro e o quarto nó, da onda estacionária obtida, é 60 cm. O comprimento de onda, em centímetros, que dá origem à onda estacionária, é de a) 15. b) 20. c) 30. d) 40. 85 – ANULADA 86 – Considere os corpos A, B, C e D, indicados na figura abaixo, colocados no vácuo. Sendo TA, TB, TC e TD as temperaturas dos corpos A, B, C e D, respectivamente, onde TA < TB, TD > TC e TC > TB. Indique a alternativa que informa o modo de propagação de calor a) somente irradiação. b) somente condução. c) condução e convecção. d) irradiação e convecção. 87 – Uma barra metálica de comprimento L0 a 0ºC sofreu um aumento de comprimento de 200 1 do seu comprimento inicial, quando aquecida a 125ºC. Pode-se dizer que o coeficiente de dilatação linear do metal, em ºC -1 , é a) 2 x 10 -10 . b) 4 x 10 -5 . c) 6 x 10 -4 . d) 1 x 10 3 . 88 – Um chuveiro quando ligado à rede de 220 V exige uma corrente igual a 11 A. A potência elétrica, em kW, desse chuveiro é a) 0,20. b) 24,20. c) 20,00. d) 2,42. H h P - p 2p V/2 V p V 1 2 - - A - - B 8µF 4µF 8µF 8µF 8µF - - 20 19 2 1 - 89 – O gráfico abaixo mostra uma expansão isotérmica de um gás ideal, de uma situação 1 para uma final 2. Supondo que a área sob o gráfico p x V (onde p é a pressão e V é o volume) possa ser aproximada para a área de um trapézio retângulo, o calor envolvido no processo, em módulo, vale a) 0,75 p·V b) p·V c) 0,5 p·V d) 1,3 p·V 90 – Um edifício projeta uma sombra horizontal de 52 m de comprimento, a partir de sua base, quando o Sol está a 60º em relação ao horizonte. Nessas condições, a altura, em metros, do edifício vale aproximadamente (considere 73 , 1 3 = e que os raios do Sol "chegam" ao edifício praticamente paralelos). a) 90 b) 100 c) 110 d) 120 91 – Uma partícula de massa m e carga Q foi colocada entre duas placas carregadas, que geram um campo elétrico vertical ascendente de intensidade E. Sendo g a aceleração da gravidade no local, é correto afirmar que para essa partícula permanecer em repouso deve se ter a) E mg Q = . b) gE m Q = . c) E Qg m = . d) QE g m = . 92 – Um objeto real de 10 cm de altura está colocado frontalmente a 30 cm de uma lente convergente, perpendicular ao eixo focal da mesma, cuja vergência é de 10 dioptrias. A distância da imagem à lente vale, em cm, a) 0,1. b) 10,0. c) 15,0. d) 30,0. 93 – Dada a associação dos capacitores abaixo, a capacidade equivalente entre os pontos A e B é dada por ______ µF. a) 2,7 b) 8,0 c) 12,0 d) 16,0 94 – Um fio reto é percorrido por uma corrente elétrica constante de intensidade 6,2 A. Calcule a intensidade do campo magnético, em 10 – 7 tesla, a 20 cm do fio.Dado: µ = t 4 x 10 – 7 T. m/A Obs.: considerar o fio infinito a) 31 b) 31 t c) 62 d) 62 t 95 – Considere as seguintes afirmações a respeito de misturas. I- Os gases sempre formam misturas homogêneas. II- Água e sal comum sempre formam misturas homogêneas. III- O álcool hidratado é uma mistura homogênea. Podemos dizer que são corretas as afirmações: a) I e III. b) I, II e III. c) II e III. d) apenas I. 96 – Em uma região onde existe um campo magnético uniforme, são lançadas, perpendicularmente às linhas de campo três partículas A, B e C, com velocidades iguais, tal como a figura. De acordo com as trajetórias descritas pelas partículas, pode-se afirmar que A, B e C tem, respectivamente, cargas elétricas a) positiva, neutra e negativa. b) positiva, negativa e neutra. c) negativa, neutra e positiva. d) negativa, positiva e neutra. 97 – A espécies químicas gás carbônico (CO2), hidrogênio H2), oxigênio (O2), e sal comum (NaCl), são classificadas, respectivamente, como substâncias puras: a) composta, simples, composta, simples. b) composta, simples, simples, composta. c) simples, composta, composta, simples. d) composta, simples, simples, simples. 98 – As reações químicas são representadas através das equações químicas, onde o número total de átomos presente nos reagentes é igual ao número total de átomos presente nos produtos. Considerando que a equação da reação entre o ácido sulfúrico e o alumínio tem a seguinte representação: x H2SO4 + y Al z Al2(SO4)3 + w H2 A soma dos valores de x, y, z e w é igual a a) 10 (dez) c) 9 (nove) b) 8 (oito) d) 7 (sete) 99 – Os elementos Ca e Cl, em certas condições podem originar os íons Ca +2 e Cl -1 . Isso ocorre porque a) o Ca "ganhou" elétrons e o Cl perdeu elétron. b) ambos os elementos "perderam" elétrons. c) ambos os elementos "ganharam" elétrons. d) o Ca "perdeu" elétrons e o Cl "ganhou" elétron. 100 – A formação das nuvens e a precipitação da chuva, entre outros fenômenos, exige que haja a) evaporação das águas dos oceanos, rios, lagos e mares e a condensação do vapor d‘água existente no ar. b) a evaporação das águas dos oceanos, rios, lagos e mares e a fusão do vapor d‘água existente no ar. c) a condensação das águas dos oceanos, rios, lagos e mares e a condensação do vapor d‘água existente no ar. d) a solidificação das águas dos oceanos, rios, lagos e mares e a condensação do vapor d‘água existente no ar. 2/2003-TURMA B 71 – Um corpo em queda livre percorre, a partir do repouso, uma certa distância 1 h nos dois primeiros segundos de queda. A distância 2 h percorrida do início do terceiro ao final do sexto segundo, será quantas vezes maior que 1 h ? a) 4 b) 6 c) 8 d) 16 72 – Uma pessoa de massa 50 kg sobe uma escada de 20 degraus, com 20 cm de altura cada degrau. A escada tem inclinação de 60º com a horizontal. Sendo g = 10 m/s 2 e cos 60º = 0,5, o trabalho resistente, em joules, realizado pelo peso do homem foi de a) 1.000. b) 2.000. c) 3.000. d) 4.000. 73 – Uma nave extraterrestre está orbitando o planeta Terra em uma altitude na qual a aceleração da gravidade é 4,2 m/s 2 . Tal altitude, em 10 3 km, vale, aproximadamente, Dados: G = const. univ. de gravitação ~ 2 2 11 kg / m . N 10 0 , 7 ÷ × M = massa da Terra ~ kg 10 0 , 6 24 × R = raio da Terra ~ 6.400 km a) 3,6. b) 4,5. c) 6,5. d) 10. 74 – Consideremos um tijolo apoiado sobre sua face maior. Colocando-o apoiado sobre sua face menor, cuja área é um terço da maior, a pressão a) triplica. c) nonuplica. b) não varia. d) reduz-se a um terço. 75 – Um cubo de 10 cm de aresta e densidade absoluta 8,0 g/cm 3 está apoiado por uma face sobre um plano inclinado de 60º em relação à horizontal. Admitindo que o cubo esteja em repouso, a pressão que este exerce sobre o plano é de ________ bárias. Dados: cos 60º = 0,5 g = 10 m/s 2 (aceleração da gravidade local) a) 40 b) 400 c) 4.000 d) 40.000 76 – ANULADA - - 30º o F  A B L 77 – Um trovão foi ouvido por uma pessoa 15 s após esta ter visto o relâmpago. A que distância aproximada, em km, "caiu" o raio? Dado: velocidade do som no ar = 340 m/s a) 2,4 b) 5,1 c) 510 d) 5.100 78 – Um objeto real está colocado frontalmente e a 30 cm de uma lente divergente de vergência 2 dioptrias. A razão tamanho de objeto por tamanho de imagem vale a) 5 / 8. b) 8 / 5. c) 15 / 8. d) 8 / 15. 79 – Calcular a quantidade de calor, em calorias, que atravessa uma placa de ferro de 3 cm de espessura em uma hora, sendo de 1 cm 2 a superfície da mesma e de 150ºC a diferença de temperatura entre as faces. Dado: coeficiente de condutibilidade 1 1 1 12 0 ÷ ÷ ÷ ° = C s m cal K . . , a) 216 b) 432 c) 648 d) 1.800 80 – Um corpo, em repouso, está apoiado em um plano inclinado. O peso deste corpo e sua componente paralela ao citado plano estão na razão 2 :1. O valor do ângulo, em graus, entre o plano inclinado e a direção horizontal vale a) 30º. b) 45º. c) 60º. d) 90º. 81 – Dois móveis A e B percorrem a mesma pista circular com movimentos uniformes, partindo do mesmo ponto e caminhando no mesmo sentido. A velocidade angular de A é o triplo da velocidade angular de B e 0,5 s após a partida eles se encontram pela primeira vez. A velocidade angular de B, em rad/s, vale Dado: t = 3,14 a) 2,00. b) 3,00. c) 3,14. d) 6,28. 82 – A figura a seguir mostra uma barra homogênea de peso P  e comprimento AB = L. Esta barra possui uma articulação na extremidade A e está em equilíbrio devido à aplicação de uma força F  na extremidade B. Qual deve ser o valor do ângulo o, em graus, para que o equilíbrio seja mantido? Dados: N 3 F e N 2 2 P = =   a) 135º b) 120º c) 60º d) 45º 83 – Considere um ônibus em movimento e um observador, externo a este, em repouso. Se, num determinado instante, o observador vê, dentro do ônibus, uma mosca voando em linha reta ao longo do comprimento deste, podemos afirmar que a mosca possui uma velocidade cujo módulo é Dado: considere que a mosca voa na mesma direção e no mesmo sentido do deslocamento do ônibus. a) nulo. b) igual ao módulo da velocidade do ônibus. c) maior que o módulo da velocidade do ônibus. d) menor que o módulo da velocidade do ônibus. 84 – Durante uma competição de natação, em uma piscina olímpica (50 metros), um nadador parte de uma das extremidades desta piscina com uma velocidade inicial nula. Admitindo que o movimento do nadador, nesta prova, seja retilíneo uniformemente variado, qual a aceleração aproxima- da, em m/s 2 , deste nadador ao chegar na outra extremidade da piscina? Considere que o tempo de percurso seja de 8 segundos. a) 1,0 b) 1,5 c) 2,0 d) 2,5 85 – Lançam-se, separadamente e em um campo magnético constante, duas partículas P e Q de mesma carga elétrica e velocidades perpendiculares a tal campo. Sabe-se que as quantidades de movimento de ambas são iguais e permanecem constantes no tempo. Admitindo que a massa de P é 10 vezes a massa de Q, pode-se afirmar que a partícula de a) maior período tem maior velocidade. b) maior freqüência tem maior massa. c) menor massa tem maior freqüência. d) maior velocidade tem maior massa. 86 – Certa onda, propagando-se no ar, possui um comprimento de onda igual a 10,0 cm e velocidade de propagação de 340 m/s. Qual será o comprimento de onda desta, em metros, ao passar para um meio onde a velocidade de propagação é de 1,36 km/s? a) 0,04 b) 0,40 c) 2,50 d) 2.500 87 – Um grande mestre na área da Física se deparou com a seguinte situação hipotética: possuía duas espiras circulares concêntricas 1 E e 2 E , com raios cm 10 r 1 = e mm 200 r 2 = , respectivamente, percorridas por correntes de sentidos opostos. Sabe-se que, na espira de diâmetro menor, a corrente é de 20 A e que a intensidade (em módulo) do campo magnético no centro das espiras é de T 10 14 , 3 5 ÷ × . Nessa situação, quais devem ser as intensidades, em ampéres, das correntes elétricas aplicadas na outra espira, visto que o mestre encontrou dois resultados possíveis para o problema. Dado: permeabilidade magnética do meio = A m T. 7 10 4 ÷ × t considere 14 , 3 = t a) 5 e 15 b) 5 e 30 c) 15 e 50 d) 30 e 50 88 – Aquece-se uma certa massa de gás ideal a volume constante de 27ºC até 127ºC. Pode-se afirmar que a razão entre as energias cinéticas médias das moléculas, depois e antes do aquecimento, é de a) 4 3 . b) 3 4 . c) 27 127 . d) 127 27 . 89 – N espiras circulares, geometricamente iguais e justapostas (bobinas chatas), são percorridas por uma corrente de 2,0 ampères. O raio dessas espiras vale 4 cm. Admitindo que a permeabilidade magnética (µO) seja A m . T 10 4 7 ÷ × t , o valor da intensidade do campo magnético por espira, em 5 10 ÷ tesla, é a) t / 2 b) t c) 2t d) 4t 90 – No gráfico a seguir, o trabalho, em atm.cm 3 , executado pelo gás entre os estados A e B vale a) 4. b) 8. c) 16. d) 32. 91 – Um corpo, inicialmente neutro, foi eletrizado negativamente com uma carga elétrica de C 15 10 2 7 ÷ × , . Sabendo que o valor da carga elementar é de C 19 10 6 1 ÷ × , , pode-se afirmar que esse corpo _____________ 3 10 × elétrons. a) ganhou 45 c) perdeu 45 b) ganhou 90 d) perdeu 90 P 4 4 8 A B v (cm 3 ) A B 92 – Dois automóveis A e B se deslocam na mesma direção, em um trecho retilíneo da estrada, em sentidos contrários e indo um ao encontro do outro, com velocidades constantes de 36 e 18 km/h, respectivamente. Após passarem um pelo outro, o motorista do carro A observa, através do retrovisor (plano), o carro B se deslocando. Nessas condições, pode-se afirmar que a velocidade, em m/s, da imagem do carro B vista pelo motorista do carro A, pelo espelho, vale (considere o motorista do carro A como referencial) a) 15. b) 30. c) 54. d) 108. 93 – Considere duas placas paralelas separadas por uma distância mm d 16 = , entre as quais se estabelece um campo elétrico uniforme C N E 4 10 2× = . Admitindo que um elétron seja liberado, a partir do repouso, na extremidade da placa negativa, determine a velocidade aproximada, em s m 7 10 , do elétron, ao chegar à placa positiva. Dado: Carga do C elétron 19 10 6 1 ÷ × = , Massa do kg elétron 31 10 9 ÷ × ~ a) 1,1 b) 2,7 c) 5,2 d) 7,2 94 – Na associação de pilhas de 1,5 V do desenho abaixo, a diferença de potencial entre os pontos A e B é, em volts, a) 1,5 b) 3,0 c) 4,5 d) 6,0 95 – Einstein supôs que a energia "E" do "pacote" (ou fóton) está relacionada com sua freqüência "v" e é dada pela equação: E = hv. Neste caso, se "v" é dada no Sistema Internacional de Unidades (SI), ou seja 1 s ÷ , a unidade de "h", também dada no SI, é a) J.s b) J. 1 s ÷ c) J. 2 s d) J. 2 s ÷ 96 – Automóveis que ficam ao "relento" da noite para o dia, amanhecem com suas superfícies "molhadas". Esse fenômeno ocorre devido à a) condensação do vapor de água que existe no ar. b) condensação do vapor de água e do gás carbônico que existem no ar. c) solidificação do vapor de água e do gás carbônico que existem no ar. d) solidificação do vapor de água e condensação do gás carbônico que existem no ar. 97 – O átomo do elemento cloro é representado por: 35 17 Cl , sendo que em certas condições forma o ânion monovalente representado por 1 Cl ÷ . Nesse ânion existem: a) 18 prótons, 18 elétrons e 18 nêutrons. b) 17 prótons, 17 elétrons e 18 nêutrons. c) 17 prótons, 18 elétrons e 18 nêutrons. d) 17 prótons, 18 elétrons e 19 nêutrons. 98 – As espécies químicas representadas pelas fórmulas 3 2 O Al , 4 3 PO H , KOH e 4 2 SO Na pertencem, respectivamente, às funções: a) base, ácido, óxido e sal. b) sal, ácido, base e óxido. c) óxido, ácido, sal e base. d) óxido, ácido, base e sal. 99 – Um tanque de um automóvel contém 50 L de álcool hidratado, que é uma mistura constituída por 96% de álcool anidro e 4% de água (porcentagens em volume). Podemos afirmar que dentro desse tanque existe aproximadamente: a) 48 L de álcool anidro e 2 L de água. b) 2 L de álcool anidro e 48 L de água. c) 25 L de álcool anidro e 25 L de água. d) 40 L de álcool anidro e 10 L de água. 100 – ANULADA GABARITO DAS PROVAS DE FÍSICA e QUÍMICA 1 1/2001 - TURMA A 26 C 27 D 28 B 29 C 30 B 31 A 32 B 33 A 34 A 35 A 36 C 37 D 38 B 39 D 40 B 41 C 42 D 43 A 44 C 45 D 46 B 47 C 48 D 49 A 50 C 1/2001 - TURMA B 26 C 27 A 28 D 29 C 30 A 31 C 32 D 33 A 34 C 35 D 36 B 37 D 38 B 39 C 40 D 41 B 42 A 43 B 44 A 45 C 46 A 47 B 48 C 49 D 50 B 2/2001 - TURMA A 26 A 27 B 28 C 29 B 30 C 31 D 32 A 33 C 34 D 35 C 36 B 37 D 38 A 39 B 40 D 41 C 42 A 43 B 44 A 45 C 46 B 47 D 48 A 49 B 50 D 2/2001 - TURMA B 26 A 27 C 28 A 29 D 30 B 31 D 32 B 33 D 34 A 35 B 36 C 37 B 38 D 39 A 40 D 41 B 42 C 43 A 44 C 45 B 46 C 47 A 48 C 49 A 50 D 1/2002 - TURMA A 01 D 02 A 03 B 04 D 05 A 06 B 07 A 08 D 09 A 10 B 11 C 12 B 13 D 14 B 15 C 16 B 17 D 18 B 19 D 20 B 21 B 22 D 23 B 24 B 25 B 26 D 27 D 28 C 29 B 30 C 31 C 32 B 33 A 34 A 35 C 36 D 37 B 38 A 39 C 40 B 1/2002 - TURMA B 72 01 D 02 C 03 A 04 B 05 C 06 A 07 D 08 B 09 B 10 C 11 A 12 C 13 D 14 B 15 A 16 C 17 C 18 B 19 B 20 B 21 B 22 D 23 D 24 A 25 ANULADA 26 C 27 B 28 A 29 C 30 A 31 A 32 D 33 B 34 D 35 B 36 A 37 D 38 D 39 B 40 A 2/2002 - TURMA A 41 B 42 D 43 D 44 B 45 C 46 A 47 C 48 C 49 D 50 A 51 B 52 C 53 A 54 A 55 B 56 D 57 C GABARITO DAS PROVAS DE FÍSICA e QUÍMICA 2 58 D 59 A 60 C 61 B 62 D 63 B 64 A 65 D 66 B 67 D 68 B 69 C 70 A 71 B 72 C 73 A 74 C 75 A 76 D 77 C 78 D 79 A 80 B 2/2002 - TURMA B 41 B 42 A 43 D 44 B 45 B 46 B 47 A 48 A 49 B 50 C 51 A 52 B 53 A 54 B 55 D 56 C 57 A 58 D 59 B 60 D 61 A 62 D 63 C 64 B 65 B 66 A 67 B 68 C 69 A 70 B 71 B 72 D 73 B 74 A 75 ANULADA 76 C 77 C 78 D 79 D 80 A 1/2003 - TURMA A 41 B 42 D 43 C 44 B 45 D 46 A 47 A 48 C 49 D 50 C 51 C 52 C 53 A 54 B 55 B 56 D 57 C 58 C 59 B 60 A 61 B 62 A 63 B 64 D 65 A 66 C 67 D 68 A 69 C 70 D 71 B 72 B 73 A 74 D 75 C 76 B 77 D 78 C 79 D 80 A 1/2003 - TURMA B 41 A 42 B 43 C 44 C 45 A 46 B 47 A 48 B 49 C 50 B 51 B 52 C 53 C 54 D 55 D 56 C 57 C 58 D 59 C 60 C 61 B 62 A 63 C 64 D 65 A 66 D 67 A 68 D 69 B 70 A 71 B 72 B 73 A 74 D 75 D 76 C 77 D 78 B 79 D 80 A 2/2003 - TURMA A 71 B 72 D 73 B 74 B 75 B 76 ANULADA 77 A 78 A 79 C 80 D 81 C 82 B 83 C 84 D 85 ANULADA 86 A 87 B GABARITO DAS PROVAS DE FÍSICA e QUÍMICA 3 88 D 89 A 90 A 91 A 92 C 93 C 94 C 95 A 96 A 97 B 98 C 99 D 100 A 2/2003 - TURMA B 71 C 72 B 73 A 74 A 75 D 76 ANULADA 77 B 78 B 79 A 80 B 81 D 82 A 83 C 84 B 85 C 86 B 87 D 88 B 89 B 90 C 91 A 92 A 93 A 94 B 95 A 96 A 97 C 98 D 99 A 100 ANULADA
Copyright © 2024 DOKUMEN.SITE Inc.